At least 100 of these will be on the OB test

Réussis tes devoirs et examens dès maintenant avec Quizwiz!

Which nursing interventions would best correlate with the nursing diagnosis of At risk for disturbed sleep patterns related to night sweats? 1. Take a cool shower before bedtime. 2. Avoid medications such as fluoxetine. 3. Eat a consistent carbohydrate prior to bedtime. 4. Avoid liquid intake 2 hours prior to bedtime.

ANS: 1 Chapter: Chapter 18 Well Women's Health Chapter Learning Objective: 5. Discuss the physical and emotional changes related to perimenopause and menopause. Page: 563 Heading: Reproductive Changes Across the Life Span > Menopause > Treatment for Specific Menopausal Discomforts Integrated Processes: Teaching and Learning Client Need: Physiological Adaptation: Basic Care and Comfort Cognitive Level: Application [Applying] Concept: Promoting Health Difficulty: Moderate Feedback 1 This is correct. This would facilitate a cooler body core prior to going to bed and may decrease hot flashes. 2 This is incorrect. Fluoxetine has been shown to decrease hot flashes. 3 This is incorrect. This would be an intervention for increased blood sugar control for a diabetic. 4 This is incorrect. This is an intervention for enuresis, not hot flashes.

The nurses in a postnatal unit are aware of the fears of new parents with regard to infant abduction. Which interventions by the nurse will alleviate the concerns of the parents? Select all that apply. 1. Allow only visitors with identification to enter the unit. 2. Use the hospital abduction alarm systems. 3. Require unit personnel to wear specific name tags. 4. Footprints and a photo of the neonate are taken for identification purposes. 5. Encourage parents to accompany persons transporting the newborn.

ANS: 1, 2, 3, 4, 5 Chapter: Chapter 15 Physiological and Behavioral Responses of the Neonate Chapter Learning Objective: 5. Describe the nursing care for neonates during the first week of life. Page: 469 Heading: Nursing Care of the Neonate > Nursing Actions From 4 Hours of Age to Discharge Integrated Processes: Nursing Process Client Need: Safe and Effective Care Environment: Safety and Infection Control Cognitive Level: Analysis [Analyzing] Concept: Ante/Intra/Post-partum Difficulty: Difficult Feedback 1 This is correct. An effective safety measure to prevent newborn abduction is to allow only visitors with identification to enter the unit. 2 This is correct. Hospital alarm systems are extremely effective in preventing newborn abduction. A neonate attachment will trigger an alarm, lock doors, and freeze elevators if the newborn comes within 4 feet of an exit or elevator. 3 This is correct. Requiring personnel working in the maternal-newborn units to wear name tags specific to that unit is reassuring and effective. Name tags should have a photo along with the name of the person. 4 This is correct. Taking footprints and a photo of the newborn for identification purposes is effective against abduction but is also appropriate for situations involving concerns about "baby switching." 5 This is correct. Encouraging parents to accompany any person who removes their infant from the mother's room is an additional action to prevent newborn abductions and alleviate parenteral concern.

The nurse is concerned about the number of infants in the community who die from SIDS even with teaching about "back to sleep" being provided. On which additional preventive measures will the nurse focus? Select all that apply. 1. During pregnancy, women should not smoke, drink alcohol, or use illegal drugs. 2. Infants need to be dressed to prevent infants from overheating during sleep. 3. Mothers need to be informed that breastfeeding reduces the risk for SIDS. 4. Parents should not smoke or allow smoking around their baby. 5. Parents need to avoid products that claim to reduce the risk of SIDS.

ANS: 1, 2, 3, 4, 5 Chapter: Chapter 16 Discharge Planning and Teaching Chapter Learning Objective: 8. Provide parents with information regarding newborn care that reflects the assessed learning needs of parents. Page: 498 Heading: Newborn Care > Safe Sleep Integrated Processes: Nursing Process Client Need: Physiological Integrity: Reduction of Risk Potential Cognitive Level: Analysis [Analyzing] Concept: Ante/Intra/Post-partum Difficulty: Difficult Feedback 1 This is correct. The National Institute of Child Health and Human Development (2017) and American Academy of Pediatrics (2016c) recommend that during pregnancy, women should not smoke, drink alcohol, or use illegal drugs. 2 This is correct. The National Institute of Child Health and Human Development (2017) and American Academy of Pediatrics (2016c) recommend that infants need to be dressed to prevent infants from overheating during sleep. 3 This is correct. The National Institute of Child Health and Human Development (2017) and American Academy of Pediatrics (2016c) recommend that mothers need to be informed that breastfeeding reduces the risk for SIDS. 4 This is correct. The National Institute of Child Health and Human Development (2017) and American Academy of Pediatrics (2016c) recommend that parents should not smoke or allow smoking around their baby. 5 This is correct. The National Institute of Child Health and Human Development (2017) and American Academy of Pediatrics (2016c) recommend that parents need to avoid products that claim to reduce the risk of SIDS.

A patient at 34 weeks gestation is in labor with twins. The primary care provider decides the fetuses need to be delivered by cesarean. Which medical and nursing interventions will be in place for this delivery? Select all that apply. 1. Delivery is attended by two medical personnel. 2. The placement of a large-bore IV access is ensured. 3. The usual personnel to attend delivery is arranged. 4. A hospital with a Level II or III nursery is selected. 5. The FHR for the two fetuses is monitored alternately.

ANS: 1, 2, 4 Chapter: Chapter 10 High-Risk Labor and Birth Chapter Learning Objective: 5. Describe the key obstetrical emergencies and the related medical and nursing care. Page: 329 Heading: Obstetric Complications > Multiple Gestation Birth Integrated Processes: Nursing Process Client Need: Physiological Integrity: Reduction of Risk Potential Cognitive Level: Analysis [Analyzing] Concept: Ante/Intra/Post-partum Difficulty: Difficult Feedback 1 This is correct. For multiple births, either two experienced obstetricians or one obstetrician and a board-certified midwife will attend the delivery. 2 This is correct. The nurse will make sure that a large-bore IV access is in place for fluid replacement in case of hemorrhage or need for emergency fluid replacement and anesthesia administration. 3 This is incorrect. For the delivery of multiple fetuses, the usual personnel for surgery will be required. Personnel will include an anesthesia provider, a scrub nurse, and a circulating nurse. 4 This is correct. Due to the possibilities of fetal distress or need of special care related to immaturity, the cesarean needs to be performed in a hospital with either a Level II or III nursery. 5 This is incorrect. The nurse will continue to monitor both fetuses simultaneously and continuously. Once membranes are ruptured, internal monitoring for twin A is initiated. The well-being of one twin does not ensure the well-being of the other twin.

The nurse-manager on a labor and delivery unit is monitoring the reasons for cesarean births at the facility. Which reasons contribute to the high rates of cesarean births? Select all that apply. 1. Fetuses in breech position unable to deliver vaginally 2. Increased number of elective or maternal request cesareans 3. Incidences of women of older maternal age getting pregnant 4. Decreasing rate of malpractice litigation with cesarean birth 5. Presence of nonreassuring fetal tracings during labor

ANS: 1, 2, 3, 5 Chapter: Chapter 11 Intrapartum and Postpartum Care of Cesarean Birth Families Chapter Learning Objective: 1. Identify factors that place a woman at risk for cesarean birth. Page: 346 Heading: Box 11-1: Indications for Cesarean Birth Integrated Processes: Nursing Process Client Need: Physiological Integrity: Reduction of Risk Potential Cognitive Level: Analysis [Analyzing] Concept: Ante/Intra/Post-partum Difficulty: Difficult Feedback 1 This is correct. Fetuses in breech position that cannot be delivered vaginally are delivered by cesarean, which accounts for 17% of the cesarean births in the United States. 2 This is correct. Increase in the number of cesarean deliveries on maternal request increases the overall rate; however, this reason accounts for only 3% of the cesarean births in the United States. 3 This is correct. Increase in the maternal age at delivery increases the rate of cesarean births. However, this reason accounts for between 4% and 5% of cesarean births. 4 This is incorrect. Cesarean births have the potential of increasing malpractice litigation because of the high-risk causes and general risks related to surgery. 5 This is correct. The presence of nonreassuring fetal tracings during labor accounts for 23% of cesarean births.

Following a cesarean birth, intrathecal morphine is administered to the patient for postoperative pain management. Of which fact about intrathecal morphine therapy is the nurse aware? Select all that apply. 1. An anesthesiologist or CRNA administers it intrathecally. 2. The nurse needs to closely monitor for common side effects. 3. The drug produces generalized CNS depression. 4. The recommended dose is 10 to 15 mg. 5. The drug alters perception of and response to painful stimuli.

ANS: 1, 2, 3, 5 Chapter: Chapter 11 Intrapartum and Postpartum Care of Cesarean Birth Families Chapter Learning Objective: 5. Identify potential intraoperative and postoperative complications related to cesarean birth and nursing actions to reduce risk. Page: 355 Heading: Postoperative Care > First 24 Hours After Birth > Anesthesia Management Integrated Processes: Nursing Process Client Need: Physiological Integrity: Reduction of Risk Potential Cognitive Level: Analysis [Analyzing] Concept: Ante/Intra/Post-partum Difficulty: Difficult Feedback 1 This is correct. Intrathecal morphine is always administered to the patient by an anesthesiologist or CRNA (certified registered nurse anesthetist). 2 This is correct. The nurse will need to closely monitor for side effects of the drug. Common side effects include respiratory depression, itching, hypotension, nausea and vomiting, and urinary retention. 3 This is correct. The nurse needs to be aware that intrathecal morphine will cause generalized CNS depression. 4 This is incorrect. Recommended dose of intrathecal morphine is 5 to 10 mg. 5 This is incorrect. Intrathecal morphine is effective because it alters perception of and response to painful stimuli.

The nurse is making a plan of care for a patient who is in the first 24-hour period past a cesarean delivery. Which interventions will the nurse include in regards to medications? Select all that apply. 1. Continue a daily stool softener. 2. Manage pain with morphine. 3. Ensure the availability of naloxone. 4. Provide prophylaxis antibiotics. 5. Administer Rhogam if needed.

ANS: 1, 2, 3, 5 Chapter: Chapter 11 Intrapartum and Postpartum Care of Cesarean Birth Families Chapter Learning Objective: 5. Identify potential intraoperative and postoperative complications related to cesarean birth and nursing actions to reduce risk. Page: 360 Heading: Clinical Pathway for Scheduled Cesarean Birth: Medication Integrated Processes: Nursing Process Client Need: Physiological Integrity: Reduction of Risk Potential Cognitive Level: Analysis [Analyzing] Concept: Ante/Intra/Post-partum Difficulty: Difficult Feedback 1 This is correct. Within the first 24 hours and beyond, the nurse will continue administration of a daily stool softener. Constipation related to opiate pain control is common. 2 This is correct. Following a cesarean, morphine is administered for management of post-surgery pain. 3 This is correct. Naloxone is administered for any signs of morphine overdose. 4 This is incorrect. Prophylaxis antibiotics are administered prior to the cesarean delivery to prevent infection. 5 This is correct. If the mother is Rh-negative, Rhogam is given within the first 24 hours.

The nurse is performing a uterus assessment on a patient who is 20 hours postpartum. The nurse finds the fundus of the uterus to be soft and boggy. In addition, the uterus is displaced to the left and moderate bleeding is noted. If the uterus does not respond to uterine massage, which actions does the nurse implement? Select all that apply. 1. Assist the patient to the bathroom to void. 2. Reassess to determine response to treatment. 3. Administer oxytocin as prescribed. 4. Place an emergency call to the HCP. 5. Make the patient NPO for surgery.

ANS: 1, 2, 3, 5 Chapter: Chapter 12 Postpartum Physiological Assessments and Nursing Care Chapter Learning Objective: 2. Identify the critical elements of assessment and nursing care during the postpartum period. Page: 368 Heading: The Reproductive System > Uterus > Nursing Actions Integrated Processes: Nursing Process Client Need: Physiological Integrity: Physiological Adaptation Cognitive Level: Analysis [Analyzing] Concept: Ante/Intra/Post-partum Difficulty: Difficult Feedback 1 This is correct. Because of the displacement of the uterus to the left, the nurse concludes that a full bladder may be the cause. The nurse needs to assist the patient to the bathroom to void. 2 This is correct. After the nurse implements all nursing and prescribed interventions, the nurse will reassess the uterus to evaluate and determine the response to treatment. 3 This is correct. When the uterus is boggy and bleeding is moderate, the nurse will administer oxytocin as prescribed in the HCP's postpartum orders. 4 This is incorrect. The nurse will contact the HCP after interventions are implemented and reassessment has taken place. There is no need to place an emergency call to the HCP prior to this. An emergency call is appropriate if improvement is not noted. 5 This is correct. If the patient does not respond to nursing and prescribed interventions, the nurse may make the patient NPO for anticipated surgery. The lack of response may indicate complications such as retained placental tissue or birth trauma. Continued uterine atony can lead to postpartum hemorrhage and requires assessment and potentially further treatment by the woman's health care provider.

The nurse is preparing to perform a visual assessment of the perineum of a postpartum patient. The nurse will use the REEDA acronym. Which specific assessments are covered by REEDA? Select all that apply. 1. Perineal coloration 2. Suture line appearance 3. Amount of swelling 4. Description of pain 5. Soft tissue trauma

ANS: 1, 2, 3, 5 Chapter: Chapter 12 Postpartum Physiological Assessments and Nursing Care Chapter Learning Objective: 2. Identify the critical elements of assessment and nursing care during the postpartum period. Page: 372 Heading: The Reproductive System > Vagina and Perineum > Nursing Actions Integrated Processes: Nursing Process Client Need: Physiological Integrity: Physiological Adaptation Cognitive Level: Analysis [Analyzing] Concept: Ante/Intra/Post-partum Difficulty: Difficult Feedback 1 This is correct. The acronym REEDA stands for redness, edema, ecchymosis, discharge, approximation of edges of episiotomy or laceration. Redness is indicative of perineal coloration. 2 This is correct. The acronym REEDA stands for redness, edema, ecchymosis, discharge, approximation of edges of episiotomy or laceration. Suture line appearance is indicative of approximation of edges of episiotomy or laceration. 3 This is correct. The acronym REEDA stands for redness, edema, ecchymosis, discharge, approximation of edges of episiotomy or laceration. The amount of swelling is indicative of edema. 4 This is incorrect. The acronym REEDA stands for redness, edema, ecchymosis, discharge, approximation of edges of episiotomy or laceration. The patient's description of pain is not specifically addressed by a REEDA assessment. 5 This is correct. The acronym REEDA stands for redness, edema, ecchymosis, discharge, approximation of edges of episiotomy or laceration. Soft tissue trauma is frequently accompanied by bruising and is indicative of ecchymosis.

The nurse is counseling a lesbian couple who have decided to have a child. Which considerations does the nurse present with regard to which partner will become pregnant? Select all that apply. 1. Consider the age and health of each partner. 2. Evaluate each partner's career goals. 3. Decide which partner has better insurance. 4. Determine who will be on the birth certificate. 5. Identify which woman desires to be pregnant.

ANS: 1, 2, 3, 5 Chapter: Chapter 13 Transition to Parenthood Chapter Learning Objective: 4. Identify factors that affect the family dynamics. Page: 403 Heading: Same-Sex Parents Integrated Processes: Nursing Process Client Need: Health: Psychosocial Integrity Cognitive Level: Analysis [Analyzing] Concept: Family Dynamics Difficulty: Difficult Feedback 1 This is correct. A lesbian couple needs to consider the age and overall health of each partner before deciding on who will be the biological mother of their child. 2 This is correct. It is important to evaluate each partner's career goals before deciding who will be the biological mother of a lesbian couple's child. More defined goals may indicate a stronger need to be a coparent instead of the biological mother. 3 This is correct. A consideration about which partner has the best health insurance coverage for the pregnancy and birth, and for the child, is a necessary financial consideration. 4 This is incorrect. There is no need for determination of which partner's name goes on the birth certificate; the biological mother's name is always recorded. 5 This is correct. An important decision for the lesbian couple is determining which partner desires to be pregnant. If both want to be pregnant, it may be a matter of which partner will become pregnant first.

The nurse is explaining to a mother that her newborn's blood test indicates a high level of unconjugated bilirubin, which causes jaundice. Which information does the nurse present to the mother? Select all that apply. 1. The blood test does not indicate a pathological disease. 2. The newborn's liver converts bilirubin to a water-soluble substance. 3. An abundance of RBCs and RBC short life span contributes to the condition. 4. The newborn's condition is also referred to as hyperbilirubinemia. 5. Elevated bilirubin can be excreted in the urine and stool.

ANS: 1, 2, 3, 5 Chapter: Chapter 15 Physiological and Behavioral Responses of the Neonate Chapter Learning Objective: 6. Describe the common laboratory and diagnostic tests for neonates. Page: 448 Heading: Transition to Extrauterine Life > The Hepatic System Integrated Processes: Nursing Process Client Need: Physiological Integrity: Physiological Adaptation Cognitive Level: Analysis [Analyzing] Concept: Ante/Intra/Post-partum Difficulty: Difficult Feedback 1 This is correct. The newborn's blood test is indicative of a type of physiological condition (jaundice). 2 This is correct. Unconjugated bilirubin, a fat-soluble substance, is produced from the breakdown of red blood cells (RBCs). It is converted to conjugated bilirubin, a water-soluble substance, by liver enzymes. 3 This is correct. Newborns are born with an abundance of RBCs, which have a shorter life span. These factors contribute to a proportionally greater amount of bilirubin production. 4 This is incorrect. Hyperbilirubinemia is a condition in which there is a high level of unconjugated bilirubin in the neonate's blood related to the immature liver function. Hyperbilirubinemia is categorized into either physiological jaundice or pathological jaundice. 5 This is correct. Unconjugated bilirubin is eventually excreted in the urine and stool.

An infertile couple voices concern to the nurse about assisted reproductive technologies (ART). The nurse agrees that ART has created numerous dilemmas. Which potential questions does the nurse discuss with the couple? Select all that apply. 1. Which partner has ownership of the embryos? 2. Should a child be told about donors or surrogates? 3. Are there parental rights for sperm donors? 4. Does a fertility donor have financial obligations? 5. What happens to any surplus embryos?

ANS: 1, 2, 3, 5 Chapter: Chapter 3 Genetics, Conception, Fetal Development, and Reproductive Technology Chapter Learning Objective: 7. Discuss the ethical and emotional implications of assisted reproductive therapies. Page: 55 Heading: Infertility and Reproductive Technology > Ethical Implications Integrated Processes: Nursing Process Client Need: Health Promotion and Maintenance Cognitive Level: Analysis [Analyzing] Concept: Sexuality Difficulty: Difficult Feedback 1 This is correct. Maternal and paternal ownership is a potential ethical issue for ART, especially if the relationship does not last. 2 This is correct. Informing the child about ART is a potential ethical issue, especially if a sperm donor or surrogate is used. 3 This is correct. Parental rights of sperm donors are a potential ethical issue. This issue is most likely in the event of a surrogate. 4 This is incorrect. ART is legal and practiced throughout the United States. 5 This is correct. The fate of embryos is a potential ethical issue, especially when the owners of the embryos are no longer able or interested in becoming pregnant.

A patient who is at 20 weeks gestation is being prepared for an MRI after a nonconclusive ultrasound testing for suspected brain abnormality related to possible zika virus exposure. Which nursing actions are appropriate for this patient? Select all that apply. 1. Provide information regarding the test. 2. Allow patient to express feelings about her high-risk pregnancy. 3. Promote open communication with her primary health care providers. 4. Encourage patient to think about resolutions for negative testing. 5. Provide psychological support to the patient and her partner.

ANS: 1, 2, 3, 5 Chapter: Chapter 6 Antepartal Tests Chapter Learning Objective: 4. Articulate the nursing responsibilities related to key antenatal tests. Page: 132 Heading: The Nurse's Role in Antepartal Testing Integrated Processes: Nursing Process Client Need: Physiological Integrity: Reduction of Risk Potential Cognitive Level: Analysis [Analyzing] Concept: Ante/Intra/Post-partum Difficulty: Difficult Feedback 1 This is correct. A nursing responsibility related to antenatal testing is to inform the patient of what to expect during the testing. 2 This is correct. A nursing responsibility related to antenatal testing because of a high-risk pregnancy is to provide the patient and her partner with psychological support. The patient is likely to be anxious and should be allowed to express feelings. 3 This is correct. A nursing responsibility related to antenatal testing is always to promote open communication with the patient's primary health care providers. This nursing action is especially important in the event of a high-risk pregnancy. 4 This is incorrect. In this scenario, the nurse needs to refrain from encouraging the patient from thinking of resolutions if the MRI test results indicate manifestations of zika virus. The nurse needs to encourage the patient to make informed decisions when all factors are available. 5 This is correct. All antenatal testing related to high-risk factors causes anxiety and distress; the nurse needs to provide psychological support.

Which actions indicate the is assessing uterine activity? Select all that apply. 1. Feeling the maternal abdomen in between contractions 2. Checking the EFM strip to determine if contractions are either 2 or 3 minutes apart 3. Evaluating that the EFM strip shows 200 MVU every 10 minutes. 4. Checking the EFM strip to see if the fetus has an elevation of 15 bpm over baseline for 15 seconds twice in a 20-minute period 5. Evaluating that the EFM strip shows that each contraction lasts 1 minute

ANS: 1, 2, 3, 5 Chapter: Chapter 9 Fetal Heart Rate Assessment Chapter Learning Objective: Identify the modes of fetal heart rate assessment: auscultation, palpation, EFM. Page: 295 Heading: FHR and Contraction Pattern Interpretation > Uterine Activity and Contraction Patterns Integrated Processes: Nursing Process Client Need: Physiological Integrity: Reduction of Risk Potential Cognitive Level: Application [Applying] Concept: Ante/Intra/Post-partum Difficulty: Moderate Feedback 1 This is correct. Resting tone is an interpretation of uterine activity. 2 This is correct. Frequency is an interpretation of uterine activity. 3 This is correct. Intensity is an interpretation of uterine activity. 4 This is incorrect. Accelerations are an interpretation of periodic and episodic fetal changes. 5 This is correct. Duration of uterine contractions is an interpretation of uterine activity.

A nurse is caring for a single teen mother who has just given birth to her first child. The patient notes that the child's father "wants no relationship with his son." The patient goes on to express concerns about the short- and long-term impact of her teen pregnancy on herself and on her child. Which statements made by the nurse are accurate? Select all that apply. 1. "You are more likely than others to have additional children before you turn 20." 2. "You may have difficulty completing high school or college." 3. "Your child is less likely to experience behavioral problems." 4. "Your child is at increased risk of abusing alcohol when he is a teenager." 5. "You may not earn enough money to independently support yourself and your child."

ANS: 1, 2, 4 Chapter: Chapter 1 Trends and Issues Chapter Learning Objective: 4. Discuss current maternal and infant health issues. Page: 10 Heading: Issues > Teen Pregnancy Integrated Processes: Nursing Process Client Need: Physiological Integrity: Physiological Adaptation Cognitive Level: Application [Applying] Concept: Health Promotion Difficulty: Moderate Feedback 1 This is correct. Teen mothers are more likely to have additional children than their peers. Roughly 17% of all teen mothers will have at least one more birth before the age of 20. 2 This is correct. Only half of teen mothers earn their high school diploma by age 22. Less than 2% of teen mothers finish college by age 30. 3 This is incorrect. Children of teen mothers are more likely to experience behavioral problems. 4 This is correct. Teenage males without an involved father have a higher risk of abusing alcohol or drugs. 5 This is correct. Around one-fourth of teen mothers begin receiving welfare within 3 years of the birth of their first child. 66% of teen mothers are poor.

The nurse is collecting information during a follow-up OB appointment with a patient who delivered 3 months ago. The patient reports her partner has become cynical, irritable, and verbally abusive. The nurse will screen for which risks related to paternal postnatal depression (PPND)? Select all that apply. 1. The father exhibited depression during the pregnancy. 2. The birth of this fourth child was unexpected and unplanned. 3. The father expresses feeling bored and underappreciated in his job. 4. The father is recently estranged from his parents and siblings. 5. The mother experienced a prolonged labor and a cesarean birth.

ANS: 1, 2, 4 Chapter: Chapter 14 High-Risk Postpartum Nursing Care Chapter Learning Objective: 3. Describe the primary postpartum psychological complications and the related nursing actions and medical care. Page: 437 Heading: Postpartum Psychological Complications > Paternal Postnatal Depression Integrated Processes: Nursing Process Client Need: Psychosocial Integrity Cognitive Level: Analysis [Analyzing] Concept: Ante/Intra/Post-partum Difficulty: Difficult Feedback 1 This is correct. Exhibiting paternal depression during the pregnancy can be a risk factor for the development of PPND. 2 This is correct. An unexpected or unplanned pregnancy can be a risk factor for the development of PPND. 3 This is incorrect. Job dissatisfaction is not likely to be a risk factor for the development of PPND. 4 This is correct. The father's estrangement from his parents and siblings can be a stressful life event and/or indicate a lack of social support. Both manifestations can be a risk factor for the development of PPND. 5 This is incorrect. The mother experiencing a prolonged labor that ended in a cesarean birth can be a source of maternal postpartum depression; however, it is not likely to be a risk factor for PPND.

The nurse is preparing to teach a class on the benefits of breastfeeding for infants. Which benefits will the nurse include in the presentation? Select all that apply. 1. Decreased incidence of SIDS 2. Fewer cases of necrotizing enterocolitis 3. Less likely to become obese adults 4. Decreased risk for developing otitis media 5. Immunity to respiratory syncytial virus

ANS: 1, 2, 4 Chapter: Chapter 16 Discharge Planning and Teaching Chapter Learning Objective: 6. Develop a teaching plan for breastfeeding. Page: 482 Heading: Newborn Nutrition and Feeding > Breastfeeding > Benefits of Breastfeeding for the Infant Integrated Processes: Teaching and Learning Client Need: Physiological Integrity: Reduction of Risk Potential Cognitive Level: Analysis [Analyzing] Concept: Ante/Intra/Post-partum Difficulty: Difficult Feedback 1 This is correct. There is a decreased incidence of SIDS in infants who are breastfed. 2 This is correct. Breastfed infants have fewer cases of necrotizing enterocolitis. 3 This is incorrect. There is no reported information that indicates that breastfed infants are less likely to become obese adults. 4 This is correct. Breastfed infants have a decreased risk for developing otitis media. 5 This is incorrect. Infants who are breastfed have a decreased risk for being hospitalized for respiratory syncytial virus but are not immune to the condition from breastfeeding.

The premature neonate is more susceptible to skin breakdown than a term neonate. Which skin care interventions will the nurse implement for the premature neonate? Select all that apply. 1. Use a neutral pH cleanser and sterile water for bathing. 2. Gently apply emollients to avoid unnecessary friction. 3. Perform daily skin assessment to identify problems early. 4. Use water, air, or gel mattresses. 5. Provide a full bath every other day.

ANS: 1, 2, 4 Chapter: Chapter 17 High-Risk Neonatal Nursing Care Chapter Learning Objective: 2. Identify critical elements of assessment and nursing care of the high-risk neonate. Page: 511 Heading: Premature Neonates > Nursing Actions Integrated Processes: Nursing Process Client Need: Physiological Integrity: Reduction of Risk Potential Cognitive Level: Analysis [Analyzing] Concept: Ante/Intra/Post-partum Difficulty: Difficult Feedback 1 This is correct. Use a neutral pH cleanser and sterile water for bathing to help prevent skin breakdown on a premature neonate. 2 This is correct. Emollients should be applied gently to avoid unnecessary friction, which can cause skin breakdown. 3 This is incorrect. Skin assessment on a premature neonate needs to be performed whenever the neonate is repositioned or the diaper is changed. Once daily is not frequently enough for early identification of potential problems. 4 This is correct. Water, gel, or air mattresses are used to help prevent skin breakdown in the premature neonate. 5 This is incorrect. Premature neonates do not receive complete baths; the soiled areas are cleaned as needed.

The nurse is providing education on the prevention of heart disease for adult females ages 35 to 44 years old. Which of the following would be most appropriate for this disease process? Select all that apply. 1. Taking a brisk walking for 50 minutes three times a week 2. Consuming a diet rich in vegetables, fruits, and whole grains 3. Obtaining a varicella vaccine, if no evidence of immunity 4. Recognizing the importance of smoking cessation 5. Taking calcium supplements, if lactose intolerant

ANS: 1, 2, 4 Chapter: Chapter 18 Well Women's Health Chapter Learning Objective: 3. Describe how lifestyle factors such as diet, exercise, and cigarette smoking influence women's health. Page: 556 Heading: Health Promotion > Risk Reduction Integrated Processes: Teaching and Learning Client Need: Physiological Integrity: Reduction of Risk Potential Cognitive Level: Application [Applying] Concept: Promoting Health Difficulty: Moderate Feedback 1 This is correct. Getting at least 150 minutes of moderate intensity physical activity per week helps reduce risk potential for heart disease. 2 This is correct. Consuming a nutrient rich diet helps reduce the risk potential for heart disease, as well as other disease processes, such as cancer. 3 This is incorrect. A varicella vaccine may prevent shingles but has little or no effect on the cardiac system. 4 This is correct. Smoking cessation is another risk factor for many disease processes, including cardiac disease. It is a leading cause of heart disease and cancer. 5 This is incorrect. Calcium supplements decrease the risk factors for osteoporosis, not cardiac disease.

A college-aged female patient states that she understands the risk of sexual assault with overdrinking. She asks the nurse what health risks are associated with excessive alcohol intake for her age. What diseases or conditions should the nurse include in her response? Select all that apply. 1. Infertility 2. Cancer of mouth 3. Hypertension 4. Brain shrinkage 5. Osteoporosis

ANS: 1, 2, 4 Chapter: Chapter 18 Well Women's Health Chapter Learning Objective: 3. Describe how lifestyle factors such as diet, exercise, and cigarette smoking influence women's health. Page: 560 Heading: Health Promotion > Risk Reduction Integrated Processes: Teaching and Learning Client Need: Physiological Integrity: Reduction of Risk Potential Cognitive Level: Application [Applying] Concept: Promoting Health Difficulty: Moderate Feedback 1 This is correct. Per the CDC, excessive alcohol intake at this age can increase a woman's risk for infertility. 2 This is correct. Per the CDC, excessive alcohol intake at this age can increase a woman's risk for cancer of the mouth. 3 This is incorrect. Although the patient may be at risk for heart damage, there is no indication of hypertension at this time. 4 This is correct. Per the CDC, excessive alcohol intake at this age can increase a woman's risk for memory loss and brain shrinkage. 5 This is incorrect. Although the patient may make poor dietary choices while drinking alcohol, this is not indicated at this time.

Parents of a neonate are grieving over their child's life-threatening disabilities. The neonate's course of treatment has changed three times in the last 24 hours due to irrational parenteral decisions. Which interventions will the NICU nurse implement in order to fulfill nursing responsibility to both the parents and the neonate? Select all that apply. 1. Inform the parents as to realistic expectations. 2. Use neonate's status with parenteral counseling. 3. Elicit parenteral input regarding medical care. 4. Advocate for medical support of the neonate. 5. Inform parents they are the final decision makers.

ANS: 1, 2, 4 Chapter: Chapter 2 Ethics and Standards of Practice Issues Chapter Learning Objective: 2. Debate ethical issues in maternity nursing. Page: 23 Heading: Ethics in Nursing > Ethics in Neonatal Care Integrated Processes: Caring Client Need: Psychosocial Integrity Cognitive Level: Analysis [Analyzing] Concept: Grief and Loss Difficulty: Difficult Feedback 1 This is correct. The neonatal nurse has a responsibility to be truthful to the parents regarding realistic expectations for their neonate. The nurse also has a responsibility to cause no additional harm to the neonate. The ethical principle is for veracity. 2 This is correct. Again, the ethical principle of veracity (truthfulness) is important when counseling the parents of the neonate. The nurse is mindful of fulfilling nursing responsibility for both the parents and the neonate. 3 This is incorrect. Eliciting parenteral input regarding medical care of the neonate may fulfill a need for the parents to be involved in decision making; however, the possibility of causing the neonate additional harm or stress in increased with this action. 4 This is correct. The nurse has a responsibility to both the parents and the neonate to advocate for appropriate medical care of the neonate. Ethically, the nurse needs to do no harm. 5 This is incorrect. Informing the parents they are the final decision makers for their compromised neonate is not ethical. The parents may inadvertently cause additional distress to the neonate, or not be able to make appropriate decisions. The nurse needs to make sure the parents have access to medical professionals who can assist with decision making.

A female patient with a history of infertility is scheduled to have a hysterosalpingogram. Which findings can be detected with this procedure? Select all that apply. 1. Tubal occlusions 2. Uterine fibroids 3. Cervical irritation 4. Bicornate uterus 5. Vaginal infection

ANS: 1, 2, 4 Chapter: Chapter 3 Genetics, Conception, Fetal Development, and Reproductive Technology Chapter Learning Objective: 5. Describe the most common diagnostic tests used to identify causes of infertility Page: 53 Heading: Infertility and Reproductive Technology > Diagnosis of Infertility Integrated Processes: Nursing Process Client Need: Physiological Integrity: Reduction of Risk Potential Cognitive Level: Analysis [Analyzing] Concept: Sexuality Difficulty: Difficult Feedback 1 This is correct. A hysterosalpingogram is a radiological examination that provides information about fallopian tubes; occlusions and adhesions can be visualized. 2 This is correct. A hysterosalpingogram is a radiological examination that provides information about the uterus; the presence of fibroids can be diagnosed. 3 This is incorrect. A hysterosalpingogram is a radiological examination that provides information about the endocervical canal, uterine cavity, and fallopian tubes. Cervical irritation cannot be diagnosed with this test. 4 This is correct. A hysterosalpingogram is a radiological examination that provides information about the uterus; a structural defect such as a bicornate uterus can be identified. 5 This is incorrect. A hysterosalpingogram is a radiological examination that provides information about the endocervical canal, uterine cavity, and fallopian tubes. Vaginal infection cannot be diagnosed with this test.

The nurse explains to a patient who has missed a second menstrual cycle that a combination of presumptive and probable signs is used to make a practical diagnosis of pregnancy. Which signs are expected by the nurse when making a practical diagnosis? Select all that apply. 1. Elevated hCG levels in blood and urine 2. Brownish pigmentation on the face 3. Fetal movement detected by the examiner 4. Bluish-purple coloration of vagina and cervix 5. Occasional mild contractions

ANS: 1, 2, 4 Chapter: Chapter 4 Physiological Aspects of Antepartum Care Chapter Learning Objective: 2. Describe methods for diagnosis of pregnancy and determination of estimated date of delivery. Page: 75 Heading: Diagnosis of Pregnancy > Probable Signs of Pregnancy Integrated Processes: Nursing Process Client Need: Physiological Integrity: Physiological Adaptation Cognitive Level: Application [Applying] Concept: Ante/Intra/Post-partum Difficulty: Moderate Feedback 1 This is correct. Elevated hCG levels in the patient's blood and urine are probable signs of pregnancy and will be considered when making a practical diagnosis of pregnancy. 2 This is correct. Brownish pigmentation on the patient's forehead, temples, cheeks, and/or upper lip is melisma (chloasma), which is a probable sign of pregnancy and will be considered when making a practical diagnosis of pregnancy. 3 This is incorrect. Fetal movement that can be observed and detected by the examiner is considered a positive sign of pregnancy, which does not occur until after or about 20 weeks gestation. The finding would be unexpected at this time. 4 This is correct. Bluish-purple coloration (Chadwick's sign) of the vaginal mucosa, cervix, and vulva occurs at 6 to 8 weeks gestation and is considered a probable sign of pregnancy and will be considered when making a practical diagnosis of pregnancy. 5 This is incorrect. Contractions are not expected, even Braxton-Hicks contractions, until long after the pregnancy is identified.

The nurse is planning an assessment on a patient in the second trimester of pregnancy. For which assessments will the nurse plan? Select all that apply. 1. Urine testing with a dipstick. 2. Presence of dependent edema. 3. Determine EDD by Naegele's rule. 4. Antibody screening for Rh?2- patient. 5. Check for chromosomal abnormalities.

ANS: 1, 2, 4 Chapter: Chapter 4 Physiological Aspects of Antepartum Care Chapter Learning Objective: 6. Identify the critical elements of assessment and nursing care during initial and subsequent prenatal visits. Page: 85 Heading: Antepartal Nursing Care: Physiology-Based Nursing Assessment and Nursing Actions > Second Trimester > Components of Second-Trimester Prenatal Assessments Integrated Processes: Nursing Process Client Need: Physiological Integrity: Physiological Adaptation Cognitive Level: Analysis [Analyzing] Core concept: Ante/Intra/Post-partum Difficulty: Difficult Feedback 1 This is correct. During the second trimester, it is common for the nurse to perform urine testing with a dipstick to check for glucose, albumin, and ketones. Mild proteinuria and glycosuria are expected. 2 This is correct. During the second trimester, the nurse should be checking the patient for slight, dependent edema in the lower extremities due to decreased venous return. Upper body edema is abnormal and requires additional evaluation. 3 This is incorrect. The EDD is estimated using Naegele's rule during the first trimester; EDD is determined in the second trimester if the patient is not aware of her last menstrual cycle. 4 This is correct. In the second trimester, the nurse will perform screening needed to determine if the Rh- patient has produced antibodies. If so, the patient will receive the first dose of Rhogam. The patient's Rh factor is determined in the first trimester. 5 This is incorrect. Chromosomal abnormalities are not routinely screened; however, during the early stage of the second trimester, all patients should be offered the screening and diagnostic testing regardless of age or other risk factors.

The nurse works in a prenatal clinic located in a multicultural city. It is important for the nurse to recognize which cultural beliefs as prescriptive? Select all that apply. 1. The mother will aid the baby's circulation by remaining active during pregnancy. 2. The satisfaction of pregnancy cravings will prevent birthmarks on the baby. 3. The mother invites harm to the fetus during the night by sleeping on her back. 4. A safety pin attached to an undergarment will prevent fetal facial deformities. 5. Drinking too much tea will stimulate the fetus and cause a premature birth.

ANS: 1, 2, 4 Chapter: Chapter 5 The Psycho-Social-Cultural Aspects of the Antepartum Period Chapter Learning Objective: 4. Identify nursing assessments and interventions that promote positive psycho-social-cultural adaptations for the pregnant woman and her family. Page: 119 Heading: Childbearing and Culture > Common Themes for the Childbearing Family Integrated Processes: Nursing Process Client Need: Psychosocial Integrity Cognitive Level: Analysis [Analyzing] Concept: Culture Difficulty: Difficult Feedback 1 This is correct. The belief of improving the baby's circulation by the mother remaining active during pregnancy is a prescriptive belief. 2 This is correct. It is a prescriptive belief that satisfying a mother's cravings will prevent birthmarks on the baby. 3 This is incorrect. It is a prescriptive belief that the mother will protect the fetus from harm by sleeping on her back, and not invite harm during the night if in this position. 4 This is correct. It is a prescriptive belief that if a mother attaches a safety pin to an undergarment, the baby will be protected from having a cleft lip or palate. 5 This is incorrect. It is not believed that drinking too much tea during pregnancy will stimulate the fetus and cause a premature birth. The prescriptive belief is that drinking chamomile tea will ensure an effective labor.

The nurse on a postpartum unit is acutely aware that cultural influences impact the patient's process of "becoming a mother." For which cultural influences does the nurse assess? Select all that apply. 1. What amount of time the mother spends in each phase 2. Differences in the mother's expectation related to ability to rest 3. How the mother physically recovers from labor and delivery 4. Mother's involvement in decision making for the first few months 5. Whether the mother seems interested in how to care for her baby

ANS: 1, 2, 4, 5 Chapter: Chapter 13 Transition to Parenthood Chapter Learning Objective: 2. Identify factors that influence women and men in their role transitions to mother and father. Page: 400 Heading: Motherhood > Maternal Phases Integrated Processes: Nursing Process Client Need: Psychosocial Integrity Cognitive Level: Analysis [Analyzing] Concept: Family Dynamics Difficulty: Difficult Feedback 1 This is correct. The amount of time the mother spends in each phase of "becoming a mother" can be influenced by culture. 2 This is correct. The expectation of some cultures is that the mother should rest in the postpartum period or longer rather than be focused on care of the baby. 3 This is incorrect. Physical recovery after childbirth is not altered by culture; however, methods of care may vary. 4 This is correct. The expectation of some cultures is that the mother does not need to be focused on decision making about the baby for the first few months of the infant's life. 5 This is correct. Whether a mother is interested in learning how to care for her neonate may be culturally influenced. In some cultures, the mother's mother and other family members will provide care for the infant for several months. The nurse does need to assess whether the lack of interest is culturally based or a sign of other issues.

The nurse is speaking with a pregnant patient who is asking what processes start labor. Which responses by the nurse are appropriate? Select all that apply. 1. There is more pressure on the cervix, which causes the start of contractions by releasing oxytocin. 2. Oxytocin stimulates uterine muscles to contract. 3. The uterus begins to contract due to a decrease in estrogen. 4. The placenta begins to age and deteriorate and this triggers the start of contractions. 5. The fetus releases a stress hormone, cortisol, and this starts contractions.

ANS: 1, 2, 4, 5 Chapter: Chapter 8 Intrapartum Assessment and Interventions Chapter Learning Objective: Describe the four stages of labor and the related nursing and medical care. Page: 220 Heading: Labor Triggers > Maternal Factors Integrated Processes: The Nursing Process Client Need: Physiological Integrity: Physiological Adaptation Cognitive Level: Application [Applying] Concept: Ante/Intra/Post-partum Difficulty: Moderate Feedback 1 This is correct. Increased pressure on the cervix stimulates the nerve plexus, causing release of oxytocin by the maternal pituitary gland, which then stimulates contractions. 2 This is correct. Oxytocin stimulates uterine muscles to contract. 3 This is incorrect. It is an increase in estrogen that stimulates a uterine response. 4 This is correct. As the placenta ages, it begins to deteriorate, triggering initiation of contractions. 5 This is correct. Fetal cortisol rises and stimulates the uterus to contract.

The nurse-educator is preparing a presentation on fetal heart monitoring. Which of the following should be included? Select all that apply. 1. Intermittent auscultation should be performed every hour in the latent phase. 2. For Category I situations, intermittent electronic fetal monitoring (EFM) should be performed for 10 to 30 minutes every 1 to 2.5 hours. 3. A patient with a previous cesarean section should have EFM for 10 to 30 minutes every 1 to 2.5 hours. 4. A patient with membranes ruptured over 24 hours should be monitored during the latent phase every 30 minutes, every 15 minutes during the active phase, and every 5 minutes during the second stage. 5. A patient with fever should be monitored during the latent phase every 30 minutes, every 15 minutes during the active phase, and every 5 minutes during the second stage.

ANS: 1, 2, 4, 5 Chapter: Chapter 9 Fetal Heart Rate Assessment Chapter Learning Objective: Identify the modes of fetal heart rate assessment: auscultation, palpation, EFM. Page: 277 Heading: AWHONN Standards for Frequency of Assessment of FHR Integrated Processes: Caring Client Need: Physiological Integrity: Reduction of Risk Potential Cognitive Level: Application [Applying] Concept: Ante/Intra/Post-partum Difficulty: Difficult Feedback 1 This is correct. Intermittent auscultation should be done every hour in the latent phase. 2 This is correct. For Category I situations, intermittent EFM should be done for 10 to 30 minutes every 1 to 2.5 hours. 3 This is incorrect. A patient with a previous cesarean section should be monitored during the latent phase every 30 minutes, every 15 minutes during the active phase, and every 5 minutes during the second stage. 4 This is correct. A patient with membranes ruptured over 24 hours should be monitored during the latent phase every 30 minutes, every 15 minutes during the active phase, and every 5 minutes during the second stage. 5 This is correct. A patient with fever should be monitored during the latent phase every 30 minutes, every 15 minutes during the active phase, and every 5 minutes during the second stage.

The nurse is providing care for a patient who is at 42 weeks gestation. The patient's primary care provider is suggesting induction, but the patient is resistant. Which facts can the nurse provide if the patient asks about allowing labor to start spontaneously? Select all that apply. 1. Stillbirth or newborn death increases in pregnancies beyond 42 weeks. 2. There is a greater chance of developing complications because of larger fetal size. 3. Maternal death rate is higher if the pregnancy is continued beyond 42 weeks. 4. Post-term fetuses are prone to developmental delays related to uterine hypoxia. 5. Postmature fetuses have decreased subcutaneous fat and lack vernix and lanugo.

ANS: 1, 2, 5 Chapter: Chapter 10 High-Risk Labor and Birth Chapter Learning Objective: 4. Identify and manage high-risk pregnancy, labor, and delivery to promote healthy outcomes for the mother and infant. Page: 327 Heading: Obstetric Complications > Post-Term Pregnancy and Birth Integrated Processes: Nursing Process Client Need: Physiological Integrity: Reduction of Risk Potential Cognitive Level: Analysis [Analyzing] Concept: Ante/Intra/Post-partum Difficulty: Difficult Feedback 1 This is correct. One of the greatest concerns about post-term pregnancy is the increased risk of stillbirth or infant death. 2 This is correct. The fetus who is post-term is most likely to exhibit macrosomia, a condition that contributes to a higher risk for complications associated with both the mother and fetus. 3 This is incorrect. There is no support for a higher maternal death rate related to post-term pregnancies. 4 This is incorrect. There is no definitive connection between post-term pregnancies and developmental delays in the child born post-term. 5 This is correct. Postmature fetuses have decreased subcutaneous fat and lack vernix and lanugo. However, although the nurse can present this as factual information, it is likely to have little influence on the patient's decision.

The nurse is assessing patients who are postpartum. Which patients does the nurse identify as being at increased risk for respiratory complications? Select all that apply. 1. The patient who was placed on bedrest for threatened abortion 2. The patient with preeclampsia treated with magnesium sulfate 3. The patient with a preexisting diagnosis of diabetes mellitus 4. The patient who delivered a neonate after regional anesthesia 5. The patient who received large amounts of IV fluid due to blood loss

ANS: 1, 2, 5 Chapter: Chapter 12 Postpartum Physiological Assessments and Nursing Care Chapter Learning Objective: 2. Identify the critical elements of assessment and nursing care during the postpartum period. Page: 376 Heading: The Respiratory System > Nursing Actions Integrated Processes: Nursing Process Client Need: Physiological Integrity: Reduction of Risk Potential Cognitive Level: Analysis [Analyzing] Concept: Ante/Intra/Post-partum Difficulty: Difficult Feedback 1 This is correct. The nurse recognizes that the patient placed on bedrest for a threatened abortion is a risk for developing a pulmonary embolus. 2 This is correct. Treating a patent with preeclampsia with magnesium sulfate places the patient at an increased risk for respiratory system complications. 3 This is incorrect. The patient with a preexisting diagnosis of diabetes mellitus is not at risk for the development of a respiratory system complication. 4 This is incorrect. The nurse does not expect the patient who delivers after the administration of regional anesthesia to develop respiratory system complications. 5 This is correct. The patient who receives a large amount of IV fluids because of blood loss is at risk for respiratory system complications related to the potential for fluid overload.

The nurse is reviewing the medical record for a patient who is postpartum. The nurse notices the patient is rubella-nonimmune. Which information does the nurse present to the patient? Select all that apply. 1. The risks to the fetuses of any future pregnancies. 2. The patient will need to be immunized before discharge. 3. Breastfeeding should be avoided for 24 hours after immunization. 4. Maternal immunization carries over to the neonate. 5. Pregnancy should be avoided for 4 weeks.

ANS: 1, 2, 5 Chapter: Chapter 12 Postpartum Physiological Assessments and Nursing Care Chapter Learning Objective: 3. Describe safe and effective nursing care during the postpartum period. Page: 376 Heading: The Immune System Integrated Processes: Nursing Process Client Need: Physiological Integrity: Reduction of Risk Potential Cognitive Level: Analysis [Analyzing] Concept: Ante/Intra/Post-partum Difficulty: Difficult Feedback 1 This is correct. Fetuses exposed to rubella during the first trimester are at risk for birth defects that include deafness, blindness, heart defects, and mental retardation. 2 This is correct. The patient should be immunized before discharge so that immunity is established before the possibility of another pregnancy. Women cannot be immunized during pregnancy. 3 This is incorrect. There is no reason to refrain from breastfeeding for any period of time after a rubella vaccine is received. 4 This is incorrect. Maternal immunity to rubella does not carry over to the neonate. The infant will be immunized for rubella and other diseases at designated times. 5 This is correct. Although the risk of a fetus developing birth defects from the vaccine is extremely low, the patient is advised to avoid pregnancy for 4 weeks.

The nurse is preparing a postpartum patient for discharge. For which reasons does the nurse instruct the patient to call the primary care provider? Select all that apply. 1. Foul-smelling lochia 2. Hot, red, painful breasts 3. Mild headache 4. Not sleeping well 5. Frequent, painful urination

ANS: 1, 2, 5 Chapter: Chapter 14 High-Risk Postpartum Nursing Care Chapter Learning Objective: 2. Describe the primary postpartum infections and the related nursing actions and medical care. Page: 428 Heading: Infections Integrated Processes: Nursing Process Client Need: Physiological Integrity: Reduction of Risk Potential Cognitive Level: Analysis [Analyzing] Concept: Ante/Intra/Post-partum Difficulty: Difficult Feedback 1 This is correct. Foul-smelling lochia is a sign of infection. 2 This is correct. Hot, red, painful breasts are a sign of infection. 3 This is incorrect. Call the provider for a severe headache. 4 This is incorrect. New parents are frequently unable to sleep due to caring for a new baby. 5 This is correct. Frequent, painful urination is a sign of infection.

The nurse is teaching new parents about the Period of PURPLE Crying Program aimed at the prevention of shaken baby syndrome. At the end of the program, the nurse evaluates the program successful if parents select which fact? Select all that apply. 1. Your baby may not stop crying no matter what you do. 2. Your baby may cry more in the late afternoon and evening. 3. A serious condition exists if crying last 5 hours a day or more. 4. Your baby will cry less each week, the least during the first 2 months. 5. A crying baby may look to be in pain, even when he or she is not.

ANS: 1, 2, 5 Chapter: Chapter 16 Discharge Planning and Teaching Chapter Learning Objective: 1. Incorporate principles of teaching and learning when providing newborn care information to parents. Page: 497 Heading: Newborn Care > Pediatric Abusive Head Trauma Integrated Processes: Teaching and Learning Client Need: Physiological Integrity: Reduction of Risk Potential Cognitive Level: Analysis [Analyzing] Concept: Ante/Intra/Post-partum Difficulty: Difficult Feedback 1 This is correct. Parents who understand the baby may not stop crying no matter what they do indicates a successful program. 2 This is correct. Parents who understand the baby may cry more in the late afternoon and evening is an indication of a successful program. 3 This is incorrect. Parents who think a serious condition exists if crying lasts 5 hours a day or more are indicative of an unsuccessful program. 4 This is incorrect. Parents who think the baby will cry less each week, the least during the first 2 months, is an indication of an unsuccessful program. A baby may cry more each week, the most at 2 months, the least in 3 to 5 months.

The nurse is providing care for the 34-year-old patient diagnosed with polycystic ovarian syndrome. Which interventions would correlate to the common symptoms of this syndrome? Select all that apply. 1. The patient has been unsuccessful with the ability to conceive. 2. The patient has a history of painful and irregular menstrual cycles. 3. The patient has noticed a drastic weight loss and dry skin. 4. The patient has chronic back pain and gastrointestinal issues. 5. The patient has heart palpitations and hypertension.

ANS: 1, 2, 5 Chapter: Chapter 18 Well Women's Health Chapter Learning Objective: 1. Identify factors that place a woman at risk for adverse health conditions. Page: 567 Heading: Specific Populations > Lesbian Health Integrated Processes: Teaching and Learning Client Need: Physiological Integrity: Physiological Adaptation Cognitive Level: Analysis [Analyzing] Concept: Promoting Health Difficulty: Moderate Feedback 1 This is correct. Due to abnormal hormonal levels, the patient may have fertility issues. 2 This is correct. Due to abnormal hormonal levels, the patient may have issues with menstrual cycles. 3 This is incorrect. Although many patients may experience weight issues, there is no indication of skin issues. 4 This is incorrect. Although this many be a symptom of painful menstrual cycles, it is not an indication of polycystic ovarian syndrome. 5 This is correct. The patient may experience cardiac issues related to the hormonal fluctuations associated with polycystic ovarian syndrome.

The nurse is arranging education for the menopausal support group regarding the approaches to treat the symptoms. Which of the following would be components of current treatments? Select all that apply. 1. Moderately intense exercises 2. Balanced diet, rich in calcium 3. Alternative medicine such as black cohosh 4. Over-the-counter sleep aid 5. Hormone therapy

ANS: 1, 2, 5 Chapter: Chapter 18 Well Women's Health Chapter Learning Objective: 5 Discuss the physical and emotional changes related to perimenopause and menopause. Page: 562 Heading: Reproductive Changes Across the Life Span > Menopause > Treating Menopausal Symptoms Integrated Processes: Teaching and Learning Client Need: Health Promotion and Maintenance Cognitive Level: Application [Applying] Concept: Promoting Health Difficulty: Moderate Feedback 1 This is correct. Moderately intense exercise will promote overall health. 2 This is correct. A balanced diet rich in calcium will help maintain bone strength (due to the risk of osteoporosis). 3 This is incorrect. Black cohosh has not been proven to alleviate symptoms, and it may increase the likelihood of liver toxicity. 4 This is incorrect. The patient should attempt sleep hygiene first, limiting electronics use prior to bedtime, and removing distractors from the bedroom. 5 This is correct. Studies have shown that hormone therapy can decrease the symptoms of menopause effectively.

The nurse is preparing to teach the community about risk factors for the second most common cancer of the female reproductive system. Which of the following groups of women are at higher risk for this cancer? Select all that apply. 1. Menopausal women with an intact uterus who receive hormonal therapy 2. Women who have undergone treatment for breast cancer 3. Women who have many pregnancies and nursed their infants 4. Women who smoke and have many sexual partners 5. Women with a long-standing history of polycystic ovarian syndrome

ANS: 1, 2, 5 Chapter: Chapter 19 Alterations in Women's Health Chapter Learning Objective: 3. Describe common alterations in women's health, including medical management and nursing actions. Page: 601 Heading: Gynecological Cancers > Endometrial Cancer Integrated Processes: Teaching and Learning Client Need: Physiological Integrity: Reduction of Risk Potential Cognitive Level: Application [Applying] Concept: Health Promotion Difficulty: Moderate Feedback 1 This is correct. Unopposed estrogen therapy in women with a uterus can cause endometrial hyperplasia in the lining of the uterus. The second most common cancer of the female reproductive system is endometrial cancer. 2 This is correct. Tamoxifen is indicated for reducing risk for breast cancer in women or as a treatment of breast cancer. The medication competes with estrogen for binding sites in the breast but increases the risk for endometrial cancer. 3 This is incorrect. Women who have had no pregnancies are at higher risk for endometrial cancer than those that have had several pregnancies and who have nursed their infants. 4 This is incorrect. Smoking and having multiple sexual partners put the patient at risk for other disease processes, not endometrial cancer. 5 This is correct. Polycystic ovarian syndrome increases the risk for endometrial cancer.

A male patient is diagnosed with low sperm count as the cause of infertility. Which interventions will the nurse recommend to improve the patient's sperm count? Select all that apply. 1. Yoga or relaxation techniques 2. Surgical repair of an inguinal hernia 3. Switch to underwear made from cotton 4. Avoidance of showers with hot water temperature 5. Consultation to change hypertension drugs

ANS: 1, 2, 5 Chapter: Chapter 3 Genetics, Conception, Fetal Development, and Reproductive Technology Chapter Learning Objective: 6. Describe the most common methods used in assisted fertility. Page: 53 Heading: Infertility and Reproductive Technology > Treatment Integrated Processes: Nursing Process Client Need: Physiological Integrity: Reduction of Risk Potential Cognitive Level: Analysis [Analyzing] Concept: Sexuality Difficulty: Difficult Feedback 1 This is correct. Stress can be a cause of infertility caused by a low sperm count. The nurse will recommend yoga, relaxation techniques, or other stress reducers. 2 This is correct. The presence of an inguinal hernia can interfere with sperm transport; the nurse will recommend surgical repair of the hernia. 3 This is incorrect. There is no particular benefit from wearing cotton underwear to improve a low sperm count. However, the nurse will recommend underwear that does not hold the testicles close to the body and increase testicular temperature. 4 This is incorrect. The patient should refrain from sitting in hot water frequently or for a prolonged period of time, which will result in a sperm count reduction. The nurse will recommend avoidance of hot baths or sitting in hot tubs or steam rooms. Hot showers should not be an issue. 5 This is correct. If the client is taking a channel blocker for management of hypertension, the nurse will recommend a consultation with the patient's HCP to change the medication. Certain medications are considered to be gonadotoxins.

The nurse is conducting a staff education session about preeclampsia and eclampsia complications. Which statements by the nurse are accurate about HELLP syndrome? Select all that apply. 1. This syndrome destroys red blood cells. 2. This syndrome impacts the amount of platelets. 3. This syndrome decreases a patient's white blood cell (WBC) count. 4. This syndrome decreases a patient's blood urea nitrogen (BUN). 5. This syndrome increases liver enzymes.

ANS: 1, 2, 5 Chapter: Chapter 7 High-Risk Antepartum Nursing Care Chapter Learning Objective: 3. Identify potential antenatal complications for the woman, the fetus, and the newborn. Page: 179 Heading: Hypertension in Pregnancy > HELLP Syndrome Integrated Processes: Nursing Process Client Need: Physiological Integrity: Reduction of Risk Potential Cognitive Level: Analysis [Analyzing] Concept: Ante/Intra/Post-partum Difficulty: Difficult Feedback 1 This is correct. Hemolysis is part of HELLP syndrome. 2 This is correct. Low platelets are part of HELLP syndrome. 3 This is incorrect. Low WBC count is not part of HELLP syndrome. 4 This is incorrect. Low BUN is not part of HELLP syndrome. 5 This is correct. Elevated liver enzymes are part of HELLP syndrome.

The nurse encourages the patient to bring loved ones to be with her during labor because the nurse knows that their presence does which of the following? Select all that apply. 1. Shortens the length of labor 2. Reduces the likelihood that a patient will require an epidural 3. Reduces the risk of infection after birth 4. Lowers the chance that the patient will breastfeed 5. Increases patients' happiness and satisfaction with the outcome of their birth

ANS: 1, 2, 5 Chapter: Chapter 8 Intrapartum Assessment and Interventions Chapter Learning Objective: Demonstrate understanding of supportive care of the laboring woman. Page: 227 Heading: Critical Component > Nursing Support of Laboring Women Integrated Processes: The Nursing Process Client Need: Safe and Effective Care Environment: Management of Care Cognitive Level: Application [Applying] Concept: Ante/Intra/Post-partum Difficulty: Moderate Feedback 1 This is correct. In 2011, the Association of Women's Health, Obstetric, and Neonatal Nurses (AWHONN) found that having support persons shortens labor. 2 This is correct. In 2011, the AWHONN found that having support persons decreases the use of analgesia. 3 This is incorrect. Having support persons has not been shown to have an effect on post-delivery infections. 4 This is incorrect. In 2011, the AWHONN found that having support persons increases the likelihood of breastfeeding. 5 This is correct. In 2011, the AWHONN found increased satisfaction with the birth experience in those with support persons.

The nurse educator is teaching the nursing student the basic principles of using analgesia by using a patient, Christa, as an example. Which of the following demonstrates the basic principles of analgesia during Christa's labor and delivery? Select all that apply. 1. Christa has been in labor for 4 hours. She has been having contractions every 3 minutes. Contractions last a minute and are intense. She has had cervical change from 3 to 5 cm over the past 4 hours. 2. The medication Christa wants is an epidural. After she receives the epidural, the FHR tracing is within normal limits. 3. Christa received butorphanol (Stadol) within 3 hours of delivery and the infant experienced neonatal depression. 4. Local anesthesia in the perineal area was used in conjunction with the epidural throughout labor and delivery. 5. Christa has a history of opiate abuse and therefore it was understood that she might require higher doses of medications to experience relief.

ANS: 1, 2, 5 Chapter: Chapter 8 Intrapartum Assessment and Interventions Chapter Learning Objective: Demonstrate understanding of supportive care of the laboring woman. Page: 258 Heading: Pharmacological Management of Labor Discomfort Integrated Processes: Teaching and Learning. Client Need: Physiological Integrity: Pharmacological and Parenteral Therapies Cognitive Level: Application [Applying] Concept: Ante/Intra/Post-partum Difficulty: Moderate Feedback 1 This is correct. Labor must be well established before administration of an analgesic is begun. 2 This is correct. Medication should provide relief to the mother with minimal risk to the baby. 3 This is incorrect. Neonatal depression may occur if medication is given within an hour before delivery. 4 This is incorrect. Local anesthesia is used at the time of delivery for episiotomy and repair. 5 This is correct. Women with a history of drug abuse may have a lessened effect from pain medication and require higher doses.

An infant was recently born weighing 1,498 grams. The nurse understands that the birth weight of this infant is an important indicator of what? Select all that apply. 1. Morbidity rate 2. Prenatal care 3. Mortality rate 4. Infant health outcome 5. Postpartum care

ANS: 1, 3, 4 Chapter: Chapter 1 Trends and Issues Chapter Learning Objective: 1. Discuss current trends in the management of labor and birth. Page: 6 Heading: Trends > Neonatal Birth Weight Rates Integrated Processes: Nursing Process Client Need: Physiological Integrity: Reduction of Risk Potential Cognitive Level: Application [Applying] Concept: Ante/Intra/Post-partum Difficulty: Moderate Feedback 1 This is correct. Morbidity rate is an outcome of low birth weight. 2 This is incorrect. Prenatal care is not an outcome of low birth weight. 3 This is correct. Mortality rate is an outcome of low birth weight. 4 This is correct. Infant health is an outcome of low birth weight. 5 This is incorrect. Postpartum care is not an outcome of low birth weight.

The postnatal nurse is providing care for a neonate being treated with phototherapy for hyperbilirubinemia. For which side effects of phototherapy will the nurse contact the neonatal care provider? Select all that apply. 1. Hyperthermia 2. Lethargy 3. Hypocalcemia 4. Thrombocytopenia 5. Bronze baby syndrome

ANS: 1, 3, 4 Chapter: Chapter 17 High-Risk Neonatal Nursing Care Chapter Learning Objective: 2. Identify critical elements of assessment and nursing care of the high-risk neonate. Page: 527 Heading: Hyperbilirubinemia > Nursing Actions for Hyperbilirubinemia Integrated Processes: Nursing Process Client Need: Physiological Integrity: Reduction of Risk Potential Cognitive Level: Analysis [Analyzing] Concept: Ante/Intra/Post-partum Difficulty: Difficult Feedback 1 This is correct. The nurse will recognize hyperthermia as an elevation of temperature in the neonate. The nurse will report this assessment finding to the neonatal care provider. 2 This is incorrect. Lethargy is a common manifestation related to hyperbilirubinemia. The nurse will report bilirubin levels to the neonatal care provider, but not necessarily the presence of lethargy. 3 This is correct. Hypocalcemia in a neonate is a serum calcium level below 7.5 mg/dL. Neonatal hypocalcemia symptoms are often similar to those of hypoglycemia and include jitteriness, tetany, and seizures. The nurse will report this assessment finding to the neonatal care provider. 4 This is correct. Thrombocytopenia is indicative of a deficiency of platelets in the blood, which can result in bruising or bleeding. The nurse will report this assessment finding to the neonatal care provider. 5 This is incorrect. Bronze baby syndrome is identified by a dark gray-brown pigmentation of skin that disappears after phototherapy is discontinued. There is no reason for the nurse to report this assessment finding to the neonatal care provider.

The nurse is assessing a 42-year-old patient who presents to the gynecology office with complaints of pelvic pressure, backaches, menorrhagia, and urinary frequency. The health care provider has diagnosed the patient with leiomyoma of the uterus. What are the treatment options for this condition? Select all that apply. 1. Routine pelvic examinations to assess rate of disease process 2. Oral contraceptives to control the bleeding and pain 3. Myomectomy for women who desire pregnancy 4. Hysterectomy for women who do not desire pregnancy 5. Antibiotic therapy with NSAIDS for symptom management

ANS: 1, 3, 4 Chapter: Chapter 19 Alterations in Women's Health Chapter Learning Objective: 3. Describe common alterations in women's health, including medical management and nursing actions. Page: 590 Heading: Leiomyoma of the Uterus > Medical Management Integrated Processes: Nursing Process Client Need: Physiological Integrity: Physiological Adaptation Cognitive Level: Analysis [Analyzing] Concept: Cellular Regulation Difficulty: Moderate Feedback 1 This is correct. Routine pelvic examinations may be performed to assess the rate of growth of the leiomyomas (fibroids). A pelvic ultrasound to confirm the diagnosis of tumors and rule out pregnancy is another option of diagnosis. 2 This is incorrect. This would be an option for another diagnosis such as PMS, not fibroids, which could increase the growth due to increase levels of estrogen and progesterone. 3 This is correct. A myomectomy may be a surgical intervention to remove the fibroids, while leaving the uterus intact for those that desire pregnancy. 4 This is correct. Hysterectomy is recommended for women who do not desire pregnancy and who are experiencing excessive bleeding. Leiomyomas are the main indicator for hysterectomy. 5 This is incorrect. This may be a treatment for an infection not of fibroids of the uterus.

A pregnant patient tells the nurse that her spouse has been diagnosed with Couvade syndrome. Which manifestations does the nurse suspect the spouse is experiencing? Select all that apply. 1. Nausea from unidentifiable causes 2. Physical rejection of sexual advances 3. Significant recent weight gain 4. Unexplainable abdominal pains 5. Self-imposed social isolation

ANS: 1, 3, 4 Chapter: Chapter 5 The Psycho-Social-Cultural Aspects of the Antepartum Period Chapter Learning Objective: 4. Identify nursing assessments and interventions that promote positive psycho-social-cultural adaptations for the pregnant woman and her family. Page: 110 Heading: Paternal Adaptation During Pregnancy > Effect of Pregnancy on Fathers Integrated Processes: Nursing Process Client Need: Psychosocial Integrity Cognitive Level: Analysis [Analyzing] Concept: Family Difficulty: Difficult Feedback 1 This is correct. Couvade syndrome is diagnosed when the male experiences some of the same manifestations of pregnancy as a pregnant partner. Nausea is a symptom of Couvade syndrome. 2 This is incorrect. The rejection of sexual advances by the male of a pregnant couple is not a manifestation of Couvade syndrome. Some causes may be fear hurting the woman and/or the fetus, or lack of interest related to the physical changes of pregnancy. 3 This is correct. Couvade syndrome is diagnosed when the male experiences some of the same manifestations of pregnancy as a pregnant partner. Weight gain is a symptom of Couvade syndrome. 4 This is correct. Couvade syndrome is diagnosed when the male experiences some of the same manifestations of pregnancy as a pregnant partner. Abdominal pains are a symptom of Couvade syndrome. 5 This is incorrect. Self-imposed social isolation is not a manifestation of Couvade syndrome. Additional assessment is needed to determine the cause of this behavior.

A patient in the third trimester of pregnancy is instructed on how to perform daily fetal movement count. The nurse needs to inform the patient what to do if fetal movement is decreased. Which patient actions are appropriately recommended by the nurse? Select all that apply. 1. Eat something. 2. Recount movements the next morning. 3. Arrange for a period of rest. 4. Focus on movement for 1 hour. 5. Exercise or take a walk.

ANS: 1, 3, 4 Chapter: Chapter 6 Antepartal Tests Chapter Learning Objective: 5. Identify patient teaching needs related to antenatal tests. Page: 142 Heading: Antenatal Fetal Surveillance > Daily Fetal Movement Count Integrated Processes: Nursing Process Client Need: Physiological Integrity: Reduction of Risk Potential Cognitive Level: Application [Applying] Concept: Ante/Intra/Post-partum Difficulty: Moderate Feedback 1 This is correct. If the fetal movement is decreased from previous counts, the nurse should instruct the patient to eat something, which may stimulate the fetus. 2 This is incorrect. Fetal movement is an indicator of fetal well-being. If the patient notices a decrease in fetal movement, measures need to be taken to stimulate and/or reassess movement. Persistent decreased movement or lack of movement needs to be reported immediately to the HCP. 3 This is correct. If the fetal movement is decreased from previous counts the nurse should instruct the patient to arrange for a period of rest. If the patient is busy or distracted, the fetal movements may not be noticed. 4 This is correct. If the fetal movement is decreased from previous counts the nurse should instruct the patient to focus on fetal movement for a period of 1 hour. Four movements in an hour is reassuring 5 This is incorrect. If the fetal movement is decreased from previous counts, the nurse should instruct the patient to rest, not exercise or take a walk.

A neonate is born after 37 weeks gestation, and the nurse is concerned about avoiding cold stress after discharge. Which suggestions does the nurse give the mother to keep the baby safe? Select all that apply. 1. Keep the baby wrapped in a warm blanket. 2. Perform the daily bath in a warm location. 3. Position the baby away from vents and drafts. 4. Place a stocking cap on the neonate's head. 5. Change wet clothing immediately.

ANS: 1, 3, 4, 5 Chapter: Chapter 15 Physiological and Behavioral Responses of the Neonate Chapter Learning Objective: 7. Discuss the nursing actions that support parents in the care of their newborn. Pages: 445 Heading: The Thermoregulatory System > Cold Stress > Nursing Actions Integrated Processes: Nursing Process Client Need: Safe and Effective Care Environment: Safety and Infection Control Cognitive Level: Analysis [Analyzing] Concept: Ante/Intra/Post-partum Difficulty: Difficult Feedback 1 This is correct. Keeping the baby swaddled in a warm blanket will decrease heat loss due to convection and radiation. 2 This is incorrect. Neonates do not need daily baths. Undressing and bathing will cause heat loss due to evaporation. 3 This is correct. Place the neonate away from air vents to decrease heat loss due to convection. 4 This is correct. Place a stocking cap on the neonate's head to decrease heat loss due to radiation and convection. 5 This is correct. Remove wet clothing from the neonate immediately to decrease heat loss due to radiation, evaporation, and conduction.

The nurse is providing care for a neonate born to a mother with preexisting diabetes mellitus. Which neonatal assessment findings do the nurse expect? Select all that apply. 1. Macrosomia 2. Hyperglycemia 3. Hypocalcemia 4. Jaundice 5. Dyspnea

ANS: 1, 3, 4, 5 Chapter: Chapter 17 High-Risk Neonatal Nursing Care Chapter Learning Objective: 2, Identify critical elements of assessment and nursing care of the high-risk neonate. Page: 531 Heading: Infants of Diabetic Mothers > Assessment Findings Integrated Processes: Nursing Process Client Need: Physiological Integrity: Physiological Adaptation Cognitive Level: Analysis [Analyzing] Concept: Ante/Intra/Post-partum Difficulty: Difficult Feedback 1 This is correct. Macrosomia is an expected assessment finding in neonates born to mothers with preexisting diabetes mellitus. 2 This is incorrect. Hypoglycemia (not hyperglycemia) is an expected assessment finding in neonates born to mothers with preexisting diabetes mellitus. 3 This is correct. Hypocalcemia is an expected assessment finding in neonates born to mothers with preexisting diabetes mellitus. 4 This is correct. Hyperbilirubinemia is an expected assessment finding in neonates born to mothers with preexisting diabetes mellitus, because of polycythemia. 5 This is correct. Dyspnea and respiratory distress syndrome are expected assessment findings in neonates born to mothers with preexisting diabetes mellitus.

The nurse is preparing for the discharge of a neonate diagnosed with a congenital breathing disorder. Which health team members does the nurse include in discharge planning? Select all that apply. 1. Respiratory therapist 2. Community agency manager 3. Social worker 4. Home health agency nurse 5. Case manager

ANS: 1, 3, 4, 5 Chapter: Chapter 17 High-Risk Neonatal Nursing Care Chapter Learning Objective: 3. Develop a discharge plan for high-risk neonates. Page: 543 Heading: Discharge Planning Integrated Processes: Nursing Process Client Need: Physiological Integrity/ Reduction of Risk Potential Cognitive Level: Analysis [Analyzing] Concept: Ante/Intra/Post-partum Difficulty: Difficult Feedback 1 This is correct. The nurse will include the respiratory therapist in the discharge planning for a neonate with a congenital breathing disorder. Respiratory therapy may be a significant lifetime need for this neonate. 2 This is incorrect. The nurse would not include a community agency manager in the discharge planning for a neonate with a congenital breathing disorder. 3 This is correct. The social worker will play a key role in helping the family find agencies that can provide support for the parents and the neonate. Consideration is focused on meeting financial, psychosocial, and medical needs. 4 This is correct. The family is likely to benefit from a home health agency who can assist with the physiological needs of the neonate. This may be a long-term or short-term need, but the nurse would include a home agency nurse in discharge planning. 5 This is correct. The case manager is included in the discharge planning because the family with a neonate with a congenital breathing disorder is likely to have needs for special equipment and/or therapies.

The nurse is preparing a prenatal plan of care for a patient who is in the first trimester of pregnancy. Which long-range goals does the nurse include in the plan of care? Select all that apply. 1. Perform an ongoing assessment of risk status 2. Determine parental outlook on immunizations 3. Build rapport with the childbearing family 4. Make referral to specific resources as needed 5. Implement a risk-appropriate intervention

ANS: 1, 3, 4, 5 Chapter: Chapter 4 Physiological Aspects of Antepartum Care Chapter Learning Objective: 6. Identify the critical elements of assessment and nursing care during initial and subsequent prenatal visits. Page: 80 Heading: Antepartal Nursing Care: Physiology-Based Nursing Assessment and Nursing Actions > Prenatal Assessment > Goals of Prenatal Care Integrated Processes: Nursing Process Client Need: Health Promotion and Maintenance Cognitive Level: Analysis [Analyzing] Concept: Ante/Intra/Post-partum Difficulty: Difficult Feedback 1 This is correct. An appropriate long-term goal in a prenatal care plan is to perform an ongoing assessment of risk status of the patient, fetus, and expectant family. 2 This is incorrect. The parenteral outlook regarding immunizations is not an appropriate long-term goal on a prenatal care plan. Attitudes about immunizations can be discussed by health care providers involved with pediatric care. 3 This is correct. An appropriate long-term goal in a prenatal care plan is for the nurse to build a rapport with the child-bearing family. Communication is an important part of prenatal care. 4 This is correct. An appropriate long-term goal in a prenatal care plan is to ascertain and make referrals to specific resources for the fetus, neonate, and family. 5 This is correct. An appropriate long-term goal in a prenatal care plan is for the nurse to implement any risk-appropriate intervention. During the prenatal period, risks can occur and interventions must be implemented in a timely manner.

The nurse is reviewing the purpose of a modified BPP for a patient at 38 weeks gestation. The nurse recognizes which determinations can be made through a modified BPP regarding fetal well-being? Select all that apply. 1. The NST is an indicator of short-term fetal well-being. 2. The test is normal if NST is considered to be nonreactive. 3. The test is considered most predictive for perinatal outcomes. 4. The AFI is an indicator of long-term placental function. 5. An AFI of 5 cm is indicative of fetal asphyxia.

ANS: 1, 3, 4, 5 Chapter: Chapter 6 Antepartal Tests Chapter Learning Objective: 5. Identify patient teaching needs related to antenatal tests. Page: 146 Heading: Antenatal Fetal Surveillance > Modified Biophysical Profile Integrated Processes: Nursing Process Client Need: Physiological Integrity: Reduction of Risk Potential Cognitive Level: Analysis [Analyzing] Concept: Ante/Intra/Post-partum Difficulty: Difficult Feedback 1 This is correct. The nurse recognizes the NST indicates short-term fetal well-being. 2 This is incorrect. The nurse recognizes that modified BPP is considered normal if the NST is noted to be reactive, not nonreactive. 3 This is correct. The nurse recognizes a modified BPP is considered to be the most predictive testing for perinatal outcomes; the test measures the two most sensitive indicators (NST and AFI) for fetal well-being. 4 This is correct. The nurse is aware a modified BPP uses the AFI to determine the long-term functionality of the placenta. 5 This is correct. The nurse understands oligohydramnios is associated with increased perinatal mortality, and decreased amniotic fluid may reflect acute or chronic fetal asphyxia. The finding is related to a decrease in renal output as blood is shifted away from the kidneys to other more vital organs in response to asphyxia. Normal AFI is greater than 5 cm.

A patient at 35 weeks gestation arrives at the prenatal clinic in physical distress. Assessment reveals hypotension, thready pulse, shallow respirations, pallor, cold and clammy skin, and anxiety. The nurse does not find evidence of vaginal bleeding but suspects placental abruption. For which reason does the nurse call for emergency transport to the hospital? Select all that apply. 1. The patient has all the symptoms of hypovolemia. 2. The patient reports a recent bout with nausea and vomiting. 3. The absence of blood can indicate a concealed hemorrhage. 4. The patient and fetus are at risk of death from hypovolemic shock. 5. The patient states a sudden onset of severe symptoms.

ANS: 1, 3, 4, 5 Chapter: Chapter 7 High-Risk Antepartum Nursing Care Chapter Learning Objective: 4. Formulate a plan of care that includes the physical, emotional, and psychosocial needs of women diagnosed with pregnancy complications. Page: 183 Heading: Placental Abnormalities and Hemorrhagic Complications > Placental Abruption Integrated Processes: Nursing Process Client Need: Physiological Integrity: Reduction of Risk Potential Cognitive Level: Analysis (Analyzing) Concept: Ante/Intra/Post-partum Difficulty: Difficult Feedback 1 This is correct. The presenting manifestations are classic for hypovolemia; the only cardinal symptom missing is the presence of bleeding. 2 This is incorrect. Placental abruption can be accompanied by nausea and vomiting; however, these manifestations alone do not indicate a need for emergency transport. 3 This is correct. In a pregnant patient, the lack of bleeding as evidence to placental abruption is indicative of concealed hemorrhage. 4 This is correct. Hypovolemic shock places both the patient and fetus at risk for death. 5 This is correct. During pregnancy, signs of shock are usually not until 25% to 30% of maternal blood loss has occurred. The patient may be unaware of life-threatening bleeding because of the concealed hemorrhage.

The labor and delivery unit nurses are adopting methods to reduce the number of women who develop postpartum depression. Research from Dennis and Dowswell (2013) provides evidence-based suggestions regarding beneficial interventions. Which suggestions do the nurses consider? Select all that apply. 1. Telephone-based peer support 2. Partner report of symptoms 3. Interpersonal psychotherapy 4. Teaching for self-recognition of problems 5. Professionally based postpartum home visits

ANS: 1, 3, 5 Chapter: Chapter 14 High-Risk Postpartum Nursing Care Chapter Learning Objective: 3. Describe the primary postpartum psychological complications and the related nursing actions and medical care. Page: 435 Heading: Postpartum Psychological Complications Integrated Processes: Nursing Process Client Need: Psychosocial Integrity Cognitive Level: Analysis [Analyzing] Concept: Ante/Intra/Post-partum Difficulty: Difficult Feedback 1 This is correct. Research from Dennis and Dowswell (2013) reveals that psychosocial and psychological interventions significantly reduce the number of women who develop postpartum depression. Beneficial interventions include telephone-based peer support. 2 This is incorrect. Research from Dennis and Dowswell (2013) does not address any benefit from partner report of symptoms. 3 This is correct. Research from Dennis and Dowswell (2013) reveals that psychosocial and psychological interventions significantly reduce the number of women who develop postpartum depression. Beneficial interventions include interpersonal psychotherapy. 4 This is incorrect. Research from Dennis and Dowswell (2013) does not address any benefit related to teaching for the self-recognition of problems. 5 This is correct. Research from Dennis and Dowswell (2013) reveals that psychosocial and psychological interventions significantly reduce the number of women who develop postpartum depression. Beneficial interventions include professionally based postpartum home visits

The nurse is preparing a talk with new parents about immunity and their newborns. Which factual information will the nurse present? Select all that apply. 1. A vaccination is an example of acquired immunity. 2. Antigens are produced as part of natural immunity. 3. Placental transfer is how newborns get natural passive immunity. 4. Gamma globulin is an example of artificial active immunity. 5. Natural passive immunity protects the baby for a few months after birth.

ANS: 1, 3, 5 Chapter: Chapter 15 Physiological and Behavioral Responses of the Neonate Chapter Learning Objective: 1. Identify the changes that occur during the transition from intrauterine to extrauterine life and the related nursing actions. Page: 449 Heading: Transition to Extrauterine Life > The Immunity System Integrated Processes: Nursing Process Client Need: Physiological Integrity: Physiological Adaptation Cognitive Level: Analysis [Analyzing] Concept: Ante/Intra/Post-partum Difficulty: Difficult Feedback 1 This is correct. A vaccination is an example of how acquired immunity is produced. 2 This is incorrect. Antibodies are produced with natural immunity, not antigens, which actually stimulate antibody production. 3 This is correct. Placental transfer of antibodies from mother to fetus is the manner in which the neonate acquires natural passive immunity. 4 This is incorrect. Gamma globulin is an example of how artificial passive immunity is acquired, not artificial active immunity. 5 This is correct. Natural passive immunity protects the baby for only a few months after birth.

The nurse is providing postoperative care to a patient who underwent a total abdominal hysterectomy 12 hours ago. Which of the following are appropriate nursing interventions? Select all that apply. 1. Assist the patient with ambulation. 2. Maintain the Foley catheter for 48 to 72 hours postoperatively. 3. Monitor intake and output and characteristics of urine. 4. Maintain bedrest while taking narcotic pain medications. 5. Initiate antiembolism therapy as ordered.

ANS: 1, 3, 5 Chapter: Chapter 19 Alterations in Women's Health Chapter Learning Objective: 3. Describe common alterations in women's health, including medical management and nursing actions. Page: 578 Heading: Hysterectomy> Postoperative Care for Abdominal Hysterectomy Integrated Processes: Nursing Process Client Need: Physiological Integrity: Reduction of Risk Potential Cognitive Level: Application [Applying] Concept: Patient-Centered Care Difficulty: Moderate Feedback 1 This is correct. Ambulation decreases the risk for deep vein thrombosis and facilitates return of peristalsis, which decreases the amount of gas buildup. 2 This is incorrect. The Foley catheter is typically removed 12 to 24 hours after surgery. 3 This is correct. Monitoring intake and output, as well as the characteristics of the urine, will facilitate monitoring for signs/symptoms of fluid imbalances and injury to the ureters or bladder. 4 This is incorrect. Although this patient may be at risk for falls due the narcotic pain control, it usually does not impede ambulation. Ambulation decreases risk for deep vein thrombosis and facilitates return of peristalsis, which decreases the amount of gas buildup. 5 This is correct. A risk related to surgical procedures is deep vein thrombosis or venous thromboembolism. This may include antiembolism stockings, ambulation, and/or anti-coagulant therapy.

The nurse is assessing a 25-year-old female patient when the patient becomes tearful. The patient states that she has thin milky discharge from her nipples and two small masses on her left breast. She has lost over 40 pounds in the past year due to intensive exercises and finds that she needs to wear a sports bra during her sessions. The patient states that she is afraid that she will become the first member of her family to have breast cancer. Besides a negative mammogram, what other symptoms would correlate with this being a benign finding? Select all that apply. 1. Milky discharge from nipples 2. Extensive weight loss 3. Painful masses 4. Mood swings 5. Multiple masses

ANS: 1, 3, 5 Chapter: Chapter 19 Alterations in Women's Health Chapter Learning Objective: 3. Describe common alterations in women's health, including medical management and nursing actions. Page: 595 Heading: Breast Disorders Integrated Processes: Teaching and Learning Client Need: Physiological Integrity: Physiological Adaptation Cognitive Level: Application [Applying] Concept: Health Promotion Difficulty: Moderate Feedback 1 This is correct. Discharge from nipples that are milky may be due to the elicited response from compression (resulting from the sports bra). 2 This is incorrect. The patient's weight loss is likely from intensive exercises. 3 This is correct. Fewer than 10% of women with breast cancer will present with pain. 4 This is incorrect. The mood swing (tearfulness) may be due to the stress and fear of a possible breast cancer diagnosis. 5 This is incorrect. The two masses are painful, and fewer than 10% of women with breast cancer will present with pain.

The nurse notes that a patient in the third trimester of pregnancy feels unable to "mother" her unborn child. Which information about the patient helps the nurse identify the sources of the patient's ambivalence? Select all that apply. 1. The patient is estranged from her mother. 2. The patient asks about classes for baby care. 3. The patient expresses a loss of independence. 4. The patient's partner is excited about a baby. 5. The patient expresses disgust about body changes.

ANS: 1, 3, 5 Chapter: Chapter 5 The Psycho-Social-Cultural Aspects of the Antepartum Period Chapter Learning Objective: 3. Identify critical variables that influence adaptation to pregnancy, including age, parity, and social, cultural, and sexual orientation. Page: 102 Heading: Maternal Adaptation to Pregnancy > Maternal Tasks of Pregnancy > Acceptance of the Pregnancy Integrated Processes: Nursing Process Client Need: Psychosocial Integrity Cognitive Level: Analysis [Analyzing] Concept: Patient-Centered Care Difficulty: Difficult Feedback 1 This is correct. Women who have a positive relationship with their own mothers more easily identify with the role of motherhood. A source of ambivalence for this patient is likely related to her estranged relationship from her own mother. 2 This is incorrect. Asking about classes for baby care is not a sign of ambivalence. 3 This is correct. When the patient expresses unresolved conflict about her loss of independence, which may relate to the demands of motherhood, the nurse identifies a source of ambivalence. 4 This is incorrect. One of the tasks of motherhood is ensuring social acceptance of the baby by significant others; it is unlikely the excitement about the baby by the patient's partner is a source of ambivalence. 5 This is correct. Acceptance of a pregnancy includes acceptance of the related body changes. The patient's expressed disgust about body changes is a likely source of ambivalence.

A patient just learns that her unborn fetus has a life-threatening condition and is not expected to survive long term. Which does the nurse include in a plan of care to meet psychological needs of the patient and her partner? Select all that apply. 1. Provide time for the patient to talk about her feelings. 2. Encourage the patient's partner to be emotionally strong. 3. Facilitate referrals related to the fetal condition. 4. Monitor patient's condition and adjust visitors accordingly. 5. Ascertain if the patient and partner have previous crisis skills.

ANS: 1, 3, 5 Chapter: Chapter 7 High-Risk Antepartum Nursing Care Chapter Learning Objective: 4. Formulate a plan of care that includes the physical, emotional, and psychosocial needs of women diagnosed with pregnancy complications. Page: 151 Heading: Gestational Complications > Risk Assessment Integrated Processes: Nursing Process Client Need: Psychological Integrity Cognitive Level: Analysis [Analyzing] Concept: Ante/Intra/Post-partum Difficulty: Difficult Feedback 1 This is correct. In a time of crisis, a patient needs to be able to express and discuss their feelings in order to meet psychological needs. Providing for adequate time is planning and implementation actions in developing a nursing plan of care. 2 This is incorrect. The patient's partner is expected to be feeling emotionally distressed. Encouraging the partner to be emotionally strong does not fulfill either the patient's or partner's needs. 3 This is correct. Facilitating referrals related to the fetus condition is an implementation action that is appropriate for the patient's plan of care. The patient and partner need reliable sources of information and support. 4 This is incorrect. Monitoring the patient's condition and adjusting visitors accordingly may isolate the patient from her family/support sources. Visitation needs to have flexible guidelines to minimize the patient's isolation from family/support sources. 5 This is correct. Formulation of a solid plan of care always involves assessment. The nurse needs to ascertain if the patient and partner have experienced previous crisis events. The knowledge will guide the nurse to either implement coping strategies or evaluate and implement previous skills.

____ 16. The nurse is providing care to the 35-year-old female patient at the family practice clinic who is in the office for her annual physical examination. Which tests should the nurse recommend are the most appropriate for this patient? Select all that apply. 1. Papanicolaou test every 5 years 2. Mammogram every 2 years 3. DEXA screen every 2 years 4. HPV every 5 years 5. HIV testing every 5 years.

ANS: 1, 4 Chapter: Chapter 18 Well Women's Health Chapter Learning Objective: 2. Discuss preventative screenings for women across the life span. Page: 559 Heading: Table 18-2: Recommended Screenings and Immunizations for Women Across the Life Span Integrated Processes: Teaching and Learning Client Need: Physiological Integrity: Reduction of Risk Potential Cognitive Level: Application [Applying] Concept: Promoting Health Difficulty: Moderate Feedback 1 This is correct. According to Healthy People 2020, the Pap test along with the HPV is recommended every 5 years for women ages 30 to 65. 2 This is incorrect. Per the American Cancer Society, annual screening should be given to women ages 45 to 54. 3 This is incorrect. The DXA screening is for osteoporosis and is based on health history and risk for osteoporosis, which typically increases after menopause. 4 This is correct. According to Healthy People 2020, the Pap test along with the HPV is recommended every 5 years for women ages 30 to 65. 5 This is incorrect. There is no indication of risk for HIV in the scenario.

In preparation for a cesarean birth, the nurse expects which medical-based preoperative interventions? Select all that apply. 1. Administration of narrow-spectrum prophylactic antibiotics 2. Verification that the woman has been NPO for 6 to 8 hours before surgery 3. Assessment of the woman's knowledge and educational needs 4. Assessment for risk of venous thromboembolism (VTE) 5. Prescription for sequential compression devices prior to surgery

ANS: 1, 4, 5 Chapter: Chapter 11 Intrapartum and Postpartum Care of Cesarean Birth Families Chapter Learning Objective: 2. Discuss the preoperative nursing care and medical and anesthesia management for cesarean births. Page: 349 Heading: Perioperative Care > Scheduled Cesarean Birth Integrated Processes: Nursing Process Client Need: Physiological Integrity: Reduction of Risk Potential Cognitive Level: Analysis [Analyzing] Concept: Ante/Intra/Post-partum Difficulty: Difficult Feedback 1 This is correct. Administration of narrow-spectrum prophylactic antibiotics should occur within 60 minutes prior to the skin incision for any cesarean. If the cesarean is emergent, the antibiotics can be administered during or immediately after the procedure. Prescription of medications is a medical-based intervention. 2 This is incorrect. Verification that the woman has been NPO for 6 to 8 hours before cesarean surgery is obtained only if the cesarean is a planned procedure. The verification is a nursing-based intervention. 3 This is incorrect. Assessment of the woman's knowledge and educational needs is performed primarily for the planned or non-urgent cesarean. The assessment is primarily a nursing-based intervention. 4 This is correct. Performing an assessment for risk of venous thromboembolism (VTE) and classifying the woman based on VTE classification guidelines is a medical-based intervention. Preoperative anticoagulant therapy may be necessary for women classified as moderate or high risk or with a history of recurrent thrombosis. 5 This is correct. Application of sequential compression devices prior to surgery is to promote lower extremity circulation and aid in the prevention of blood clots. The nurse performs the action based on a medical prescription.

During the fourth stage of labor, which actions by the nurse will promote parent-newborn attachment? Select all that apply. 1. Delay administration of eye ointment until parents have held newborn. 2. Stay close with the couple and the neonate in case of an emergency. 3. Space out necessary assessments to prevent prolonged interruptions. 4. Initiate skin-to-skin contact with a warm blanket over the neonate and parent. 5. Explain expected neonatal characteristics such as molding, milia, and lanugo.

ANS: 1, 4, 5 Chapter: Chapter 15 Physiological and Behavioral Responses of the Neonate Chapter Learning Objective: 7. Discuss the nursing actions that support parents in the care of their newborn. Page: 469 Heading: Nursing Care of the Neonate > Nursing Actions During the Fourth Stage of Labor Integrated Processes: Nursing Process Client Need: Physiological Integrity: Physiological Adaptation Cognitive Level: Analysis [Analyzing] Concept: Ante/Intra/Post-partum Difficulty: Difficult Feedback 1 This is correct. Once ointment is administered, the neonate is less likely to open his or her eyes and make eye contact with parents. The administration can be delayed. 2 This is incorrect. The nurse will still be in the room with the parents and the neonate; however, the nurse does not need to stay close by. The parents will need alone time to get acquainted with the neonate. 3 This is incorrect. The nurse should actually cluster nursing interventions together in order to provide for longer interrupted time for the parents and the neonate. Many assessments can be performed while a parent holds the neonate. 4 This is correct. The nurse can initiate skin-to-skin contact with a warm blanket over the neonate and parent. 5 This is correct. The nurse can point out and explain expected neonatal characteristics such as molding, milia, and lanugo. Understanding the characteristics of their neonate will aid in bonding. The parents may be reluctant to ask about physical characteristics.

The nurse is caring for a pregnant patient who expresses concern about the effects of electronic fetal monitoring (EFM) on her labor and delivery. Which responses by the nurse would be appropriate in this situation? Select all that apply. 1. "There is a reduced rate of seizures if a patient has EFM during labor." 2. "There is a decrease in the incidence of cerebral palsy if a patient has EFM during labor." 3. "There is a link between decreased infant mortality and EFM during delivery." 4. "There is a link between the rate of cesarean sections and continuous EFM." 5. "There is an increase in operative vaginal births and the use of continuous EFM."

ANS: 1, 4, 5 Chapter: Chapter 9 Fetal Heart Rate Assessment Chapter Learning Objective: Identify the modes of fetal heart rate assessment: auscultation, palpation, EFM. Page: 278 Heading: AWHONN Standards for Frequency of Assessment of FHR Integrated Processes: Caring Client Need: Physiological Integrity: Reduction of Risk Potential Cognitive Level: Application [Applying] Concept: Ante/Intra/Post-partum Difficulty: Moderate Feedback 1 This is correct. EFM during labor is associated with reduced rates of neonatal seizures. 2 This is incorrect. EFM during labor has not been shown to reduce cerebral palsy. 3 This is incorrect. EFM during labor has not been shown to reduce infant mortality. 4 This is correct. Continuous EFM increases the rate of cesarean sections. 5 This is correct. Continuous EFM increases the rate of operative vaginal births.

The nurse is providing care to a patient who is in labor. The patient's membranes rupture spontaneously, and the nurse notices meconium-stained amniotic fluid. Which actions does the nurse immediately perform? Select all that apply. 1. Alert the neonatal team of a possible meconium aspiration neonate. 2. Promote fetal well-being by placing the patient on her left side. 3. Test the stained fluid for percentage of meconium content. 4. Administer oxygen to the mother to help prevent fetal hypoxia. 5. Notify the primary care provider about the presence of meconium.

ANS: 1, 5 Chapter: Chapter 10 High-Risk Labor and Birth Chapter Learning Objective: 4. Identify and manage high-risk pregnancy, labor, and delivery to promote healthy outcomes for the mother and infant. Page: 328 Heading: Obstetric Complications > Meconium-Stained Fluid and Birth Integrated Processes: Nursing Process Client Need: Physiological Integrity: Reduction of Risk Potential Cognitive Level: Analysis [Analyzing] Concept: Ante/Intra/Post-partum Difficulty: Difficult Feedback 1 This is correct. The nurse will immediately inform the neonatal resuscitation team that they may expect a meconium aspiration neonate. It is imperative that resuscitation occur immediately in order to avoid or decrease respiratory complications. 2 This is incorrect. The nurse will need to monitor for fetal well-being since the presence of meconium in the amniotic fluid can be indicative of fetal distress. However, placing the patient on her left side is not necessary. 3 This is incorrect. There is no reason for or means to test the amniotic fluid to determine the percentage of meconium. All infants with meconium in the amniotic fluid should have their nose, mouth, and pharynx suctioned as soon as the head is delivered (intrapartum suctioning) regardless of whether the meconium is thin or thick. 4 This is incorrect. Administering oxygen to the patient may or may not be necessary. Fetal monitoring is performed to help identify any indications of fetal hypoxia. 5 This is correct. The nurse immediately notifies the primary care provider about the meconium-stained amniotic fluid, which may be an indicator of fetal distress.

The nurse is counseling a female patient diagnosed with anovulation as a cause of infertility. Which information does the nurse provide to the patient? Select all that apply. 1. Methods for dietary planning 2. Consultation for myomectomy 3. Refrain from unprotected sex 4. Surgical correction of fallopian tubes 5. Instructions for clomiphene citrate

ANS: 1, 5 Chapter: Chapter 3 Genetics, Conception, Fetal Development, and Reproductive Technology Chapter Learning Objective: 6. Describe the most common methods used in assisted fertility. Page: 53 Heading: Infertility and Reproductive Technology > Treatment Integrated Processes: Nursing Process Client Need: Physiological Integrity: Reduction of Risk Potential Cognitive Level: Analysis [Analyzing] Concept: Sexuality Difficulty: Difficult Feedback 1 This is correct. One method of treating anovulation is to improve the patient's nutrition. The nurse will provide the patient with a variety of methods to promote a healthier dietary intake. 2 This is incorrect. A myomectomy is performed to remove uterine fibroids that can interfere with embryo implantation and/or maintenance of a pregnancy. This procedure is not used to treat anovulation. 3 This is incorrect. Refraining from unprotected sex will interfere with fertilization. 4 This is incorrect. Surgical correction of fallopian tubes may be necessary if there is an obstruction or other abnormality. However, this is not an appropriate treatment for anovulation. 5 This is correct. One drug used to treat anovulation is clomiphene citrate, which has a very high success rate. The nurse will provide the patient with information regarding this therapy. The routine patient teaching about drug therapy will be included.

The nurse works in a prenatal clinic that serves a multicultural population. The nurse is culturally aware, and so, which behaviors by a patient are expected due to common restrictive beliefs? Select all that apply. 1. A pregnant woman denies sexual intercourse during her third trimester. 2. A pregnant woman allows a clinic staff member to take a photo of her. 3. A pregnant woman reaches to an overhead shelf to collect her belongings. 4. A pregnant woman avoids sitting in front of a fan or air conditioner. 5. A pregnant woman refuses to watch a televised eclipse of the moon.

ANS: 1, 5 Chapter: Chapter 5 The Psycho-Social-Cultural Aspects of the Antepartum Period Chapter Learning Objective: 4. Identify nursing assessments and interventions that promote positive psycho-social-cultural adaptations for the pregnant woman and her family. Page: 119 Heading: Childbearing and Culture > Common Themes for the Childbearing Family Integrated Processes: Nursing Process Client Need: Psychosocial Integrity Cognitive Level: Analysis [Analyzing] Concept: Culture Difficulty: Difficult Feedback 1 This is correct. The nurse expects a pregnant woman to deny sexual intercourse in the last trimester of pregnancy; it is a restrictive belief aimed at preventing respiratory distress in a newborn. 2 This is incorrect. The nurse does not expect to see a pregnant woman allowing a staff member to take her photo; there is a restrictive belief the action may cause a stillbirth. 3 This is incorrect. The nurse does not expect to see a pregnant woman reach over her head for any reason; there is a restrictive belief the action will cause the cord to wrap around the baby's neck. 4 This is incorrect. The nurse does not expect to see a pregnant woman expose herself to cold in any form due to it causing arthritis or other chronic illness. This is an example of a restrictive belief. 5 This is correct. A restrictive belief states that a pregnant woman should not see an eclipse of the moon; to do so will cause the baby to have a cleft lip or palate.

The nurse is obtaining a baseline fetal heart rate (FHR). At 1:00 pm the baseline FHR was 130, at 1:20 pm FHR baseline was 166, and at 1:40 pm the baseline FHR was 204. What should the nurse assess from this trend of fetal baselines? Select all that apply. 1. The 1:00 pm FHR baseline warranted no further action. 2. The 1:20 pm FHR baseline warranted immediate fetal resuscitation. 3. The 1:00 pm FHR baseline warranted immediate maternal resuscitation 4. The 1:20 pm FHR baseline should be corrected immediately with delivery. 5. The 1:40 pm FHR baseline should be corrected immediately.

ANS: 1, 5 Chapter: Chapter 9 Fetal Heart Rate Assessment Chapter Learning Objective: Articulate the physiology of FHR patterns. Page: 283 Heading: Fetal Reserves > NICHD Criteria for Interpretation of FHR Patterns Integrated Processes: Nursing Process Client Need: Physiological Integrity: Reduction of Risk Potential Cognitive Level: Application [Applying] Concept: Ante/Intra/Post-partum Difficulty: Moderate Feedback 1 This is correct. The normal range is 110 to 160 bpm. 2 This is incorrect. FHR baseline above 160 bpm for at least 10 minutes is tachycardic; however, maternal position change may correct the problem before fetal resuscitation. 3 This is incorrect. The normal range is 110 to 160 bpm and therefore no maternal resuscitation was needed at that time. 4 This is incorrect. An FHR that is tachycardic should be corrected but other measured should be performed before delivery. 5 This is correct. If tachycardia persists above 200 to 220 bpm, fetal demise may occur.

The nurse is counseling a female patient about alcohol use during pregnancy. Which statement by the patient demonstrates successful patient teaching? 1. "I will limit my drinking to just one alcoholic beverage per day." 2. "It's best for my baby if I avoid drinking during pregnancy." 3. "An occasional drink on special occasions is okay." 4. "Drinking alcohol is only acceptable in the first trimester."

ANS: 2 Chapter: Chapter 1 Trends and Issues Chapter Learning Objective: 3. Identify leading causes of infant death. Page: 13 Heading: Issues > Substance Abuse During Pregnancy Integrated Processes: Nursing Process Client Need: Safe and Effective Care Environment: Safety and Infection Control Cognitive Level: Application [Applying] Concept: Ante/Intra/Post-partum Difficulty: Moderate Feedback 1 This is incorrect. Alcohol should not be consumed while pregnant. 2 This is correct. Drinking alcohol while pregnant can cause low birth weight, fetal alcohol syndrome, mental retardation, and intrauterine growth restriction. 3 This is incorrect. Alcohol should not be consumed while pregnant. 4 This is incorrect. Alcohol should not be consumed while pregnant.

The nurse is caring for a 23-year-old patient who arrives at the clinic for a pregnancy test. The test confirms the patient is pregnant. The patient states, "I do not need to stop smoking my electronic cigarette because it will not harm my baby." Which is the best response by the nurse? 1. "You are correct. Electronic cigarettes are not harmful during pregnancy." 2. "Tobacco products, including electronic cigarettes, should not be used during pregnancy due to risking nicotine toxicity." 3. "According to the FDA, although electronic cigarettes are safe for you, they can cause harm to the fetus during pregnancy." 4. "Electronic cigarettes are considered harmful only in the first trimester."

ANS: 2 Chapter: Chapter 1 Trends and Issues Chapter Learning Objective: 4. Discuss current maternal and infant health issues. Page: 12 Heading: Issues > Tobacco and Electronic Cigarette Use During Pregnancy Integrated Processes: Nursing Process Client Need: Safe and Effective Care Environment: Safety and Infection Control Cognitive Level: Application [Applying] Concept: Ante/Intra/Post-partum Difficulty: Moderate Feedback 1 This is incorrect. Electronic cigarettes can be harmful during pregnancy. 2 This is correct. Pregnant women should not use tobacco products or electronic cigarettes during pregnancy. 3 This is incorrect. Electronic cigarettes are not controlled by the FDA and may be harmful to both mother and fetus. 4 This is incorrect. Electronic cigarettes are considered harmful during pregnancy.

A nursing student is asked to set goals that will decrease the fetal death outcomes during delivery. What guidelines will the nursing student use to assist in setting her goals? 1. WHO Maternal care guidelines 2. Healthy People 2020 3. AWHONN white papers 4. State Practice Act

ANS: 2 Chapter: Chapter 1 Trends and Issues Chapter Learning Objective: 5. Identify the primary maternal and infant goals of Healthy People 2020. Page: 15 Heading: Maternal and Child Health Goals Integrated Processes: Nursing Process Client Need: Health Promotion and Maintenance Cognitive Level: Application [Applying] Concept: Health Promotion Difficulty: Moderate Feedback 1 This is incorrect. The WHO guidelines are too broad for this purpose and the nurse will need to use national goals. 2 This is correct. The national goals for improving maternal and infant health are found in Healthy People 2020. 3 This is incorrect. AWHONN white papers will present positions but not necessarily detail health promotion goals. 4 This is incorrect. State practice acts specify legal requirements rather than health promotion goals.

A patient who is pregnant expresses a desire to attempt a vaginal delivery after a cesarean birth 2 years before. The primary care provider initiates trial of labor after cesarean (TOLAC) and vaginal birth after cesarean (CVAC) screening. The nurse is aware that which patient information will likely disqualify the patient for CVAC? 1. A low transverse uterine scar 2. Cesarean due to pelvic abnormalities 3. First labor needed to be induced 4. Patient asks multiple questions

ANS: 2 Chapter: Chapter 10 High-Risk Labor and Birth Chapter Learning Objective: 5. Describe the key obstetrical emergencies and the related medical and nursing care. Page: 325 Heading: Operative Birth > Vaginal Birth After a Cesarean > Contraindications Integrated Processes: Nursing Process Client Need: Physiological Integrity: Reduction of Risk Potential Cognitive Level: Application [Applying] Concept: Ante/Intra/Post-partum Difficulty: Moderate Feedback 1 This is incorrect. A low transverse uterine scar is the safest for TOLAC/CVAC. 2 This is correct. The fact that the patient had a cesarean for pelvic abnormalities will disqualify her for TOLAC/CVAC; pelvic abnormalities are permanent. 3 This is incorrect. When the first labor is induced, the second birth may also need induction; however, this information does not disqualify the patient for TOLAC/CVAC. 4 This is incorrect. The patient considering TOLAC/CVAC is making a decision about both her safety and the safety of her fetus. Asking multiple questions is not a reason to disqualify the patient from TOLAC/CVAC.

The nurse is providing care for a prenatal patient who is told she will require a cesarean delivery because of cephalopelvic disproportion. Which explanation of the condition will the nurse provide to the patient? 1. The patient has a preexisting medical condition that supports cesarean birth. 2. The size and/or shape of either the fetal head or patient pelvis is an issue. 3. The placenta is implanted in an unfavorable position in the uterus. 4. The patient had a surgery with an incision through the myometrium of the uterus.

ANS: 2 Chapter: Chapter 11 Intrapartum and Postpartum Care of Cesarean Birth Families Chapter Learning Objective: 1. Identify factors that place a woman at risk for cesarean birth. Page: 346 Heading: Box 11-1: Indications for Cesarean Birth Integrated Processes: Nursing Process Client Need: Physiological Integrity: Reduction of Risk Potential Cognitive Level: Analysis [Analyzing] Concept: Ante/Intra/Post-partum Difficulty: Difficult Feedback 1 This is incorrect. A preexisting medical condition may be reason for a cesarean, but it does not fit with the cephalopelvic disproportion condition diagnosed. 2 This is correct. The patient requires a definition of cephalopelvic disproportion, which prior to labor includes the determination that the size and/or shape of either the fetal head or patient pelvis is an issue. 3 This is incorrect. The location of the placenta in the uterus can be a reason for a cesarean; however, it does not fit with the cephalopelvic disproportion condition diagnosed. 4 This is incorrect. Previous surgery with an incision through the myometrium of the uterus can be a reason for a cesarean. However, it does not fit with the cephalopelvic disproportion condition diagnosed.

The nurses in a labor and delivery unit are concerned about the high incidence of cesarean deliveries at their facility and initiate an internal study. Which is the most likely condition the nurses will recognize as a contributor to the rate of cesarean births? 1. The facility has a high rating for managing high-risk pregnancies. 2. Policies and parameters for cesarean need to be reviewed and refined. 3. Community education about the advantages of vaginal birth is deficient. 4. The incidence of maternal requests for cesarean delivery is increasing.

ANS: 2 Chapter: Chapter 11 Intrapartum and Postpartum Care of Cesarean Birth Families Chapter Learning Objective: 1. Identify factors that place a woman at risk for cesarean birth. Page: 347 Heading: Indications for Cesarean Birth > Preventing the First Cesarean Birth Integrated Processes: Nursing Process Client Need: Physiological Integrity: Reduction of Risk Potential Cognitive Level: Analysis [Analyzing] Concept: Ante/Intra/Post-partum Difficulty: Difficult Feedback 1 This is incorrect. The high rating of the facility for management of high-risk pregnancies may or may not impact the number of cesareans performed; not every high-risk pregnancy ends with a cesarean birth. 2 This is correct. Review of the current literature demonstrates the importance of adhering to appropriate definitions for failed induction and arrest of labor progress as a means to reduce the numbers of cesarean births. Clinical improvement strategies with careful examination of labor management practices is important. 3 This is incorrect. It is important that community education be provided explaining the advantages of vaginal births. However, this is not the lone action that will reduce the numbers of cesarean births. 4 This is incorrect. Cesarean delivery by maternal request (CDMR) accounts for only 3% of cesarean births in the United States.

The nurse in the post-delivery unit is encouraging skin-to-skin contact for a mother and neonate after cesarean delivery. Which action, if noticed by the nurse, requires immediate intervention by the nurse? 1. Mother is sitting up with the neonate prone on her chest. 2. Mother is supine with the neonate prone on her chest. 3. The neonate is prone on mother's chest and facing to the side. 4. Neonate is prone with mother resting in semi-Fowler's position.

ANS: 2 Chapter: Chapter 11 Intrapartum and Postpartum Care of Cesarean Birth Families Chapter Learning Objective: 5. Identify potential intraoperative and postoperative complications related to cesarean birth and nursing actions to reduce risk. Page: 356 Heading: Postoperative Care > First 24 Hours After Birth Integrated Process: Nursing Process Client Need: Physiological Integrity: Reduction of Risk Potential Cognitive Level: Analysis [Analyzing] Concept: Ante/Intra/Post-partum Difficulty: Difficult Feedback 1 This is incorrect. There is no need to intervene if the mother is sitting up with the neonate prone on her chest. 2 This is correct. Neonates are susceptible to sudden respiratory and cardiac arrest during the first few hours of life. Newborns in prone position on the mother's chest, especially if the mother is on her back, are especially susceptible to sudden unexpected newborn collapse (SUNC). The nurse will intervene immediately. 3 This is incorrect. There is no need to intervene if the neonate is prone on mother's chest and facing to the side. 4 This is incorrect. There is no need to intervene if the neonate is prone with the mother resting in semi-Fowler's position.

The nurse is providing care to a patient who is postpartum. Using anatomy and physiology knowledge, which expectation does the nurse relate to the cardiovascular system? 1. Patient reporting of being cold related to blood loss 2. WBC laboratory level of 30,000/mm a few hours after delivery 3. Risk for hemorrhage due to decrease in circulating clotting factors 4. A normal postpartum hemoglobin laboratory value of less than 11 g/dL

ANS: 2 Chapter: Chapter 12 Postpartum Physiological Assessments and Nursing Care Chapter Learning Objective: 1. Describe the physiological changes that occur during the postpartum period. Pages: 374 Heading: The Cardiovascular System Integrated Processes: Nursing Process Client Need: Physiological Integrity: Physiological Adaptation Cognitive Level: Analysis [Analyzing] Concept: Ante/Intra/Post-partum Difficulty: Difficult Feedback 1 This is incorrect. An average blood loss of 200 to 500 mL is expected with a vaginal birth, which has a minimal effect on a woman's system due to pregnancy-induced hypervolemia. 2 This is correct. The nurse is aware that a WBC lab level of 30,000/mm a few hours after delivery is normal and as the result of the stress of labor and birth. 3 This is incorrect. The nurse understands that postpartum patients are at risk for thromboembolism related to the increase of circulating clotting factors during pregnancy. 4 This is incorrect. Hemoglobin and hematocrit are expected to be within normal ranges postpartum. Anemia is diagnosed if the hemoglobin is less than 11 g/dL and the hematocrit is less than 32%.

The nurse is collecting the urine of a postpartum patient who is passing large clots. For which reason does the nurse examine the large collected clots? 1. To validate the presence of clotting 2. To determine the presence of tissue 3. To obtain an accurate description 4. To document the number of clots

ANS: 2 Chapter: Chapter 12 Postpartum Physiological Assessments and Nursing Care Chapter Learning Objective: 2. Identify the critical elements of assessment and nursing care during the postpartum period. Page: 370 Heading: The Reproductive System > The Endometrium > Nursing Actions Integrated Processes: Nursing Process Client Need: Physiological Integrity: Physiological Adaptation Cognitive Level: Application [Applying] Concept: Ante/Intra/Post-partum Difficulty: Moderate Feedback 1 This in incorrect. The nurse does not collect the large clots to validate the presence of clotting. 2 This is correct. The nurse collects the large clots in order to examine them for the presence of tissue, which indicates retained placenta tissue. Retained placental tissue can interfere with uterine involution and lead to excessive bleeding. 3 This is incorrect. The nurse can determine the size and appearance of the clots through a visual examination. 4 This in incorrect. The nurse can document the number of clots after visual examination.

A postpartum patient calls the OB office 8 days following a vaginal delivery. The patient reports concern regarding vaginal bleeding. Which patient-reported symptom causes the nurse concern? 1. Increased flow noticed with physical activity 2. A description of the lochia as being red in color 3. Discharge that is noted to have a fleshy odor 4. Bleeding that is described as scant

ANS: 2 Chapter: Chapter 12 Postpartum Physiological Assessments and Nursing Care Chapter Learning Objective: 2. Identify the critical elements of assessment and nursing care during the postpartum period. Pages: 370 Heading: Table 12-2: Stages and Characteristics of Lochia Integrated Processes: Nursing Process Client Need: Physiological Integrity: Physiological Adaptation Cognitive Level: Analysis [Analyzing] Concept: Ante/Intra/Post-partum Difficulty: Moderate Feedback 1 This is incorrect. When the patient reports an increased flow with physical activity, the nurse recognizes this as being normal in the period of 4 to 10 days after delivery. 2 This is correct. The lochia during the period of 4 to 10 days is described as lochia serosa (pink or brown color). The nurse will be concerned if the patient reports lochia that is red in color, which is indicative of bleeding. 3 This is incorrect. Normally, lochia has a fleshy odor. A foul odor is indicative of an infection. 4 This is incorrect. During this postpartum stage, the lochia is expected to be scant in amount.

The nurse is preparing a postpartum patient for discharge. Which patient teaching is most important for the nurse to provide? 1. The signs and symptoms of uterine infection 2. The signs and symptoms of secondary hemorrhage 3. The signs and symptoms of postpartum depression 4. The signs and symptoms of a boggy uterus

ANS: 2 Chapter: Chapter 12 Postpartum Physiological Assessments and Nursing Care Chapter Learning Objective: 4. Describe the critical elements of discharge teaching. Page: 371 Heading: The Reproductive System > The Endometrium > Patient Education Integrated Processes: Nursing Process Client Need: Physiological Integrity: Reduction of Risk Potential Cognitive Level: Analysis [Analyzing] Concept: Ante/Intra/Post-partum Difficulty: Difficult Feedback 1 This is incorrect. It is important for the nurse to provide teaching regarding the signs and symptoms of infection; however, one other topic is most important. 2 This is correct. It is most important for the nurse to provide teaching regarding the signs and symptoms of secondary hemorrhage, which often occurs after the patient is discharged. The patient needs to understand the normal progression of lochia and uterine involution, and report abnormal amounts of bleeding. 3 This is incorrect. It is important for the nurse to provide teaching regarding the signs and symptoms of postpartum depression. However, one other topic is most important. 4 This is incorrect. It is important for the nurse to provide teaching to the patient about how to identify a boggy uterus and how to perform uterine massage. The patient also needs to know that the health care provider should be contacted. However, this is not the most important teaching.

Prior to discharge from the birthing center, the nurse informs the patient that she will receive vaccines for rubella, hepatitis B, pertussis, and influenza. For which reason does the nurse explain the need for the vaccinations? 1. Discharge with a neonate is discouraged if the mother is not vaccinated. 2. Vaccinating the mother will protect the neonate from serious illnesses. 3. The mother's immune system has been suppressed during pregnancy. 4. Vaccination is more easily accomplished while the mother is under medical care.

ANS: 2 Chapter: Chapter 12 Postpartum Physiological Assessments and Nursing Care Chapter Learning Objective: 4. Describe the critical elements of discharge teaching. Page: 376 Heading: The Immune System Integrated Processes: Nursing Process Client Need: Physiological Integrity: Reduction of Risk Potential Cognitive Level: Analysis [Analyzing] Concept: Ante/Intra/Post-partum Difficulty: Difficult Feedback 1 This is incorrect. The nurse does not inform the patient that discharge with a neonate is discouraged if the mother is not vaccinated. This reason threatens the patient's autonomy. 2 This is correct. When the mother is vaccinated for rubella, hepatitis B, pertussis, and influenza, the neonate is also less likely to be infected with or affected by these diseases. Hepatitis B, pertussis, and influenza can be life-threatening for a neonate. The mother should be immunized for rubella to avoid contracting the disease during a future pregnancy. 3 This is incorrect. The mother's immune system has been suppressed during pregnancy; however, it is expected to return to normal during the postpartum period. 4 This is incorrect. Vaccination may or may not be more easily accomplished while the mother is under medical care. This depends entirely on the attitude of the mother.

The nurse is researching for evidence-based practice related to a mother's response during the postpartum period. Based on research by Rubin and Mercer, which finding will the nurse be able to easily implement to change the culture of the unit? 1. Satisfaction questionnaires 2. Alterations in terminology 3. Decrease nurse/patient ratios 4. Soliciting paternal expectations

ANS: 2 Chapter: Chapter 13 Transition to Parenthood Chapter Learning Objective: 1. Describe the process of "becoming a mother." Page: 400 Heading: Motherhood Integrated Processes: Nursing Process Client Need: Psychosocial Integrity Cognitive Level: Analysis [Analyzing] Concept: Family Dynamics Difficulty: Difficult Feedback 1 This is incorrect. Rubin and Mercer do not address the use of satisfaction questionnaires. 2 This is correct. Rubin and Mercer have addressed the terminology used regarding the mother during early postpartum. From "maternal phases" and "maternal touch" (Rubin), terminology was changed by Mercer to "maternal role attainment" and finally to "becoming a mother." This change can be easily implemented by the nurse and promote a change in the unit culture. 3 This is incorrect. Decreasing nurse/patient ratios are not part of the Rubin and Mercer research. 4 This is incorrect. Rubin and Mercer do not address the process of soliciting for paternal expectations.

The nurse is aware that some parenting skills are acquired through the process of intentional learning. Which activity does the nurse associate with intentional learning? 1. The couple observes other individuals who are mothers and fathers. 2. The couple attends hospital classes addressing newborn and infant care. 3. The couple discusses with each other how they were parented. 4. The couple watches media containing parenting roles.

ANS: 2 Chapter: Chapter 13 Transition to Parenthood Chapter Learning Objective: 2. Identify factors that influence women and men in their role transitions to mother and father. Page: 399 Heading: Transition to Parenthood > Parenting Roles Integrated Processes: Nursing Process Client Need: Psychosocial Integrity Cognitive Level: Analysis [Analyzing] Concept: Family Dynamics Difficulty: Moderate Feedback 1 This is incorrect. When the couple observes other individuals who are mothers and fathers, the couple is experiencing incidental learning. 2 This is correct. The couple who attends hospital classes addressing newborn and infant care is experiencing intentional learning. 3 This is incorrect. When a couple discusses with each other how they were parented, the couple is experiencing incidental learning. 4 This is incorrect. When the couple watches media containing parenting roles, the couple is experiencing incidental learning.

Which behavior does the nurse identify as a demonstration of unidirectional bonding between a parent and infant? 1. The parents respond to the baby's cry. 2. The parents call the baby by name. 3. The baby responds to comforting measures. 4. The parents stimulate and entertain the baby.

ANS: 2 Chapter: Chapter 13 Transition to Parenthood Chapter Learning Objective: 3. Discuss bonding and attachment. Pages: 404 Heading: Table 13-2: Bonding and Attachment Behaviors Integrated Processes: Nursing Process Client Need: Psychosocial Integrity Cognitive Level: Analysis [Analyzing] Concept: Family Dynamics Difficulty: Difficult Feedback 1 This is incorrect. When the parents respond to the baby's cry, the parents are demonstrating bidirectional attachment behaviors. 2 This is correct. When the parents call the baby by name, they are demonstrating unidirectional bonding. 3 This is incorrect. When the crying baby responds to comforting measures, the parents are demonstrating bidirectional attachment behaviors. 4 This is incorrect. When the parents stimulate and entertain the baby, they are demonstrating bidirectional attachment behaviors.

Dayton et al. (2016) performed qualitative research regarding expectant fathers' beliefs and expectations. The nurse identifies which theme as emerging from this research? 1. Men felt that the role of being a father can be learned. 2. Men described fathering as an extremely difficult task. 3. Men rely on other men to support the fathering role. 4. Men believe that the nurturing role is always the mother's.

ANS: 2 Chapter: Chapter 13 Transition to Parenthood Chapter Learning Objective: 4. Identify factors that affect the family dynamics. Page: 401 Heading: Fatherhood Integrated Processes: Nursing Process Client Need: Psychosocial Integrity Cognitive Level: Analysis [Analyzing] Concept: Family Dynamics Difficulty: Difficult Feedback 1 This is incorrect. An identified theme of men feeling that the role of father can be learned is not addressed in the referenced research. 2 This is correct. The referenced research did result in men describing fathering as being an extremely difficult task. Fathers felt the task included being responsible for another life and the importance of providing financial and concrete support to their children. 3 This is incorrect. The referenced research does not indicate that men rely on other men to support the fathering role. Men indicated that they relied on women versus men for support in their role as father. 4 This is incorrect. The referenced research does not indicate that men believe the nurturing role is always the mother's. However, implications of the study do support the involvement of the father during the pregnancy and postpartum period. Early parenting behaviors include rocking, soothing, and carrying their infant.

The nurse is observing a new mother interact with her baby and notices the mother holding the baby close to her body. However, the nurse also notices that the mother does not hold the baby in an enface position. Which question is most appropriate for the nurse to ask? 1. "Can I help you with a nice position in which to hold your baby?" 2. "What can you tell me about your family's beliefs with new babies?" 3. "Is there some reason that I have not seen you look into your baby's eyes?" 4. "Your baby is so expressive, have you looked into his eyes yet?"

ANS: 2 Chapter: Chapter 13 Transition to Parenthood Chapter Learning Objective: 5. Describe nursing actions that support couples during their transition to parenthood. Page: 406 Heading: Parent-Infant Contact > Maternal Touch Integrated Processes: Nursing Process Client Need: Culture Cognitive Level: Application [Applying] Concept: Family Dynamics Difficulty: Moderate Feedback 1 This is incorrect. The nurse first needs to find out if there is a reason for the mother avoiding gazing into the baby's eyes. It may be culturally insensitive to place the mother in the position of performing an act that is culturally avoided. 2 This is correct. Asking the mother about her family's beliefs with new babies allows the mother to provide cultural information that may be influencing the mother's interactions with her baby. 3 This is incorrect. It is confrontational for the nurse to ask specifically why the mother does not look into the baby's eyes. 4 This is incorrect. When the nurse introduces the question with a positive statement, the nurse makes the question of "why" even more uncomfortable for the mother.

The nurse works in a labor and delivery facility with new protocols for estimating postpartum blood loss. Which method for estimating blood loss is implemented in the delivery room? 1. Ask the patient how many peripads she considered to be "soaked." 2. Collect blood in calibrated, under-buttocks drapes for vaginal birth. 3. Place a basin at the foot of the delivery table to catch any blood. 4. Rely on the primary health care provider's estimate of blood loss.

ANS: 2 Chapter: Chapter 14 High-Risk Postpartum Nursing Care Chapter Learning Objective: 1. Describe the primary causes of postpartum hemorrhage and the related nursing actions and medical care. Page: 419 Heading: Hemorrhage Integrated Processes: Nursing Process Client Need: Physiological Integrity: Reduction of Risk Potential Cognitive Level: Application [Applying] Concept: Ante/Intra/Post-partum Difficulty: Moderate Feedback 1 This is incorrect. The inquiry to the patient calls for subjective judgment. The nurse is responsible for patient assessment. The situation is not related to blood loss in the delivery room. 2 This is correct. Collecting blood in calibrated, under-buttocks drapes for vaginal birth and then weighing the drapes is the easiest way to estimate blood loss in the delivery room. 3 This is incorrect. Placing a basin at the foot of the delivery table may result in a collection of no blood. During delivery, most blood will be on the drapes used on and under the patient. 4 This is incorrect. Regardless of the experience of the primary health care provider, visual estimation of blood loss is subjective and at risk for being inaccurate.

The nurse is preparing discharge teaching for a postpartum patient who exhibits signs and symptoms of an episiotomy infection and is on oral antibiotic therapy. Which discharge teaching will the nurse provide regarding pain management? 1. Application of hot packs to the perineal area 2. Information applicable to medication therapy 3. Instructions to improve circulation by ambulating 4. Medicating for pain above level 4 on a 0 to 10 scale

ANS: 2 Chapter: Chapter 14 High-Risk Postpartum Nursing Care Chapter Learning Objective: 2. Describe the primary postpartum infections and the related nursing actions and medical care. Page: 431 Headings: Infections > Wound Infections Integrated Processes: Nursing Process Client Need: Physiological Integrity: Reduction of Risk Potential Cognitive Level: Analysis [Analyzing] Concept: Ante/Intra/Post-partum Difficulty: Difficult Feedback 1 This is incorrect. With a wound infection, the nurse will advise hot packs to abdominal wounds and sitz baths for perineal wounds to promote comfort and circulation. 2 This is correct. The nurse will need to provide applicable discharge teaching for both antibiotic and analgesic therapy. Antibiotics need to be taken as ordered and until they are gone. 3 This is incorrect. The nurse does not need to advise the patient to ambulate frequently to promote circulation. Sitz baths will promote circulation without increasing pain from movement. 4 This is incorrect. The nurse does not necessarily advise the patient to take medication when the pain reaches a specific level of intensity. The patient needs to take pain medication when she feels it is necessary.

A patient delivers a term neonate and expresses concern about the reason for giving the neonate an injection. Which information from the nurse is accurate? 1. Neonates will hemorrhage without vitamin K. 2. Vitamin K is needed to activate clotting factors. 3. Mothers are unable to supply vitamin K to the fetus. 4. Breastfeeding is an excellent source of vitamin K.

ANS: 2 Chapter: Chapter 15 Physiological and Behavioral Responses of the Neonate Chapter Learning Objective: 1. Identify the changes that occur during the transition from intrauterine to extrauterine life and the related nursing actions. Page: 448 Heading: Transition to Extrauterine Life > The Hepatic System Integrated Processes: Nursing Process Client Need: Physiological Integrity: Physiological Adaptation Cognitive Level: Application [Applying] Concept: Ante/Intra/Post-partum Difficulty: Moderate Feedback 1 This is incorrect. After birth, the neonate experiences a decrease in vitamin K and is at risk for delayed clotting and hemorrhage; it is not definitive that neonates hemorrhage without receiving vitamin K after birth. 2 This is correct. Vitamin K is given to the neonate in order to activate coagulation factors II, VII, IX, and X, which are synthesized in the liver. 3 This is incorrect. During intrauterine life, the fetus receives vitamin K from its mother. After birth, the neonate experiences a decrease in vitamin K. 4 This is incorrect. The decline of maternally acquired vitamin K levels is greater in breastfed neonates, neonates with a history of perinatal asphyxia, and neonates of mothers who are on warfarin.

The lactation nurse visits the room of a patient who is postpartum and being prepared for discharge. The nurse plans to provide breastfeeding information aimed at assisting the patient to continue breastfeeding her newborn. Which observation by the nurse indicates a possible disruption to the planned teaching? 1. The patient is currently breastfeeding her baby. 2. The patient is excited about taking her baby home. 3. The patient's partner is in the patient's room. 4. The patient states she has no questions or concerns.

ANS: 2 Chapter: Chapter 16 Discharge Planning and Teaching Chapter Learning Objective: 1. Incorporate principles of teaching and learning when providing newborn care information to parents. Page: 482 Heading: Box 16-1: The Five Rights of Teaching Integrated Processes: Teaching and Learning Client Need: Physiological Integrity: Physiological Adaptation Cognitive Level: Analysis [Analyzing] Concept: Ante/Intra/Post-partum Difficulty: Difficult Feedback 1 This is incorrect. The fact that the patient is currently breastfeeding her baby will not present a disruption in the nurse performing teaching. 2 This is correct. When the patient is distracted by feelings and/or activities, there is the possibility for disruption of the nurse's teaching. The right time for teaching is imperative. 3 This is incorrect. The presence of the patient's partner will not disrupt the nurse's teaching plan. The partner can be included in the goals of the teaching. 4 This is incorrect. If the patient has no questions or concerns, the nurse may or may not decide to continue with the teaching plan. The nurse can share the teaching plan topics to elicit patient interest.

The postnatal nurse is making a newborn visit to the parents who are from a different country. The nurse finds the newborn swaddled in a heavy blanket and wearing a knitted cap. The newborn has wet hair and is restless with rapid breathing. Which initial comment from the nurse is appropriate? 1. "Your baby is exhibiting some concerning symptoms." 2. "Share with me how babies are cared for in your country." 3. "I would like to explain how to dress your baby correctly." 4. "Let me explain the baby's symptoms of being overheated."

ANS: 2 Chapter: Chapter 16 Discharge Planning and Teaching Chapter Learning Objective: 3. Demonstrate awareness of cultural values in care of newborns. Page: 494 Heading: Newborn Care > Clothing Integrated Processes: Nursing Process Client Need: Physiological Integrity: Physiological Adaptation Cognitive Level: Analysis [Analyzing] Concept: Ante/Intra/Post-partum Difficulty: Difficult Feedback 1 This is incorrect. If the nurse's initial comment expresses concern over the baby's well-being, the nurse may be interfering with open sharing and conversation with the parents. The baby is not in a life-threatening situation. 2 This is correct. When the nurse asks the parents to share how babies are cared for in their country, the nurse is showing interest and respect to the parents' culture. The nurse needs to understand the motivation behind dressing the baby in the current manner. 3 This is incorrect. When the nurse expresses a desire to explain how to correctly dress the baby, the nurse is suggesting the parents are lacking knowledge. The nurse needs to gain information and understanding about cultural differences. 4 This is incorrect. Explaining the symptoms that indicate the baby is over-dressed may be appropriate later in the conversation; however, the nurse's initial comment is aimed at achieving cultural understanding.

A breastfeeding mother is planning to return to work 3 months after her baby is born. The mother is planning to use an electric breast pump and freeze some breast milk for use later. Which information does the nurse need to provide? 1. Frozen breast milk can be defrosted in a microwave. 2. Breast milk can be kept in a deep freezer for 6 to 12 months. 3. The freezer door shelf decreases the chance of milk contamination. 4. Breast milk can only be frozen in special plastic freezer bags.

ANS: 2 Chapter: Chapter 16 Discharge Planning and Teaching Chapter Learning Objective: 8. Provide parents with information regarding newborn care that reflects the assessed learning needs of parents. Page: 490 Heading: Newborn Nutrition and Feeding > Breastfeeding > Expressing and Storing Breast Milk Integrated Processes: Teaching and Learning Client Need: Physiological Integrity: Basic Care and Comfort Cognitive Level: Application [Applying] Concept: Ante/Intra/Post-partum Difficulty: Moderate Feedback 1 This is incorrect. Breast milk in any form cannot be either thawed or warmed in a microwave. The microwave can cause uneven heating or overheating. Overheating by either microwave or stovetop can destroy antibodies within the breast milk. 2 This is correct. Breast milk can be safely kept in a deep freezer for 6 to 12 months; in a freezer attached to a refrigerator, it can be safely stored for 3 to 6 months. 3 This is incorrect. Freezer door storage causes a fluctuation of temperature every time the freezer is opened. The milk needs to be stored far back in the freezer/compartment. 4 This is incorrect. Breast milk can be stored in plastic bags, in glass bottles, or in hard BPA-free plastic containers.

The parents of a newborn male are concerned about providing care for the baby's new circumcision performed with a Plastibell. Which information will the nurse include in the teaching plan for the parents? 1. Apply lubricants to the penis to keep the diaper from sticking. 2. Report if penis is red, warm, and swollen and/or there is surgical site drainage. 3. Remove the plastic ring gently on the fifth day after surgery. 4. Contact the health care provider if newborn does not void for 36 hours.

ANS: 2 Chapter: Chapter 16 Discharge Planning and Teaching Chapter Learning Objective: 8. Provide parents with information regarding newborn care that reflects the assessed learning needs of parents. Page: 494 Heading: Newborn Care > Circumcision Care Integrated Processes: Nursing Process Client Need: Physiological Integrity: Reduction of Risk Potential Cognitive Level: Analysis [Analyzing] Concept: Ante/Intra/Post-partum Difficulty: Moderate Feedback 1 This is incorrect. With a Plastibell, lubricant is not applied to the penis; the Gomco or Mogen clamp needs a protective lubricant over the circumcision site after each diaper change for the first week. 2 This is correct. The nurse will include information to the parents that if the entire penis is red, warm, and swollen and/or there is drainage from the surgical site (signs of infection), it should be reported immediately to the health care provider. 3 This is incorrect. With the Plastibell method, the plastic ring falls off in 7 to 10 days. Parents should not pull it off. 4 This is incorrect. The health care provider should be contacted if the newborn does not void within 24 hours.

The nurse is teaching the mother of a neonate the benefits of kangaroo care. Which action is explained to the mother regarding the procedure? 1. The neonate is tucked into the front of a parent's shirt. 2. A bare-chested neonate is held against a bare-chested parent. 3. A pouch is formed from a blanket for carrying the neonate. 4. The neonate is placed in a sling and placed on a parent's side.

ANS: 2 Chapter: Chapter 16 Discharge Planning and Teaching Chapter Learning Objective: 8. Provide parents with information regarding newborn care that reflects the assessed learning needs of parents. Page: 496 Heading: Newborn Care > Kangaroo Care Integrated Processes: Nursing Process Client Need: Physiological Integrity: Physiological Adaptation Cognitive Level: Applying [Applying] Concept: Ante/Intra/Post-partum Difficulty: Moderate Feedback 1 This is incorrect. When the nurse teaches a mother the benefit of initiating kangaroo care, the neonate is not tucked into the front of a parent's shirt. 2 This is correct. When the nurse teaches a mother the benefit of initiating kangaroo care, a bare-chested neonate is held against a bare-chested parent and both the neonate and parent are covered with a warm blanket. 3 This is incorrect. When the nurse teaches a mother the benefit of initiating kangaroo care, a pouch is not formed from a blanket for carrying the neonate. 4 This is incorrect. When the nurse teaches a mother the benefit of initiating kangaroo care, the neonate is not placed in a sling and placed on a parent's side.

The nurse is collecting information from a parent whose infant has frequent diaper dermatitis. Which comment by the parent indicates a possible cause of the condition? Select all that apply. 1. "I use disposable wipes to clean the diaper area." 2. "I buy an antibiotic ointment specified for skin rashes." 3. "I leave the diaper off while the baby is sleeping." 4. "I treat any sign of a rash immediately with zinc oxide." 5. "I even get up and change the baby's diaper during the night."

ANS: 2 Chapter: Chapter 16 Discharge Planning and Teaching Chapter Learning Objective: 8. Provide parents with information regarding newborn care that reflects the assessed learning needs of parents. Page: 499 Heading: Newborn Care > Skin Care Integrated Processes: Nursing Process Client Need: Physiological Integrity: Reduction of Risk Potential Cognitive Level: Analysis [Analyzing] Concept: Ante/Intra/Post-partum Difficulty: Difficult Feedback 1 This is incorrect. The use of disposable wipes or clear water to clean an infant's diaper area is not a cause for diaper dermatitis. 2 This is correct. When an infant has diaper dermatitis, the use of antibiotic ointments, which can increase the risk of allergic skin reactions, should be avoided. This statement alerts the nurse to a possible cause of the infant's diaper dermatitis. 3 This is incorrect. Leaving the infant's diaper off during sleeping and exposing the diaper area to air helps to clear or prevent diaper dermatitis. 4 This is incorrect. Prevention and management of diaper dermatitis is most effective if the first signs are treated immediately with zinc oxide or a thin layer of petroleum-based ointment. 5 This is incorrect. The infant's diaper should be changed every 1 to 3 hours during the day. Changing the diaper at least once during the night will also help prevent diaper dermatitis.

A mother of a premature neonate in NICU asks the nurse when her baby will begin getting oral feedings. The nurse is aware that multiple conditions are desired. Which condition is most essential? 1. The neonate demonstrates proper feeding actions. 2. The neonate exhibits cardiorespiratory regulation. 3. The neonate is able to demonstrate hunger cues. 4. The neonate is able to maintain a quiet alert state.

ANS: 2 Chapter: Chapter 17 High-Risk Neonatal Nursing Care Chapter Learning Objective: 2. Identify critical elements of assessment and nursing care of the high-risk neonate. Page: 510 Heading: Preterm Neonates > Nursing Actions Integrated Processes: Nursing Process Client Need: Physiological Integrity: Reduction of Risk Potential Cognitive Level: Analysis [Analyzing] Concept: Ante/Intra/Post-partum Difficulty: Moderate Feedback 1 This is incorrect. A premature neonate needs to be able to suck, swallow, and breathe before oral feedings can be initiated. However, this is not the most essential condition. 2 This is correct. The nurse will observe the neonate for respiratory status, apnea, bradycardia, oxygenation, and feeding tolerance. The neonate needs to exhibit cardiorespiratory regulation before oral feedings are started. This is the most essential condition for oral feedings. 3 This is incorrect. When a neonate demonstrates hunger cues (bringing hand to the mouth, sucking on fingers), oral feedings can be considered. However, it is not the most essential condition. 4 This is incorrect. The ability of a neonate to maintain a quiet alert state is important for successful oral feeding; however, it is not the most essential condition.

The nurse notices that a neonate being treated for hyperbilirubinemia with phototherapy has had a daily increase of total bilirubin serum levels greater than 5 mg/dL for the past 2 days. The neonatal care provider prescribes an exchange transfusion. Which knowledge does the nurse apply to the procedure? 1. The bilirubin indicates a severe hemolytic disease. 2. Approximately 85% of the neonate's RBCs are replaced. 3. Donor RBCs are obtained from the neonate's mother. 4. The procedure is exclusive to pathological jaundice.

ANS: 2 Chapter: Chapter 17 High-Risk Neonatal Nursing Care Chapter Learning Objective: 2. Identify critical elements of assessment and nursing care of the high-risk neonate. Page: 525 Heading: Hyperbilirubinemia > Pathological Jaundice Integrated Processes: Nursing Process Client Need: Physiological Integrity: Reduction of Risk Potential Cognitive Level: Application [Applying] Concept: Ante/Intra/Post-partum Difficult: Difficult Feedback 1 This is incorrect. Exchange transfusion is used in cases where phototherapy is not effective for hyperbilirubinemia or severe hemolytic disease is present. Hyperbilirubinemia is identified in the scenario; however, there is no reference to severe hemolytic disease. 2 This is correct. The nurse is aware that approximately 85% of the neonate's RBCs are replaced with donor cells. 3 This is incorrect. Donor cells are not specifically obtained from the neonate's mother. 4 This is incorrect. This procedure reduces bilirubin, removes RBCs coated with a maternal antibody, corrects anemia, and removes other toxins associated with hemolysis. The procedure is not exclusive to the treatment of pathological jaundice.

The nurse is providing care to the adult female patient who presents to the emergency department with a suspected myocardial infarction (MI). Which symptom indicates a possible MI for the female patient? 1. Chest pain that radiates to the right arm and jaw 2. Episodic nausea/indigestion and palpitations 3. Sudden onset of trouble walking, and loss of balance 4. Swelling of the feet and shortness of breath

ANS: 2 Chapter: Chapter 18 Well Women's Health Chapter Learning Objective: 1. Identify factors that place a woman at risk for adverse health conditions. Page: 556 Heading: Health Promotion > Risk Reduction Integrated Processes: Teaching and Learning Client Need: Physiological Integrity: Basic care and comfort Cognitive Level: Analysis [Analyzing] Concept: Promoting Health Difficulty: Moderate Feedback 1 This is incorrect. The pain is typically felt in the left arm and jaw, but this type of pain presents more often to men than in women. If the woman is expecting this type of pain to present, she may suffer an MI without proper treatment. 2 This is correct. Often, the female patient will experience gastrointestinal issues as a sign of a MI and will often ignore these symptoms, believing them to be heartburn. 3 This is incorrect. This is a cardinal sign of a stroke and needs to be addressed immediately. 4 This is incorrect. Swelling of the feet and shortness of breath may be a sign of another disease process, such as renal failure or congestive heart failure, not MI.

The nurse is assembling data for an education session at the local women's shelter. The majority of the guests at the shelter are 25- to 35-year-old American Indian women with young children. What is the leading cause of death for this population? 1. Situational depression 2. Risk-taking behaviors 3. Poor prenatal care 4. Poor dietary choices

ANS: 2 Chapter: Chapter 18 Well Women's Health Chapter Learning Objective: 1. Identify factors that place a woman at risk for adverse health conditions. Page: 557 Heading: Table 18-1: Top 3 Leading Causes of Death in Females by Age Group and Race Integrated Processes: Teaching and Learning Client Need: Health Promotion and Maintenance Cognitive Level: Comprehension [Understanding] Concept: Promoting Health Difficulty: Moderate Feedback 1 This is incorrect. Although the patient may experience situational depression, it does not preclude any risks to health. 2 This is correct. This could increase the likelihood for accidents, such as not wearing seat belts. Native American women have a high risk for death from accidents at this age. 3 This is incorrect. This would increase the likelihood for fetal development issues or maternal health issues. 4 This is incorrect. Although poor dietary choices could cause disease processes, such as diabetes, obesity, and cardiac issues, it is not a leading cause of death among this age group.

A 48-year-old female patient presents to the OB/GYN clinic for her annual examination. She states that she has had the following symptoms: mood swings, irregular menstrual cycles, forgetfulness, food cravings, and a decrease in libido. Which of the following does the nurse suspect the patient is experiencing? 1. Menopause 2. Perimenopause 3. Postmenopause 4. Pregnancy

ANS: 2 Chapter: Chapter 18 Well Women's Health Chapter Learning Objective: 5. Discuss the physical and emotional changes related to perimenopause and menopause. Page: 562 Heading: Reproductive Changes Across the Life Span > Menopause Integrated Processes: Teaching and Learning Client Need: Physiological Integrity: physiological adaptation Cognitive Level: Analysis [Analyzing] Concept: Promoting Health Difficulty: Difficult Feedback 1 This is incorrect. These are classic signs of the beginnings of menopause. Had the patient stated no menstrual cycles for 12 months, then the patient would have been in menopause. 2 This is correct. Given the patient's age and current symptoms, the patient is experiencing perimenopause. 3 This is incorrect. The patient did not specify absence of menstrual cycles. 4 This is incorrect. The patient did not specify absence of menstrual cycles. Although the patient may become pregnant at this time, there is no indication of pregnancy.

A 55-year-old postmenopausal female patient presents to the family practice clinic for her annual examination. The nurse notes that the patient has lost inch in height. Upon further investigation, the nurse discovers that the patient has a slow-healing fracture of the left radius. What screening will most likely be initiated at this time for the patient? 1. Assessment for domestic violence 2. Dual-energy x-ray absorptiometry scan 3. Follicle-stimulating hormone 4. Serum levels of calcium and vitamin D

ANS: 2 Chapter: Chapter 18 Well Women's Health Chapter Learning Objective: 5. Discuss the physical and emotional changes related to perimenopause and menopause. Page: 563 Heading: Osteoporosis Integrated Processes: Teaching and Learning Client Need: Physiological Integrity: Reduction of Risk Potential Cognitive Level: Analysis [Analyzing] Concept: Promoting Health Difficulty: Moderate Feedback 1 This is incorrect. Although the fracture may be an indication of domestic violence, the rest of the descriptors do not correlate with the problem. 2 This is correct. The postmenopausal patient has experienced height loss and has a fracture. This indicates a need to screen for osteoporosis. 3 This is incorrect. There is no indication for this test to verify whether the patient is menopausal. 4 This is incorrect. Although this may be a risk factor for pathological fractures, this would not be indicated as a screening tool.

A 32-year-old female patient arrives to the emergency department with heavy uterine bleeding. The nurse finds that the patient had a positive pregnancy test 1 month prior to the visit. What is the potential medical management for this patient's issue? 1. Endometrial biopsy 2. Dilation and curettage 3. Colposcopy 4. Laparoscopy

ANS: 2 Chapter: Chapter 19 Alterations in Women's Health Chapter Learning Objective: 3. Describe common alterations in women's health, including medical management and nursing actions. Page: 576 Heading: Table 19-1: Common Diagnostic Procedures Integrated Processes: Nursing Process Client Need: Safe and Effective Care Environment: Coordinated Care Cognitive Level: Analysis [Analyzing] Concept: Patient-Centered Care Difficulty: Moderate Feedback 1 This is incorrect. This diagnostic procedure would be appropriate to determine the cause of the uterine bleeding. 2 This is correct. The patient may be experiencing an incomplete abortion and will need the remnants of the pregnancy removed. 3 This is incorrect. A colposcopy is a procedure to rule out cancer of the cervix after an abnormal Pap smear. 4 This is incorrect. Although this patient may need a laparoscopic procedure if the dilation and curettage (D&C) is not indicated, the D&C is often first in treatment of an incomplete abortion.

The telephone triage nurse receives a call from a patient who is 5 days postoperative total abdominal hysterectomy. The patient states that her pain is not relieved with the medications and that she has noticed blood in her urine. The nurse instructs the patient to report immediately to the emergency department. What does the nurse suspect as the surgical complication? 1. Possible complication related to the anesthesia 2. Possible injury to the ureters or bladder 3. Possible hemorrhage from the internal incision 4. Possible peritoneal venous thromboembolism

ANS: 2 Chapter: Chapter 19 Alterations in Women's Health Chapter Learning Objective: 3. Describe common alterations in women's health, including medical management and nursing actions. Page: 578 Heading: Hysterectomy > Risks Related to Surgical Procedure Integrated Processes: Nursing Process Client Need: Physiological Integrity: Reduction of Risk Potential Cognitive Level: Analysis [Analyzing] Concept: Safety Difficulty: Moderate Feedback 1 This is incorrect. A complication from anesthesia often effects the cardiovascular or pulmonary systems, not the urinary system. 2 This is correct. Bloody urine and unmanaged pain could indicate a ureter injury from the surgery. 3 This is incorrect. Although unmanaged pain would indicate a buildup of blood in the peritoneal cavity, the bloody urine is a cue that the ureters may be injured and the patient is bleeding within the bladder. 4 This is incorrect. A venous thromboembolism would occur in the calves as a result of the surgical procedure.

The nurse is assessing the 19-year-old female patient in the women's health clinic. She reports that for the past several months, she has had lower abdominal pain, elimination issues, and mood swings, and her acne is worse the week before menstruation. The health care provider has prescribed oral contraceptives. The patient asks the nurse for the reason for the prescription. Which response by the nurse is appropriate? 1. "Pregnancy should be avoided because the hormonal shift would place the fetus at risk for birth defects." 2. "The hormones in the oral contraceptives can maintain estrogen-progesterone hormones to alleviate discomfort." 3. "Oral contraceptives may provide an increase in the hormones to help with your possible infertility issues." 4. "Oral contraceptives may balance the chemical changes in your brain to alleviate the depression symptoms."

ANS: 2 Chapter: Chapter 19 Alterations in Women's Health Chapter Learning Objective: 3. Describe common alterations in women's health, including medical management and nursing actions. Page: 581 Heading: Table 19-2: Menstrual Disorders Integrated Processes: Nursing Process Client Need: Health Promotion and Maintenance Cognitive Level: Application [Applying] Concept: Health Promotion Difficulty: Moderate Feedback 1 This is incorrect. This treatment plan is based on the patient's symptoms, which correlate with premenstrual syndrome (PMS). 2 This is correct. PMS may be controlled, with oral contraceptives, as PMS might be related to hormonal changes related to the menstrual cycle. Estrogen-progesterone hormones found in oral contraceptives may provide relief from the symptoms. 3 This is incorrect. The patient did not claim any issues with fertility. The fertility issues may occur due to the patient's sexual dysfunction, which was not listed in her statement. 4 Incorrect. Although some of the symptoms of PMS can include mental health/mood disorders, the chemical changes in the brain are only part of the patient's symptoms. The health care provider may include antidepressants or antianxiety medications along with oral contraceptives.

The nurse is assessing a 22-year-old female patient who arrives at the family clinic. The patient has finished her prescription for a urinary tract infection and now reports a milky discharge from her vagina and has a fishy odor. After microscopic examination of the vaginal discharge, the health care provider prescribed metronidazole gel to be inserted vaginally. What is the suspected medical diagnosis for this patient? 1. The patient has gonorrhea due to lowered immunity after antibiotic therapy. 2. The patient has bacterial vaginosis due to recent disruption of normal vaginal flora. 3. The patient has genital condylomas due to the recent UTI treatment. 4. The patient has candida vaginitis due to recent disruption of normal vaginal flora.

ANS: 2 Chapter: Chapter 19 Alterations in Women's Health Chapter Learning Objective: 3. Describe common alterations in women's health, including medical management and nursing actions. Page: 588 Heading: Table 19-3: Sexually Transmitted Infections (STI) Integrated Processes: Nursing Process Client Need: Physiological Integrity: Pharmacological Therapies Cognitive Level: Analysis [Analyzing] Concept: Health Promotion Difficulty: Moderate Feedback 1 This is incorrect. Although gonorrhea may have vaginal discharge, it is not caused by lowered immunity after antibiotic therapy. 2 This is correct. Disruption of normal vaginal flora can increase the likelihood of bacterial vaginosis, and the symptoms are often described as milky vaginal discharge with a distinct "fishy" odor. 3 This is incorrect. Genital condylomas (genital warts) are caused by the human papillomavirus (HPV), not bacterial or fungal infections. 4 This is incorrect. Although some of the symptoms are the same, such as vaginal discharge, the key term in this answer is the "fishy odor."

The nurse is assisting a patient in the use of an ethical decision-making model related to quality of life. Which question is inappropriate when using this model? 1. What are the prospects for a normal life? 2. Are arrangements made for prolonging life? 3. Which type of deficits for the patient are likely? 4. Does a present or future condition make life undesirable?

ANS: 2 Chapter: Chapter 2 Ethics and Standards of Practice Issues Chapter Learning Objective: 1. Define key terms. Page: 25 Heading: Ethics in Nursing Practice > Ethical Decision-Making Models > Quality of Life Integrated Processes: Nursing Process Client Need: Physiological Integrity: Physiological Adaptation Cognitive Level: Analysis [Analyzing] Concept: Patient-Centered Care Difficulty: Moderate Feedback 1 This is incorrect. With the use of the ethical decision-making model, a quality of life consideration is whether the patient will experience a normal life. 2 This is correct. When using the ethical decision-making model for quality of life, the question needs to address if there are plans for comfort or palliative care. It is inappropriate to question about the prolongation of life. 3 This is incorrect. Consideration must be given to the deficits that the patient is likely to exhibit and the impact on the patient's quality of life. Physical, mental, and social deficits should all be considered. 4 This is incorrect. The question of how present or future condition may affect the patient is important. The quality of life must be considered over a potential life span.

The nurse is providing care for a patient in active labor. The patient continuously asks the nurse for medication to "stop the pain." Which ethical principle does the nurse use when replying, "We need to protect the baby from being overmedicated. Let me help you with some breathing and relaxation techniques"? 1. Veracity 2. Beneficence 3. Nonmaleficence 4. Fidelity

ANS: 2 Chapter: Chapter 2 Ethics and Standards of Practice Issues Chapter Learning Objective: 2. Debate ethical issues in maternity nursing. Page: 22 Heading: Ethics in Nursing Practice > Ethical Principles Integrated Processes: Caring Client Need: Safe and Effective Care Environment: Management of Care Cognitive Level: Application [Applying] Concept: Patient-Centered Care Difficulty: Moderate Feedback 1 This is incorrect. Veracity is the ethical principle dealing with being truthful with the patient about risks or admitting that the nurse needs assistance in providing appropriate care. Veracity is not the best description of the nurse's statement. 2 This is correct. Beneficence is the ethical principle related to doing good. The nurse is describing why additional medication is not given but is also offering to assist with other methods of pain management. 3 This is incorrect. Nonmaleficence is the ethical principle that supports avoidance of actions that will cause harm. The first part of the nurse's statement does address nonmaleficence, but the principle of beneficence is best supported by the complete statement. 4 This is incorrect. Fidelity is the ethical principle of being accountable and responsible for the care of the patient. This principle is most strongly related to handing off patient care to another nurse.

A nurse is suspended for refusal to participate in the performance of an elective termination of a pregnancy. Which specific group's standards does the nurse use for job reinstatement? 1. American Nurses Association (ANA) 2. Association of Women's Health, Obstetric and Neonatal Nurses (AWHONN) 3. State board of nursing 4. Facility ethics committee

ANS: 2 Chapter: Chapter 2 Ethics and Standards of Practice Issues Chapter Learning Objective: 3. Explore standards of practice in maternity nursing. Page: 25 Heading: Ethics in Nursing Practice > Ethics and Practice: Nurses' Rights and Responsibilities Integrated Processes: Nursing Process Client Need: Safe and Effective Care Environment: Management of Care Cognitive Level: Analysis [Analyzing] Concept: Ethics Difficulty: Moderate Feedback 1 This is incorrect. American Nurses Association (ANA) has a code of ethics, which describes the goals, values, and obligations of nursing. The code applies to all nurses and is intended to be adaptable to areas of specialty. However, this group does not specifically address the nurse's concern. 2 This is correct. The Association of Women's Health, Obstetric and Neonatal Nurses (AWHONN) supports the protection of an individual nurse's right to choose to participate or decline in any reproductive health care service or research. The nurse needs to seek specific support from this group. 3 This is incorrect. State boards of nursing are responsible for defining the requirements of nursing education, licensure standards, and the scope of practice for all nurses within that specific state. 4 This is incorrect. The ethics committee for the facility is responsible for reviewing patient care cases that present ethical dilemmas or legal concerns.

The nurse is counseling a patient regarding pregnancy. The patient lost her first two pregnancies in the second trimester of gestation for undetermined reasons. Which initial advice does the nurse provide for this patient? 1. Consider adoption. 2. Seek genetic studies. 3. Plan for fertility tests. 4. Attend grief counseling.

ANS: 2 Chapter: Chapter 3 Genetics, Conception, Fetal Development, and Reproductive Technology Chapter Learning Objective: 1. Discuss the relevance of genetics within the context of the care of the childbearing family. Page: 37 Heading: Genetics and the Childbearing Family Integrated Processes: Nursing Process Client Need: Health Promotion and Maintenance Cognitive Level: Analysis [Analyzing] Concept: Ante/Intra/Post-partum Difficulty: Moderate Feedback 1 This is incorrect. At some point, the nurse may suggest the patient consider adoption. However, because the patient is currently being counseled about pregnancy, the timing about adoption is questionable. 2 This is correct. Following two spontaneous abortions, the nurse initially suggests genetic studies in order to identify any genetic factors that may have caused the loss of previous pregnancies. 3 This is incorrect. The patient has been pregnant twice and fertility is not a concern. 4 This is incorrect. The patient may or may not need grief counseling. This suggestion does not directly address the patient's interest or need at this point.

The nurse works in an infertility clinic and is interviewing a male patient whose partner has been unable to conceive. Which finding obtained during a health history will cause the nurse greatest concern? 1. The patient is a passionate gardener. 2. The patient had a vasectomy reversed. 3. The patient rides a bicycle daily to work. 4. The patient is concerned about infertility.

ANS: 2 Chapter: Chapter 3 Genetics, Conception, Fetal Development, and Reproductive Technology Chapter Learning Objective: 4. List the common causes of infertility. Page: 52 Heading: Infertility and Reproductive Technology > Causes of Infertility Integrated Processes: Nursing Process Client Need: Physiological Integrity: Reduction of Risk Potential Cognitive Level: Analysis [Analyzing] Concept: Sexuality Difficulty: Difficult Feedback 1 This is incorrect. Exposure to pesticides can affect the development of mature sperm and cause male infertility. However, having a passion for gardening does not necessarily indicate an exposure to pesticides. 2 This is correct. The fact that the patient had a vasectomy reversed causes the nurse greatest concern about male infertility. A vasectomy reversal can result in the development of sperm antibodies that decreases sperm motility. 3 This is incorrect. The fact that the patient rides a bicycle to work daily is not alone a reason for concern. The nurse needs to determine the total distance ridden on a daily basis. 4 This is incorrect. Psychosocial factors can interfere with fertility; however, the nurse needs additional information about how the patient's concern impacts sexual functioning.

After counseling with an obstetrician about infertility, a couple is advised to undergo testicular sperm aspiration. The nurse is aware that the procedure may be recommended for which infertility? 1. Blocked fallopian tubes 2. Unsuccessful vasectomy reversal 3. Poor cervical mucus production 4. Diminished sperm motility

ANS: 2 Chapter: Chapter 3 Genetics, Conception, Fetal Development, and Reproductive Technology Chapter Learning Objective: 6. Describe the most common methods used in assisted fertility. Page: 54 Heading: Table 3.5: Common Assisted Fertility Technologies Integrated Processes: Nursing Process Client Need: Physiological Integrity: Physiological Adaptation Cognitive Level: Analysis [Analyzing] Concept: Sexuality Difficulty: Difficult Feedback 1 This is incorrect. Blocked fallopian tubes occur in females and are best treated with embryo transfer. 2 This is correct. An unsuccessful vasectomy reversal is treated with testicular sperm aspiration. Other reasons for this infertility treatment are the absence of a vas deferens, an extremely low sperm count, or absence of sperm in ejaculated semen. 3 This is incorrect. Poor cervical mucus production is a result of previous surgery to the cervix; it is treated with artificial insemination. 4 This is incorrect. Diminished sperm motility is a male infertility condition that is treated with artificial insemination.

The nurse is counseling a patient who shares the intention to become pregnant. Which finding during the collection of health information will the nurse feel the least concern to address? 1. The patient smokes a pack of cigarettes a week. 2. The patient lives in a recently renovated house. 3. The patient travels outside the country for work. 4. The patient has a family history of diabetes mellitus.

ANS: 2 Chapter: Chapter 4 Physiological Aspects of Antepartum Care Chapter Learning Objective: 1. Identify the major components of preconception health care. Page: 68 Heading: Preconception Health Care Integrated Processes: Nursing Process Client Need: Health Promotion and Maintenance Cognitive Level: Analysis [Analyzing] Concept: Ante/Intra/Post-partum Difficulty: Moderate Feedback 1 This is incorrect. The nurse needs to express the need for the patient to stop smoking; there is no amount of smoking that is harmless during pregnancy. 2 This is correct. The fact that the patient lives in a recently renovated house is the least concern to the nurse. If renovation was in process or the house was old and not renovated, the nurse would be concerned about exposure to environmental hazards. 3 This is incorrect. The nurse needs to further assess where the patient travels for work. Greatest concerns are about sanitation, environmental issues, exposure to potential diseases, and availability of health care if needed. 4 This is incorrect. The nurse needs to further assess the patient's family history of diabetes mellitus. The nurse needs to determine the type of diabetes and any predisposing factors such as obesity. The nurse will provide information for prevention and address a need for additional monitoring.

The nurse is collecting health information from a patient who is early in the first trimester of pregnancy. Which topic is most important for the nurse to discuss with the patient after learning that the patient works for a commercial cleaning company? 1. Risk related to exposure to environmental toxins 2. Weight limit for lifting during the patient's pregnancy 3. Importance of resting with feet up during the day 4. Reasons for the patient to look for a safer job

ANS: 2 Chapter: Chapter 4 Physiological Aspects of Antepartum Care Chapter Learning Objective: 2. Describe methods for diagnosis of pregnancy and determination of estimated date of delivery. Page: 74 Heading: Diagnosis of Pregnancy > Presumptive Signs of Pregnancy Integrated Processes: Nursing Process Client Need: Physiological Integrity: Physiological Adaptation Cognitive Level: Analysis [Analyzing] Concept: Ante/Intra/Post-partum Difficulty: Difficult Feedback 1 This is incorrect. A positive result on a home pregnancy test is a probable sign of pregnancy, not a presumptive sign, which is primarily subjective information provided by the patient. 2 This is correct. If the patient experiences breast enlargement, tenderness, and tingling after missing a period, the patient has a presumptive sign of pregnancy. This is considered a subjective finding that occurs 2 to 3 weeks after conception. 3 This is incorrect. After missing one period, it is not likely the patient will experience the first awareness of fetal movement, which is a presumptive sign but does not occur until 18 to 20 weeks after conception. 4 This is incorrect. An increase in appetite is not a presumptive sign of pregnancy. A more likely sign is nausea and vomiting, which can occur from 2 through 12 weeks.

A patient arrives for her fourth month prenatal visit and expresses concern because of a leakage of yellow fluid from her breasts. Which topic does the nurse discuss during this visit? 1. Signs of infection 2. Breast changes 3. A change in EDD 4. Support bras

ANS: 2 Chapter: Chapter 4 Physiological Aspects of Antepartum Care Chapter Learning Objective: 3. Identify the anatomical and physiological changes over the course of pregnancy. Page: 58 Heading: Table 4-1: Physiological Changes in Pregnancy Integrated Processes: Nursing Process Client Need: Physiological Integrity: Physiological Adaptation Cognitive Level: Analyzing [Analysis] Concept: Ante/Intra/Post-partum Difficulty: Moderate Feedback 1 This is incorrect. There is no need to cover signs of infection because of the patient's concern. 2 This is correct. The leakage of yellow fluid from the patient's breasts is a normal change during pregnancy. The patient is experiencing a leakage of colostrum, which is rich in antibodies for the neonate. This manifestation can begin as early as 16 weeks. 3 This is incorrect. The presentation of colostrum does not affect the EDD. 4 This is incorrect. The topic of support bras should take place early in the first trimester due to expected breast enlargement. Covering this topic does not address the patient's concern.

The nurse is providing care for a patient at 30 weeks gestation. Which topic related to patient concern or discomfort is most important for the nurse to address? 1. Increased breast enlargement 2. Dizziness when lying supine 3. Dependent edema and varicosities 4. Hyperpigmentation on the face

ANS: 2 Chapter: Chapter 4 Physiological Aspects of Antepartum Care Chapter Learning Objective: 5. Describe appropriate interventions to relieve common discomforts of pregnancy. Page: 64 Heading: Physiological Progression of Pregnancy > Cardiovascular System Integrated Processes: Nursing Process Client Need: Physiological Integrity: Reduction of Risk Potential Cognitive Level: Analysis [Analyzing] Concept: Ante/Intra/Post-partum Difficulty: Moderate Feedback 1 This is incorrect. If the patient is experiencing increased breast enlargement, the nurse should reiterate the importance of a well-fitting bra. This not the most important issue for the nurse to address. 2 This is correct. The most important issue for the nurse to address is the patient's experience of dizziness when lying supine. The nurse will provide education about supine and orthostatic hypotension and advise the patient to refrain from supine positioning. The patient needs to be instructed to use side-lying positions. 3 This is incorrect. The nurse will need to instruct the patient about the management of dependent edema and varicosities, which is to sit or lie with the feet and legs elevated several times daily. This is not the most important issue for the nurse to address. 4 This is incorrect. Because it is so noticeable, many patients will express concern over hyperpigmentation on the face. The nurse needs to review the cause and remind the patient that the coloration is likely to be temporary. This is not the most important issue for the nurse to address.

A patient is confirmed to be pregnant. Obstetric history includes two sets of twins born at 30 and 32 weeks gestation, respectively, a singleton birth born at 39 weeks gestation, and two pregnancies lost in the first trimester. In which way will the nurse define the patient's obstetrical history? 1. G4, T3, P2, A2, L3 2. G6, T1, P4, A2, L5 3. G5, T1, P2, A2, L5 4. G6, T4, P0, A4, L3

ANS: 2 Chapter: Chapter 4 Physiological Aspects of Antepartum Care Chapter Learning Objective: 6. Identify the critical elements of assessment and nursing care during initial and subsequent prenatal visits. Page: 77 Heading: Pregnancy > Due Date Calculation > Prenatal Assessment Terminology Integrated Processes: Nursing Process Client Need: Health Promotion and Maintenance Cognitive Level: Analysis [Analyzing] Concept: Ante/Intra/Post-partum Difficulty: Difficult Feedback 1 This is incorrect. The only correct information in this option is the one indicating two spontaneous abortions. 2 This is correct. The nurse will correctly determine that the patient has been pregnant six times; delivered one term neonate; had two set of twins born prematurely for a total of four births; had two spontaneous abortions before 20 weeks gestation; and currently has five living children. 3 This is incorrect. In this option, the number of pregnancies and number of premature neonates are wrong. 4 This is incorrect. In this option, all determinations are incorrect except for the number of pregnancies, which is correct at six.

A patient who is pregnant shares details of being in a physically and psychologically abusive relationship with her baby's father. Which statement by the nurse is indicative of AWHONN's standing regarding intimate partner violence (IPV)? 1. "If you are all alone, you need to make arrangements for someone to stay with you." 2. "Your partner needs to come to the office so that we can confront his behavior." 3. "I will call a women's shelter to make arrangement for you to move in immediately." 4. "Let's explore ways to protect you and stop the abuse you have been enduring."

ANS: 2 Chapter: Chapter 4 Physiological Aspects of Antepartum Care Chapter Learning Objective: 7. Describe the elements of patient education and anticipatory guidance appropriate for each trimester of pregnancy. Page: 87 Heading: Antepartal Nursing Care: Physiology-Based Nursing Assessment and Nursing Actions >Third Trimester Integrated Processes: Teaching/Learning Client Need: Physiological Integrity: Reduction of Risk Potential Cognitive Level: Applying [Application] Core concept: Ante/Intra/Post-partum Difficulty: Moderate Feedback 1 This is incorrect. Update of fetal growth and development is appropriate during all trimesters of pregnancy and not specific to the third trimester. 2 This is correct. The patient education most likely needed in the third trimester is related to the management of commonly experienced discomforts. 3 This is incorrect. General health promotion and health maintenance education is appropriate during all trimesters of pregnancy and not specific to the third trimester. 4 This is incorrect. Counseling about diet and exercise is appropriate during all trimesters of pregnancy and not specific to the third trimester.

The nurse is providing care for a patient in the third trimester of pregnancy. Which topic of patient education is most likely to be needed during this time? 1. Update on fetus growth and behavioral patterns 2. Management for commonly experienced discomforts 3. General health maintenance and promotion topics 4. Counseling and guidance about diet and exercise

ANS: 2 Chapter: Chapter 4 Physiological Aspects of Antepartum Care Chapter Learning Objective: 7. Describe the elements of patient education and anticipatory guidance appropriate for each trimester of pregnancy. Page: 87 Heading: Antepartal Nursing Care: Physiology-Based Nursing Assessment and Nursing Actions >Third Trimester Integrated Processes: Teaching/Learning Client Need: Physiological Integrity: Reduction of Risk Potential Cognitive Level: Applying [Application] Core concept: Ante/Intra/Post-partum Difficulty: Moderate Feedback 1 This is incorrect. Update of fetal growth and development is appropriate during all trimesters of pregnancy and not specific to the third trimester. 2 This is correct. The patient education most likely needed in the third trimester is related to the management of commonly experienced discomforts. 3 This is incorrect. General health promotion and health maintenance education is appropriate during all trimesters of pregnancy and not specific to the third trimester. 4 This is incorrect. Counseling about diet and exercise is appropriate during all trimesters of pregnancy and not specific to the third trimester.

A mother has a child who is 4 years of age, and she is expecting another child. The mother expresses concern to the nurse about how the older sibling will receive the newborn. Which intervention shared by the mother does the nurse discourage? 1. "I plan to let him hear the baby's heartbeat at the next prenatal visit." 2. "I think that I will just bring the new baby home as a surprise." 3. "I have enrolled him in a sibling preparation class at the hospital." 4. "I let him pick out a gift for the baby, and have one for him from the baby."

ANS: 2 Chapter: Chapter 5 The Psycho-Social-Cultural Aspects of the Antepartum Period Chapter Learning Objective: 2. Identify the major developmental tasks of pregnancy as they relate to maternal, paternal, and family adaptation. Page: 114 Heading: Family Adaptation During Pregnancy > Sibling Adaptation Integrated Processes: Nursing Process Client Need: Psychosocial Integrity Cognitive Level: Analysis [Analyzing] Concept: Family Dynamics Difficulty: Moderate Feedback 1 This is incorrect. A good tip for sibling preparation for a newborn is to allow the sibling to hear the baby's heartbeat during a prenatal visit. 2 This is correct. The nurse will discourage the mother from bringing a newborn home as a surprise. The older sibling needs time to adjust to the prospect of having a new baby. This action is likely to create a greater lack of acceptance in the older sibling. 3 This is incorrect. Taking the sibling on a tour of the hospital or the birthing center will help the child develop a sense of reality and understanding. A sibling preparation class is designed to help the older sibling prepare for the presence of a newborn. 4 This is incorrect. Having the older sibling pick out a present for the newborn, and having a gift for the sibling from the newborn will help alleviate feelings of animosity and foster a caring relationship.

The nurse works in a prenatal clinic and interacts with multiple patients from various socioeconomic backgrounds. Which patient does the nurse assess most carefully for a mental health issue? 1. A woman who chooses single parenthood 2. A military veteran who was deployed twice 3. The pregnant partner of a lesbian relationship 4. The mother who is multigestational with triplets

ANS: 2 Chapter: Chapter 5 The Psycho-Social-Cultural Aspects of the Antepartum Period Chapter Learning Objective: 3. Identify critical variables that influence adaptation to pregnancy, including age, parity, and social, cultural, and sexual orientation. Page: 104 Heading: Maternal Adaptation to Pregnancy > Factors That Influence Maternal Adaptation Integrated Processes: Nursing Process Client Need: Psychosocial Integrity Cognitive Level: Analysis [Analyzing] Concept: Patient-Centered Care Difficulty: Moderate Feedback 1 This is incorrect. The woman who chooses single parenthood will require routine assessment; however, the nurse does not need to assess this patient carefully for mental health issues. 2 This is correct. A military veteran who was deployed twice is at greater risk for depression. The nurse ascertains if the patient was treated for PTSD and any signs of mental health issues. 3 This is incorrect. The pregnant partner of a lesbian relationship does not necessarily need to be assessed for signs of mental health issues. However, the nurse is aware this couple may require additional social and emotional support. 4 This is incorrect. The mother who is multigestational with triplets will require careful physiological monitoring. In addition, the mother may need emotional support related to a higher risk for fetal well-being.

A couple is planning for the birth of their first child and is discussing the difference between a physician and a midwife. Which information presented by the couple does the nurse validate as being true? 1. Midwives are commonly self-taught without formal training. 2. Physicians provide care for both low- and high-risk patients. 3. Midwives primarily deliver babies in the home setting. 4. Physicians rely on the use of technological procedures for birth.

ANS: 2 Chapter: Chapter 5 The Psycho-Social-Cultural Aspects of the Antepartum Period Chapter Learning Objective: 5. Analyze critical factors in preparing for birth, including choosing a provider and birth setting and creating a birth plan. Page: 122 Heading: Planning for Birth > Choosing a Provider Integrated Processes: Nursing Process Client Need: Safe and Effective Care Environment: Management of Care Cognitive Level: Analysis [Analyzing] Concept: Ante/Intra/Post-partum Difficulty: Moderate Feedback 1 This is incorrect. Midwives can be self-taught without formal training, but this group manages about 1% of the deliveries in the United States and Canada. Midwives can also be RNs with advanced training who are licensed in midwifery. 2 This is correct. It is true that physicians are able to manage both low- and high-risk patients during childbirth. 3 This is incorrect. Only lay midwives deliver primarily in the home setting. Other midwives can deliver in hospitals and alternative birth centers, as well as in family homes. 4 This is incorrect. Physicians have access to technological procedures for birth, but they do not rely on or use the procedures for uncomplicated births.

A patient who is at 30 weeks gestation is involved in a car crash. The nurse recognizes that which initial testing will be used to assess fetal well-being? 1. Ultrasonography 2. Nonstress testing 3. Contraction stress test 4. Fetal movement counting

ANS: 2 Chapter: Chapter 6 Antepartal Tests Chapter Learning Objective: 2. Identify the purpose and indication for key antenatal tests. Page: 142 Heading: Antenatal Fetal Surveillance Integrated Processes: Nursing Process Client Need: Physiological Integrity: Reduction of Risk Potential Cognitive Level: Analysis [Analyzing] Concept: Ante/Intra/Post-partum Difficulty: Moderate Feedback 1 This is incorrect. Ultrasonography is useful for assessing the uterine and fetal structures; however, this test does not specifically indicate fetal well-being. 2 This is correct. Initially, nonstress testing is used to monitor fetal heart rate patterns and accelerations as an indication of fetal well-being. The heart rate of a physiologically normal fetus with adequate oxygenation and an intact autonomic nervous system accelerates in response to movement. This test is the most widely accepted method to assess fetal well-being after maternal trauma, among other conditions. 3 This is incorrect. A contraction stress test is used to ascertain fetal well-being in response to uterine contractions. This testing is primarily used at term pregnancy if the mother has a nonreactive NST. 4 This is incorrect. Fetal movement counting is a procedure used to routinely check for fetal well-being by measuring fetal movement in a specific time frame. This test is for general well-being and does not monitor fetal heart rate, which may be compromised due to trauma.

An adolescent patient who is 15 weeks pregnant refuses to have an alpha-fetoprotein test performed because, "I don't like needles." Which initial approach does the nurse take to achieve the testing? 1. Insist that testing will be done with or without her cooperation. 2. Explain the testing is important in detecting serious birth defects. 3. Ask an accompanying parent to help persuade the patient. 4. Notify the health care provider of the patient's refusal.

ANS: 2 Chapter: Chapter 6 Antepartal Tests Chapter Learning Objective: 5. Identify patient teaching needs related to antenatal tests. Page: 141 Heading: Maternal Assays > Alpha-Fetoprotein/1-Fetoprotein/Maternal Serum Alpha-Fetoprotein Integrated Processes: Nursing Process Client Need: Physiological Integrity: Reduction of Risk Potential Cognitive Level: Analysis [Analyzing] Concept: Ante/Intra/Post-partum Difficulty: Difficult Feedback 1 This is incorrect. Insisting the testing will be done with or without the patient's cooperation is inappropriate and exemplifies disregard for the patient's rights to refuse medical care. 2 This is correct. The initial approach for the nurse to take is to appeal to the patient about the importance of ensuring that her baby is healthy. 3 This is incorrect. When an adolescent patient is pregnant, she becomes independent and able to make her own decisions. Asking the parent to intervene is not appropriate and may not be effective. 4 This is incorrect. The nurse's second action will be to notify the health care provider that the patient is refusing the test. As always, the event needs to be documented.

A patient at 36 weeks gestation reports a constant dull backache, regular frequent contractions that are painless, and lower abdominal pressure. Physical examination reveals intact membranes and cervical dilation of 3 cm. Which order by the health care provider is unexpected by the nurse? 1. Administer antenatal steroids 2. Obtain fetal fibronectin levels 3. Beta-adrenergic agonist therapy 4. Monitor blood glucose levels

ANS: 2 Chapter: Chapter 7 High-Risk Antepartum Nursing Care Chapter Learning Objective: 2. Delineate clinical features indicative of pregnancy complications and tests to predict, screen for, diagnose, and manage high-risk conditions. Page: 154 Heading: Gestational Complications > Preterm Labor and Birth > Medical Management Integrated Processes: Nursing Process Client Need: Physiological Integrity: Reduction of Risk Potential Cognitive Level: Analysis [Analyzing] Concept: Ante/Intra/Post-partum Difficulty: Difficult Feedback 1 This is incorrect. The nurse should expect a prescription of antenatal steroids to allow time and facilitate fetal lung maturity. 2 This is correct. Fetal fibronectin is a previously used test, which has a low positive predictive value but a high negative predictive value, thereby making it a useful test to predict those women who will not deliver preterm. The test is considered to be unsuitable for wide-spread testing; the nurse would not expect the health care provider to prescribe this test. 3 This is incorrect. Evidence supports the use of first-line tocolytic treatment, such as beta-adrenergic agonist therapy, for short-term prolongation of pregnancy (up to 48 hours) to allow for the administration of antenatal steroids. 4 This is incorrect. Beta-adrenergic agonist drugs can cause hyperglycemia; therefore, frequent monitoring of blood glucose levels is appropriate.

The nurse educator is preparing a presentation on preterm labor (PTL) and birth (PTB). Which information does the nurse recognize as being inaccurate? 1. PTB is the leading cause of neonatal mortality and for antenatal hospitalization. 2. PTL is defined as regular uterine contractions resulting in cervical changes before 40 weeks gestation. 3. PTBs result in increased numbers of neonatal and infant deaths and long-term neurological impairment. 4. Average costs for premature/low birthweight infants are more than 10 times as high than for other newborns.

ANS: 2 Chapter: Chapter 7 High-Risk Antepartum Nursing Care Chapter Learning Objective: 5. Describe the key aspects of teaching for women with antenatal complications. Page: 152 Heading: Gestational Complications > Preterm Labor and Birth Integrated Processes: Teaching/Learning Client Need: Physiological Integrity: Reduction of Risk Potential Cognitive Level: Analysis [Analyzing] Concept: Ante/Intra/Post-partum Difficulty: Moderate Feedback 1 This is incorrect. PTB is the leading cause of neonatal mortality and the most common reason for antenatal hospitalization. 2 This is correct. This is the information the nurse needs to recognize as being inaccurate. PTL is defined as regular contractions of the?9?uterus?9?resulting in changes in the?9?cervix before 37 weeks gestation, not before 40 weeks gestation. After 37 weeks, the fetus is no longer considered premature. 3 This is incorrect. PTBs account for approximately 70% of neonatal deaths and 36% of infant deaths, as well as 25% to 50% of cases of long-term neurological impairment in children. 4 This is incorrect. Average expenditures for premature/LBW infants were more than 10 times as high as for uncomplicated newborns. PTS: 1 CON: Ante/Intra/Post-partum

The nursing is caring for a patient in active labor with significant back pain. The patient has requested nonpharmacologic methods of pain relief. The nurse thinks that sterile water injections may help with her pain management. Which of the following explanations should the nurse explain to the patient? 1. "We will inject 1 mL of sterile water intramuscularly, and it will last for about an hour." 2. "We will inject 0.5 mL of sterile water subcutaneously, and it will last for an hour to 90 minutes." 3. "We will inject 1 mL of sterile water subcutaneously, and it will last for about a half hour." 4. "We will inject 0.5 mL of sterile water intramuscularly, and it will last for about 3 hours."

ANS: 2 Chapter: Chapter 8 Intrapartum Assessment and Interventions Chapter Learning Objective: Demonstrate understanding of supportive care of the laboring woman. Page: 257 Heading: Non-Pharmacological Management of Labor Discomfort >Complementary Therapies Integrated Processes: The Nursing Process Client Need: Physiological Integrity: Pharmacological and Parenteral Therapies Cognitive Level: Application [Applying] Concept: Ante/Intra/Post-partum Difficulty: Moderate Feedback 1 This is incorrect. Sterile water injections are subcutaneous injections of 0.5 ml sterile water and provide 60 to 90 minutes of lower back pain relief. 2 This is correct. Sterile water injections are subcutaneous injections of 0.5 ml sterile water and provide 60 to 90 minutes of lower back pain relief. 3 This is incorrect. Sterile water injections are subcutaneous injections of 0.5 ml sterile water and provide 60 to 90 minutes of lower back pain relief. 4 This is incorrect. Sterile water injections are subcutaneous injections of 0.5 ml sterile water and provide 60 to 90 minutes of lower back pain relief.

The nurse preceptor is teaching a nursing student about the physiology of the fetal heart rate (FHR) pattern. Which statement by the student indicates successful teaching about this concept? 1. "Vagus nerve stimulation increases FHR and helps maintain variability." 2. "The sympathetic nervous system is responsible for heart rate variability." 3. "Action of the FHR occurs through the absence of norepinephrine." 4. "Baroreceptors are responsible for increasing FHR and fetal blood pressure."

ANS: 2 Chapter: Chapter 9 Fetal Heart Rate Assessment Chapter Learning Objective: Articulate the physiology of FHR patterns. Pages: 279 Heading: Influences on Fetal Heart Rate > Autonomic Nervous System Integrated Processes: Teaching and Learning Client Need: Safe and Effective Care Environment: Management of Care Cognitive Level: Application [Applying] Concept: Patient-Centered Care Difficulty: Moderate Feedback 1 This is incorrect. Vagus nerve stimulation slows FHR and helps maintain variability. 2 This is correct. The sympathetic nervous system is responsible for heart rate variability. 3 This is incorrect. Action occurs through the release of norepinephrine. 4 This is incorrect. Baroreceptors are stretch receptors that protect homeostasis and regulate heart rate by stimulating a vagal response and decreasing FHR, fetal blood pressure, and cardiac output.

While providing care for a patient, the nurse notices an erratic FHR recording. What action should the nurse take next? 1. Help the patient move around to help obtain the signal. 2. Place the transducer in a different position. 3. Check the mother's cervical progress to see if she is in the second stage of labor. 4. Remove some of the ultrasound gel from the transducer.

ANS: 2 Chapter: Chapter 9 Fetal Heart Rate Assessment Chapter Learning Objective: Define terms used in electronic fetal monitoring (EFM). Page: 276 Heading: Modes or Types of Fetal and Uterine Monitoring: External Electronic Fetal and Uterine Monitoring Integrated Processes: Teaching and Learning Client Need: Safe and Effective Care Environment: Management of Care Cognitive Level: Application (Applying) Concept: Ante/Intra/Post-partum Difficulty: Moderate Feedback 1 This is incorrect. Fetal or maternal movement may cause erratic FHR recordings; increasing movement would not improve the recording. 2 This is correct. Fetal or maternal movement may cause erratic FHR recordings, and moving the transducer would help the recording if fetal movement causes it. 3 This is incorrect. Erratic FHR recordings are not usually due to moving into second stage. 4 This is incorrect. A lack of gel can cause the recording to be erratic, and if this is the case, more gel should be added, not less.

The nurse is looking at an EFM strip and sees that the patient is having contractions that are measuring 150 MVU every 10 minutes for the past 2 hours and the fetus is in fetal distress. What would this indicate for next steps? 1. The patients' contractions are adequate, so the main focus should be on resuscitating the fetus with maternal oxygen and maternal position change. 2. The patients' contractions are inadequate; the provider could consider an amnioinfusion through the IUPC, and once the fetus has improved, contractions need to be augmented to be more effective. 3. The patients' contractions are adequate, so the main focus should be on determining her progress through cervical change. 4. The patients' contractions are inadequate; the provider could consider augmenting with Pitocin to be more effective.

ANS: 2 Chapter: Chapter 9 Fetal Heart Rate Assessment Chapter Learning Objective: Identify the modes of fetal heart rate assessment: auscultation, palpation, EFM. Page: 277 Heading: Modes or Types of Fetal and Uterine Monitoring > Internal Electronic Fetal and Uterine Monitoring Integrated Processes: Nursing Process Client Need: Safe and Effective Care Environment: Management of Care Cognitive Level: Application [Applying] Concept: Ante/Intra/Post-partum Difficulty: Moderate Feedback 1 This is incorrect. Although the fetus should be resuscitated, the patients' contractions are inadequate. 2 This is correct. The patients' contractions are inadequate, so the provider could consider an amnioinfusion through the IUPC. Then, once the fetus has improved, contractions need to be augmented to be more effective. 3 This is incorrect. The patients' contractions are inadequate, and determining her progress through cervical change would not improve the mother's or fetus's condition. 4 This is incorrect. Although the patients' contractions are inadequate, augmenting the patients' contractions with Pitocin might put more undue stress on the fetus.

The nurse is providing pre-amniocentesis teaching for a patient who is at 18 weeks gestation. Which information does the nurse provide? Select all that apply. 1. Positioning on the left side will avoid injury to the fetus. 2. A full bladder will assist in ultrasound visualization. 3. Discomfort will be minimized with a local anesthetic. 4. Avoid lifting heavy objects for a period of 2 weeks. 5. Abdominal cramping and bleeding is normal for 24 hours.

ANS: 2, 3 Chapter: Chapter 6 Antepartal Tests Chapter Learning Objective: 4. Articulate the nursing responsibilities related to key antenatal tests. Page: 139 Heading: Biochemical Assessment > Amniocentesis Integrated Processes: Nursing Process Client Need: Physiological Integrity: Reduction of Risk Potential Cognitive Level: Analysis [Analyzing] Concept: Ante/Intra/Post-partum Difficulty: Difficult Feedback 1 This is incorrect. Injury to the fetus and placenta is avoided through the use of ultrasonography during needle insertion. Placing the patient on her left side is not sufficient. 2 This is correct. Because the patient is less than 20 weeks gestation, a full bladder will assist with ultrasound visualization. 3 This is correct. To minimize discomfort as the needle is inserted, the patient will receive local anesthesia. 4 This is incorrect. The nurse will instruct the patient to avoid heavy lifting for a period of 2 days following the procedure. 5 This is incorrect. The nurse teaches the patient to report abdominal pain or cramping, fluid leakage, bleeding, decrease in fetal movement, fever, or chills to the HCP. The listed manifestations are not expected or normal.

The postpartum nurse-manager wants the unit to become active as a supporter of the Baby-Friendly Hospital Initiative. Which nursing actions will be initiated? Select all that apply. 1. Give pacifiers to infants on demand. 2. Help mothers initiate breastfeeding within 1 hour of birth. 3. Teach breastfeeding and promote lactation to mothers separated from infants. 4. Refer mothers to support group resources on discharge. 5. Provide infants with water until a milk supply is established.

ANS: 2, 3, 4 Chapter: Chapter 16 Discharge Planning and Teaching Chapter Learning Objective: 6. Develop a teaching plan for breastfeeding. Page: 483 Heading: Newborn Nutrition and Feeding > Breastfeeding Integrated Processes: Nursing Process Client Need: Physiological Integrity: Physiological Adaptation Cognitive Level: Analysis [Analyzing] Concept: Ante/Intra/Post-partum Difficulty: Difficult Feedback 1 This is incorrect. Giving babies nipples and pacifiers causes confusion for breastfed babies and interferes with breastfeeding. 2 This is correct. Initiating breastfeeding early is helpful in establishing breastfeeding. The ideal time line is to initiate breastfeeding within the first hour after birth. 3 This is correct. Even if separated from their infants for a period of time, mothers are offered information about breastfeeding and maintaining lactation. If the separation is related to a medical condition, the mother can pump and supply her infant with breastmilk unless it is medically contraindicated. 4 This is correct. For continuing care, breastfeeding mothers need to be offered the support of other breastfeeding mothers. 5 This is incorrect. While establishing breastfeeding, the infant will not be given supplements of formula or water. This is especially important if the mother plans to exclusively breastfeed for a period of 6 months.

The nurse is providing education to the patient that is receiving a bisphosphate medication for osteoporosis. Which instructions should the nurse include in the teaching plan to reduce side effects and enhance absorption while taking bisphosphates? Select all that apply. 1. Take the medication with a small meal at the same time. 2. Take the medication with at least 8 oz. of water. 3. Remain upright for at least 30 minutes after taking a dose. 4. Take the medication on an empty stomach. 5. Take the medication with a calcium supplement.

ANS: 2, 3, 4 Chapter: Chapter 18 Well Women's Health Chapter Learning Objective: 7. Develop a health promotion-teaching plan for an older adult based on normal physiological changes. Page: 564 Heading: Osteoporosis > Pharmacotherapy > Bisphosphonates Integrated Processes: Teaching and Learning Client Need: Physiological Integrity: Pharmacological Therapies Cognitive Level: Application [Applying] Concept: Promoting Health Difficulty: Moderate Feedback 1 This is incorrect. This medication should be given on an empty stomach 30 minutes before breakfast. The patient must follow it with at least 8 oz. of only water and remain upright for 30 minutes. 2 This is correct. This medication should be given on an empty stomach 30 minutes before breakfast. The patient must follow it with at least 8 oz. of only water and remain upright for 30 minutes. 3 This is correct. This medication should be given on an empty stomach 30 minutes before breakfast. The patient must follow it with at least 8 oz. of only water and remain upright for 30 minutes. 4 This is correct. This medication should be given on an empty stomach 30 minutes before breakfast. The patient must follow it with at least 8 oz. of only water and remain upright for 30 minutes. 5 This is incorrect. This medication should be given on an empty stomach 30 minutes before breakfast. The patient must follow it with at least 8 oz. of only water and remain upright for 30 minutes.

The nurse is aware that there are multiple classifications for cesarean deliveries. Which situations does the nurse classify as an unscheduled cesarean birth? Select all that apply. 1. Patient had a previous cesarean delivery. 2. There is evidence of a prolapsed cord with membrane rupture. 3. The cervix fails to fully dilate after prolonged labor. 4. Patient has a preexisting cardiac health condition. 5. There is recognition of placenta previa with mild bleeding.

ANS: 2, 3, 4, 5 Chapter: Chapter 11 Intrapartum and Postpartum Care of Cesarean Birth Families Chapter Learning Objective: 1. Identify factors that place a woman at risk for cesarean birth. Pages: 347 Heading: Classification of Cesarean Births Integrated Processes: Nursing Process Client Need: Physiological Integrity: Reduction of Risk Potential Cognitive Level: Analysis [Analyzing] Concept: Ante/Intra/Post-partum Difficulty: Difficult Feedback 1 This is incorrect. When a patient has a cesarean, a repeat cesarean is commonly scheduled for any additional pregnancies. 2 This is correct. Evidence of a prolapsed cord with membrane rupture is a complication requiring an emergent cesarean delivery. 3 This is correct. When the cervix fails to fully dilate after prolonged labor, an unplanned nonurgent cesarean birth is performed. 4 This is correct. The patient with a preexisting cardiac health condition is usually scheduled for a planned cesarean delivery. The patient is at risk during labor and/or vaginal birth. 5 This is correct. Recognition of placenta previa with mild bleeding is a reason for an urgent cesarean birth because of need for rapid delivery of the fetus.

The postpartum nurse is preparing to present infant care information to a couple who expresses concern about when to bathe their newborn. Which behaviors will the nurse present as general guidelines? Select all that apply. 1. Bathing is best after a feeding when newborn is relaxed. 2. Daily bathing with soap is not necessary for the newborn. 3. Use a mild preservative-free soap with a neutral pH. 4. Avoid the use of soap on the face of the newborn. 5. Genital and rectal areas should be cleaned at each diaper change.

ANS: 2, 3, 4, 5 Chapter: Chapter 16 Discharge Planning and Teaching Chapter Learning Objective: 8. Provide parents with information regarding newborn care that reflects the assessed learning needs of parents. Page: 493 Heading: Newborn Care > Bathing Integrated Processes: Teaching and Learning Client Need: Physiological Integrity: Physiological Adaptation Cognitive Level: Analysis [Analyzing] Concept: Ante/Intra/Post-partum Difficulty: Difficult Feedback 1 This is incorrect. Bathing of a newborn is best performed before a feeding to decrease the risk of emesis related to jostling during bathing. 2 This is correct. Daily bathing of a newborn with soap is not necessary and can contribute to skin irritation. Between baths with soap, the newborn can be cleaned with clear water. 3 This is correct. When bathing with soap, a mild preservative-free soap with a neutral pH is used to prevent skin irritation. 4 This is correct. The use of soap on the face is not recommended. The face and neck areas should be cleaned after feedings with plain water. 5 This is correct. Genital and rectal areas should be cleaned at each diaper change with water or diaper wipes.

A patient with pregestational diabetes mellitus delivers a neonate who is diagnosed with macrosomia. The nurse is aware that the neonate is at risk for additional long-term conditions related to maternal diabetes mellitus. Which long-term effects may occur? Select all that apply. 1. Shoulder injury related to birth size 2. Development of metabolic syndrome 3. Impaired intellectual development 4. Changes in genetic expression 5. Increased risk for chronic illnesses

ANS: 2, 3, 4, 5 Chapter: Chapter 7 High-Risk Antepartum Nursing Care Chapter Learning Objective: 6. Demonstrate understanding of knowledge related to preexisting medical complications of pregnancy and related management. Page: 168 Heading: Diabetes Mellitus > Pregestational Diabetes Integrated Processes: Nursing Process Client Need: Physiological Integrity: Reduction of Risk Potential Cognitive Level: Analysis [Analyzing] Concept: Ante/Intra/Post-partum Difficulty: Difficult Feedback 1 This is incorrect. The risk for injury to a newborn whose mother is a pregestational diabetic is related to macrosomia. Common injury is brachial plexus injuries. 2 This is correct. Long-term risks for a newborn whose mother has pregestational diabetes is for the development of metabolic syndrome. 3 This is correct. Long-term risks for a newborn whose mother has pregestational diabetes is for impaired intellectual development. There can also be an impairment of psychomotor development. 4 This is correct. Long-term risks for a newborn whose mother has pregestational diabetes is for changes in genetic expression related to exposure to hyperglycemia in utero. 5 This is correct. Long-term risks for a newborn whose mother has pregestational diabetes is for an increase in chronic illnesses in later life. This change is related to the previously listed changes in genetic expression.

The nurse is assessing a patient at 26 weeks gestation. The patient has chronic hypertension and exhibited hypertension and proteinuria prior to 20 weeks gestation. Previous blood pressure (BP) readings have been in the range of 130 to 140/88 to 90 mm Hg. Due to superimposed preeclampsia, for which additional manifestations will the nurse immediately contact the health care provider? Select all that apply. 1. Laboratory report that shows an elevation of liver enzymes 2. Current blood pressure reading of 162/102 mm Hg 3. Evident pulmonary edema noted with auscultation. 4. Subjective report of severe headache and photophobia 5. Lack of response to verbal and tactile stimulation

ANS: 2, 3, 4, 5 Chapter: Chapter 7 High-Risk Antepartum Nursing Care Chapter Learning Objective: 6. Demonstrate understanding of knowledge related to preexisting medical complications of pregnancy and related management. Page: 172 Heading: Hypertension in Pregnancy Integrated Processes: Nursing Process Client Need: Physiological Integrity: Reduction of Risk Potential Cognitive Level: Analysis [Analyzing] Concept: Ante/Intra/Post-partum Difficulty: Difficult Feedback 1 This is incorrect. In itself, elevated liver enzymes will not prompt the nurse to contact the health care provider. 2 This is correct. The patient has significant increase in blood pressure; due to a risk for stroke, the nurse is prompted to immediately contact the health care provider. 3 This is correct. Pulmonary edema can be a symptom of organ failure (cardiac, renal, hepatic). This manifestation will prompt the nurse to contact the health care provider. 4 This is correct. If the client is expressing the presence of a severe headache and photophobia, the nurse needs to be aware of worsening preeclampsia symptoms. The nurse will be prompted to notify the health care provider of these manifestations. 5 This is correct. When the nurse notices a lack of response from the patient to verbal and tactile stimulation, the nurse needs to recognize the possibility of seizure activity. The HCP needs to be notified immediately.

The nursing preceptor is teaching the nursing student about fetal bradycardia. Which is true of the maternal reasons for fetal bradycardia? Select all that apply. 1. A maternal fever in labor is usually due to dehydration and therefore should be treated with a fluid bolus. 2. Administering terbutaline to the mother for uterine tachysystole; this is self-limiting to when the drug is affecting the mother. 3. A urine toxicology screen may reveal recent cocaine use; the nurse should also monitor for placental abruption. 4. Check the chart for a history of maternal mental illness, particularly maternal anxiety; speak with the patient regarding her anxiety and take steps to ease her anxieties. 5. Check the maternal blood pressure, as hypertension is linked to fetal tachycardia; identify the on-call provider and correct with lisinopril as necessary.

ANS: 2, 3, 4, 5 Chapter: Chapter 9 Fetal Heart Rate Assessment Chapter Learning Objective: Articulate the physiology of FHR patterns. Page: 285 Heading: FHR and Contraction Pattern Interpretation> Baseline Fetal Heart Rate Integrated Processes: Teaching and Learning Client Need: Physiological Integrity: Reduction of Risk Potential Cognitive Level: Application (Applying) Concept: Ante/Intra/Post-partum Difficulty: Difficult Feedback 1 This is incorrect. Maternal fever is usually caused by an infectious agent. Antibiotics, antipyretics, and ice packs should be used to combat it. 2 This is correct. Terbutaline is given to mothers for uterine tachysystole in labor, and its effect is limited to how quickly the drug is cleared from the maternal system. 3 This is correct. Illicit drugs such as cocaine can cause fetal tachycardia and additionally can cause placental abruption. 4 This is correct. Fetal tachycardia can be linked to maternal anxiety, and taking steps to reduce it can correct the FHR. 5 This is incorrect. There is no link between hypertension and fetal tachycardia. Additionally, lisinopril is an ACE inhibitor that is not recommended in pregnancy.

The nurse is educating a 24-year-old female patient who was newly diagnosed with polycystic ovary syndrome (PCOS). The patient was prescribed an antidiabetic medication. Which medication effects should the nurse educate the patient about? Select all that apply. 1. A period of initial weight gain 2. A change in her integumentary system 3. Low blood sugar until her hormones stabilize 4. Decreased likelihood of pregnancy 5. A decrease in abdominal obesity and weight

ANS: 2, 3, 5 Chapter: Chapter 19 Alterations in Women's Health Chapter Learning Objective: 3. Describe common alterations in women's health, including medical management and nursing actions. Page: 584 Heading: Polycystic Ovary Syndrome > Medical Management Integrated Processes: Nursing Process Client Need: Physiological Integrity: Physiological adaptation Cognitive Level: Analysis [Analyzing] Concept: Patient-Centered Care Difficulty: Difficult Feedback 1 This is incorrect. The patient may experience a weight loss due to the decreased blood glucose levels. 2 This is correct. The patient may notice loss of hirsutism and acne while taking the antidiabetic medication. This is because this medication decreases testosterone levels. 3 This is correct. The nature of the antidiabetic medication can cause lower blood sugar, and the patient will need to take the proper precautions. 4 This is incorrect. This medication is actually used to treat infertility. 5 This is correct. This may lower the testosterone level, which may decrease the degree of acne, hirsutism, and abdominal obesity.

The nurse educator is teaching about regional and local anesthesia in a childbirth education class. The nurse should teach that regional anesthesia is used for which of the following? Select all that apply. 1. Lidocaine 1% for episiotomy 2. Epidural with lidocaine for pain in labor 3. Spinal block with lidocaine for a cesarean delivery 4. Lidocaine 1% for episiotomy repair 5. Lidocaine without epinephrine for forceps use

ANS: 2, 3, 5 Chapter: Chapter 8 Intrapartum Assessment and Interventions Chapter Learning Objective: Demonstrate understanding of supportive care of the laboring woman. Page: 259 Heading: Table 8-4 Anesthesia in Labor and Delivery Integrated Processes: Teaching and Learning Client Need: Physiological Integrity: Pharmacological and Parenteral Therapies Cognitive Level: Application [Applying] Concept: Ante/Intra/Post-partum Difficulty: Moderate Feedback 1 This is incorrect. A local anesthesia can be used for episiotomy. 2 This is correct. Regional anesthesia can be used for labor pain in the form of a pudendal block, epidural block, or spinal block. 3 This is correct. Regional anesthesia can be used for surgical pain in cesarean section. 4 This is incorrect. A local anesthesia is used for episiotomy repair. 5 This is correct. A pudendal block (which is regional anesthesia) is used for forceps use.

The nursing department of a large facility is interested in improving clinical care with the introduction of EBP. Which barriers to EBP does the nursing department expect within the facility? Select all that apply. 1. Changes in nursing academia to include how to integrate research into practice 2. Outdated facility politics and policies that do not embrace research-based change 3. A general lack of information addressing nursing-focused research and change 4. Limited resources available to clinical care providers from nursing administration 5. Resistance by other health care professionals to providing nursing with autonomy

ANS: 2, 4 Chapter: Chapter 2 Ethics and Standards of Practice Issues Chapter Learning Objective: 5. Analyze concepts related to evidence-based practice. Page: 32 Heading: Evidence-Based Practice > Evidence-Based Nursing > Barriers to Evidence-Based Practice Integrated Processes: Nursing Process Client Need: Safe and Effective Care Environment: Management of Care Cognitive Level: Analysis [Analyzing] Concept: Evidence-Based Practice Difficulty: Difficult Feedback 1 This is incorrect. It is believed that nursing educational programs need to learn how to include EBP research in clinical care. Currently, the focus is on the rigorous process of performing research. However, this barrier is not managed within the health care facility. 2 This is correct. Many health care facilities have outdated political and policies, which can be a barrier to EBP. Nurses need the support, approval, and process by which to make EBP changes. This barrier can be managed within the facility. 3 This is incorrect. EBP applicable to the clinical site is readily available at this time and is not necessarily influenced by health care facilities. Barriers to EBP change is not based on available research, but on reluctance for facility involvement and management. 4 This is correct. A very real barrier to EBP at the clinical setting is the possibility of nursing administration failing to provide the resources needed to make changes in clinical care delivery. EBP may include provisions for resources, personnel, and training. 5 This is incorrect. Inadequate EBP knowledge and skills may exist with nurses and other health care professionals; however, it is not expected that resistance to nursing autonomy will become a barrier to EBP. The focus of all health care providers is on improvement of patient care and outcomes.

A couple informs the nurse they have decided to make arrangements for a home birth. Which criteria will the nurse share with the couple regarding a safe home birth? Select all that apply. 1. The couple must be trained on how to be in control of the birth. 2. The pregnant woman must be in good health with a normal pregnancy. 3. The birthing home must be within a 1-hour drive of a hospital. 4. Adequate medical supplies and resuscitation equipment is available. 5. The birthing room needs to be sterile prior to labor and delivery.

ANS: 2, 4 Chapter: Chapter 5 The Psycho-Social-Cultural Aspects of the Antepartum Period Chapter Learning Objective: 5. Analyze critical factors in preparing for birth, including choosing a provider and birth setting and creating a birth plan. Page: 122 Heading: Planning for Birth > Choosing a Place of Birth Integrated Processes: Nursing Process Client Need: Safe and Effective Care Environment: Management of Care Cognitive Level: Analysis [Analyzing] Concept: Ante/Intra/Post-partum Difficulty: Difficult Feedback 1 This is incorrect. The couple planning a home birth do not need training about how to control the birth, but they do need a solid understanding of the process. 2 This is correct. The nurse is correct in informing the couple that the woman must be in good health with a normal pregnancy. High-risk pregnancies are not suitable for home births. 3 This is incorrect. The nurse needs to share that in case of an emergency or complications, a good method of transportation should be accessible. The couple should evaluate possible consequences of living 1 hour away from emergency assistance. 4 This is correct. The nurse needs to inform the couple that in addition to being attended by a well-trained health care provider, adequate medical supplies and resuscitation equipment need to be in the home. 5 This is incorrect. There is no need for the birthing room in the home to be sterile prior to labor and delivery. The risk for infection in a home birth may be lower than in a hospital.

The nurse is caring for a woman who is pregnant. The patient reports that she has also smoked less than five cigarettes per day and that she has continued to smoke during her pregnancy. Routine prenatal examination and diagnostics have indicated she tested negative for sexually transmitted infections and has a BMI of 25. Based on statistics, which is likely true about the patient and the developing fetus? Select all that apply. 1. She will breastfeed her infant. 2. She is probably aged 20-24. 3. She likely has earned a college degree. 4. Her child may have impaired brain development. 5. Her child is more likely to be born prematurely.

ANS: 2, 4, 5 Chapter: Chapter 1 Trends and Issues Chapter Learning Objective: 3. Identify leading causes of infant death. Page: 12 Heading: Issues > Tobacco and Electronic Cigarette Use During Pregnancy Integrated Processes: Nursing Process Client Need: Physiological Integrity: Reduction of Risk Potential Cognitive Level: Application [Applying] Concept: Ante/Intra/Post-partum Difficulty: Moderate Feedback 1 This is incorrect. Mother who smoke during pregnancy are less likely to breastfeed their infants. 2 This is correct. Only 4.5% of mothers over 35 smoke during pregnancy, which is the lowest smoking prevalence rate of age ranges listed. Mothers aged 20-24 have the highest prevalence of smoking during pregnancy (13%). 3 This is incorrect. Statistics indicate that mothers with less than a high school diploma have the highest prevalence of smoking during pregnancy (14.1%). Patients with a bachelor's degree or higher have the lowest prevalence of smoking during pregnancy (0.9%). 4 This is correct. Fetal brain development may be impaired when the mother continues to smoke throughout pregnancy. 5 This is correct. Tobacco exposure during development is toxic to developing fetuses. It may contribute to a variety adverse effects, including abruptio placenta and premature birth.

The nurse is providing support to parents of a premature neonate in NICU. Which actions by the nurse will best provide psychosocial support to the parents? Select all that apply. 1. Assess the parents' ability to care for their neonate. 2. Ask the parents how they are coping with the experience. 3. Provide equipment for breast pumping and storage of milk. 4. Encourage parents to take photos to share with family and friends. 5. Praise parents for their involvement in the care of their neonate.

ANS: 2, 4, 5 Chapter: Chapter 17 High-Risk Neonatal Nursing Care Chapter Learning Objective: 4. Describe the loss and grief process experienced by parents whose infant has died. Page: 545 Heading: Psychosocial Needs of Parents with High-Risk Neonates > Nursing Actions Integrated Processes: Nursing Process Client Need: Physiological Psychosocial Integrity Cognitive Level: Analysis [Analyzing] Concept: Ante/Intra/Post-partum Difficulty: Difficult Feedback 1 This is incorrect. Assessing the parents' ability to provide care for their neonate is meeting a physiological need. 2 This is correct. Asking the parents how they are coping with the experience provides the parents with an opportunity to talk about their feelings and responses. This action by the nurse is providing psychosocial support. 3 This is incorrect. When the nurse provides equipment for pumping breast milk and storage, the nurse is meeting a physiological need. 4 This is correct. Encouraging parents to take photos of their neonate to share with family and friends is providing psychosocial support. 5 This is correct. Praising the parents for their involvement in providing care for their neonate is meeting the need for psychosocial support.

The labor and delivery nurse is providing care to a patient in active labor. The nurse notes EFM changes that suggest fetal distress and monitors the fetus for an additional 20 minutes before calling the health care provider. The health care provide determines that an emergency cesarean delivery is required. During transport, EFM is interrupted and not resumed due to expectations of an emergent procedure. A stillborn fetus is delivered. For which actions can the nurse be held legally responsible? Select all that apply. 1. Inappropriate use of oxytocin, causing fetal distress. 2. Lack of appropriate response to fetal compromise. 3. Inability to initiate resuscitation to a compromised fetus. 4. Delayed communication resulting in a delay of cesarean. 5. Performance of a technical error related to monitoring.

ANS: 2, 4, 5 Chapter: Chapter 2 Ethics and Standards of Practice Issues Chapter Learning Objective: 4. Describe legal issues in maternity nursing. Page: 27 Heading: Legal Issues in Delivery of Care Integrated Processes: Nursing Process Client Need: Physiological Integrity: Reduction of Risk Potential Cognitive Level: Analyze [Analyzing] Concept: Legal Difficulty: Difficult Feedback 1 This is incorrect. There is no indication in the scenario that oxytocin was used during labor. 2 This is correct. The nurse can be held legally responsible for the lack of an appropriate response to fetal compromise. The nurse needs to immediately report indications of fetal compromise to colleagues, charge nurses, and/or health care providers. 3 This is incorrect. Prior to birth, resuscitation on a distressed fetus is not possible. 4 This is correct. Communication was delayed by at least 20 minutes of additional monitoring, which prolonged fetal distress. The nurse can be held legally responsible for the delay in emergency care. 5 This is correct. EFM was interrupted during transport and not reestablished for continued monitoring. The nurse can be held legally responsible for a technology error.

The nurse is teaching the patient about the trends and risks of epidural anesthesia. Which of the following should the nurse relay about the risks of epidural anesthesia? Select all that apply. 1. There is a higher rate of instrumental vaginal delivery. 2. When using an epidural, women are less likely to sweat with a fever. 3. There is a higher rate of spontaneous vaginal delivery. 4. Labor may be longer with an epidural. 5. There is a higher rate of fever and sepsis.

ANS: 2, 4, 5 Chapter: Chapter 8 Intrapartum Assessment and Interventions Chapter Learning Objective: Demonstrate understanding of supportive care of the laboring woman. Page: 260 Heading: Pharmacological Management of Labor Discomfort Integrated Processes: Teaching and Learning Client Need: Physiological Integrity: Pharmacological and Parenteral Therapies Cognitive Level: Application (Applying) Concept: Ante/Intra/Post-partum Difficulty: Moderate Feedback 1 This is correct. Epidurals are associated with higher rates of forceps and vacuum suction rates. 2 This in correct. An elevation in maternal temperature is associated with regional anesthesia, and it is also associated with reduced perspiration. 3 This is incorrect. Epidurals are associated with lower rates of spontaneous vaginal delivery. 4 This is correct. Epidurals are associated with longer labor. 5 This is correct. Epidurals are associated with higher rates of fever and sepsis.

The nurse is attending to a patient who just delivered a term fetus who was stillborn. Which nursing interventions will the nurse use to provide emotional support to the couple? Select all that apply. 1. Express the belief that a little angel was sent to heaven. 2. Cut a lock of the neonate's hair and get foot and hand prints. 3. Ask the parents what name they are giving their baby. 4. Inquire if the patient had any warning of fetal death. 5. Allow parents unlimited time to hold and touch the neonate.

ANS: 2, 5 Chapter: Chapter 10 High-Risk Labor and Birth Chapter Learning Objective: 5. Describe the key obstetrical emergencies and the related medical and nursing care. Page: 330 Heading: Obstetric Complications > Stillbirth/Intrauterine Fetal Demise > Nursing Actions Integrated Processes: Culture and Spirituality Client Need: Physiological Integrity: Reduction of Risk Potential Cognitive Level: Analysis [Analyzing] Concept: Ante/Intra/Post-partum Difficulty: Difficult Feedback 1 This is incorrect. It is important for health care professionals to not use any clichés that belittle the situation. 2 This is correct. The parents need to have mementoes of the baby. Items may include photographs of the infant, locks of hair, name bracelets, footprints, measuring tape, name certificates, quilts, clothing, poems, or sympathy cards. 3 This is incorrect. Nurses can build trust with the women and families they care for while keeping in mind that decisions made are based on each woman's or family's unique cultural and individual identity. Some parents may name the neonate and arrange for specific rituals. 4 This is incorrect. Helping grieving families includes listening more than talking, allowing for silence, being genuine and caring, allowing them to express their feelings, and listening to their story without passing judgment. Asking the patient about warning signs of fetal death may foster feelings of guilt. 5 This is correct. The neonate is wrapped in a blanket and given to the parents to hold and touch for as long as they wish. Other family members may be included as the parents wish.

The nurse is taking a history of a mother who admits to cocaine drug use. Which action should the nurse take first? 1. Refer the patient to a drug abuse program. 2. Screen the infant for side effects associated with cocaine use. 3. Educate the patient of the risks associated with cocaine use during pregnancy. 4. Advise the patient that her baby will be okay even with the history of cocaine use.

ANS: 3 Chapter: Chapter 1 Trends and Issues Chapter Learning Objective: 3. Identify leading causes of infant death. Page: 13 Heading: Issues > Substance Abuse During Pregnancy Integrated Processes: Nursing Process Client Need: Safe and Effective Care Environment: Safety and Infection Control Cognitive Level: Application [Applying] Concept: Ante/Intra/Post-partum Difficulty: Difficult Feedback 1 This is incorrect. This is appropriate, but not the first action the nurse should take. 2 This is incorrect. The infant is not screened for side effects of maternal drug use until delivery. 3 This is correct. The patient should be educated on possible risks associated with drug use. 4 This is incorrect. It is not appropriate to tell a patient "your baby will be okay" in any circumstance.

The nurse is caring for a patient at 7 weeks gestation. The nurse suspects that a pregnant patient may have been using marijuana. With consent, the nurse confirms via urine drug screen. Which statement by the nurse is most appropriate? 1. "Did you smoke marijuana when pregnant with your other child?" 2. "To avoid negative effects on your baby, you'll need to stop using marijuana during your last trimester." 3. "Using marijuana while pregnant can have a negative effect on the neurological development of your baby." 4. "Marijuana use while pregnant greatly increases your risk of miscarriage."

ANS: 3 Chapter: Chapter 1 Trends and Issues Chapter Learning Objective: 3. Identify leading causes of infant death. Page: 13 Heading: Issues > Substance Abuse During Pregnancy Integrated Processes: Nursing Process Client Need: Safe and Effective Care Environment: Safety and Infection Control Cognitive Level: Application [Applying] Concept: Ante/Intra/Post-partum Difficulty: Moderate Feedback 1 This is incorrect. Whether or not the woman used marijuana during her previous pregnancy is not relevant to her current care. 2 This is incorrect. Marijuana should not be used at any point during pregnancy. 3 This is correct. Marijuana use during pregnancy may have a negative effect on the neurological development of the fetus. 4 This is incorrect. There currently is no research linking marijuana use to increased risk of miscarriage.

The nurse is caring for a 16-year-old patient who is 32 weeks pregnant with her first child, who is male. The patient's mother has accompanied her to today's visit. During the nursing assessment, the patient mentions that she is no longer in a relationship with the baby's father but her mother plans to help her. However, the patient's mother asks whether this will have any impact on the child. Which should the nurse indicate the child is at increased risk of during his adolescence? 1. Hypertension 2. Diabetes 3. Alcohol abuse 4. Intraventricular bleeding

ANS: 3 Chapter: Chapter 1 Trends and Issues Chapter Learning Objective: 4. Discuss current maternal and infant health issues. Page: 12 Heading: Issues > Teen Pregnancy > Implications of Teen Pregnancy and Birth Integrated Processes: Nursing Process Client Need: Physiological Integrity: Reduction of Risk Potential Cognitive Level: Application [Applying] Concept: Ante/Intra/Post-partum Difficulty: Moderate Feedback 1 This is incorrect. Teen mothers, rather than their children, are at increased risked of hypertension during pregnancy. 2 This is incorrect. Children born to mothers who are obese have an increased risk of developing childhood obesity and childhood diabetes. 3 This is correct. Statistics have shown that adolescent boys without an involved father may be at higher risk of incarceration, dropping out of school, and abusing drugs or alcohol. 4 This is incorrect. Children born to teen mothers are at increased risk for health problems associated with low birth weight, including intraventricular bleeding.

A pregnant patient with a BMI of 35 is concerned about health effects she and her baby may face during pregnancy. During routine testing, the patient tested negative for sexually transmitted illnesses (STIs) and indicated that she is in a committed, long-term relationship with the child's father. Which of the following is accurate? 1. The patient's infant is at increased risk of neonatal blindness. 2. The patient's infant has a decreased risk of birth injury. 3. The patient will have increased risk of wound infection. 4. The patient will have a decreased risk of preeclampsia.

ANS: 3 Chapter: Chapter 1 Trends and Issues Chapter Learning Objective: 4. Discuss current maternal and infant health issues. Page: 14 Heading: Issues > Obesity Integrated Processes: Nursing Process Client Need: Physiological Integrity: Reduction of Risk Potential Cognitive Level: Application [Applying] Concept: Ante/Intra/Post-partum Difficulty: Difficult Feedback 1 This is incorrect. Infants born to mothers with certain sexually transmitted illnesses (STIs) are at increased risk of neonatal blindness. 2 This is incorrect. Infants born to obese pregnant women have increased risk of birth injury related to macrosomia. 3 This is correct. Obese pregnant patients are at increased risk for wound infections. 4 This is incorrect. Obese pregnant patients have an increased risk of developing certain conditions, including gestational diabetes, gestational hypertension, and preeclampsia.

The nurse is teaching a prenatal class. For which reason does the nurse emphasize the importance of managing maternal fear during labor? 1. Fear promotes feelings of exhaustion. 2. Mothers cannot enjoy the actual birth. 3. Dystocia is associated with extreme fear. 4. Fear during labor causes postpartum depression.

ANS: 3 Chapter: Chapter 10 High-Risk Labor and Birth Chapter Learning Objective: 1. Describe the primary causes of dystocia and the related nursing and medical care. Page: 302 Heading: Dystocia Integrated Processes: Nursing Process Client Need: Physiological Integrity: Reduction of Risk Potential Cognitive Level: Application [Applying] Concept: Ante/Intra/Post-partum Difficulty: Moderate Feedback 1 This is incorrect. Fear and exhaustion are often experienced together by the patient in labor. However, there is a more important reason to focus on fear. 2 This is incorrect. The nurse will focus on the physiological impact of fear during labor rather on the extent to which a patient will enjoy the birthing process. 3 This is correct. Extreme maternal fear or exhaustion can result in catecholamine release interfering with uterine contractility, a condition that will cause dystocia. 4 This is incorrect. Postpartum depression is related to hormonal changes after birth.

The nurse is providing care for a primip patient in active labor. Cervical dilation has progressed 0.5 cm in 2 hours. Intrauterine pressure catheter reading is 20 mm Hg. Which action does the nurse anticipate next? 1. Rupture of uterine membranes by the nurse 2. Preparation for a cesarean delivery due to signs of fetal distress 3. Augmentation of labor with oxytocin per health care provider's order 4. Medicating the patient with pain medication to promote uterine rest

ANS: 3 Chapter: Chapter 10 High-Risk Labor and Birth Chapter Learning Objective: 1. Describe the primary causes of dystocia and the related nursing and medical care. Page: 303 Heading: Dystocia > Uterine Dystocia > Hypotonic Uterine Dysfunction Integrated Processes: Nursing Process Client Need: Physiological Integrity: Reduction of Risk Potential Cognitive Level: Analysis [Analyzing] Concept: Ante/Intra/Post-partum Difficulty: Difficult Feedback 1 This is incorrect. Rupture of the uterine membranes can improve dystocia-related hypotonic uterine dysfunction; however, this is not a nursing action. The health care provider will rupture the membranes and react accordingly if umbilical cord prolapse or fetal distress occurs. In this scenario, the membranes are likely ruptured because of the use in an intrauterine pressure catheter. 2 This is incorrect. A cesarean birth may be necessary; however, other actions to resolve the hypotonic uterine dysfunction will be tried first unless there is fetal distress. 3 This is correct. The action the nurse will anticipate is the augmentation of labor with administration of the prescribed oxytocin. 4 This is incorrect. Medicating the patient with pain medication to promote uterine rest is not an appropriate action for a patient with hypoactive uterine dysfunction. This is an appropriate action for the patient with hyperactive uterine dysfunction.

The nurse is providing care for a patient who is admitted for cervical ripening. The health care provider has prescribed the use of a hygroscopic dilator. Which conclusion is the nurse likely to draw from the prescribed method of cervical ripening? 1. This method is quicker than hormonal ripening. 2. The patient has a history of cesarean childbirth. 3. The method may be indicative of fetal demise. 4. This patient is being treated for active herpes.

ANS: 3 Chapter: Chapter 10 High-Risk Labor and Birth Chapter Learning Objective: 2. Demonstrate understanding of knowledge related to induction of labor and augmentation of labor and vaginal birth after cesarean birth. Page: 315 Heading: Labor Interventions > Cervical Ripening Integrated Processes: Nursing Process Client Need: Physiological Integrity: Reduction of Risk Potential Cognitive Level: Analysis [Analyzing] Concept: Ante/Intra/Post-partum Difficulty: Difficult Feedback 1 This is incorrect. The process of cervical ripening using a hygroscopic dilator occurs over a period of 12 to 24 hours as the inserted material absorbs water. 2 This is incorrect. Hydroscopic dilators for cervical ripening are not used specifically for women who have a history of cesarean childbirth. Pharmacological ripening can be achieved through the use of Cervidil. Misoprostol is avoided with a patient with a history of cesarean due to the risk of uterine rupture. 3 This is correct. Currently, hygroscopic dilators are used primarily during pregnancy termination rather than for cervical ripening in term pregnancies. Pregnancy termination may be necessary due to fetal demise. 4 This is incorrect. Being treated for active herpes is a contraindication for any method of cervical ripening. The patient with active herpes will require a cesarean delivery.

The nurse is providing care to a patient who is diagnosed with dystocia related to hypertonic uterine dysfunction. Which medical intervention does the nurse implement for this patient? 1. Explain to the family that the patient needs rest before labor continues. 2. Assist the patient to relax by providing back and neck massage. 3. Administer morphine to decrease contractions and promote uterine rest. 4. Discuss how the patient's fear is interfering with the progression of labor.

ANS: 3 Chapter: Chapter 10 High-Risk Labor and Birth Chapter Learning Objective: 3. Identify potential complications of dystocia in labor and related nursing and medical care. Page: 302 Heading: Dystocia > Uterine Dystocia > Hypertonic Uterine Dysfunction Integrated Processes: Nursing Process Client Need: Physiological Integrity: Reduction of Risk Potential Cognitive Level: Analysis [Analyzing] Concept: Ante/Intra/Post-partum Difficulty: Moderate Feedback 1 This is incorrect. Explaining to the family that the patient needs rest is applicable; however, this action is not considered a medical intervention. 2 This is incorrect. A back and neck massage may help the patient to relax; however, this action is not considered a medical intervention. The patient may also benefit from a warm shower or tub bath, a quiet environment, and a long period of sleep. 3 This is correct. The medical intervention of administering a pain medication, such as morphine, will decrease uterine contractions and allow the uterus to rest. 4 This is incorrect. In this scenario, the patient's dystocia is not related to fear. Hypertonic uterine dysfunction is uncoordinated uterine activity. Contractions are frequent and painful but ineffective in promoting dilation and effacement.

The nurse is monitoring a patient who has been in prolonged labor. Which assessment finding will result in the nurse notifying the health care provider about the development of an emergent situation requiring a cesarean delivery? 1. Maternal blood pressure indicative of hypotension 2. Maternal exhaustion from prolonged uterine activity 3. Recognition of a Category II fetal heart rate pattern 4. Increased maternal temperature related to infection

ANS: 3 Chapter: Chapter 11 Intrapartum and Postpartum Care of Cesarean Birth Families Chapter Learning Objective: 1. Identify factors that place a woman at risk for cesarean birth. Page: 350 Heading: Perioperative Care > Unscheduled Cesarean Birth > Nursing Actions Integrated Processes: Nursing Process Client Need: Physiological Integrity: Reduction of Risk Potential Cognitive Level: Analysis [Analyzing] Concept: Ante/Intra/Post-partum Difficulty: Moderate Feedback 1 This is incorrect. After a prolonged labor, the maternal blood pressure may be low (hypotension) due to dehydration. This finding alone does not warrant the nurse notifying the health care provider of a developing emergent situation. The nurse will initiate fluid replacement protocol. 2 This is incorrect. Maternal exhaustion related to a prolonged labor is not unexpected. However, this finding alone does not warrant the nurse notifying the health care provider of a developing emergent situation. 3 This is correct. The nurse will notify the health care provider about recognition of a Category II fetal heart rate pattern, which is an indication of fetal intolerance of labor. 4 This is incorrect. The increase in temperature for the patient in prolonged labor may be related to dehydration, and not infection. This finding alone does not warrant the nurse notifying the health care provider of a developing emergent situation. The nurse will initiate fluid replacement protocol.

A patient is being prepared for an unplanned cesarean section. Which pre-procedure information is most important for the nurse to report before the administration of regional anesthesia? 1. Hypovolemia corrected with IV fluid administration 2. Inability of the patient to sit on the bedside and flex forward 3. Laboratory value indicating a low platelet count 4. History of patient experiencing headaches after a spinal

ANS: 3 Chapter: Chapter 11 Intrapartum and Postpartum Care of Cesarean Birth Families Chapter Learning Objective: 3. Describe the intraoperative nursing care and medical and anesthesia management for cesarean births. Page: 351 Heading: Intraoperative Care > Anesthesia Management Integrated Processes: Nursing Process Client Need: Physiological Integrity: Reduction of Risk Potential Cognitive Level: Analysis [Analyzing] Concept: Ante/Intra/Post-partum Difficulty: Difficult Feedback 1 This is incorrect. The nurse will review laboratory results in order to report the correction of hypovolemia through IV fluid administration. This information is important, but other information is more important. 2 This is incorrect. The patient's inability to sit on the side of the bed and flex forward is important; however, the anesthesia can still be administered with the patient in a side-lying position. This is not the most important information to be reported. 3 This is correct. The most important information to be reported by the nurse is the laboratory value that indicates a low platelet count. It is the most common contraindication, especially with women who have preeclampsia and/or HELLP (hemolysis, elevated liver enzymes, and low platelets) syndrome. 4 This is incorrect. The fact that the patient experienced headaches after a spinal is information most likely ascertained by the anesthesiologist during pre-anesthesia assessment. Interventions will be initiated to help avoid the same manifestation.

The nurse is providing postpartum care to a patient 24 hours after a vaginal delivery. Which action does the nurse perform prior to assessing the patient's uterus? 1. Place the patient on the left side. 2. Assess the passage of lochia. 3. Ask the patient to void. 4. Administer a dose of oxytocin.

ANS: 3 Chapter: Chapter 12 Postpartum Physiological Assessments and Nursing Care Chapter Learning Objective: 1. Describe the physiological changes that occur during the postpartum period. Page: 368 Heading: The Reproductive System > Uterus > Nursing Actions Integrated Processes: Nursing Process Client Need: Physiological Integrity: Physiological Adaptation Cognitive Level: Application [Applying] Concept: Ante/Intra/Post-partum Difficulty: Moderate Feedback 1 This is incorrect. The patient is positioned in a flat supine position. Side-lying will interfere with the assessment of the uterus. 2 This is incorrect. The passage of lochia is assessed as the nurse is palpating the uterus. 3 This is correct. The nurse needs to have the patient void prior to palpating the uterus in order to accurately assess uterine placement and tone. An overdistended bladder can result in uterine displacement and atony. 4 This is incorrect. Oxytocin is administered as prescribed if the uterus is boggy and uncontracted. The drug is not administered prior to uterus palpation.

The nurse in a postpartum unit frequently teaches patients regarding breast care. Which teaching is most helpful to the breastfeeding patient? 1. Run warm water over breasts while in the shower. 2. Wear a supportive bra for 24 hours a day. 3. Express milk by a breast pump or manually. 4. Take analgesics for breast pain management.

ANS: 3 Chapter: Chapter 12 Postpartum Physiological Assessments and Nursing Care Chapter Learning Objective: 3. Describe safe and effective nursing care during the postpartum period. Page: 374 Heading: The Reproductive System > Breasts > Patient Education Integrated Processes: Nursing Process Client Need: Physiological Integrity: Physiological Adaptation Cognitive Level: Analysis [Analyzing] Concept: Ante/Intra/Post-partum Difficulty: Moderate Feedback 1 This is incorrect. Running warm water over the breasts in the shower will stimulate milk production, which should be avoided by the patient who is not breastfeeding. However, the breastfeeding patient can achieve the same stimulation with warm compresses and breast massage. 2 This is incorrect. Wearing a supportive bra 24 hours a day is helpful to both the breastfeeding patient and the non-breastfeeding patient. The breastfeeding patient may wear bras until breastfeeding is discontinued; the non-breastfeeding patient until the breasts soften. 3 This is correct. Expressing milk with a breast pump or manually is specifically helpful to the breastfeeding patient. The process can help relieve breast engorgement, maintain a milk supply for the neonate/infant who cannot suckle at the breast, and/or provide breast milk for the neonate/infant who is separated from the mother. 4 This is incorrect. Managing breast pain with the use of analgesics is appropriate for both the breastfeeding and non-breastfeeding patient.

In an attempt to improve the effectiveness of postpartum teaching, the nurse uses the AWHONN acronym POST BIRTH. Which teaching points require the patient to call for 911 assistance? Select all that apply. 1. Bleeding that soaks a pad per hour 2. A bad headache with vision changes 3. Thoughts of hurting self or baby 4. Signs an incision is not healing 5. A red, swollen leg painful to touch

ANS: 3 Chapter: Chapter 12 Postpartum Physiological Assessments and Nursing Care Chapter Learning Objective: 4. Describe the critical elements of discharge teaching. Page: 382 Heading: Discharge Teaching Integrated Processes: Nursing Process Client Need: Physiological Integrity: Reduction of Risk Potential Cognitive Level: Analysis [Analyzing} Concept: Ante/Intra/Post-partum Difficulty: Difficult Feedback 1 This is incorrect. The AWHONN acronym states the health care provider is called if bleeding that soaks one pad an hour occurs. 2 This is incorrect. The AWHONN acronym states the health care provider is called if the mother experiences a bad headache with vision changes that does not respond to headache medication. 3 This is correct. The AWHONN acronym states the mother should call 911 immediately if she has thoughts of self-harm or harm to her baby. This can be indicative of severe postpartum depression or psychosis. 4 This is incorrect. The AWHONN acronym states the health care provider is called about signs indicative of an incision not healing. This is true of a perineal incision as well as one from a cesarean delivery. 5 This is incorrect. The AWHONN acronym states the health care provider is called if the mother notices a red or swollen leg that is painful to the touch; this is indicative of a blood clot in a lower extremity.

The nurse is discussing contraception with a couple before discharge following the birth of a first child. The couple are uncertain about the method but are certain about avoiding pregnancy for at least 2 years. Which method does the nurse recommend? 1. Emergency contraceptives 2. Oral estrogen/progesterone pill 3. Depo-Provera 4. Natural family planning

ANS: 3 Chapter: Chapter 12 Postpartum Physiological Assessments and Nursing Care Chapter Learning Objective: 4. Describe the critical elements of discharge teaching. Page: 383 Heading: Table 12-3: Methods of Contraception Integrated Processes: Nursing Process Client Need: Physiological Integrity: Basic Care and Comfort Cognitive Level: Application [Applying] Concept: Ante/Intra/Post-partum Difficulty: Difficult Feedback 1 This is incorrect. The nurse does not suggest emergency contraceptives for this couple. The failure rate is only 9%, but it is not recommended as a regular form of birth control. 2 This is incorrect. The oral estrogen/progesterone combination contraceptive has many side effects (some are serious). There also some non-contraceptive advantages such as reduced risk for endometrial and ovarian cancer, benign breast disease, anemia, and may improve acne. Failure rate is 9%. This method may or may not be recommended depending on the patient's health and identified risk factors. 3 This is correct. Depo-Provera is a likely suggestion by the nurse. This method has a 3% failure rate, is injectable every 3 months, and has few non-life-threatening side effects. It may result in delayed fertility. 4 This is incorrect. Natural family planning has a 24% failure rate and requires a regular menstrual cycle and the knowledge/willingness to frequently monitor body functions such as temperature and vaginal mucus production and consistency.

The postpartum nurse is planning a home visit to a mother who delivered her baby 1 week ago. Which finding indicates to the nurse a possible problem with mother-infant bonding? 1. The mother is pleased to have the nurse visit her home and baby. 2. The baby's grandmother is present and involved with mother/baby care. 3. The mother focuses the visit on her physical recovery and concerns. 4. The baby's father is on "paternity leave" and involved with the baby.

ANS: 3 Chapter: Chapter 13 Transition to Parenthood Chapter Learning Objective: 3. Discuss bonding and attachment. Page: 401 Heading: Table 13-1: Maternal Phases Integrated Processes: Nursing Process Client Need: Psychological Integrity Cognitive Level: Analysis [Analyzing] Concept: Family Dynamics Difficulty: Moderate Feedback 1 This is incorrect. The mother being pleased to have the nurse visit her home and baby does not indicate a problem with mother-infant bonding. 2 This is incorrect. At 1 week, it is not unusual for the baby's grandmother to be present and helping with mother and infant care. This finding alone does not indicate a problem with mother-infant bonding. 3 This is correct. After the first 48 hours postpartum, the mother moves into the "taking hold" phase when the mother's focus moves from self to the infant. When the mother focuses the nurse's attention on the mother's physical recovery and concerns, the nurse needs to assess for problems with mother-infant bonding. 4 This is incorrect. It is not unusual for fathers to take some time to be involved during the postpartum period. This finding alone does not indicate a problem with mother-infant bonding.

The nurse is providing care for a new mother during a follow-up visit 6 weeks after a vaginal delivery. The mother begins to cry and reports difficulty with eating and sleeping. The nurse identifies postpartum blues and cites which reason as the most likely cause? 1. Fatigue related to a "fussy" baby 2. Frustration over physical appearance 3. Changes in hormonal levels 4. Stress related to new mother role

ANS: 3 Chapter: Chapter 13 Transition to Parenthood Chapter Learning Objective: 5. Describe nursing actions that support couples during their transition to parenthood. Page: 412 Heading: Postpartum Blues Integrated Processes: Nursing Process Client Need: Psychosocial Integrity Cognitive Level: Application [Applying] Concept: Family Dynamics Difficulty: Moderate Feedback 1 This is incorrect. Fatigue is a possible cause of postpartum blues, but the scenario does not validate the presence of a "fussy" baby. 2 This is incorrect. Six weeks after childbirth, some mothers become frustrated about their physical appearance; however, changes in hormone levels are still the most likely cause of postpartum blues. 3 This is correct. Although the other options can contribute to postpartum blues, the most likely cause is changes in hormone levels. 4 This is incorrect. Stress and/or anxiety over the new role of being a mother can be a cause of postpartum blues. In this scenario, the most likely cause is changes in hormone levels.

The nurse in a labor and delivery department carefully assesses postpartum patients for signs of complications related to hemorrhage. Which factor makes it most difficult to identify the risk of hemorrhage through vital sign evaluation? 1. Blood pressure may be elevated from prenatal conditions. 2. Respirations are increased due to activity of labor. 3. Changes in blood pressure may not be an immediate sign. 4. Heart rate may increase with intensity of labor.

ANS: 3 Chapter: Chapter 14 High-Risk Postpartum Nursing Care Chapter Learning Objective: 1. Describe the primary causes of postpartum hemorrhage and the related nursing actions and medical care. Page: 419 Heading: Hemorrhage Integrated Processes: Nursing Process Client Need: Physiological Integrity: Reduction of Risk Potential Cognitive Level: Analysis [Analyzing] Concept: Ante/Intra/Post-partum Difficulty: Difficult Feedback 1 This is incorrect. If patients have a prenatal condition that causes an increase in blood pressure, the postpartum blood pressure will be compared with the patient's baseline readings. 2 This is incorrect. Respiration rates can be elevated during labor, but the rate should be within normal limits for the postpartum patient. 3 This is correct. Changes in blood pressure may not be an immediate sign of hemorrhage in a postpartum patient. OB patients may not show the same signs and symptoms observed in nonpregnant patients during hemorrhage until approximately one-third of the woman's entire blood volume is lost. The postpartum patient has an increased blood volume from pregnancy, which delays vital sign indications. A decrease in BP is a late sign of postpartum hemorrhage. 4 This is incorrect. Pulse rates can be elevated during labor, but the rate should be within normal limits for the postpartum patient. Tachycardia is a late sign of primary postpartum hemorrhage.

The nurse continues to monitor a patient after a vaginal delivery with an estimated blood loss of 1,000 mL. Which assessment finding does the nurse recognize as requiring Stage 3 hemorrhage protocol? 1. Increased patient restlessness. 2. Manifestations of severe pain. 3. Development of abnormal vital signs. 4. Patient requests water for extreme thirst.

ANS: 3 Chapter: Chapter 14 High-Risk Postpartum Nursing Care Chapter Learning Objective: 1. Describe the primary causes of postpartum hemorrhage and the related nursing actions and medical care. Page: 421 Heading: Box 14-2: Staging Obstetric Hemorrhage: Elements of Common Protocols Integrated Processes: Nursing Process Client Need: Physiological Integrity: Reduction of Risk Potential Cognitive Level: Analysis [Analyzing] Concept: Ante/Intra/Post-partum Difficulty: Difficult Feedback 1 This is incorrect. Increased restlessness can be a sign of hypovolemia; however, this is not an indicator used in the protocol for staging postpartum hemorrhage. 2 This is incorrect. Severe pain may indicate the source of hemorrhage, but it is not an indicator used in the protocol for staging postpartum hemorrhage. 3 This is correct. Vital signs will remain normal during Stages 1 and 2. The evidence of abnormal vital signs is one indicator of Stage 3 hemorrhage. Other Stage 3 indicators include continued bleeding, more than 2 units red blood cells (RBCs) given, patient at risk for occult bleeding/coagulopathy, abnormal laboratory values, or oliguria. 4 This is incorrect. Patients will experience extreme thirst as a result of hemorrhage; however, this is not used in the protocol for staging postpartum hemorrhage.

The nurse is assisting the primary care provider with the third stage of a vaginal delivery. The patient is multiparous, experienced a precipitous birth, and has a history of hypertension. Which medical prescription does the nurse anticipate for this patient? 1. Methylergonovine 2. Fresh frozen plasma 3. Carboprost-tromethamine 4. Magnesium sulfate

ANS: 3 Chapter: Chapter 14 High-Risk Postpartum Nursing Care Chapter Learning Objective: 1. Describe the primary causes of postpartum hemorrhage and the related nursing actions and medical care. Page: 423 Heading: Hemorrhage > Uterine Atony > Nursing Actions Integrated Processes: Nursing Process Client Need: Physiological Integrity: Reduction of Risk Potential Cognitive Level: Analysis [Analyzing] Concept: Ante/Intra/Post-partum Difficulty: Difficult Feedback 1 This is incorrect. Methylergonovine directly stimulates smooth and vascular smooth muscles causing sustaining uterine contractions. This drug is used to prevent or treat PP hemorrhage/uterine atony/subinvolution. However, it is contraindicated in hypertensive patients. 2 This is incorrect. Fresh frozen plasma is administered when the patient is experiencing postpartum hemorrhage. This client is at risk but does not have the manifestations of PP. 3 This is correct. Carboprost-tromethamine is classified as a prostaglandin and is prescribed to maintain contraction of the uterine muscles. It is injected into a large muscle or directly into the uterine muscle. The nurse will expect this prescription because the patient has multiple risks for PP. 4 This is incorrect. Magnesium sulfate is administered to prevent uterine contractions and is contraindicated for this patient and this stage of labor.

The nurse is providing care for a patient who is 8 hours postpartum after a vaginal delivery. The patient reports severe perineal pain unaffected by pain medication. The nurse notices a 4 cm area of discoloration on the labia that is tender to the touch. Which action does the nurse take? 1. Continue to apply ice to the area for 24 hours. 2. Monitor vital signs and report any abnormal readings. 3. Contact the primary care provider for further evaluation. 4. Relieve pressure by placing patient in a side-lying position.

ANS: 3 Chapter: Chapter 14 High-Risk Postpartum Nursing Care Chapter Learning Objective: 1. Describe the primary causes of postpartum hemorrhage and the related nursing actions and medical care. Page: 425 Heading: Hematomas > Nursing Action Integrated Processes: Nursing Process Client Need: Physiological Integrity: Reduction of Risk Potential Cognitive Level: Application [Applying] Concept: Ante/Intra/Post-partum Difficulty: Moderate Feedback 1 This is incorrect. Ice can be applied to the perineal area for 24 hours after delivery to help avoid swelling; however, there are other nursing actions relative to the presence of a hematoma. 2 This is incorrect. The presence of a hematoma is indicative of hemorrhage into soft tissue. Reporting abnormal vital signs is a late sign of hemorrhage; the nurse can take other proactive actions to prevent complications. 3 This is correct. The primary care provider needs to be contacted about assessment findings; the hematoma may need to be evaluated further and/or evacuation of the hematoma performed. 4 This is incorrect. The patient may or may not be more comfortable in a side-lying position; however, this action does not address the issue of hemorrhage.

The nurse is providing care for a patient who is 1 day postpartum and exhibiting symptoms of postpartum psychosis. Which medical management does the nurse expect for this patient? 1. Prescriptions for antidepressant/antipsychotic drugs 2. Discharge to home with 24-hour observation in place 3. Immediate hospitalization in a psychiatric unit 4. Prescribed neonate visits during in-patient treatment

ANS: 3 Chapter: Chapter 14 High-Risk Postpartum Nursing Care Chapter Learning Objective: 3. Describe the primary postpartum psychological complications and the related nursing actions and medical care. Page: 436 Heading: Postpartum Psychological Complications > Postpartum Psychosis Integrated Processes: Nursing Process Client Need: Psychosocial Integrity Cognitive Level: Analysis [Analyzing] Concept: Ante/Intra/Post-partum Difficulty: Moderate Feedback 1 This is incorrect. The patient's current health care provider may or may not prescribe antidepressant/antipsychotic drugs; the medication regimen may be deferred to the psychiatric care provider. 2 This is incorrect. The patient is at risk for self-harm or harm to others, including the baby. The family is not responsible for managing the intensity of the manifestations or the safety issues in the home. 3 This is correct. The nurse expects the health care provider to immediately hospitalize the patient in a psychiatric unit. Maintaining the patient in the postpartum unit delays necessary psychiatric treatment. 4 This is incorrect. It is highly unlikely that neonatal visits will be initially prescribed while the patient is on the psychiatric unit. Neonatal care will be managed by other family members; the focus for the patient is psychiatric care.

The nurse on a postpartum unit observes a patient who delivered 2 days ago. The nurse notices extreme agitation and depressed mood. The patient states, "I think that my baby is deformed inside and we have to fix him." Which risk factor is most strongly related to possible postpartum psychosis (PPP)? 1. Separation from the baby's father 2. Personal history of bipolar disorder 3. Prolonged labor resulting in cesarean 4. Loss of first child from a heart defect

ANS: 3 Chapter: Chapter 14 High-Risk Postpartum Nursing Care Chapter Learning Objective: 3. Describe the primary postpartum psychological complications and the related nursing actions and medical care. Page: 436 Heading: Postpartum Psychological Complications > Postpartum Psychosis Integrated Processes: Nursing Process Client Need: Psychosocial Integrity Cognitive Level: Application [Applying] Concept: Ante/Intra/Post-partum Difficulty: Moderate Feedback 1 This is incorrect. Separation from the baby's father can be a contributing factor for postpartum depression. 2 This is correct. A patient history of either bipolar disorder or affective disorder can result in postpartum psychosis (PPP). 3 This is incorrect. Prolonged labor resulting in a cesarean delivery is not a risk factor for PPP; the mother is more likely to experience postpartum depression. 4 This is incorrect. The loss of a previous child from a heart defect can cause escalated concern about the well-being for the neonate. However, the other manifestations are indicative of PPP.

The labor and delivery nurse understands that some neonates spontaneously take a breath once the head and chest is delivered. Which understanding does the nurse have for the neonate that requires chemical stimuli to breathe? 1. Oxygen is applied immediately to start respirations. 2. Carbon dioxide is administered in small doses. 3. Mild hypoxia and decreased pH stimulates the brain. 4. Suctioning is used to stimulate breathing efforts.

ANS: 3 Chapter: Chapter 15 Physiological and Behavioral Responses of the Neonate Chapter Learning Objective: 1. Identify the changes that occur during the transition from intrauterine to extrauterine life and the related nursing actions. Page: 444 Heading: Transition to Extrauterine Life > Respiratory System Integrated Processes: Nursing Process Client Need: Physiological Integrity: Physiological Adaptation Cognitive Level: Analysis [Analyzing] Concept: Ante/Intra/Post-partum Difficulty: Moderate Feedback 1 This is incorrect. Oxygen is not used to stimulate breathing in the neonate who does not begin respirations from mechanical stimulation. 2 This is incorrect. Carbon dioxide is never administered to a neonate. The natural buildup of carbon dioxide is sufficient to stimulate the respiratory center of the medulla. 3 This is correct. The essence of chemical stimulation to initiate neonate breathing is the mild hypoxia that occurs when placental blood flow stops. Hypoxia causes an increase in carbon dioxide and decrease in blood pH, a chemical reaction that stimulates the respiratory center in the medulla. 4 This is incorrect. Suctioning is a mechanical process which occurs when the head and chest are delivered. Some neonates will begin breathing after both mechanical and chemical stimulation.

The postpartum nurse notices that a new mother has her neonate unwrapped and undressed "to check out the baby." For which reason does the nurse conclude the neonate is at risk for cold stress? 1. The neonate has an increased metabolic rate. 2. The neonate's respiratory rate has dropped. 3. The neonate is moving extremities about. 4. The neonate's skin is cool and clammy.

ANS: 3 Chapter: Chapter 15 Physiological and Behavioral Responses of the Neonate Chapter Learning Objective: 1. Identify the changes that occur during the transition from intrauterine to extrauterine life and the related nursing actions. Page: 445 Heading: Transition to Extrauterine Life > The Thermoregulatory System Integrated Processes: Nursing Process Client Need: Physiological Integrity: Physiological Adaptation Cognitive Level: Analysis [Analyzing] Concept: Ante/Intra/Post-partum Difficulty: Moderate Feedback 1 This is incorrect. If the neonate exhibits a thermoregulation problem, the nurse knows an increase in metabolism occurs; however, this is not a visible manifestation. 2 This is incorrect. Typically, the neonate's respiratory rate will increase with thermoregulatory issues. 3 This is correct. A visible manifestation that indicates the neonate may be approaching cold stress is movement of the extremities in an effort to produce body heat. 4 This is incorrect. During cold stress, the neonate's skin will feel cool, but not clammy.

The nurse is assisting a newborn's primary care provider with the performance of a circumcision. Which intervention is used to manage the neonate's pain? 1. A Velcro tourniquet is loosely wrapped around the penis. 2. The neonate is breastfed first to promote a sense of calmness. 3. A sucrose-dipped pacifier is offered during the nerve block. 4. The foreskin is numbed with ice before the nerve block.

ANS: 3 Chapter: Chapter 15 Physiological and Behavioral Responses of the Neonate Chapter Learning Objective: 4. Discuss methods used in neonatal pain management. Pages: 473 Heading: Therapeutic and Surgical Procedures > Circumcision Integrated Processes: Nursing Process Client Need: Physiological Integrity: Physiological Adaptation Cognitive Level: Application [Applying] Concept: Ante/Intra/Post-partum Difficulty: Moderate Feedback 1 This is incorrect. There is no reason to wrap a Velcro tourniquet around the penis, even loosely. Obstruction of blood flow can result in complications. 2 This is incorrect. The neonate does not eat for 2 to 3 hours before the procedure to decrease the risk of vomiting and aspiration during the procedure. 3 This is correct. A sucrose-dipped pacifier is offered during the nerve block as a procedure for pain management. The sucrose entices the neonate to suck, which is a comforting activity. 4 This is incorrect. An ice pack is not used to numb the foreskin before the nerve block. The nerve block is not administered into or around the foreskin.

Postnatal nurses expressed concern about neonatal pain management during painful interventions. Using evidence-based practice from research performed by Thakkar, Arora, Das, Javadekar, and Panigrahi (2016), which method of pain control will be used for heel sticks? 1. An anesthetic gel will be applied 20 minutes before the stick. 2. The stick will be administered while the neonate is breastfeeding. 3. A combination of stimulated sucking and receiving sucrose orally. 4. The neonate is stuck while the mother and neonate are en face.

ANS: 3 Chapter: Chapter 15 Physiological and Behavioral Responses of the Neonate Chapter Learning Objective: 8. Describe the most common therapeutic and surgical procedures used for neonates and the related nursing care. Page: 473 Heading: Therapeutic and Surgical Procedures > Surgical Procedure Integrated Processes: Nursing Process Client Need: Physiological Integrity: Physiological Adaptation Cognitive Level: Application [Applying] Concept: Ante/Intra/Post-partum Difficulty: Moderate Feedback 1 This is incorrect. The study did not include the use of an anesthetic gel. 2 This is incorrect. The study did not include sticking the neonate during breastfeeding. 3 This is correct. The study concluded that pain was decreased the most with a combined intervention of having sterile gauze held gently in the neonate's mouth and the palate tickled to stimulate sucking, and administering 30% sucrose solution PO by sterile syringe. 4 This is incorrect. The study did not include sticking the neonate while en face with the neonate's mother.

A patient in the first stage of pregnancy is discussing the options for feeding her infant, and asks the nurse, "Which is the most important reason I should consider breastfeeding my baby?" Which is the most significant reason the nurse presents? 1. Human milk proteins are easier to digest than protein in prepared formula. 2. The amount of cholesterol in human milk is essential for the baby. 3. Human milk contains multiple antibodies, enzymes, and immune factors. 4. Vitamins and minerals are transferred to human milk from the mother.

ANS: 3 Chapter: Chapter 16 Discharge Planning and Teaching Chapter Learning Objective: 2. Discuss the nutritional needs of newborns and infants. Page: 484 Heading: Newborn Nutrition and Feeding > Breastfeeding > Composition of Human Milk Integrated Processes: Nursing Process Client Need: Physiological Integrity: Physiological Adaptation Cognitive Level: Analysis [Analyzing] Concept: Ante/Intra/Post-partum Difficulty: Difficult Feedback 1 This is incorrect. Human milk proteins are easier to digest than protein in prepared formula; however, this is not the most significant reason the nurse recommends breastfeeding. 2 This is incorrect. Cholesterol is high in human milk and is essential for brain development; however, this is not the most significant reason the nurse recommends breastfeeding. 3 This is correct. Human milk contains multiple antibodies, enzymes, and immune factors that help protect the infant from common infections. It also stimulates the growth of healthy bacteria in the intestinal system, which inhibits growth of bacteria that can cause diseases. The factors are not found in formula, and this is the most significant reason for the nurse to recommend breastfeeding. 4 This is incorrect. Vitamins and minerals are transferred to the human milk from the maternal plasma; however, this is not the most significant reason for the nurse to recommend breastfeeding.

Which information is important for the nurse to provide to mothers of infants of 3 months of age regardless of the method of infant feeding? 1. Why breastfeeding delays the need for solid foods 2. When and what order solid foods are introduced 3. When growth spurts and dietary increases are expected 4. Why the babies are most likely to prefer food over milk

ANS: 3 Chapter: Chapter 16 Discharge Planning and Teaching Chapter Learning Objective: 2. Discuss the nutritional needs of newborns and infants. Page: 492 Heading: Newborn Nutrition and Feeding > Nutritional Needs > Birth to 4 Months Integrated Processes: Nursing Process Client Need: Physiological Integrity: Basic Care and Comfort Cognitive Level: Analysis [Analyzing] Concept: Ante/Intra/Post-partum Difficulty: Moderate Feedback 1 This is incorrect. At the age of 3 months, the nutritional requirements for infants are ideally met by breast milk. Iron-fortified infant formula is substituted when the woman is not breastfeeding. 2 This is incorrect. The order and types of solid food introduction do not need to be taught to the mothers of 3-month-old infants, regardless of the current methods of feeding. Introduction of semisolid foods is determined by the physician or nurse practitioner in collaboration with parents. The AAP and WHO guidelines recommend waiting to start solids until 6 months of age to reduce allergy risks. 3 This is correct. Mothers need to be aware of probable growth spurts regardless of the method of feeding. Infants experience growth spurts at 3 to 5 days, 1 week, 6 weeks, 3 months, and 6 months and require more frequent feedings during these time periods. 4 This is incorrect. Infants at the age of 3 months are not likely to prefer food over milk. Before 4 to 6 months, the sucking reflex forces semisolid food out of the mouth instead of moving it to the back of the mouth.

The nurse is collecting information from a new mother who is bottle-feeding her infant. Which comment, if made by the mother, requires the nurse to provide patient teaching? 1. "I wish that I had tried breastfeeding because formula is expensive." 2. "At least I get a break every evening when my spouse feeds the baby." 3. "Sometimes I will add a little water to the formula if I am running low." 4. "I get frustrated if the last bottle is fed to the baby late at night."

ANS: 3 Chapter: Chapter 16 Discharge Planning and Teaching Chapter Learning Objective: 7. Develop a teaching plan for formula feeding. Page: 491 Heading: Newborn Nutrition and Feeding > Bottle Feeding Integrated Processes: Nursing Process Client Need: Physiological Integrity: Reduction of Risk Potential Cognitive Level: Analysis [Analyzing] Concept: Ante/Intra/Post-partum Difficulty: Difficult Feedback 1 This is incorrect. When the mother expresses a reconsideration about breastfeeding, the nurse can only plan teaching if the next pregnancy occurs. 2 This is incorrect. When the mother expresses positive feelings about the ability of her spouse to feed the baby, there is no need for teaching. 3 This is correct. If the mother states a practice of diluting the baby's formula if her supply is low, the nurse needs to provide teaching. Prolonged over-dilution of formulas can cause water intoxication, as well as decrease the caloric intake by the baby. 4 This is incorrect. The nurse may make suggestions about planning specific times for the preparation of formula. However, this is not a statement that requires teaching.

The nurse in NICU is assessing a neonate delivered at 32 weeks gestation. Which pathophysiological manifestation is the nurse's greatest concern? 1. Absent or weak reflexes 2. Presence of a heart murmur 3. Apnea 20 seconds or longer 4. Low hemoglobin lab level

ANS: 3 Chapter: Chapter 17 High-Risk Neonatal Nursing Care Chapter Learning Objective: 1. Describe the physiology and pathophysiology associated with selected complications of the neonatal period. Page: 507 Heading: Preterm Neonates > Assessment Findings Integrated Processes: Nursing Process Client Need: Physiological Integrity: Physiological Adaptation Cognitive Level: Analysis [Analyzing] Concept: Ante/Intra/Post-partum Difficulty: Moderate Feedback 1 This is incorrect. Absent or weak reflexes are expected in a premature neonate because of neurologic immaturity. 2 This is incorrect. In a premature neonate, the presence of a heart murmur is related to a patent ductus arteriosus. This finding is indicative of circulation status; however, it is still not the nurse's greatest concern. 3 This is correct. Apnea for 20 seconds or longer is the nurse's greatest concern. Even though this is expected in premature neonates, the nurse will still focus on ABCs. 4 This is incorrect. Anemia is not unexpected in a premature neonate. While this finding is a concern because of a reduction in the blood's ability to carry sufficient oxygen, the greatest concern is still breathing issues.

The nurse is providing care for a premature neonate in the NICU nursery. The neonate is diagnosed with bronchopulmonary dysplasia (BPD) and patent ductus arteriosus (PDA). Which specific intervention does the nurse expect for this neonate? 1. Monitor of hemoglobin and hematocrit levels. 2. Obtain blood glucose levels. 3. Maintain fluid restrictions. 4. Administer enteral feedings.

ANS: 3 Chapter: Chapter 17 High-Risk Neonatal Nursing Care Chapter Learning Objective: 2. Identify critical elements of assessment and nursing care of the high-risk neonate. Page: 509 Heading: Preterm Neonates > Nursing Actions Integrated Processes: Nursing Process Client Need: Physiological Integrity: Reduction of Risk Potential Cognitive Level: Analysis [Analyzing] Concept: Ante/Intra/Post-partum Difficulty: Difficult Feedback 1 This is incorrect. Monitoring hemoglobin and hematocrit levels is an expected intervention for premature neonates. 2 This is incorrect. Obtaining blood glucose levels is an expected intervention for premature neonates. 3 This is correct. Maintaining fluid restrictions is specific for this neonate due to bronchopulmonary dysplasia (BPD) and patent ductus arteriosus (PDA). Fluid restrictions are appropriate for premature neonates with BPD, PDA, or other complications that can lead to pulmonary edema. 4 This is incorrect. Administering enteral feedings is an expected intervention for premature neonates.

The nurse is preparing for the discharge of a premature neonate to home with the parents. The nurse explains the neonate must be able to pass the infant car seat challenge before discharge. For which reason would the neonate be considered unsafe in a car seat? 1. Inability to remain at a 45-degree angle for a period of 1 hour 2. Reluctance of parents to use the car seat because of the small size of the baby 3. Inability to maintain adequate oxygenation, heart rate, and respiratory rate during trial 4. Inability to continue prescribed oxygen therapy for the neonate while in a car seat

ANS: 3 Chapter: Chapter 17 High-Risk Neonatal Nursing Care Chapter Learning Objective: 3. Develop a discharge plan for high-risk neonates. Page: 543 Heading: Discharge Planning Integrated Processes: Nursing Process Client Need: Physiological Integrity: Reduction of Risk Potential Cognitive Level: Application [Applying] Concept: Ante/Intra/Post-partum Difficulty: Moderate Feedback 1 This is incorrect. The infant car seat challenge actually requires the neonate to be secured snugly in an appropriate-sized car seat at a 45-degree angle for a specified amount of time, which may be related to the distance between the neonate's home, or the distance to medical facilities. 2 This is incorrect. The parents need to be advised that car seats, or inserts, are available to accommodate the small size of a premature neonate. 3 This is correct. In order to pass the infant care seat challenge, the premature neonate must be able to maintain adequate oxygenation, heart rate, and respiratory rate during trial. 4 This is incorrect. If the premature neonate is prescribed oxygen therapy, the therapy can and must be maintained while the neonate is in a car seat.

The nurse is providing support to a mother whose newborn is diagnosed with a life-threatening defect. The mother states, "I just want to go home and never come back." Which reaction by the mother does the nurse recognize? 1. Guilty feelings by the mother 2. Delay of attachment process 3. Maternal emotional distancing 4. Disruption of family life

ANS: 3 Chapter: Chapter 17 High-Risk Neonatal Nursing Care Chapter Learning Objective: 4. Describe the loss and grief process experienced by parents whose infant has died. Page: 545 Heading: Psychosocial Needs of Parents with High-Risk Neonates Integrated Processes: Nursing Process Client Need: Psychosocial Integrity Cognitive Level: Comprehension [Understanding] Concept: Ante/Intra/Post-partum Difficulty: Moderate Feedback 1 This is incorrect. Guilty feelings by the mother, who may feel she did something wrong to cause her newborn to be ill, may be an expected reaction. However, the mother is not voicing feelings of guilt. 2 This is incorrect. Delay of attachment process is due to the separation of parent and newborn, which can place the newborn at risk for abuse and neglect. However, there is no information supporting a delay of attachment in this scenario. 3 This is correct. Emotional distancing of parents from their newborn is a protective mechanism related to fear of their child's death. The mother's comment reflects emotional distancing. 4 This is incorrect. Disruption of family life occurs because of parents needing to return to work, caring for other children, and at the same time wanting to spend time in the hospital with their newborn. There is no information supporting the disruption of family life in the scenario or in the mother's comment.

A patient who is at 41 weeks gestation is concerned when the primary care provider decides to induce labor. Which reason does the nurse explain as the most important need for this procedure? 1. Increasing size of the neonate 2. Ability to deliver vaginally 3. Risk for placental dysfunction 4. Likelihood of meconium aspiration

ANS: 3 Chapter: Chapter 17 High-Risk Neonatal Nursing Care Chapter Learning Objective: Describe the physiology and pathophysiology associated with selected complications of the neonatal period. Page: 519 Heading: Postmature Neonates > Complications Integrated Processes: Nursing Process Client Need: Physiological Integrity: Reduction of Risk Potential Cognitive Level: Analysis [Analyzing] Concept: Ante/Intra/Post-partum Difficulty: Moderate Feedback 1 This is incorrect. The postmature neonate may or may not exhibit an increase in size. The postmature neonate can have a thin, wasted appearance if placental insufficiency has caused the fetus to use its subcutaneous fat stores and glycemic stores. Increasing size is not the most important need for labor induction. 2 This is incorrect. Macrosomia may interfere with the mother's ability to vaginally deliver a postmature fetus, and the fetus is at risk for birth trauma. However, this is not the most important need for labor induction. 3 This is correct. With the postmature fetus, the greatest reason to induce labor is to minimize complications related to placental dysfunction. With postmaturity, placental function decreases, resulting in altered oxygenation and nutrient transport, which increases the risk for hypoxia and hypoglycemia at the onset of labor. This is the most important reason for labor induction. 4 This is incorrect. The risk for meconium aspiration may or may not occur as a result of postmaturity. This is not the most important reason for labor induction.

The nurse is providing education for disease prevention to the adult female patient. Which factor puts the patient at a higher risk for multisystem disease processes such as cardiac issues, gynecological issues, and cancers? 1. Consuming two glasses of wine a week 2. Smoking two packs of cigarettes a day 3. Having a body mass index of over 32 4. Having poor intake of calcium and vegetables

ANS: 3 Chapter: Chapter 18 Well Women's Health Chapter Learning Objective: 3. Describe how lifestyle factors such as diet, exercise, and cigarette smoking influence women's health. Page: 558 Heading: Health Promotion > Risk Reduction Integrated Processes: Teaching and Learning Client Need: Physiological Integrity: Reduction of Risk Potential Cognitive Level: Analysis [Analyzing] Concept: Promoting Health Difficulty: Moderate Feedback 1 This is incorrect. Although alcohol consumption may put the patient at risk for disease processes, they often do not include gynecological issues. Two glasses of wine a week is not indicative of alcohol abuse nor of binge drinking. 2 This is incorrect. Although smoking increases the risks for cardiac and pulmonary diseases, it does not increase the risk for gynecological issues. 3 This is correct. A BMI of over 30 indicates obesity, which increases the risks for many disease processes, such as cardiac, endocrine, musculoskeletal issues, and gynecological disorders. 4 This is incorrect. This lifestyle may increase the risk for cancer and cardiac issues and may in an indirect route increase the BMI of the patient.

During the nurse's assessment of a 44-year-old female patient in the family medicine clinic, the patient becomes tearful and states she may be pregnant, as she has some unintentional weight gain, mood swings, and irregular menstrual cycles. The urine hCG reveals that the patient is not pregnant. What is the possible reason for her symptoms at this time? 1. The patient has a false pregnancy. 2. The patient is menopausal. 3. The patient is perimenopausal. 4. The patient may be pregnant.

ANS: 3 Chapter: Chapter 18 Well Women's Health Chapter Learning Objective: 5. Discuss the physical and emotional changes related to perimenopause and menopause. Page: 562 Heading: Reproductive Changes Across the Life Span > Menopause Integrated Processes: Teaching and Learning Client Need: Physiological Integrity: Physiological Adaptation Cognitive Level: Application [Applying] Concept: Promoting Health Difficulty: Moderate Feedback 1 This is incorrect. There is no indication of a false pregnancy in the descriptor. 2 This is incorrect. The patient's age and symptoms are indicative of perimenopause, not menopause. 3 This is correct. The patient's age and symptoms are indicative of perimenopause. 4 This is incorrect. The patient's age, symptoms, and negative hCG are not indicative of pregnancy.

The nurse is providing care to a 75-year-old female patient diagnosed with osteoporosis. Which of the following would be the priority nursing diagnosis? 1. At risk for falls related to impaired balance 2. Knowledge deficit related to new medication regimen 3. Impaired physical mobility related to pain and skeletal changes 4. Ineffective health maintenance related to continued immobility

ANS: 3 Chapter: Chapter 18 Well Women's Health Chapter Learning Objective: 7. Develop a health promotion-teaching plan for an older adult based on the normal physiological changes related to aging. Page: 569 Heading: Table 18-4: Age-Related Physiological Changes and Health Promotion Integrated Processes: Teaching and Learning Client Need: Physiological Integrity: Reduction of Risk Potential Cognitive Level: Application [Applying] Concept: Promoting Health Difficulty: Difficult Feedback 1 This is incorrect. Although the patient is at risk for falls, this is not a priority 2 This is incorrect. Although the patient may be prescribed a medication to treat the osteoporosis, this is not a priority. 3 This is correct. The patient will experience skeletal changes, and often pain due to pathological fractures. In order to prevent further exacerbation of the disease process, the nurse will need to develop a plan of care that includes increasing physical mobility, especially weight-bearing exercises to increase bone density. 4 This is incorrect. Although the patient typically experiences immobility, this is not a priority, compared to the root cause of the issue, the disease process.

The nurse is providing preoperative education on the laparoscope-assisted vaginal hysterectomy. Which statement by the patient verifies understanding of the procedure? 1. "I will use a mild douche solution to keep the surgical area clean." 2. "I'm relieved that I won't have any visible scars on my abdomen." 3. "I understand that there will be some light vaginal bleeding for several days." 4. "I understand that I must remain on bedrest until cleared by the surgeon."

ANS: 3 Chapter: Chapter 19 Alterations in Women's Health Chapter Learning Objective: 3. Describe common alterations in women's health, including medical management and nursing actions. Page: 579 Heading: Hysterectomy > Postoperative Care for Abdominal Hysterectomy > Nursing Actions Integrated Processes: Communication and Documentation Client Need: Safe and Effective Care Environment: Safety and Infection Control Cognitive Level: Application [Applying] Concept: Patient-Centered Care Difficulty: Moderate Feedback 1 This is incorrect. Although there will be small abdominal incisions, anything introduced into the vagina can cause an infection. 2 This is incorrect. Although the incisions will be small, they may be visible. The laparoscopic procedure includes several small "stab-like" incisions in the abdomen to guide the surgical removal of the uterus. 3 This is correct. There will be some light bleeding for several days, and the patient should be instructed to notify the surgeon if bleeding increases. 4 This is incorrect. Walking is important to prevent postoperative complications, such as constipation and blood clots. Walking can facilitate the patient's gradual return to her presurgery activity level.

The nurse is assessing a 16-year-old sexually active patient in the family practice clinic. The patient's symptoms include breakthrough vaginal bleeding, abdominal pain, nausea, and fever. The health care provider obtains cultures of the cervical epithelial cells during a Pap smear and orders doxycycline 100 mg orally twice a day for 7 days for her and her partner. The nurse understands that this prescription is consistent with the treatment of which medical diagnosis? 1. Herpes 2. Condylomas 3. Chlamydia 4. Gonorrhea

ANS: 3 Chapter: Chapter 19 Alterations in Women's Health Chapter Learning Objective: 3. Describe common alterations in women's health, including medical management and nursing actions. Page: 586 Heading: Table 19-3: Sexually Transmitted Infections Integrated Processes: Nursing Process Client Need: Physiological Integrity: Pharmacological Therapies Cognitive Level: Application [Applying] Concept: Infection Difficulty: Difficult Feedback 1 This is incorrect. The signs and symptoms of genital herpes vary from no symptoms to lesions in the area of virus entry. Treatment for herpes includes antiviral mediations such as acyclovir. 2 This is incorrect. Genital warts/condylomas are caused by human papillomavirus. They are a painless warty growth in the vagina, vulva, perineum, or anal areas. Treatment for this disease is topical application of trichloroacetic acid, cryotherapy, or surgical removal. 3 This is correct. Chlamydia is the most common bacterial STI in the United States and the leasing cause of preventable infertility. This is diagnosed by cultures of cervical epithelial cells and is treated with antibiotics. The partner needs to be treated to prevent reinfection. 4 This is incorrect. The symptoms for this disease vary from being absent to painful sexual intercourse and bleeding. It is treated with a single intramuscular dose of ceftriaxone with azithromycin or doxycycline, and the partner should be treated to prevent reinfection.

The nurse is assessing a 64-year-old female patient. The patient states that she is able to reduce the risk of urinary tract infections (UTIs) by drinking a quart of cranberry juice a day. Which health condition, if present in this patient, contraindicates the use of cranberry juice? 1. Hypertension, managed with lisinopril 2. Diabetes type 1, managed with insulin 3. Atrial fibrillation, managed with warfarin 4. COPD, managed with inhaled steroids

ANS: 3 Chapter: Chapter 19 Alterations in Women's Health Chapter Learning Objective: 3. Describe common alterations in women's health, including medical management and nursing actions. Page: 590 Heading: Vaginitis > Urinary Tract Infections > Nursing Actions Integrated Processes: Nursing Process Client Need: Physiological Integrity: Reduction of Risk Potential Cognitive Level: Application [Applying] Concept: Infection Difficulty: Moderate Feedback 1 This is incorrect. Although there is no evidence that cranberry juice will prevent UTIs, it does not interact with lisinopril. 2 This is incorrect. Although cranberry juice may be high in sugar content and increase blood glucose levels, it is not contraindicated with insulin. 3 This is correct. Studies have shown that cranberry juice can affect the liver and interfere with warfarin (Coumadin) levels. Cranberry juice should not be taken with warfarin (Coumadin), aspirin, or medications that effect the liver. 4 This is incorrect. Inhaled steroids affect the pulmonary system, not hepatic.

The nurse is providing care for a patient in labor. The unborn fetus was diagnosed with severe microcephaly at 20 weeks' gestation. The patient tells the nurse, "We want everything done to save our baby who has as much right to a good life as anyone else." Which ethical approach does this represent? 1. Autonomy 2. Libertarianism 3. Egalitarianism 4. Utilitarianism

ANS: 3 Chapter: Chapter 2 Ethics and Standards of Practice Issues Chapter Learning Objective: 2. Debate ethical issues in maternity nursing. Page: 23 Heading: Ethics in Nursing Practice > Ethical Approaches Integrated Processes: Nursing Process Client Need: Safe and Effective Care Environment: Management of Care Cognitive Level: Analysis [Analyzing] Concept: Ethics Difficulty: Moderate Feedback 1 This is incorrect. Autonomy is a patient's right to make health care decisions regardless of the outcome; however, autonomy is not considered specifically to be an ethical approach. 2 This is incorrect. Libertarianism is the ethical principle that promotes the idea that some people are more valuable to society than others, and those persons should be given the resources needed to survive. To do otherwise is seen as a waste of resources. 3 This is correct. Egalitarianism is the ethical principle being expressed by the patient. The principle focuses on the belief that all people are equal and resources should be distributed according to need. The focus is to protect the marginal or vulnerable members of society. 4 This is incorrect. Utilitarianism is the ethical principle that supports distributing resources to produce the greatest good for the greatest number of people. This principle opposes distribution of resources to only a few.

The nurse in an obstetrician's office is discussing a patient's request for legally terminating an unwanted pregnancy after the fetus tests positive for Down syndrome. The woman and her spouse have five children from ages 11 to 17 years, and the family lives in an isolated rural area. Which action does the nurse take? 1. Inform the patient of programs to meet the needs of special children. 2. Share that children with Down syndrome can be high functioning. 3. Tell the physician of contextual factors identifying an ethical dilemma. 4. Inquire about the presence of an extended family support system

ANS: 3 Chapter: Chapter 2 Ethics and Standards of Practice Issues Chapter Learning Objective: 2. Debate ethical issues in maternity nursing. Page: 25 Heading: Ethics in Nursing Practice > Ethical Decision-Making Models > Contextual Features Integrated Processes: Caring Client Need: Physiological Integrity: Reduction of Risk Potential Cognitive Level: Analysis [Analyzing] Concept: Ethics Difficulty: Difficult Feedback 1 This is incorrect. The location of the family in an isolated rural area may add complexity and problems with regard to acquiring assistance with meeting the needs of a special needs child. 2 This is incorrect. It is true that some children with Down syndrome are high functioning. However, many are also born with other developmental and medical issues, which can cause financial, emotional, and medical care burdens. 3 This is correct. The situation includes conditions that cause an ethical dilemma. The nurse will inform the health care provider about the patient's request in regard to the ethical decision-making model related to contextual features. 4 This is incorrect. Family support of any kind may or may not exist. The availability of long-term care and support may not be possible.

The nurse manager in the labor and delivery unit decides that all unit nurses are to take a course in electronic fetal monitoring (EFM) as recommended by AWHONN. Which is the most important issue related to EFM the nurse manager is expecting to address? 1. Eliminate the amount of litigation related to fetal injury. 2. Strengthen the staff's level of confidence with monitoring. 3. Reduce failure to accurately assess maternal and fetal status. 4. Improve the lack of communication with health care providers.

ANS: 3 Chapter: Chapter 2 Ethics and Standards of Practice Issues Chapter Learning Objective: 3. Explore standards of practice in maternity nursing. Page: 29 Heading: Legal Issues in Delivery of Care > Fetal Monitoring Integrated Processes: Nursing Process Client Need: Safe and Effective Care Environment: Management of Care Cognitive Level: Analysis [Analyzing] Concept: Patient-Centered Care Difficulty: Difficult Feedback 1 This is incorrect. One result of nurses taking a course of EFM may be a reduction (not elimination) of litigation related to fetal injury. However, a more important result will be the decrease in fetal injury. 2 This is incorrect. A course in EFM should increase the confidence of the labor and delivery nursing staff, but this is not the most important result the nurse manager is expecting. 3 This is correct. The most important issue the nurse manager expects to address with an EFM course is to reduce or eliminate the failure of nursing staff to accurately assess maternal and fetal status. 4 This is incorrect. Improvement of communication between the nurses and health care providers is not expected from a course in EFM; however, the quality of communication may improve.

An experienced OB nurse has accepted a position in labor and delivery of an inner-city hospital providing care for multicultural clients. Which action by the nurse is most helpful in preparation for this position? 1. Reviewing quantitative research focused on current birth procedures 2. Comparing facility policies with recent research recommendations 3. Intensely studying qualitative research focused on practice enhancement 4. Obtaining access to the department's recent research of evidence-based practice

ANS: 3 Chapter: Chapter 2 Ethics and Standards of Practice Issues Chapter Learning Objective: 5. Analyze concepts related to evidence-based practice. Page: 31 Heading: Evidence-Based Practice > Evidence-Based Nursing > Utilization of Research in Clinical Practice Integrated Processes: Nursing Process Client Need: Safe and Effective Care Environment: Management of Care Cognitive Level: Analysis [Analyzing] Concept: Evidence-Based Practice Difficulty: Difficult Feedback 1 This is incorrect. Reviewing quantitative research is one aspect of obtaining information regarding evidence-based practice (EBP). Although this research is valuable in keeping pace with current practice, it is not the action that will provide this nurse with information needed in preparation for this position. 2 This is incorrect. The nurse needs to be aware of facility policies; however, a comparison of the policies with recent research recommendations does not specifically prepare the nurse for this new position. 3 This is correct. The most helpful action by the nurse is to be aware of how the new workplace will affect the nurse's practice. It is important that the nurse review quantitative research addressing multicultural attitudes, understanding, and practices during childbirth. This is the most helpful action for preparation related to this job. 4 This is incorrect. At some point, the nurse wants to evaluate and/or access the department's recent research of EBP. However, this is not the most helpful action for job preparation.

The nurse is discussing genetically linked diseases with a couple planning a pregnancy. The female states, "I am concerned because there is a history of sickle-cell disease in my family." Which information from the nurse is correct? 1. Any offspring will have the disease. 2. Only male children will have the disease. 3. The male partner needs genetic testing. 4. The couple should not become pregnant.

ANS: 3 Chapter: Chapter 3 Genetics, Conception, Fetal Development, and Reproductive Technology Chapter Learning Objective: 1. Discuss the relevance of genetics within the context of the care of the childbearing family. Page: 38 Heading: Genetics and the Childbearing Family > Dominant and Recessive Inheritance Integrated Processes: Nursing Process Client Need: Health Promotion and Maintenance Cognitive Level: Application [Applying] Concept: Ante/Intra/Post-partum Difficulty: Moderate Feedback 1 This is incorrect. Sickle-cell anemia is a recessive gene, and the disease occurs only if both partners carry the gene. Offspring from this couple will result in either carriers or noncarriers of the gene if one of the partners is negative. 2 This is incorrect. The gene for sickle-cell anemia is recessive but not sex-linked; therefore, the disease does not appear only in male or female offspring. 3 This is correct. The nurse needs to advise the male partner to have genetic testing to determine whether he is also a carrier of the sickle-cell gene. If both partners have the gene, 25% of the offspring will have the disease, 50% will be carriers, and 25% will be neither. 4 This is incorrect. The nurse does not need to advise the couple not to become pregnant. After genetic testing of the male partner, the couple will need to consider the possible outcomes of pregnancy.

A female carries one positive genetic marker for Huntington's disease. Her partner does not have any markers for the disease. For which reason is it so important for this family to identify persons with the genetic marker? 1. The Huntington gene is recessive. 2. All of the couple's offspring will have the disease. 3. Symptoms do not manifest until 30 to 50 years of age. 4. The expression of the disease is varied by individual.

ANS: 3 Chapter: Chapter 3 Genetics, Conception, Fetal Development, and Reproductive Technology Chapter Learning Objective: 1. Discuss the relevance of genetics within the context of the care of the childbearing family. Page: 38 Heading: Table 3-1: Genetic Diseases Integrated Processes: Nursing Process Client Need: Physiological Integrity: Reduction of Risk Potential Cognitive Level: Analyzing [Analysis] Concept: Health Promotion Difficulty: Difficult Feedback 1 This is incorrect. The Huntington gene is dominant so each recipient of the Huntington gene will be able to pass the gene; any recipients will develop Huntington disease. 2 This is incorrect. Fifty percent of the couple's offspring will receive the gene and develop Huntington's disease. 3 This is correct. Huntington's disease does not exhibit symptoms until the gene carrier is between 20 and 50 years of age, which is after many of the carriers have already had children. The disease is perpetuated by lack of knowledge regarding the presence of the gene. 4 This is incorrect. Huntington's disease is progressively debilitating and most frequently disrupts psychomotor functions.

The nurse is teaching a class about embryonic and fetal development to couples in the early stage of pregnancy. For which reason does the nurse emphasize the first 8 weeks of gestation? 1. Pregnancies often abort before or at this time of development. 2. Lack of size and movement prevents confirmation of pregnancy. 3. All organ systems are developing during this period. 4. Factors that can interrupt the pregnancy are no longer a concern.

ANS: 3 Chapter: Chapter 3 Genetics, Conception, Fetal Development, and Reproductive Technology Chapter Learning Objective: 2. Identify critical components of conception, embryonic development, and fetal development. Page: 47 Heading: Embryonic and Fetal Development > Embryo Integrated Processes: Nursing Process Client Need: Physiological Integrity: Reduction of Risk Potential Cognitive Level: Analysis [Analyzing] Concept: Ante/Intra/Post-partum Difficulty: Moderate Feedback 1 This is incorrect. Early pregnancies can be spontaneously aborted for a variety of reasons; however, this is not a reason for the nurse to emphasize the first 8 weeks of gestation. 2 This is incorrect. Pregnancy can be confirmed early after fertilization and implantation occurs. Embryo size and movement can aid in determining the period of gestation but are not components used to confirm pregnancy. 3 This is correct. At 8 weeks the primary germ layers have transformed into a clearly defined human. The embryo is now a fetus with all major organ systems formed. The nurse emphasizes this period because interfering factors for development should be avoided up to this point. 4 This is incorrect. Even though all major physical development is complete, there are other factors that can interrupt the pregnancy after this point. Some concerns are related to the health of the mother and the viability of the placenta and fetal circulation.

A female patient is concerned about an inability to become pregnant after trying for 1 year. Which information collected during a health history causes the nurse the greatest amount of concern about possible infertility? 1. The patient works as a ballroom dance instructor. 2. The patient is turning 37 years old this year. 3. The patient has hot flashes and mood swings. 4. The patient was treated for a pelvic inflammatory disease while in college.

ANS: 3 Chapter: Chapter 3 Genetics, Conception, Fetal Development, and Reproductive Technology Chapter Learning Objective: 4. List the common causes of infertility. Page: 52 Heading: Infertility and Reproductive Technology > Causes of Infertility Integrated Processes: Nursing Process Client Need: Physiological Integrity: Reduction of Risk Potential Cognitive Level: Analysis [Analyzing] Concept: Sexuality Difficulty: Moderate Feedback 1 This is incorrect. Excessive exercise can cause infertility in a female; however, the patient's job may or may not be a contributing factor. This is not the nurse's greatest concern. 2 This is incorrect. A female patient older than 35 years of age is considered to have fertility issues after trying to get pregnant for 6 months. However, this is not the nurse's greatest concern. 3 This is correct. Premature ovarian failure, which is menopause prior to age 40, is a cause of female infertility. At age 37, the client is experiencing some manifestations of early menopause (hot flashes and mood swings). This finding causes the nurse greatest concern and prompts additional assessment. 4 This is incorrect. STIs are a major cause of female infertility. However, one episode of treated pelvic inflammatory disease (PID) is not likely to cause infertility. This is not the nurse's greatest concern.

The nurse works in a urologist's office. A male patient is scheduled for routine fertility testing. For which test does the nurse refrain from making preparation? 1. STI screening 2. Hormonal levels 3. Sexual functioning 4. Ejaculate analysis

ANS: 3 Chapter: Chapter 3 Genetics, Conception, Fetal Development, and Reproductive Technology Chapter Learning Objective: 5. Describe the most common diagnostic tests used to identify causes of infertility Page: 53 Heading: Infertility and Reproductive Technology > Diagnosis of Infertility Integrated Processes: Nursing Process Client Need: Physiological Integrity: Physiological Adaptation Cognitive Level: Application [Applying] Concept: Sexuality Difficulty: Moderate Feedback 1 This is incorrect. STI testing is routine for both the male and female of a possible infertile couple. 2 This is incorrect. Hormonal levels are routinely performed on both the male and female of a possible infertile couple. 3 This is correct. Sexual functioning is not routinely measured by a test. If the patient voices concern about sexual functioning, vascular studies may be performed. 4 This is incorrect. An ejaculation analysis, or sperm count, is a routine test when checking for male infertility.

The nurse is counseling a female patient who has unsuccessfully attempted to become pregnant through a variety of methods and treatments for infertility. Which psychosocial manifestation is the nurse most likely to recognize? 1. Close connections with extended family 2. Strong intimate relationship with her partner 3. Difficulty accepting pregnancy if it does occur 4. Greater focus on career and job opportunities

ANS: 3 Chapter: Chapter 3 Genetics, Conception, Fetal Development, and Reproductive Technology Chapter Learning Objective: 7. Discuss the ethical and emotional implications of assisted reproductive therapies. Page: 55 Heading: Infertility and Reproductive Technology > Emotional Implications Integrated Processes: Nursing Process Client Need: Psychosocial Adaptation Cognitive Level: Analysis [Analyzing] Concept: Evidence-Based Practice Difficulty: Moderate Feedback 1 This is incorrect. Women with persistent infertility are frequently socially isolated; they become consumed by their infertility and tend to neglect family, friends, and relationships. 2 This is incorrect. Infertility becomes the focus of the couple's relationship, and other aspects of the relationship are often ignored. The result is often a dysfunctional marriage or couple relationship. 3 This is correct. Once pregnancy is achieved, the woman often has difficulty perceiving herself as a pregnant woman. 4 This is incorrect. Career women may experience a sense of loss of self, due to the shift from career to concern about infertility.

The nurse at a prenatal clinic is aware of the important tasks that each expectant mother will need to address. When an expectant mother states, "I will give up everything I have to make sure this baby is safe and well-cared for," which task is the mother addressing? 1. Expressing an attachment to the child 2. Ensuring safe birth for mother and child 3. Stating a willingness to give of oneself 4. Ensuring social acceptance of the child

ANS: 3 Chapter: Chapter 5 The Psycho-Social-Cultural Aspects of the Antepartum Period Chapter Learning Objective: 1. Describe expected emotional changes of the pregnant woman and appropriate nursing responses to these changes. Page: 102 Heading: Maternal Adaptation to Pregnancy > Maternal Tasks of Pregnancy Integrated Processes: Nursing Process Client Need: Psychosocial Integrity Cognitive Level: Analysis [Analyzing] Concept: Family Difficulty: Moderate Feedback 1 This is incorrect. Attaching or "binding-in" to the child is the development of maternal-fetal attachment. The mother's statement does not address attachment. 2 This is incorrect. Ensuring a safe passage for herself and her child involves the mother's knowledge and care-seeking behaviors to ensure that both she and the newborn emerge from pregnancy healthy. The mother's statement does not specifically address this task. 3 This is correct. The mother's statement specifically addresses the mother's willingness and efforts to make personal sacrifices for the child. The task is that of giving oneself to the demands of motherhood. 4 This is incorrect. Social acceptance of the child by significant others is important to the pregnant woman. However, the mother's statement does not address the woman's engagement of her family and social network in the pregnancy.

The nurse is providing care for a patient who is 42 years of age and in the first trimester of her pregnancy. For which possible complication will the nurse closely monitor the patient and fetus? 1. Elevated blood pressure and proteinuria 2. Indications of maladaptation to pregnancy 3. Alterations in fetal chromosomal studies 4. Subtle indicators of menopause occurring

ANS: 3 Chapter: Chapter 5 The Psycho-Social-Cultural Aspects of the Antepartum Period Chapter Learning Objective: 3. Identify critical variables that influence adaptation to pregnancy, including age, parity, and social, cultural, and sexual orientation Page: 106 Heading: Maternal Adaptation to Pregnancy > Factors That Influence Maternal Adaptation > Maternal Age > Older Mothers Integrated Processes: Nursing Process Client Need: Physiological Integrity: Reduction of Risk Potential Cognitive Level: Analysis [Analyzing] Concept: Ante/Intra/Post-partum Difficulty: Moderate Feedback 1 This is incorrect. The nurse will need to monitor the patient for signs of preeclampsia, but not in the first trimester of the pregnancy. 2 This is incorrect. Indicators for maladaptation to pregnancy should be monitored for throughout the pregnancy. 3 This is correct. Due to the patient's age, the nurse will closely monitor for chromosomal alterations in the fetus. Older mothers are a greater risk for fetal chromosome defects. 4 This is incorrect. There is no true reason why the nurse will monitor the patient for any indications of menopause; the nurse is focused on the existing pregnancy and the well-being of the mother and fetus.

A patient is scheduled for transvaginal ultrasound testing. Which preparation by the nurse is appropriate? 1. Place the patient supine with a pillow beneath her head. 2. Explain that pain at 4 or less on a 0 to10 scale is expected. 3. Ascertain whether the patient has a latex or banana allergy. 4. Request that the patient's partner leave the testing room.

ANS: 3 Chapter: Chapter 6 Antepartal Tests Chapter Learning Objective: 4. Articulate the nursing responsibilities related to key antenatal tests. Page: 136 Heading: Biophysical Assessment > Fetal Ultrasound Imaging > Nursing Actions Integrated Processes: Nursing Process Client Need: Physiological Integrity: Reduction of Risk Potential Cognitive Level: Application [Applying] Concept: Ante/Intra/Post-partum Difficulty: Moderate Feedback 1 This is incorrect. For a transvaginal ultrasound, the patient must be in a lithotomy position in order to insert the probe into the vagina. A pillow beneath the patient's head is appropriate. 2 This is incorrect. The nurse should tell the patient that she may feel some pressure when the ultrasound probe is inserted into the vagina, but pain is not expected. 3 This is correct. Because the transvaginal ultrasound probe is covered by a latex sheath, the nurse needs to ascertain whether the patient has a latex allergy or has exhibited an allergic response to specific foods such as bananas. 4 This is incorrect. Unless it is the patient's wish, there is no need for the nurse to ask the patient's partner to leave the testing room

A patient in the second trimester of pregnancy becomes upset when the health care provider (HCP) schedules several screening tests. The patient voices concern that something is wrong with her baby. Which statement by the nurse will reduce the patient's anxiety? 1. "Multiple screening tests are ordered for every pregnancy." 2. "It is better to identify problems before birth than afterward." 3. "Screening tests are primarily to identify those without disease or abnormality." 4. "Diagnostic testing is a reason for worry because they indicate fetal problems."

ANS: 3 Chapter: Chapter 6 Antepartal Tests Chapter Learning Objective: 5. Identify patient teaching needs related to antenatal tests. Page: 133 Heading: Screening and Diagnostic Tests Integrated Processes: Nursing Process Client Need: Physiological Integrity: Reduction of Risk Potential Cognitive Level: Analysis [Analyzing] Concept: Ante/Intra/Post-partum Difficulty: Moderate Feedback 1 This is incorrect. Telling the patient multiple screening tests are ordered for every pregnancy may reduce the patient's anxiety, but it is not totally true. Nurses need to be truthful. 2 This is incorrect. Telling the patient that it is better to identify problems before birth than afterward will not reduce the patient's anxiety. The nurse's comment is not therapeutic or sensitive. 3 This is correct. The truthful statement that screening tests are primarily to identify those without disease or abnormality will alleviate the patient's anxiety. 4 This is incorrect. Telling the patient diagnostic testing is a reason for worry because they indicate fetal problems may alleviate the patient's anxiety over screening tests, but it is not necessarily truthful. The patient's anxiety will be worse if additional screening or diagnostic testing is needed.

A patient undergoes chorionic villa sampling to rule out the presence of a genetic disorder. Following the procedure, the patient experiences iatrogenic PPROM. Which explanation does the nurse provide to promote patient understanding? 1. The rupture of the membranes is from a bacterial infection. 2. The membranes ruptured because the test caused fetal death. 3. The premature rupture of the membranes is a known risk to the test. 4. The membranes ruptured due to the presence of a genetic disorder.

ANS: 3 Chapter: Chapter 7 High-Risk Antepartum Nursing Care Chapter Learning Objective: 2. Delineate clinical features indicative of pregnancy complications and tests to predict, screen for, diagnose, and manage high-risk conditions. Page: 158 Heading: Gestational Complications > Preterm Premature Rupture of Membranes/Chorioamnionitis Integrated Processes: Nursing Process Client Need: Physiological Integrity: Reduction of Risk Potential Cognitive Level: Application [Applying] Concept: Ante/Intra/Post-partum Difficulty: Moderate Feedback 1 This is incorrect. Iatrogenic PPROM is not due to a bacterial infection. However, a bacterial infection is a risk factor with an invasive procedure such as chorionic villus sampling. The timing does not support the development of an infection. 2 This is incorrect. There is no evidence that the procedure caused fetal death. 3 This is correct. Iatrogenic PPROM is associated with a medical intervention such as the patient's procedure, chorionic villus sampling. The preterm rupture of membranes is a known risk and unpreventable complication related to invasive testing. 4 This is incorrect. A spontaneous abortion can result from the presence of a non-viable pregnancy; however, the scenario does not validate the presence of a genetic disorder.

A patient who is in the third trimester of pregnancy is informed that she will need a cesarean hysterectomy and bladder reconstruction due to a placenta defect. Which medical condition does the nurse explain to the patient? 1. Placenta accreta 2. Placenta increta 3. Placenta percreta 4. Placenta previa

ANS: 3 Chapter: Chapter 7 High-Risk Antepartum Nursing Care Chapter Learning Objective: 2. Delineate clinical features indicative of pregnancy complications and tests to predict, screen for, diagnose, and manage high-risk conditions. Page: 184 Heading: Placental Abnormalities and Hemorrhagic Complications > Placenta Accreta Integrated Processes: Nursing Process Client Need: Physiological Integrity: Reduction of Risk Potential Cognitive Level: Analysis [Analyzing] Concept: Ante/Intra/Post-partum Difficulty: Difficult Feedback 1 This is incorrect. Placenta accrete is when invasion of the trophoblast is beyond the normal boundary (80% of cases). 2 This is incorrect. Placenta increta is when invasion of the trophoblast extends into uterine myometrium (15% of cases). 3 This is correct. Placenta percreta is when invasion of the trophoblast extends into the uterine musculature and can adhere to other pelvic organs (5% of cases). 4 This is incorrect. Placenta previa is when the placenta is located low on the uterine wall and either partially or completely covers the internal opening of the cervix.

The nurse in a prenatal clinic is reviewing the files of four patients scheduled for visits. Which patient does the nurse identify as having the highest-risk pregnancy? 1. The patient who is 16 years of age just diagnosed with gestational diabetes 2. The patient with preexisting hypertension who is currently pregnant with twins 3. The patient who is 37 years of age, obese, and experiencing pregnancy-induced hypertension 4. The patient who is 28 years of age who delivered a premature neonate 3 years prior

ANS: 3 Chapter: Chapter 7 High-Risk Antepartum Nursing Care Chapter Learning Objective: 3. Identify potential antenatal complications for the woman, the fetus, and the newborn. Page: 150 Heading: Box 7-1: Common Risk Factors Integrated Processes: Nursing Process Client Need: Physiological Integrity: Reduction of Risk Potential Cognitive Level: Analysis [Analyzing] Concept: Ante/Intra/Post-partum Difficulty: Moderate Feedback 1 This is incorrect. The patient who is 16 years of age and newly diagnosed with gestational diabetes has two risk factors: age and a medical complication related to pregnancy. 2 This is incorrect. The patient with preexisting hypertension and pregnant with twins has two risk factors: an existing medical condition and multiple gestation. 3 This is correct. The patient who is 37 years of age, obese, and experiencing pregnancy-induced hypertension has three risk factors: age over 35 years, excessive weight, and a pregnancy-related complication. This is the patient with the highest-risk pregnancy. 4 This is incorrect. The patient who is 28 years of age who delivered a premature neonate 3 years prior has one complication: a history of prior pregnancy complications.

The patient is having an unmedicated childbirth and has begun to bear down. She vocalizes, "The baby is coming!" Which action should the nurse take? 1. Help the patient in a lithotomy position. 2. Help the patient onto all fours. 3. Have the patient assume a comfortable and upright position. 4. Help the patient into a knee-chest position.

ANS: 3 Chapter: Chapter 8 Intrapartum Assessment and Interventions Chapter Learning Objective: Demonstrate understanding of supportive care of the laboring woman. Page: 222 Heading: Factors Affecting Labor > Evidence- Based Practice: Labor Down or Bear Down Integrated Processes: Caring Client Need: Safe and Effective Care Environment: Management of Care Cognitive Level: Application [Applying] Concept: Ante/Intra/Post-partum Difficulty: Moderate Feedback 1 This is incorrect. Although regularly used, evidence has shown the lithotomy position is not the best position. 2 This is incorrect. Although sometimes used, recent research has shown that being on all fours is not the best position 3 This is correct. An upright position allows gravity to assist with the descent of the baby. 4 This is incorrect. A knee-chest position does not facilitate birth.

The nurse is reading the patient's chart, which indicates the patient has a "gynecoid pelvis." What finding is expected in this patient? 1. Narrower pubic arch 2. Shorter diameter between her coccyx and ischium 3. Wider outlet 4. Smaller inlet

ANS: 3 Chapter: Chapter 8 Intrapartum Assessment and Interventions Chapter Learning Objective: Demonstrate understanding of supportive care of the laboring woman. Page: 222 Heading: Factors Affecting Labor > Passage Integrated Processes: Teaching and Learning Client Need: Physiological Integrity: Reduction of Risk Potential Cognitive Level: Analysis [Analyzing] Concept: Ante/Intra/Post-partum Difficulty: Moderate Feedback 1 This is incorrect. A gynecoid pelvis has a wider pubic arch than an android pelvis. 2 This is incorrect. A gynecoid pelvis has a longer diameter between the coccyx and the ischium than an android pelvis 3 This is correct. A gynecoid pelvis has a wider outlet than an android pelvis. 4 This is incorrect. A gynecoid pelvis has a larger inlet than an android pelvis

The nursing preceptor asks the nursing student how to best determine the intensity of contractions before placing the patient on an electronic fetal monitoring strip. How would the nurse assess this? 1. Time the amount of time in between the ending of one contraction and the beginning of another. 2. Palpate the maternal abdomen right after a contraction ceases. 3. Palpate the maternal abdomen during a contraction. 4. Monitor the patients' vocalizations and facial expressions.

ANS: 3 Chapter: Chapter 8 Intrapartum Assessment and Interventions Chapter Learning Objective: Describe the four stages of labor and the related nursing and medical care. Page: 221 Heading: Factors Effecting Labor> Powers > Uterine Contractions Integrated Processes: Teaching and Learning Client Need: Physiological Integrity: Physiological Adaptation Cognitive Level: Application [Applying] Concept: Ante/Intra/Post-partum Difficulty: Moderate Feedback 1 This is incorrect. Timing the amount of time between the ending of one contraction and the beginning of another would determine frequency. 2 This is incorrect. Palpating the maternal abdomen right after a contraction ceases would determine resting tone. 3 This is correct. Intensity is evaluated by palpating the fingertips on the maternal abdomen. 4 This is incorrect. Depending on the patients' pain tolerance and cultural background, each patient will have different vocalizations and facial expressions that do not necessarily correlate to the intensity of the contraction.

A 40-year-old G5P1031 is attending a childbirth education class. During the class, the woman says, "The worst part of contractions with my last labor was when they were at the peak of intensity." The nurse educator correctly explains that the woman is identifying which part of a contraction? 1. When the uterine wall is resistant to indentation 2. The longest part of the contraction 3. The shortest part of the contraction 4. Descending of contraction

ANS: 3 Chapter: Chapter 8 Intrapartum Assessment and Interventions Chapter Learning Objective: Describe the four stages of labor and the related nursing and medical care. Page: 221 Heading: Factors Effecting Labor> Powers > Uterine Contractions Integrated Processes: Teaching and Learning Client Need: Physiological Integrity: Physiological Adaptation Cognitive Level: Application [Applying] Concept: Ante/Intra/Post-partum Difficulty: Moderate Feedback 1 This is incorrect. When a nurse palpates moderate contractions, they are resistant to indentation; the acme of a contraction should ideally be strong to palpation. 2 This is incorrect. The longest part of the contraction is the increment, which is the buildup of the contraction. 3 This is correct. The acme is the peak of the contraction's intensity but also is the shortest part of the contraction. 4 This is incorrect. The decrement is the relaxation of the uterine muscle and the end of the intensity of the contraction.

The nurse is reviewing the chart of a 35-year-old G4P2012 woman. The patient is at 38 weeks, 4 days and is in active labor with SROM clear fluid 2 hours ago. What action should the nurse take? 1. Monitor the patient's blood pressure, temperature, and respirations every 2 hours. 2. Have the patient rate her pain on a scale from 1-10 roughly every hour. 3. Help the patient change her position from side to side every 30 minutes. 4. Monitor the fetal heart tones every hour.

ANS: 3 Chapter: Chapter 8 Intrapartum Assessment and Interventions Chapter Learning Objective: Describe the four stages of labor and the related nursing and medical care. Page: 264 Heading: Clinical Pathway for Intrapartum Maternal and Fetal Assessment Integrated Processes: Nursing Process Client Need: Safe and Effective Care Environment: Management of Care Cognitive Level: Application [Applying] Concept: Ante/Intra/Post-partum Difficulty: Moderate Feedback 1 This is incorrect. When the patient is in active labor, the nurse should monitor the patient's vital signs every hour. Once rupture of the membranes has occurred, the nurse should monitor the patient's temperature every hour. 2 This is incorrect. When the patient is in active labor, the nurse should assess the patient's pain status every 30 minutes and as needed. 3 This is correct. When the patient is in active labor, the nurse should help the woman change position every 30 minutes and as needed. 4 This is incorrect. When the patient is in active labor, the nurse should monitor FHR every 15 to 30 minutes.

The patient is a 26-year-old G1P0 at 38 weeks, 2 days of gestation. She is at her provider's office for a visit and complains to the nurse of wrist pain, fatigue, increased discharge, and "feeling heavy." Which complaint could be a sign of impending labor? 1. Wrist pain 2. Fatigue 3. Increased discharge 4. Heavy feeling

ANS: 3 Chapter: Chapter 8 Intrapartum Assessment and Interventions Chapter Learning Objective: Describe the mechanism of spontaneous vaginal delivery and related nursing care. Pages: 220 Heading: Signs of Impending Labor Integrated Processes: Nursing Process Client Need: Physiological Integrity: Physiological Adaptation Cognitive Level: Application [Applying] Concept: Ante/Intra/Post-partum Difficulty: Moderate Feedback 1 This is incorrect. Wrist pain is common in pregnancy but is not a sign of impending labor. 2 This is incorrect. Fatigue is common in pregnancy but is not a sign of impending labor. 3 This is correct. When labor is impending, the patient may lose her mucous plug or have a change in discharge. 4 This is incorrect. Although a heavy feeling is common in pregnancy, it is not a sign of impending labor.

The nursing is caring for a 31-year-old female patient who is pregnant at 37 weeks and 5 days gestation. The patient is having contractions every 3 minutes and was found to have a platypelloid pelvis upon examination. The fetus has an estimated fetal weight of 7 lbs and is in the LOA position. This patient is laboring on the birth ball, and her mother-in-law is helping her labor. The nurse is concerned about the five Ps and their effect on the patient's labor. Which P is the nurse most likely concerned about based on the patient's history? 1. Passenger 2. Position 3. Passage 4. Psyche

ANS: 3 Chapter: Chapter 8 Intrapartum Assessment and Interventions Chapter Learning Objective: Identify the five Ps of labor. Pages: 220 Heading: Factors Affecting Labor Integrated Processes: Nursing Process Client Need: Safe and Effective Care Environment: Management of Care Cognitive Level: Analysis [Analyzing] Concept: Ante/Intra/Post-partum Difficulty: Moderate Feedback 1 This is incorrect. The passenger is the fetus; the patient's fetus is of average estimated fetal weight and in an optimal position for labor. 2 This is incorrect. This patient is on the birth ball, which is an excellent laboring position. 3 This is correct. A platypelloid pelvis is found in only 3% of women and is not an optimal pelvis for the passage of a vaginal delivery. 4 This is incorrect. Having a support person during labor has been shown to improve a woman's psyche during labor.

While reviewing the birth plan of an uncomplicated and healthy patient in active labor, the nurse notices that she would like to have a natural labor and potentially experience hydrotherapy. Which option should the nurse suggest for the patient? 1. IUPC to make sure that her contractions are adequate to keep labor progressing 2. FSE to make sure that her fetus is tolerating the hydrotherapy 3. Telemetry to allow for the patient to accomplish her birth plan 4. External EFM to make sure that there is continuous monitoring

ANS: 3 Chapter: Chapter 9 Fetal Heart Rate Assessment Chapter Learning Objective: Describe the components of fetal heart rate (FHR) and uterine contraction (UC) patterns essential to interpretation of monitor strips. Page: 277 Heading: Modes or Types of Fetal and Uterine Monitoring> Telemetry Integrated Processes: Nursing Process Client Need: Safe and Effective Care Environment: Management of Care Cognitive Level: Application [Applying] Concept: Ante/Intra/Post-partum Difficulty: Moderate Feedback 1 This is incorrect. An intrauterine pressure catheter (IUPC) is only used in a patient who is bed bound, which would not allow for a patient to move around or use hydrotherapy. 2 This is incorrect. A fetal scalp electrode (FSE) is only used in a patient who is bed bound, which would not allow for a patient to move around or use hydrotherapy. 3 This is correct. Telemetry would allow for continuous monitoring while allowing patients free movement and the ability to use hydrotherapy. 4 This is incorrect. External EFM is best used in a patient who is bed bound, which would not allow for a patient to move around or use hydrotherapy.

The nurse is assessing a patient who is 36 hours postpartum following a cesarean delivery. Which findings cause the nurse to conclude that a wound infection is developing? Select all that apply. 1. Temperature increase from 99.8°F to 100.5°F 2. Incisional tenderness with palpation 3. Increased margins of incisional redness 4. Notably warm skin around the incision 5. Serosanguinous drainage from the suture line

ANS: 3, 4 Chapter: Chapter 14 High-Risk Postpartum Nursing Care Chapter Learning Objective: 2. Describe the primary postpartum infections and the related nursing actions and medical care. Page: 430 Heading: Infections > Wound Infection Integrated Processes: Nursing Process Client Need: Physiological Integrity: Reduction of Risk Potential Cognitive Level: Analysis [Analyzing] Concept: Ante/Intra/Post-partum Difficulty: Difficult Feedback 1 This is incorrect. An increase in temperature from 99.8°F to 100.5°F may or may not indicate a developing wound infection. The patient is expected to have a low-grade fever related to surgical trauma; however, wound infections may cause a low-grade fever. The finding is inconclusive. 2 This is incorrect. At 36 hours, incisional tenderness with palpation is likely to be related to surgical trauma. This finding is inconclusive. 3 This is correct. An increase in redness in the incisional margins is a likely sign of developing wound infection. 4 This is correct. When the skin around a surgical incision is notably warm to the touch, it is likely a sign of a developing wound infection. 5 This is incorrect. Serosanguinous drainage from the suture line 36 hours after the surgery is an expected finding and is not indicative of a developing wound infection.

A new mother states, "I don't want anyone around my baby. I need to protect him from getting sick." Which statement by the nurse will help the mother to understand neonatal immunity? Select all that apply. 1. "I agree with you; the baby's sterile environment is gone." 2. "The baby will have acquired immunity soon from vaccinations." 3. "The baby has natural passive immunity from you for a few months." 4. "We will give the baby gamma globin for short-term immediate protection." 5. "Your baby was exposed to some pretty serious pathogens in your birth canal."

ANS: 3, 4 Chapter: Chapter 15 Physiological and Behavioral Responses of the Neonate Chapter Learning Objective: 1. Identify the changes that occur during the transition from intrauterine to extrauterine life and the related nursing actions. Page: 450 Heading: Transition to Extrauterine Life > The Immune System Integrated Processes: Nursing Process Client Need: Physiological Integrity: Physiological Adaptation Cognitive Level: Analysis [Analyzing] Concept: Ante/Intra/Post-partum Difficulty: Difficult Feedback 1 This is incorrect. The nurse's agreement with the mother does not help the mother understand neonatal immunity. 2 This is incorrect. The nurse's statement that the baby will soon be protected by vaccinations is not completely true; immunizations are not started immediately. 3 This is correct. The neonate does have natural passive immunity from the mother for the first few months. Natural passive immunity is the placental transmission of antibodies from the mother to the fetus. 4 This is correct. An example of artificial passive immunity is gamma globulin, which provides immediate protection for a short time. 5 This is incorrect. Neonates are first exposed to organisms from the maternal genital tract during the birthing process. The maternal genital track may contain group B streptococcus and Escherichia coli, which can result in neonatal sepsis. This is a true statement but does not help the mother to understand neonatal immunity.

The nurse is assessing a patient who just received confirmation of pregnancy. While collecting information about the patient's medical history, which information alerts the nurse to biophysical risk factors? Select all that apply. 1. The patient is primip who is 38 years of age. 2. The patient smokes two packs of cigarettes weekly. 3. The patient has been a strict vegetarian for 25 years. 4. The patient works as a nuclear medicine technician. 5. The patient is medically treated for rheumatoid arthritis.

ANS: 3, 5 Chapter: Chapter 6 Antepartal Tests Chapter Learning Objective: 1. Define terms used in antenatal tests. Page: 131 Heading: Assessment for Risk Factors Integrated Processes: Nursing Process Client Need: Physiological Integrity: Reduction of Risk Potential Cognitive Level: Analysis [Analyzing] Concept: Ante/Intra/Post-partum Difficulty: Difficult Feedback 1 This is incorrect. Age and parity are sociodemographic factors that place this patient and/or her fetus at risk for adverse outcomes. 2 This is incorrect. Smoking is a psychosocial factor that places this patient and/or her fetus at risk for adverse outcomes. 3 This is correct. Nutritional practices are a biological factor that can place this patient and/or her fetus at risk for adverse outcomes. Special considerations are needed to make sure iron and protein requirements are met. 4 This is incorrect. Environmental factors can place this patient and/or her fetus at risk for adverse outcomes. Risk factors include exposure to chemicals, radiation, and pollutants. 5 This is correct. A patient with a diagnosis of rheumatoid arthritis who is medically treated has biological factors that place this patient and/or her fetus at risk for adverse outcomes.

The nurse is caring for a patient who is in labor with her first child. The patient's mother is present for support and notes that things have changed in the delivery room since she last gave birth in the early 1980s. Which current trend or intervention may the patient's mother find most different? 1. Fetal monitoring throughout labor 2. Postpartum stay of 10 days 3. Expectant partner and family in operating room for cesarean birth 4. Hospital support for breastfeeding

ANS: 4 Chapter: Chapter 1 Trends and Issues Chapter Learning Objective: 1. Discuss current trends in the management of labor and birth Page: 4 Heading: Table 1-1: Past and Present Trends Integrated Processes: Nursing Process Client Need: Health Promotion and Maintenance Cognitive Level: Application [Applying] Concept: Evidence-Based Practice Difficulty: Moderate Feedback 1 This is incorrect. Fetal monitoring during labor began in the late 1970s. As such, this likely would have occurred during the mother's labor and delivery during the 1980s. 2 This is incorrect. In the past, the average hospital postpartum stay was 10 days. Presently, the average postpartum stay is 48 hours or less. 3 This is incorrect. In the past, expectant partners and families were excluded from the labor and birth experience. Present trends involve the expectant partner and family in the labor and birth experience, including presence in the operating room for cesarean births. 4 This is correct. Hospital support for breastfeeding, including a lactation consultant and employment of the Baby-Friendly Hospital Initiative, were both enacted during the early 1990s.

The nurse is providing education to a patient who has given birth to her first child and is being discharged home. The patient expressed concern regarding infant mortality and sudden infant death syndrome (SIDS). The patient had an uncomplicated pregnancy, labor, and vaginal delivery. She has a body mass index of 25 and has no other health conditions. The infant is healthy and was delivered full-term. What will be most helpful thing to explain to the patient? 1. Uses of extracorporeal membrane oxygenation therapy (ECMO) 2. Uses of exogenous pulmonary surfactant 3. The Baby-Friendly Hospital Initiative 4. The Safe to Sleep campaign

ANS: 4 Chapter: Chapter 1 Trends and Issues Chapter Learning Objective: 3. Identify leading causes of infant death. Page: 7 Heading: Trends > Infant Mortality Rates Integrated Processes: Nursing process Client Need: Safe and Effective Care Environment: Safety and Infection Control Cognitive Level: Application [Applying] Concept: Health Promotion Difficulty: Moderate Feedback 1 This is incorrect. EMCO has been cited as one of the factors that has reduced infant mortality among preterm infants. 2 This is incorrect. Although advances in medical treatments have decreased infant mortality, exogenous pulmonary surfactant is primarily used to reduce mortality of preterm infants. 3 This is incorrect. The Baby-Friendly Hospital Initiative was developed to support breastfeeding and is not directly linked to reduced infant mortality or SIDS. 4 This is correct. The Back to Sleep campaign and the Safe to Sleep campaigns were designed to promote healthy infant sleeping habits. The decrease in SIDS from 1995 to 2015 was attributed to the Safe to Sleep campaign.

The nurse is caring for a 14-year-old patient who is 32 weeks pregnant. After complaining of genital sores and discomfort, the patient tests positive for syphilis. The fetus is at increased risk of which condition? 1. Diabetes 2. Blindness 3. Pneumonia 4. Hypertension

ANS: 4 Chapter: Chapter 1 Trends and Issues Chapter Learning Objective: 4. Discuss current maternal and infant health issues. Page: 10 Heading: Issues > Teen Pregnancy > Implications of Teen Pregnancy and Birth Integrated Processes: Nursing Process Client Need: Physiological Integrity: Reduction of Risk Potential Cognitive Level: Application [Applying] Concept: Ante/Intra/Post-partum Difficulty: Moderate Feedback 1 This is incorrect. Maternal obesity increases a child's risk of developing childhood obesity and diabetes. 2 This is correct. Neonatal blindness, maternal death, and neonatal death are all associated with a patient who contracts syphilis during pregnancy. 3 This is incorrect. Chlamydial pneumonia is associated with maternal chlamydia. 4 This is incorrect. Teen mothers may have a higher risk of contracting sexually transmitted illnesses and hypertension during pregnancy; however, maternal syphilis is not associated with fetal hypertension.

The nurse has made it a goal to increase the rate at which women begin prenatal care in the first trimester. The nurse relates this decision to national goals for better maternal and infant outcomes. What guidelines will the nurse use to guide her maternal health goals? 1. WHO Maternal care guidelines 2. State Practice Acts 3. AWHONN white papers 4. Healthy People 2020

ANS: 4 Chapter: Chapter 1 Trends and Issues Chapter Learning Objective: 5. Identify the primary maternal and infant goals of Healthy People 2020. Page: 15 Heading: Maternal and Child Health Goals Integrated Processes: Nursing Process Client Need: Health Promotion and Maintenance Cognitive Level: Application [Applying] Concept: Health Promotion Difficulty: Moderate Feedback 1 This is incorrect. The WHO guidelines are too broad for this purpose, and the nurse will need to use national goals. 2 This is incorrect. State practice acts specify legal requirements rather than health promotion goals. 3 This is incorrect. AWHONN white papers will present positions but not necessarily detail health promotion goals. 4 This is correct. The national goals for improving maternal and infant health are found in Healthy People 2020.

The nurse is assisting the primary care provider with a vacuum-assisted delivery because of a prolonged second stage of labor. The nurse will inform the primary care provider when which guideline of the procedure is met? 1. Extension of the episiotomy is performed. 2. Signs of fetal compromise have resolved. 3. Patient is under full anesthesia status. 4. The "three-pull rule" has been achieved.

ANS: 4 Chapter: Chapter 10 High-Risk Labor and Birth Chapter Learning Objective: 4. Identify and manage high-risk pregnancy, labor, and delivery to promote healthy outcomes for the mother and infant. Page: 321 Heading: Operative Vaginal Delivery > Vacuum-Assisted Delivery Integrated Processes: Nursing Process Client Need: Physiological Integrity: Reduction of Risk Potential Cognitive Level: Application [Applying] Concept: Ante/Intra/Post-partum Difficulty: Moderate Feedback 1 This is incorrect. Extension of the patient's episiotomy is not a guideline for performance of vacuum-assisted delivery. 2 This is incorrect. A vacuum-assisted delivery is performed in the presence or suspicion of immediate or potential fetal compromise. Stopping the procedure for resolved fetal compromised is not a guideline for vacuum-assisted delivery. 3 This is incorrect. One advantage to vacuum-assisted delivery is a decrease in the amount of anesthesia required for the patient. 4 This is correct. The nurse will inform the care provider when the "three-pull rule" guideline for vacuum-assisted delivery has be achieved. There should be a maximum of three attempts for a period of 15 minutes.

The nurse is assisting with the preparation of a patient admitted for a planned cesarean birth. The patient has signed the consent form and discussed the elected regional anesthesia with the nurse anesthetist. Which is the most important action for the nurse related to anesthesia? 1. Verify the patient has been NPO for 6 to 8 hours. 2. Start an IV line and administer an IV fluid as ordered. 3. Administer preoperative medications per orders. 4. Obtain a baseline fetal heart rate monitor strip.

ANS: 4 Chapter: Chapter 11 Intrapartum and Postpartum Care of Cesarean Birth Families Chapter Learning Objective: 2. Discuss the preoperative nursing care and medical and anesthesia management for cesarean births. Page: 348 Heading: Perioperative Care: Integrated Processes: Nursing Process Client Need: Physiological Integrity: Reduction of Risk Potential Cognitive Level: Analysis [Analyzing] Concept: Ante/Intra/Post-partum Difficulty: Moderate Feedback 1 This is incorrect. Verifying the patient has been NPO for 6 to 8 hours before surgery is important; however, there is another more important nursing action. 2 This is incorrect. Starting an IV line and administering an IV fluid as ordered is important; however, there is another more important nursing action. 3 This is incorrect. Administering preoperative medications per orders is important; however, there is another more important nursing action. 4 This is correct. Obtaining a baseline fetal heart rate monitor strip for at least 20 minutes before the administration of anesthesia is the most important nursing action. Another 20-minute strip will be obtained after the administration of the anesthesia. This comparison makes fetal monitoring more effective.

A patient who is expecting her first baby tells the nurse, "I am afraid of the whole birth experience and plan to ask the doctor for a cesarean delivery." Which response by the nurse is most appropriate? 1. "I will get you some material about how labor pain is managed." 2. "Most women avoid cesarean births unless it is an emergency." 3. "I suggest you talk with the physician and get another opinion." 4. "Cesarean will cause you issues with additional pregnancies."

ANS: 4 Chapter: Chapter 11 Intrapartum and Postpartum Care of Cesarean Birth Families Chapter Learning Objective: 5. Identify potential intraoperative and postoperative complications related to cesarean birth and nursing actions to reduce risk. Page: 346 Heading: Introduction Integrated Processes: Nursing Process Client Need: Physiological Integrity: Reduction of Risk Potential Cognitive Level: Analysis [Analyzing] Concept: Ante/Intra/Post-partum Difficulty: Moderate Feedback 1 This is incorrect. It is appropriate for the nurse to supply the patient with material about pain management during labor and delivery; however, it is most important for the nurse to explain the risks associated with CDMR. 2 This is incorrect. It is true that most women attempt to avoid cesarean delivery unless it is an emergency. However, the patient may not be interested in decisions made by other women. 3 This is incorrect. It is within the scope of practice for the nurse to provide patient teaching, and the nurse can also suggest the patient explore the topic further with the health care provider. However, it is most important for the nurse to explain risks related to CDMR. 4 This is correct. The nurse needs to inform the patient that if additional pregnancies are desired, cesarean delivery risks increase with each additional pregnancy. Risk for placenta previa, placenta accreta, and gravid hysterectomy rises with each cesarean delivery.

In a research study performed by Schneuder, L., Crenshaw, J., and Gilder, R. (2017), which action by the nurse will be implemented following a cesarean delivery? 1. Allow the birth partner to hand the neonate to the mother. 2. Assist the mother and partner to cut the umbilical cord. 3. Move the neonate into the visual field of the mother. 4. Encourage skin-to-skin contact between the mother and neonate.

ANS: 4 Chapter: Chapter 11 Intrapartum and Postpartum Care of Cesarean Birth Families Chapter Learning Objective: 5. Identify potential intraoperative and postoperative complications related to cesarean birth and nursing actions to reduce risk. Page: 354 Heading: Intraoperative Care > Anesthesia Management > Nursing Actions Integrated Processes: Nursing Process Client Need: Physiological Integrity: Reduction of Risk Potential Cognitive Level: Comprehension [Understanding] Concept: Ante/Intra/Post-partum Difficulty: Moderate Feedback 1 This is incorrect. Evidence-based practice from a research study performed by Schneuder, L., Crenshaw, J., and Gilder, R. (2017) does not support allowing the partner to hand the neonate to the mother. 2 This is incorrect. Evidence-based practice from a research study performed by Schneuder, L., Crenshaw, J., and Gilder, R. (2017) does not support assisting the mother and partner to cut the umbilical cord. 3 This is incorrect. Evidence-based practice from a research study performed by Schneuder, L., Crenshaw, J., and Gilder, R. (2017) does not support moving the neonate into the visual field of the mother. 4 This is correct. Evidence-based practice from a research study performed by Schneuder, L., Crenshaw, J., and Gilder, R. (2017) strongly supports skin-to-skin contact between the mother and neonate in the operating room. The action significantly decreases the percentage of newborns who are separated from their parents due to transfers to the NICU for observation.

The nurse is palpating a patient's uterus 12 hours after a vaginal delivery. For which reason does the nurse place one hand just above the symphysis pubis? 1. To prevent uterine prolapse. 2. To prevent uterine movement 3. To prevent uterine hemorrhage 4. To prevent uterine inversion

ANS: 4 Chapter: Chapter 12 Postpartum Physiological Assessments and Nursing Care Chapter Learning Objective: 2. Identify the critical elements of assessment and nursing care during the postpartum period. Page: 367 Heading: The Reproductive System > Uterus > Nursing Actions Integrated Processes: Nursing Process Client Need: Physiological Integrity: Physiological Adaptation Cognitive Level: Analysis [Analyzing] Concept: Ante/Intra/Post-partum Difficulty: Moderate Feedback 1 This is incorrect. The nurse does not place a hand just above the symphysis pubis to prevent uterine prolapse. 2 This is incorrect. The nurse does not place a hand just above the symphysis pubis to prevent uterine movement. 3 This is incorrect. The process of assessing the uterus through palpation will provide information that can prevent uterine hemorrhage. 4 This is correct. When palpating the patient's uterus 12 hours postpartum, the nurse supports the lower uterine segment by placing one hand just above the symphysis pubis. Pregnancy stretches the ligaments that hold the uterus in place, and fundal pressure could result in uterine inversion.

A patient who is 12 hours postpartum after a vaginal delivery continues to have difficulty in initiating urination. The nurse is aware that an integrative method used when a woman is unable to void is peppermint oil. In which manner will the peppermint oil be used? 1. A thin layer is applied to the urinary meatus. 2. A small amount on a cotton ball is left at the bedside. 3. A small amount is added to the water of a vaporizer. 4. A saturated cotton ball is placed in a "hat" on the toilet.

ANS: 4 Chapter: Chapter 12 Postpartum Physiological Assessments and Nursing Care Chapter Learning Objective: 3. Describe safe and effective nursing care during the postpartum period. Page: 377 Heading: The Urinary System > Bladder Distension > Nursing Actions Integrated Processes: Nursing Process Client Need: Physiological Integrity: Physiological Adaptation Cognitive Level: Application [Applying] Concept: Ante/Intra/Post-partum Difficulty: Moderate Feedback 1 This is incorrect. The nurse understands that peppermint oil aids in urination because the vapors of the peppermint oil have a relaxing effect on the urinary sphincter. The oil is not applied to the urinary meatus. 2 This is incorrect. Leaving a small amount of peppermint oil on a cotton ball at the bedside is not effective; the peppermint vapors are effective when attempting to void. 3 This is incorrect. Adding a small amount of peppermint oil to the water of a vaporizer is not effective; the peppermint vapors are effective when attempting to void. 4 This is correct. The nurse will place a cotton ball saturated with peppermint oil in a "hat" used to collect urine when the patient voids in the toilet. A small amount of water is added to the "hat," and the resulting vapors have a relaxing effect on the urinary sphincter.

A new mother expresses frustration about how to know what her baby wants. The mother states, "I don't know what I expect, but then, the baby doesn't know either." Which situation does the nurse use as an example of neonate communication? 1. The baby is content to lie still on the mother's abdomen. 2. The baby is easily awakened if irritated by loud noises. 3. The baby resists eye contact if bored or disinterested. 4. The baby roots for the breast when the cheek is stroked.

ANS: 4 Chapter: Chapter 13 Transition to Parenthood Chapter Learning Objective: 3. Discuss bonding and attachment. Page: 408 Heading: Communication Between Parent and Child Integrated Processes: Nursing Process Client Need: Psychosocial Integrity Cognitive Level: Application [Applying] Concept: Family Dynamics Difficulty: Moderate Feedback 1 This is incorrect. Infants who are placed on their mothers' abdomens will crawl to the breast. 2 This is incorrect. Babies are more likely to awaken easily if they are drowsy. At this point, the baby can either be rocked back to sleep or made more awake. Even noise will not necessarily awaken a baby who is in deep sleep. 3 This is incorrect. Being en face is an initial interaction. Ongoing interactions lead to synchrony events, which are reciprocal actions between parents and infants that show mutual expressions of contentment. These interactions are very pleasurable for parents and infants. 4 This is correct. Rooting is an initial interaction that elicits the desire/need to eat. When a neonate's cheek is stroked, the neonate turns the head toward the touch and begins to root for the breast. The mother needs to understand this is the neonate's method of communication.

As a result of the previously mentioned research study, the nurses in a postpartum facility will implement which evidence-based change? 1. Continue to assess the level of fatigue for the mother during postpartum period. 2. Assist fathers in recognizing and managing stress and depressive symptoms. 3. Encourage the father to go home and rest while the mother is hospitalized. 4. Promote strategies to decrease fatigue during both prenatal and postnatal periods.

ANS: 4 Chapter: Chapter 13 Transition to Parenthood Chapter Learning Objective: 5. Describe nursing actions that support couples during their transition to parenthood. Page: 398 Heading: Transition to Parenthood > Evidence-Based Practice > Maternal and Paternal Fatigue Integrated Processes: Nursing Process Client Need: Psychosocial Integrity Cognitive Level: Application [Applying] Concept: Family Dynamics Difficulty: Moderate Feedback 1 This is incorrect. The nurses will assess levels of fatigue for both the mother and father during the postpartum period. 2 This is incorrect. The nurse will assist mothers in managing stress and depressive symptoms. This is more beneficial to improving her sleep quality than focusing on infant sleep patterns. 3 This is incorrect. The nurses will promote rest by providing uninterrupted time during the postpartum hospitalization. Part of this effort is enhanced by clustering nursing care to allow for periods of uninterrupted rest. 4 This is correct. Nurses will provide information on strategies to decrease fatigue during the prenatal period and postnatal periods.

The nurse is providing postpartum care for an adolescent mother and her family. Which factor is most important for the nurse to consider when planning teaching about neonatal care? 1. The grandparents decided they want to be involved. 2. The parents need to discuss their expectations of each other. 3. The mother is determined the father should be involved. 4. Information must be presented on an age-appropriate level.

ANS: 4 Chapter: Chapter 13 Transition to Parenthood Chapter Learning Objective: 5. Describe nursing actions that support couples during their transition to parenthood. Page: 402 Heading: Adolescent Parents > Nursing Actions Integrated Processes: Nursing Process Client Need: Psychosocial Integrity Cognitive Level: Analysis [Analyzing] Concept: Family Dynamics Difficulty: Difficult Feedback 1 This is incorrect. Grandparents need a review of infant care since most teen mothers live with their parents during the first year. However, there is a factor that is more important for the nurse to consider. 2 This is incorrect. Discuss with the adolescent parents their expectations of each other regarding child care and support. Realistic and mutually agreed-upon expectations decrease the level of stress within the relationship. However, there is a factor that is more important for the nurse to consider. 3 This is incorrect. Involve the adolescent father in prenatal care based on the adolescent mother's comfort level. However, there is one factor that is more important for the nurse to consider. 4 This is correct. Learning styles and teaching strategies are different for young teens and older teens. Information needs to be provided in a manner that will engage the adolescent parent in the learning process. This is the most important factor for the nurse to consider.

The nurse is closely monitoring a patient who is postpartum and at risk for PPH. Which assessment finding will cause the nurse to contact the primary care provider immediately? 1. The uterus is displaced. 2. The uterine fundus is boggy. 3. Small clots are expressed with massage. 4. Peripad weighs 100 g within 15 minutes.

ANS: 4 Chapter: Chapter 14 High-Risk Postpartum Nursing Care Chapter Learning Objective: 1. Describe the primary causes of postpartum hemorrhage and the related nursing actions and medical care. Page: 423 Heading: Hemorrhage > Uterine Atony > Nursing Actions Integrated Processes: Nursing Process Client Need: Physiological Integrity: Reduction of Risk Potential Cognitive Level: Analysis [Analyzing] Concept: Ante/Intra/Post-partum Difficulty: Moderate Feedback 1 This is incorrect. When the uterus is displaced, there is an increased risk for PPH. However, the cause is often related to a full urinary bladder. The nurse needs to assist the patient to the bathroom to void. 2 This is incorrect. When the uterine fundus is boggy, the uterine muscle is not contracted and the patient is at risk for PPH. The nurse will massage and reassess every 5 to 15 minutes. Uterotonic therapy may be initiated as per facility protocol. 3 This is incorrect. Small clots may be expelled when the fundus of the uterus is massaged. The nurse will closely assess for uterine tone, amount of bleeding, and large clots. Clots the size of an egg would warrant calling the primary care provider. 4 This is correct. The nurse will monitor the amount and characteristics of each patient's lochia. If bleeding seems excessive, the nurse will weigh peripads to ascertain the amount of blood loss. This patient's EBL is 100 mL in 15 minutes (1 g = 1 mL of blood). The nurse will contact the primary care provider and report postpartum hemorrhage.

The lactation nurse takes a phone call from a mother who is breastfeeding her 2-month-old infant. The mother reports an area of redness and warmth on the breast and a painful burning sensation when breastfeeding. Which statement by the nurse is correct if mastitis is suspected? 1. "If your nipples are cracked, you will need to stop breastfeeding." 2. "Pump your milk and throw it away until the infection is gone." 3. "The baby gave you an infection and needs to be on antibiotics." 4. "Continuing to breastfeed will help clear up the condition."

ANS: 4 Chapter: Chapter 14 High-Risk Postpartum Nursing Care Chapter Learning Objective: 2. Describe the primary postpartum infections and the related nursing actions and medical care. Page: 430 Heading: Infections > Mastitis Integrated Processes: Nursing Process Client Need: Physiological Integrity: Reduction of Risk Potential Cognitive Level: Analysis [Analyzing] Concept: Ante/Intra/Post-partum Difficulty: Moderate Feedback 1 This is incorrect. Cracked nipples are a port of entry for pathogens that cause mastitis; however, the mother does not necessarily need to stop breastfeeding. The lactation nurse will provide methods of management. 2 This is incorrect. When the mother has mastitis, the milk is unaffected and the baby will not be harmed with continuing breastfeeding. There is no reason to pump and discard the milk. 3 This is incorrect. The most common organism reported in mastitis is Staphylococcus aureus. The organism usually comes from the breastfeeding infant's mouth or throat but does not require placing the infant on antibiotics. 4 This is correct. Mastitis is generally self-limiting, and continued breastfeeding can help clear up the infection and condition. If antibiotic therapy is indicated, the infection generally resolves within 24 to 48 hours of antibiotic therapy.

A mother who is 2 weeks postpartum asks the nurse lactation specialist how she knows if her baby is hungry. Which hunger indicator does the nurse discuss? 1. Crying when all other physical needs are met 2. If 2 to 3 hours have passed since feeding 3. When the mother experiences a let-down sensation 4. Opening the mouth in response to tactile stimulation

ANS: 4 Chapter: Chapter 16 Discharge Planning and Teaching Chapter Learning Objective: 8. Provide parents with information regarding newborn care that reflects the assessed learning needs of parents Page: 485 Heading: Newborn Nutrition and Feeding > Breastfeeding > Process of Breastfeeding Integrated Processes: Nursing Process Client Need: Physiological Integrity: Physiological Adaptation Cognitive Level: Application [Applying] Concept: Ante/Intra/Post-partum Difficulty: Moderate Feedback 1 This is incorrect. There are signals for hunger that occur without the baby crying, which will reduce the baby's stress and facilitate bonding. 2 This is incorrect. The breastfed baby will not always follow a set feeding schedule. Most mothers are taught to breastfeed on demand, which will keep an adequate milk supply. 3 This is incorrect. The mother may experience a let-down sensation for a variety of reasons, including hearing an infant cry. It can also occur during sexual arousal or activity due to the natural release of oxytocin in response to an orgasm. The let-down reflex can be inhibited by stress, anxiety, pain, and fatigue. 4 This is correct. Opening the mouth in response to tactile stimulation is the best way to determine if a baby is hungry; the rooting reflex is not solicited in a baby who is not hungry.

The nurses in a NICU are concerned about the appropriate levels of oxygen therapy during the care of premature neonates. The nurses referenced an article by Newman (2014) titled, "Oxygen Saturation Limits and Evidence supporting the Targets." On which evidence-based conclusion will the nurses develop guidelines? 1. Oxygen saturation limits of 85% to 89% are effective. 2. Oxygen saturation rates of 91% to 95% are effective. 3. Infants are within saturation limits about 75% of the time. 4. Oxygen saturation limits need to be between 87% to 94%.

ANS: 4 Chapter: Chapter 17 High-Risk Neonatal Nursing Care Chapter Learning Objective: 1. Describe the physiology and pathophysiology associated with selected complications of the neonatal period. Page: 518 Heading: Retinopathy of Prematurity Integrated Processes: Nursing Process Client Need: Physiological Integrity: Reduction of Risk Potential Cognitive Level: Analysis [Analyzing] Concept: Ante/Intra/Post-partum Difficulty: Difficult Feedback 1 This is incorrect. Lower oxygen saturation limits of 85% to 89% were associated with increased risk for neonatal death. 2 This is incorrect. Higher oxygen saturation rates of 91% to 95% were associated with increased ROP rates. 3 This is incorrect. Infants are within saturation limits about 31% of the time and require multiple oxygen adjustments hourly. 4 This is correct. Rapid and consistent assessment with appropriate interventions are required to maintain oxygen saturation limits of 87% to 94% to decrease risk of ROP and neonatal death.

The labor and delivery nurse is present for the delivery of a premature neonate. Which action by the nurse is most important? 1. Stabilize and transfer neonate to NICU. 2. Review pregnancy history for risk factors. 3. Maintain fluid and electrolyte balance. 4. Provide a neutral temperature environment.

ANS: 4 Chapter: Chapter 17 High-Risk Neonatal Nursing Care Chapter Learning Objective: 2. Identify critical elements of assessment and nursing care of the high-risk neonate. Page: 508 Heading: Preterm Neonates > Nursing Actions Integrated Processes: Nursing Process Client Need: Physiological Integrity: Reduction of Risk Potential Cognitive Level: Analysis [Analyzing] Concept: Ante/Intra/Post-partum Difficulty: Difficult Feedback 1 This is incorrect. The premature neonate will need to be stabilized and moved to NICU for specialized care; however, this is not the nurse's most important action at the immediate time of birth. 2 This is incorrect. Review of the mother's pregnancy history in order to identify risk factors related to prematurity is not an immediate concern. 3 This is incorrect. The nurse in NICU is likely to be involved in maintaining fluid and electrolyte balance. 4 This is correct. When attending a premature birth, the most important nursing action is to provide a neutral temperature environment (NTE). The premature neonate is at risk for increased loss of heat because of diminished amounts of subcutaneous fat. The nurse needs to take measures to prevent cold stress, which can be fatal.

The NICU nurse encourages the mother of a premature neonate to bring breast milk to the unit for enteral feedings to her baby. For which reason does the nurse make this suggestion? 1. The baby will be more likely to breastfeed later. 2. The mother will feel more involved with the baby. 3. The neonate will gain weight faster on breast milk. 4. Breast milk helps prevent necrotizing enterocolitis.

ANS: 4 Chapter: Chapter 17 High-Risk Neonatal Nursing Care Chapter Learning Objective: 2. Identify critical elements of assessment and nursing care of the high-risk neonate. Page: 510 Heading: Premature Neonates > Nursing Actions Integrated Processes: Nursing Process Client Need: Physiological Integrity: Reduction of Risk Potential Cognitive Level: Application [Applying] Concept: Ante/Intra/Post-partum Difficulty: Moderate Feedback 1 This is incorrect. The premature neonate is more likely to breastfeed later if the mother and neonate are able to breastfeed before discharge from NICU. 2 This is incorrect. Bringing breast milk to her neonate is likely to make the mother feel involved. However, the mother can be involved regardless of the source of nutrition. 3 This is incorrect. Human milk requires fortification because it does not provide the calories, protein, fat, carbohydrate, potassium, calcium, sodium, and phosphorus that the premature infant needs 4 This is correct. It is a known fact that babies fed on breast milk are less likely to develop necrotizing enterocolitis.

The nurse is developing a plan of care for the 65-year-old obese female patient who states she wishes she could lose the weight but has no stamina for activity. Which nursing intervention would best correlate with the patient's statements? 1. Provide information on local gyms and exercise groups. 2. Provide information on choosemyplate.gov. 3. Provide information on the overall health risks of obesity. 4. Provide guidelines on how to increase daily activity as tolerated.

ANS: 4 Chapter: Chapter 18 Well Women's Health Chapter Learning Objective: 3. Describe how lifestyle factors such as diet, exercise, and cigarette smoking influence women's health. Page: 568 Heading: Specific Populations > Geriatric Health > Age-Related Physiological Changes and Health Promotion Integrated Processes: Teaching and Learning Client Need: health promotion and maintenance Cognitive Level: Application [Applying] Concept: Promoting Health Difficulty: Moderate Feedback 1 This is incorrect. Although the patient may request this later, this is not the best answer. 2 This is incorrect. This would facilitate improved nutrition. The patient did not request this. 3 This is incorrect. This would be education on obesity, not inactivity, which may increase the likelihood of obesity. 4 This is correct. By facilitating an incremental increase in activity, the patient's stamina may increase, which was the statement made by the patient.

The nurse is assessing a 70-year-old female patient in the family practice clinic. The patient states that she has been experiencing sudden heavy vaginal bleeding the past 3 months. What is the expected diagnostic procedure for this patient? 1. Cervical conization 2. Colposcopy 3. Dilation and curettage 4. Endometrial biopsy

ANS: 4 Chapter: Chapter 19 Alterations in Women's Health Chapter Learning Objective: 1. Describe diagnostic procedures commonly used in women's health care. Page: 576 Heading: Table 19-1: Common Diagnostic Procedures Integrated Processes: Nursing Process Client Need: Safe and Effective Care Environment: Coordinated Care. Cognitive Level: Analysis [Analyzing] Concept: Patient-Centered Care Difficulty: Difficult Feedback 1 This is incorrect. Cervical conization would be indicated for an abnormal Pap smear, but there is no indication of any cervical intraepithelial lesions. 2 This is incorrect. There is no indication of dysplasia or an abnormal Pap smear. This would be used to rule out cancer of the cervix. 3 This is incorrect. Although this could be used to detect uterine malignancy or to evaluate dysfunction uterine bleeding, this would not be indicated as a first step in diagnostics. 4 This is correct. The patient will need to have an endometrial biopsy to determine the cause of postmenopausal bleeding. The patient's age is past the average age of menopause.

The nurse in an OB's office is evaluating a patient who exhibits vaginal bleeding at 30 weeks gestation. The patient is prescribed bedrest at home and instructed to avoid lifting. The patient states, "I cannot go to bed, I have an 18-month-old at home." On which topic of the Jonsen model for ethical decision making will the nurse focus? 1. Contextual features 2. Quality of life 3. Patient preferences 4. Medical indications

ANS: 4 Chapter: Chapter 2 Ethics and Standards of Practice Issues Chapter Learning Objective: 2. Debate ethical issues in maternity nursing. Page: 25 Heading: Ethics in Nursing Practice > Ethical Decision-Making Models > Medical Indications Integrated Processes: Nursing Process Client Need: Safe and Effective Care Environment: Management of Care Cognitive Level: Application [Applying] Concept: Ethics Difficulty: Moderate Feedback 1 This is incorrect. Contextual features are determined by multiple social factors, including, but not limited to, family dynamics, living situation of the patient, and cultural and religious beliefs of the patient and family. This topic does not specifically address the well-being of the patient and fetus. 2 This is incorrect. Quality of life requires consideration of how the proposed treatment will affect the patient's quality of life. This topic does not specifically address the well-being of the patient and fetus. 3 This is incorrect. Patient preference focuses on the expressed or presumed wishes of the patient, and includes respect for the patient's autonomy. This topic does not specifically address the well-being of the patient and fetus. 4 This is correct. Medical indications involve medical facts, including diagnosis, prognosis, treatment options, and how the patient can benefit, if at all, from the prescribed treatment. Of all four topics, this is the one that specifically addresses the well-being of the patient and the fetus.

The nurse works in the maternal-newborn unit dedicated to management of high-risk pregnancy and delivery. A patient has delivered two children who died from a genetic disorder. The current pregnancy tests positive for the same disorder. Which ethical decision-making model will the nurse use with this patient? 1. The model that focuses on medical treatment 2. The model that focuses on patient preference 3. The model that focuses on religion and culture 4. The model that focuses on the quality of life

ANS: 4 Chapter: Chapter 2 Ethics and Standards of Practice Issues Chapter Learning Objective: 2. Debate ethical issues in maternity nursing. Page: 25 Heading: Ethics in Nursing Practice > Ethical Decision-Making Models > Quality of Life Integrated Processes: Caring Client Need: Physiological Integrity: Reduction of Risk Potential Cognitive Level: Application [Applying] Concept: Ethics Difficulty: Moderate Feedback 1 This is incorrect. The model that focuses on medical treatment may or may not be applicable for this patient. The patient has already lost two children to the same genetic disorder. Unless a new method of treatment is available, this is not the appropriate decision-making model. 2 This is incorrect. The actual patient in this scenario is uncertain, making the use of the patient preference model questionable. With a genetic disorder, the possibility of a lose-lose outcome is high. 3 This is incorrect. The model that focuses on finances, religion, and culture is based on contextual features. While consideration is necessary, this is not the most appropriate model for the nurse to use in this scenario. 4 This is correct. The most appropriate decision-making model in this scenario is the one that considers the quality of life. The important considerations involve the well-being of both the patient and the unborn fetus.

The nurse works in the labor and delivery department. Which action by the nurse indicates a breach in the nursing care principles outlined by AWHONN? 1. Assists with an emergency delivery of a woman in a homeless shelter 2. Informs employer of religious beliefs against pregnancy termination 3. Suggests a nurse care for a patient who speaks the same native language 4. Declines a patient assignment because of a history of illegal drug use

ANS: 4 Chapter: Chapter 2 Ethics and Standards of Practice Issues Chapter Learning Objective: 3. Explore standards of practice in maternity nursing. Page: 26 Heading: Ethics in Nursing Practice > Ethics and Practice: Nurses' Rights and Responsibilities Integrated Processes: Nursing Process Client Need: Safe and Effective Care Environment: Management of Care Cognitive Level: Analysis [Analyzing] Concept: Professionalism Difficulty: Moderate Feedback 1 This is incorrect. One AWHONN principle states nurses have a responsibility to provide high-quality, professional nursing care to all patients in emergency situations, regardless of the nurse's personal beliefs. The nurse's action is not a breach of this principle. 2 This is incorrect. An AWHONN principle states nurses have a professional obligation to inform their employers of any attitude or belief that can interfere with the nurse's essential job function. The nurse's action is not a breach of this principle. 3 This is incorrect. The nurse's action is not specifically addressed by an AWHONN principle; however, suggesting a nurse care for a patient who speaks a language familiar to the nurse is not inappropriate. An interpreter can still be obtained if needed. 4 This is correct. An AWHONN principle states that the nurse should not abandon a patient or refuse to provide care based on prejudice or bias. The nurse's action is a breach of this principle.

A patient who has just received confirmation that she is pregnant is distressed because she has a seizure disorder that she manages with carbamazepine. Which is the nurse's greatest concern? 1. The carbamazepine may be discontinued. 2. The pregnancy is likely to end with fetal demise. 3. The fetus will experience loss of vision and hearing. 4. Carbamazepine is teratogenic and causes neural and facial defects.

ANS: 4 Chapter: Chapter 3 Genetics, Conception, Fetal Development, and Reproductive Technology Chapter Learning Objective: 2. Identify critical components of conception, embryonic development, and fetal development. Page: 40 Heading: Teratogens Integrated Processes: Nursing Process Client Need: Physiological Integrity: Reduction of Risk Potential Cognitive Level: Analysis [Analyzing] Concept: Ante/Intra/Post-partum Difficulty: Difficult Feedback 1 This is incorrect. Seizure activity can be managed with different medications. This is not the nurse's greatest concern. 2 This is incorrect. Carbamazepine is not noted to cause intrauterine fetal demise; warfarin can be an agent causing fetal demise. 3 This is incorrect. Carbamazepine is not noted to cause vision and hearing loss. Sensory defects are more often related to infections and viruses. 4 This is correct. Carbamazepine is prescribed as an anticonvulsant medication that is classified as a teratogenic drug that causes neural tube defects, craniofacial defects, and intrauterine growth restriction. This is the nurse's greatest concern.

A patient at 13 weeks gestation asks the nurse how her baby is nourished during pregnancy. Which information does the nurse use to explain the process to the mother? 1. Fetal waste products and CO2 pass through the placenta to the mother. 2. The placenta is a special organ developed to create nutrients and oxygen. 3. The mother's blood and fetus's blood mix for an exchange of nutrients. 4. Glucose, amino acids, and oxygen pass through the placenta from mother to baby.

ANS: 4 Chapter: Chapter 3 Genetics, Conception, Fetal Development, and Reproductive Technology Chapter Learning Objective: 3. Describe the development and function of the placenta and amniotic fluid. Page: 49 Heading: Placenta, Membranes, Amniotic Fluid, and Umbilical Cord > Placenta > Function of the Placenta Integrated Processes: Nursing Process Client Need: Physiological Integrity: Physiological Adaptation Cognitive Level: Application [Applying] Concept: Ante/Intra/Post-partum Difficulty: Moderate Feedback 1 This is incorrect. It is true that fetal waste products and CO2 pass through the placenta to the mother for elimination. However, this information does not answer the patient's inquiry about nourishment of the fetus. 2 This is incorrect. The placenta does not create nutrients and oxygen. The placenta is the organ of exchange of those substances from the mother to the fetus. The placenta also produces hormones needed to sustain the pregnancy. 3 This is incorrect. Nutrients and oxygen pass through the placenta membrane from the mother to the fetus through a mechanism of diffuse and active transport. There is no mixing of the maternal and fetal blood. 4 This is correct. Glucose, amino acids, and oxygen are transported across the placenta membrane from the mother to the fetus by a mechanism of diffuse and active transport.

An infertile couple learns that the female is unable to produce viable eggs. The male partner suggests the use of a surrogate as a means of having a child. The female states, "I don't want your baby with another woman!" The nurse is aware of which psychosocial issue with this couple? 1. The male blames the female for the infertility. 2. The female is jealous of the surrogate's fertility. 3. The male needs to have a child with his genes. 4. The female is experiencing self-esteem issues.

ANS: 4 Chapter: Chapter 3 Genetics, Conception, Fetal Development, and Reproductive Technology Chapter Learning Objective: 7. Discuss the ethical and emotional implications of assisted reproductive therapies. Page: 54 Heading: Infertility and Reproductive Technology > Emotional Implications Integrated Processes: Nursing Process Client Need: Psychosocial Integrity Cognitive Level: Analysis [Analyzing] Concept: Self Difficulty: Difficult Feedback 1 This is incorrect. There is no information in the scenario that indicates the male partner is blaming the female for the infertility. 2 This is incorrect. The female may or may not be jealous of the surrogate's fertility. There is a more likely psychosocial issue with the female partner. 3 This is incorrect. There is no information in the scenario that indicates the male partner specifically needs to have a child with his genes. 4 This is correct. The nurse is aware that the diagnosis and treatment of infertility can cause self-esteem issues. The female partner is likely to be feeling "less of a woman" because of the inability to conceive. She may also have feelings of shame for having a "defective" body.

A patient expresses a desire to become pregnant for a second child. The nurse notes that the patient's first child was born with a serious neural tube defect (NTD) and died of complications at 18 months of age. Which recommendation does the nurse make to this client? 1. Folic acid 0.6 mg/day orally 1 month before conception and throughout pregnancy. 2. Folic acid 0.4 mg/day orally started when pregnant and continued throughout pregnancy 3. Folic acid 4 mg/day orally started when pregnant and continued throughout pregnancy 4. Folic acid 4 mg/day orally for 1 month prior to conception through first trimester of pregnancy

ANS: 4 Chapter: Chapter 4 Physiological Aspects of Antepartum Care Chapter Learning Objective: 1. Identify the major components of preconception health care. Page: 72 Heading: Preconception Health Care > Preconception Education > Prenatal Vitamins Integrated Processes: Nursing Process Client Need: Physiological Integrity: Reduction of Risk Potential Cognitive Level: Application [Applying] Concept: Ante/Intra/Post-partum Difficulty: Difficult Feedback 1 This is incorrect. Folic acid at a dose of 0.6 mg/day is recommended for women who have not delivered a neonate with a neural tube defect (NTD). 2 This is incorrect. After a patient delivers a neonate with an NTD, a dosage of 0.4 mg/day is prescribed for the second and third trimesters. 3 This is incorrect. For a patient who has delivered a neonate with NTD, folic acid is prescribed at 4 mg/day only for 1 month prior to conception and through the first trimester. It is not necessary to continue this dosage through the entire pregnancy. 4 This is correct. The correct dose of folic acid for the patient who previously delivered a neonate with NTD is 4 mg/day for 1 month prior to conception, which is continued through the first trimester. The dose is then reduced to 0.4 mg/day for the remainder of the pregnancy.

A patient arrives at a maternal health client and tells the nurse she has missed a period and thinks she is pregnant. Which information shared with the nurse is a presumptive sign of pregnancy? 1. Positive results on a home pregnancy test 2. Breast enlargement, tenderness, and tingling 3. First awareness of fetal movements 4. Increased appetite

ANS: 4 Chapter: Chapter 4 Physiological Aspects of Antepartum Care Chapter Learning Objective: 2. Describe methods for diagnosis of pregnancy and determination of estimated date of delivery. Page: 75 Heading: Pregnancy > Due Date Calculation Integrated Processes: Nursing Process Client Need: Health Promotion and Maintenance Cognitive Level: Application [Applying] Concept: Ante/Intra/Post-partum Difficulty: Moderate Feedback 1 This is incorrect. Naegele's rule requires counting back 3 months from the LMP and adding 7 days. This answer indicates that 3 months forward were considered. 2 This is incorrect. Naegele's rule requires counting back 3 months from the LMP and adding 7 days. In this calculation, 7 days were not added to the LMP. 3 This is incorrect. Naegele's rule requires counting back 3 months from the LMP and adding 7 days. Both the month and the date were miscalculated in this option. 4 This is correct. Naegele's rule requires counting back 3 months from the LMP and adding 7 days. This is the correct calculation and EDD.

A patient who is pregnant does not remember the last date of her menstrual period. In which manner does the nurse expect the estimated date of delivery (EDD) to be determined for this patient? 1. Having an ultrasound examination 2. Using the gestational wheel 3. Asking when previous babies were born 4. Obtaining a history of gestational length

ANS: 4 Chapter: Chapter 4 Physiological Aspects of Antepartum Care Chapter Learning Objective: 6. Identify the critical elements of assessment and nursing care during initial and subsequent prenatal visits. Page: 80 Heading: Antepartal Nursing Care: Physiology-Based Nursing Assessment and Nursing Actions > Prenatal Assessment > Nursing Actions Integrated Processes: Caring Client Need: Physiological Integrity: Reduction of Risk Potential Cognitive Level: Analysis [Analyzing] Core concept: Ante/Intra/Post-partum Difficulty: Moderate Feedback 1 This is incorrect. AWHONN's view about being alone does not focus on arranging for someone to stay with the patient; the focus is on relaying that the patient is not alone in her situation. Many women experience abuse and receive protection from agencies and individuals. 2 This is incorrect. Stating that the patient's partner needs to come to the office in order to have his behavior confronted is a potential breach in confidentiality. This action is not supported by AWHONN. 3 This is incorrect. The patient needs to be aware of agencies that are available to protect her safety. However, beyond providing the contact information, the nurse is not the decision maker for moving to a shelter. 4 This is correct. AWHONN promotes safety, support, education, and confidentiality as part of the interventions to protect the woman who is experiencing partner abuse; this statement covers the patient's needs.

The nurse is providing prenatal care for a patient who is pregnant with a second child. Which understanding about complexity of a second pregnancy does the nurse use to assist the patient with the acceptance of this pregnancy? 1. Point out that the financial obligation is always less with a second child. 2. Make suggestions of how the first child will be a "helper" with the new baby. 3. Recommend career decisions needed because of additional parenting tasks. 4. Offer strategies for working out a new relationship with the first child.

ANS: 4 Chapter: Chapter 5 The Psycho-Social-Cultural Aspects of the Antepartum Period Chapter Learning Objective: 3. Identify critical variables that influence adaptation to pregnancy, including age, parity, and social, cultural, and sexual orientation. Page: 104 Heading: Maternal Adaptation to Pregnancy > Factors That Influence Maternal Adaptation > Multiparity Integrated Processes: Nursing Process Client Need: Psychosocial Integrity Cognitive Level: Application [Applying] Concept: Family Difficulty: Moderate Feedback 1 This is incorrect. Pregnancy tasks may be more complex for the multipara. The nurse is not necessarily knowledgeable about the financial obligations related to a second child. Income can be affected by a second child, and the needs of the second child may be greater for a variety of reasons. 2 This is incorrect. Nursing knowledge about how to make the first child a "helper" with the new baby may not be appropriate. The first child's age and demeanor need to be considered. 3 This is incorrect. It is not the responsibility of the nurse to recommend career changes to a patient expecting a second child. 4 This is correct. Pregnancy tasks are more complex for the multipara due to the relationship between the mother and the first child. The nurse can offer strategies for remodeling this relationship and help the mother with feelings associated with the needed change.

A pregnant patient and her spouse live in the same home as the spouse's family who is not supportive of the pregnancy. The patient feels the family is ruining the happiness about the pregnancy. Which is the most important determination for the nurse to make? 1. What the potential for improving the current support network is 2. Who will provide the patient the greatest amount of support 3. Whether the couple's finances support moving into a separate location 4. If threatened or actual abuse from household members occur toward the patient

ANS: 4 Chapter: Chapter 5 The Psycho-Social-Cultural Aspects of the Antepartum Period Chapter Learning Objective: 7. Analyze and critique current evidence-based research in psycho-social-cultural adaptation to pregnancy. Page: 117 Heading: Social Support During Pregnancy > Assessing Social Support Integrated Processes: Nursing Process Client Need: Psychosocial Integrity Cognitive Level: Analysis [Analyzing] Concept: Family Dynamics Difficulty: Difficult Feedback 1 This is incorrect. The nurse does eventually need to examine the potential for improving the current social network. However, another option is most important. 2 This is incorrect. It is helpful for the nurse to know the persons who provide the patient with the most support. However, another option is more important. 3 This is incorrect. The nurse needs to know if the couple's financial situation will support them moving to a separate location. However, there is another option that is most important. 4 This is correct. The most important determination for the nurse to make in this scenario is whether the patient is experiencing a threat or actual abuse from the family members.

A patient has experienced an uneventful pregnancy but begins to have vaginal spotting at 38 weeks gestation. The health care provider (HCP) suspects placenta previa initiated by cervical thinning. Which testing does the nurse expect the HCP to schedule? 1. Doppler flow studies 2. Nonstress testing 3. Magnetic resonance imaging 4. Ultrasonography studies

ANS: 4 Chapter: Chapter 6 Antepartal Tests Chapter Learning Objective: 2. Identify the purpose and indication for key antenatal tests. Page: 134 Heading: Biophysical Assessment > Fetal Ultrasound Imaging Integrated Processes: Nursing Process Client Need: Physiological Integrity: Reduction of Risk Potential Cognitive Level: Analysis [Analyzing] Concept: Ante/Intra/Post-partum Difficulty: Moderate Feedback 1 This is incorrect. Doppler flow studies are performed to evaluate placental profusion. The study does not validate placental placement. 2 This is incorrect. Nonstress testing is used to determine fetal well-being and measures cardiac function during fetal movement or contractions. 3 This is incorrect. Magnetic resonance imaging (MRI) is performed to obtain detailed imaging when screening tests indicate possible abnormalities. It is most commonly performed for suspected brain abnormality. 4 This is incorrect. Ultrasonography studies are appropriate in determining placental placement and possible abnormalities.

The nurse is assisting a patient who is pregnant to prepare for an MRI scheduled to assess fetal brain development. Which situation causes the nurse to notify the radiology department personnel? 1. The patient had breakfast before the test. 2. The patient reports having an iodine allergy. 3. The patient expresses concern about pain. 4. The patient has a permanent body piercing.

ANS: 4 Chapter: Chapter 6 Antepartal Tests Chapter Learning Objective: 4. Articulate the nursing responsibilities related to key antenatal tests. Page: 137 Heading: Biophysical Assessment > Fetal Ultrasound Imaging > Magnetic Resonance Imaging Integrated Processes: Nursing Process Client Need: Physiological Integrity: Reduction of Risk Potential Cognitive Level: Application [Applying] Concept: Ante/Intra/Post-partum Difficulty: Moderate Feedback 1 This is incorrect. Patients do not need to be NPO prior to MRI testing. This is not a reason for the nurse to notify the radiology department. 2 This is incorrect. An MRI can be performed without the injection of iodinated contrast. The patient's allergy to iodine is not a reason for the nurse to notify the radiology department. 3 This is incorrect. Pain is not associated with MRI testing. There is no reason to notify the radiology department; however, the nurse needs to provide patient teaching. 4 This is correct. Part of the preparation for an MRI is to have the patient remove all metallic objects before the testing. The fact that the patient has a permanent body piercing will present a problem. The nurse needs to notify the radiology department for the situation.

A patient at 30 weeks gestation is exhibiting signs of preterm labor and delivery. The health care provider (HCP) informs the patient that nothing can be done to disrupt this process. The patient is in distress and states, "Why can't something be done to save my baby?" The nurse understands the HCP's decision is likely based on which finding? 1. Ruptured membranes caused an infection. 2. The patient is unstable due to hemorrhage. 3. Fetal age is incompatible with survival. 4. A fetal heartbeat could not be obtained.

ANS: 4 Chapter: Chapter 7 High-Risk Antepartum Nursing Care Chapter Learning Objective: 1. Describe the primary complications of pregnancy and related nursing and medical care. Page: 156 Heading: Gestational Complications > Preterm Labor and Birth > Contraindications Integrated Processes: Nursing Process Client Need: Physiological Integrity: Reduction of Risk Potential Cognitive Level: Analysis [Analyzing] Concept: Ante/Intra/Post-partum Difficulty: Moderate Feedback 1 This is incorrect. There is no mention of ruptured membranes or infection in the scenario. 2 This is incorrect. There is no mention of maternal hemorrhage in the scenario. 3 This is incorrect. At 30 weeks gestation, the fetus has a possibility of survival. 4 This is correct. The inability to obtain a fetal heart beat is indicative of fetal demise, which does not support measures to stop preterm labor and delivery.

The nurse in a prenatal clinic is assessing a patient who is at 37 weeks gestation for twins. The patient reports increased discomfort and increased lower pelvic pressure. Which action does the nurse take with this patient? 1. After examination, assures the patient of the absence of contractions 2. Explains to the patient that increased discomfort is expected with twins 3. Performs a digital cervical examination to determine if dilation is occurring 4. Sends the patient to the hospital to be checked for possible signs of labor

ANS: 4 Chapter: Chapter 7 High-Risk Antepartum Nursing Care Chapter Learning Objective: 2. Delineate clinical features indicative of pregnancy complications and tests to predict, screen for, diagnose, and manage high-risk conditions. Page: 164 Heading: Gestational Complications > Multiple Gestation > Intrapartum Medical Management Integrated Processes: Nursing Process Client Need: Physiological Integrity: Reduction of Risk Potential Cognitive Level: Analysis [Analyzing] Concept: Ante/Intra/Post-partum Difficulty: Moderate Feedback 1 This is incorrect. It is difficult to determine the presence of uterine contractions for a patient pregnant with twins because of more discomfort, stretching, and pressure with multiple fetuses, and over-distention of the uterus results in more uterine irritability. Due to the circumstances, this is a questionable action by the nurse. 2 This is incorrect. Increased discomfort is expected with multiple gestations; however, the patient is reporting changes and the nurse needs to consider the possibility of labor occurring. 3 This is incorrect. With multiple fetuses, the performance of a digital cervical examination can cause rupture of the membranes. The procedure should be avoided unless labor is confirmed. 4 This is correct. With so many variables related to multiple gestation, the nurse needs to send the patient to the hospital to be checked for possible signs of labor.

The nurse is interviewing a new patient who is in the first trimester of her second pregnancy. The patient shares that her first child was born at 36 weeks gestation. Which information does the patient share that places the patient at risk for a second premature birth? 1. The first labor was induced due to unresponsive management of hypertension. 2. The health care provider induced labor at the patient's request to avoid holiday interruptions. 3. Labor was induced when the fetus moved from a posterior to an anterior position. 4. The premature labor and birth was unexpected and followed a normal pregnancy.

ANS: 4 Chapter: Chapter 7 High-Risk Antepartum Nursing Care Chapter Learning Objective: 6. Demonstrate understanding of knowledge related to preexisting medical complications of pregnancy and related management. Page: 151 Heading: Gestational Complications > Preterm Labor and Birth Integrated Processes: Nursing Process Client Need: Physiological Integrity: Reduction of Risk Potential Cognitive Level: Analysis [Analyzing] Concept: Ante/Intra/Post-partum Difficulty: Moderate Feedback 1 This is incorrect. When labor is induced early in response to a medical condition, the premature birth is a medical decision. Unresponsive management of hypertension is indicative of a risk for preeclampsia/eclampsia. The health care provider makes a determination of the action necessary to benefit the mother and fetus the most. This condition will not necessarily occur during the second pregnancy. 2 This is incorrect. When a premature labor and birth are the result of nonmedically indicated (elective) preterm delivery, the second pregnancy is not expected to end with premature labor and birth. Elective preterm deliveries are not encouraged and should be delayed until at least 39 weeks gestation. 3 This is incorrect. When preterm labor is induced because of a fetus change in position, the decision falls somewhere between medical and elective. This condition will not necessarily occur during the second pregnancy. 4 This is correct. When premature labor and birth occurs spontaneously, as this one did, there is a good chance the second pregnancy will end in the same manner. A history of delivering preterm is one of the strongest predictors for subsequent preterm births.

The nurse is caring for a 34-year-old woman who is G2P1001 at 39 weeks and 1 day. The patient is an observant Muslim and is in active labor. Which outcome would be the most effective form of applying culturally sensitive communication? 1. Ensuring that the health care provider is present every 2 hours to examine labor progress 2. Making sure that the patient has Kosher meat for her postpartum meal 3. Making sure that the patient's hair is covered constantly 4. Assuring the patient that her baby is less than 9 lbs

ANS: 4 Chapter: Chapter 8 Intrapartum Assessment and Interventions Chapter Learning Objective: Demonstrate understanding of supportive care of the laboring woman. Page: 226 Heading: Factors Affecting Labor > Psyche > Culture Integrated Processes: Caring Client Need: Psychosocial Integrity Cognitive Level: Application [Applying] Concept: Ante/Intra/Post-partum Difficulty: Moderate Feedback 1 This is incorrect. The health care providers' presence influences the women's psyche but is not culturally sensitive communication, which acknowledges the nurse is willing to learn. 2 This is incorrect. Eating affects energy in labor, but it is not usually psyche, and Kosher meat is appropriate for an observant Jewish patient. Muslim patients eat Halal meat. 3 This is correct. Modesty is influenced by culture. 4 This is incorrect. The estimated size of the baby may affect the psyche of the patient, but it is not usually related specifically to culture.

The nurse is counseling a 15-year-old pregnant patient at her first prenatal appointment. Which patient teaching by the nurse is appropriate? 1. "Adolescents usually have very effective coping mechanisms." 2. "Teenagers always have larger babies than older patients." 3. "Adolescents' ego integrity is less easily threatened by stress and pain." 4. "Your response to pregnancy, labor, and delivery will differ from that of an adult because your cognitive development is not yet complete."

ANS: 4 Chapter: Chapter 8 Intrapartum Assessment and Interventions Chapter Learning Objective: Demonstrate understanding of supportive care of the laboring woman. Page: 228 Heading: Critical Component > Supportive Care for Adolescents in Labor Integrated Processes: Teaching and Learning Client Need: Safe and Effective Care Environment: Management of Care Cognitive Level: Application [Applying] Concept: Ante/Intra/Post-partum Difficulty: Moderate Feedback 1 This is incorrect. According to AWHONN, adolescents tend to have fewer coping mechanisms than adults. 2 This is incorrect. There is no evidence that teenagers have larger babies. 3 This is incorrect. According to AWHONN, ego integrity of adolescents is more easily threatened by stress and pain. 4 This is correct. According to AWHONN, teenagers have incomplete cognitive development.

The nurse educator is in a childbirth education class discussing nonpharmacological ways nurses can assist to enhance labor and spontaneous vaginal delivery. Which of the following facts regarding nonpharmacologic approaches will the nurse use to help illustrate its benefits? 1. Using peppermint aromatherapy oils will help promote relaxation. 2. The client can achieve self-hypnosis with the Bradley Method. 3. Sterile water injections can be very useful for pelvic pain. 4. Effleurage is performed in rhythm with breathing during a contraction.

ANS: 4 Chapter: Chapter 8 Intrapartum Assessment and Interventions Chapter Learning Objective: Describe the mechanism of spontaneous vaginal delivery and related nursing care. Page: 257 Heading: Non-Pharmacological Management of Labor Discomfort Integrated Processes: Caring Client Need: Health Promotion and Maintenance Cognitive Level: Application [Applying] Concept: Ante/Intra/Post-partum Difficulty: Moderate Feedback 1 This is incorrect. Although aromatherapy is a nonpharmacologic method of promoting relaxation and decreasing pain perception, peppermint is used to help with nausea, not to enhance labor and spontaneous vaginal delivery. 2 This is incorrect. Although the Bradley Method is a nonpharmacologic way to help manage labor pain; it is husband-coached. 3 This is incorrect. Sterile water injections are used for lower back pain relief. 4 This is correct. Effleurage is cutaneous stimulation by lightly stroking the maternal abdomen in rhythm with breathing during contractions.

The nurse-educator is instructing on the physiology of fetal heart rate (FHR) patterns. He is showing the students an EFM strip, and there is a tracing that is classified as baseline 140 bpm, moderate variability, accelerations, and 2 decelerations. A half hour later the baseline is 150 bpm, there is minimal variability, accelerations, and 3 decelerations. Which of these findings would the nurse attribute to the parasympathetic nervous system? 1. The baseline changes from 140 bpm to 150 bpm. 2. The change from moderate variability to minimal variability. 3. The consistent presence of accelerations. 4. The presence of 2 and then 3 decelerations.

ANS: 4 Chapter: Chapter 9 Fetal Heart Rate Assessment Chapter Learning Objective: Articulate the physiology of FHR patterns. Page: 279 Heading: Influences on Fetal Heart Rate > Automatic Nervous System Integrated Processes: Nursing Process Client Need: Safe and Effective Care Environment: Management of Care Cognitive Level: Application (Applying) Concept: Ante/Intra/Post-partum Difficulty: Moderate Feedback 1 This is incorrect. The sympathetic nervous system is responsible for increasing the FHR and FHR variability. 2 This is incorrect. The central nervous system is responsible for variations in FHR and variability related to fetal activity. 3 This is incorrect. The sympathetic nervous system is responsible for increasing FHR. 4 This is correct. The parasympathetic nervous system is responsible for slowing the FHR and maintaining variability.

The nurse is monitoring a patient when the EFM strip conveys fetal bradycardia. Which action would be the most urgent for the nurse to take? 1. Check the patient's input and output. 2. Take a blood pressure to determine if the mother has hypotension. 3. Change the mother's position from supine to left lateral. 4. Check the mother for vaginal bleeding and severe abdominal pain.

ANS: 4 Chapter: Chapter 9 Fetal Heart Rate Assessment Chapter Learning Objective: Articulate the physiology of FHR patterns. Page: 285 Heading: FHR and Contraction Pattern Interpretation > Baseline Fetal Heart Rate Integrated Processes: Nursing Process Client Need: Physiological Integrity: Reduction of Risk Potential Cognitive Level: Application [Applying] Concept: Ante/Intra/Post-partum Difficulty: Moderate Feedback 1 This is incorrect. Dehydration is a possible cause of fetal bradycardia but is not the most urgent action. 2 This is incorrect. Hypotension is a possible cause of fetal bradycardia but is not the most urgent action. 3 This is incorrect. Supine position is a possible cause of fetal bradycardia, so it may help to place the patient in a left lateral position, but it is not the most urgent action.

The obstetric nurse is managing her patients while covering for another nurse who is on a break. Which patient is the lowest priority? 1. A patient with a previous cesarean section 2. A patient with an epidural in place 3. A patient with decreased fetal activity 4. A patient with Category I FHR tracings

ANS: 4 Chapter: Chapter 9 Fetal Heart Rate Assessment Chapter Learning Objective: Describe the components of fetal heart rate (FHR) and uterine contraction (UC) patterns essential to interpretation of monitor strips. Page: 277 Heading: AWHONN Standards for Frequency of Assessment of FHR Integrated Processes: Caring Client Need: Physiological Integrity: Reduction of Risk Potential Cognitive Level: Application [Applying] Concept: Ante/Intra/Post-partum Difficulty: Difficult Feedback 1 This is incorrect. A patient with a previous cesarean section needs to be monitored every 30 minutes during the latent phase, every 15 minutes during the active phase, and every 5 minutes during the second stage. 2 This is incorrect. A patient with an epidural in place needs to be monitored every 30 minutes during the latent phase, every 15 minutes during the active phase, and every 5 minutes during the second stage. 3 This is incorrect. A patient with decreased fetal activity needs to be monitored every 30 minutes during the latent phase, every 15 minutes during the active phase, and every 5 minutes during the second stage. 4 This is correct. This patient is low risk.

The nurse is explaining telemetry to the patient, who has just begun active labor. The patient would like to have a labor in which she is mobile, able to change positions, and use hydrotherapy. Which response by the nurse is most appropriate? 1. "Telemetry is used mostly for women who are laboring in bed and changing positions every half hour or so." 2. "Unfortunately, you will not be able to use the shower while using telemetry." 3. "The nurses will need to come in and check your telemetry reading every half hour." 4. "We can start using telemetry now, and if there are no problems with the signal, we can continue it throughout your labor until delivery."

ANS: 4 Chapter: Chapter 9 Fetal Heart Rate Assessment Chapter Learning Objective: Describe the components of fetal heart rate (FHR) and uterine contraction (UC) patterns essential to interpretation of monitor strips. Page: 277 Heading: Modes or Types of Fetal and Uterine Monitoring > Telemetry Integrated Processes: Nursing Process Client Need: Safe and Effective Care Environment: Management of Care Cognitive Level: Application [Applying] Concept: Patient-Centered Care Difficulty: Moderate Feedback 1 This is incorrect. The patient will be able to walk while using telemetry. 2 This is incorrect. The patient will be able to bathe while using telemetry. 3 This is incorrect. Nurses can assess the patient remotely. 4 This is correct. Telemetry can be used in all phases of labor.

The EFM tracing shows the following: FHR baseline 166 bpm, moderate variability, and recurrent late decelerations to 100 bpm. Using the five-tier FHR interpretation system, how should the nurse interpret this tracing? 1. Green: very low risk of evolution, no action 2. Red: unacceptably high risk of acidemia, deliver 3. Yellow: moderate risk of evolution, increase surveillance 4. Orange: acceptable low risk of acidemia, prepare for possible urgent delivery

ANS: 4 Chapter: Chapter 9 Fetal Heart Rate Assessment Chapter Learning Objective: Describe the components of fetal heart rate (FHR) and uterine contraction (UC) patterns essential to interpretation of monitor strips. Page: 282 Heading: Fetal Reserves > NICHD Criteria for Interpretation of FHR Patterns Integrated Processes: Nursing Process Client Need: Physiological Integrity: Reduction of Risk Potential Cognitive Level: Application [Applying] Concept: Ante/Intra/Post-partum Difficulty: Difficult Feedback 1 This is incorrect. Based on the five-tier FHR interpretation system, this tracing is classified as orange, which means there is an acceptable low risk of acidemia and the nurse should prepare for possible urgent delivery. 2 This is incorrect. Based on the five-tier FHR interpretation system, this tracing is classified as orange, which means there is an acceptable low risk of acidemia and the nurse should prepare for possible urgent delivery. 3 This is incorrect. Based on the five-tier FHR interpretation system, this tracing is classified as orange, which means there is an acceptable low risk of acidemia and the nurse should prepare for possible urgent delivery. 4 This is correct. Based on the five-tier FHR interpretation system, this tracing is classified as orange, which means there is an acceptable low risk of acidemia and the nurse should prepare for possible urgent delivery.

An internal fetal monitor has been ordered for Chrissy, a 24-year-old G2P0010 at 38 weeks and 1 day gestation. Her medical history is significant for a history of pregnancy-induced hypertension. Her laboratory values are as follows: H/H 11/30, O negative, RPR negative, GBS positive. Based on Chrissy's history and presentation, what action should the nurse take next? 1. Prepare Chrissy for the placement of an internal monitor. 2. Take the required two blood pressure readings every 15 minutes prior to insertion of the internal fetal monitor due to her pregnancy-induced hypertension. 3. Discuss with the health care provider the fact that Chrissy's blood type is O negative and she should therefore receive Rhogam before insertion of an internal monitor. 4. Discuss with the health care provider that Chrissy is GBS positive and therefore should not receive an internal monitor.

ANS: 4 Chapter: Chapter 9 Fetal Heart Rate Assessment Chapter Learning Objective: Identify the modes of fetal heart rate assessment: auscultation, palpation, EFM. Page: 276 Heading: Modes or Types of Fetal and Uterine Monitoring > Internal Electronic Fetal and Uterine Monitoring Integrated Processes: Nursing Process Client Need: Physiological Integrity: Reduction of Risk Potential Cognitive Level: Application [Applying] Concept: Ante/Intra/Post-partum Difficulty: Moderate Feedback 1 This in incorrect. Chrissy is GBS positive and therefore cannot receive an internal monitor. 2 This is incorrect. There are no required blood pressure readings for a pregnancy-induced hypertensive patient prior to the insertion of an internal monitor. 3 This is incorrect. Chrissy does not need to receive Rhogam prior to the insertion of an internal monitor. She should have received it earlier in the pregnancy, but it should have no effect on an internal monitor. 4 This is correct. GBS positivity is a contraindication to the placement of an internal monitor.

The nurse is explaining to the new breastfeeding mother the types of neonatal stools the mother can expect. Which examples does the nurse provide? Select all that apply. 1. Residual meconium is passed as loose watery stool. 2. Sticky, thick, black stools indicate a presence of blood. 3. Stools will eventually become drier and more formed. 4. Golden yellow, a pasty consistency, and sour odor is expected. 5. Neonate's first stool is passed within the first 24 to 48 hours.

ANS: 4, 5 Chapter: Chapter 15 Physiological and Behavioral Responses of the Neonate Chapter Learning Objective: 1. Identify the changes that occur during the transition from intrauterine to extrauterine life and the related nursing actions. Page: 449 Heading: Transition to Extrauterine Life > Gastrointestinal System Integrated Processes: Nursing Process Client Need: Physiological Integrity: Physiological Adaptation Cognitive Level: Analysis [Analyzing] Concept: Ante/Intra/Post-partum Difficulty: Difficult Feedback 1 This is incorrect. Residual meconium is not passed as loose, watery stool. Diarrheal stools are noted to be loose and green in color. 2 This is incorrect. Sticky, thick, black stools do not indicate a presence of blood. The description fits normal meconium, which is the first stool eliminated by the neonate. It is sticky, thick, black, and odorless. 3 This is incorrect. Stools will eventually become drier and more formed when the neonate is fed formula. 4 This is correct. The stool of a breastfed baby later becomes a golden yellow with a pasty consistency and a sour odor. 5 This is correct. Meconium stool begins to form during the fourth gestational month and is the first stool eliminated by the neonate. It is first passed within 24 to 48 hours.

A patient in the third trimester of pregnancy reports having heartburn nearly every day. Which recommendations does the nurse make to alleviate the problem? Select all that apply. 1. Consume three moderate-sized meals daily. 2. Sip clear, carbonated beverages when eating. 3. Assume a low Fowler position after meals. 4. Avoid eating 3 hours prior to bedtime. 5. Avoid consuming spicy, fatty, or fried food.

ANS: 4, 5 Chapter: Chapter 4 Physiological Aspects of Antepartum Care Chapter Learning Objective: 5. Describe appropriate interventions to relieve common discomforts of pregnancy. Page: 88 Heading: Table 4-5: Self-Care/Relief Measures for Physical Changes and Common Discomforts of Pregnancy Integrated Processes: Nursing Process Client Need: Health Promotion and Maintenance Cognitive Level: Analyzing [Analysis] Core concept: Ante/Intra/Post-partum Difficulty: Difficult Feedback 1 This is incorrect. Heartburn during the third trimester of pregnancy is managed by eating small, frequent meals. Eating three moderate-sized meals daily is a normal eating pattern and will not be effective in managing the discomfort of heartburn. 2 This is incorrect. When a patient experiences heartburn in the third trimester of pregnancy, fluid intake should be avoided during meals. 3 This is incorrect. If a patient experiences heartburn in the third trimester of pregnancy, the patient needs to remain upright for 30 to 45 minutes after eating; a low Fowler position will only increase the incidence of discomfort. 4 This is correct. When a patient experiences heartburn in the third trimester of pregnancy, the patient should avoid eating at least 3 hours prior to bedtime. 5 This is correct. Spicy, fatty, and/or fried foods can contribute to heartburn, especially in the patient who is in the third trimester of pregnancy. These foods need to be eliminated from the diet.

A patient is interested in a pain relief option that she can control during labor. The nurse explains to the patient what pain relief options are available that would meet her needs, and they mutually decide on nitrous oxide. Which of the following does the nurse describe to the patient about the use of nitrous oxide in labor? Select all that apply. 1. "If started, it should be used throughout the entire labor." 2. "It has no trade name and is only available in generic forms." 3. "The nurses will administer it to you during every contraction so we will need to be with you during the entire labor." 4. "The gas takes effect in about 50 seconds after the first breath, and the patient will feel calm." 5. "There is no effect on the labor progress from active labor to delivery, and therefore you do not need to worry about longer labors with the medication."

ANS: 4, 5 Chapter: Chapter 8 Intrapartum Assessment and Interventions Chapter Learning Objective: Demonstrate understanding of supportive care of the laboring woman. Page: 258 Heading: Pharmacological Management of Labor Discomfort > Nitrous Oxide Integrated Processes: Teaching and Learning Client Need: Physiological Integrity: Pharmacological and Parenteral Therapies Cognitive Level: Application [Applying] Concept: Ante/Intra/Post-partum Difficulty: Moderate Feedback 1 This is incorrect. It can be started and stopped at any point in labor. 2 This is incorrect. It is called by the trade name Nitronox. 3 This is incorrect. It is self-administered by the patient. 4 This is correct. It takes effect in about 50 seconds after the first breath, and the effect is transient. 5 This is correct. It has no adverse effects on the normal physiology and progress of labor.

A patient arrives at labor and delivery for the induction labor for her first child. The patient tells the nurse, "I can't believe how easy this is just to pick a day, sign a paper, and have a baby." Which action does the nurse take before the induction process? 1. Call the health care provider to validate patient understanding. 2. Check the patient's chart for an informed consent. 3. Explain the possible complications of induction to the patient. 4. Report an incidence of probable malpractice by the health care provider.

NS: 1 Chapter: Chapter 10 High-Risk Labor and Birth Chapter Learning Objective: 2. Demonstrate understanding of knowledge related to induction of labor and augmentation of labor and vaginal birth after cesarean birth. Page: 309 Heading: Box 10-2: Criteria, Indications, and Contraindications for Labor Induction and Cervical Ripening Integrated Processes: Nursing Process Client Need: Physiological Integrity: Reduction of Risk Potential Cognitive Level: Analysis [Analyzing] Concept: Ante/Intra/Post-partum Difficulty: Difficult Feedback 1 This is correct. The nurse needs to call the health care provider and validate patient understanding about potential risks and complications related to labor induction. 2 This is incorrect. The nurse can check the patient's chart for a signed consent; however, the patient verbally states a paper was signed. There is a more appropriate nursing action. 3 This is incorrect. It is not within the scope of nursing for the nurse to explain possible complications of any procedure to a patient. Medical information can be reinforced by the nurse. 4 This is incorrect. It is not appropriate at this time to report a probable malpractice incidence. The nurse has an obligation to contact the health care provider.

An emergency cesarean is being implemented. The patient describes tingling in her ears and a metallic taste with the administration of regional anesthesia. The nurse is aware that which incidence has occurred? 1. Manifestation of maternal respiratory depression related to anesthesia 2. Inadvertent injection of the anesthetic agent into the maternal bloodstream 3. Maternal hypotension is occurring related to administration of anesthesia 4. Expected manifestations related to anesthetic medications are present

NS: 2 Chapter: Chapter 11 Intrapartum and Postpartum Care of Cesarean Birth Families Chapter Learning Objective: 3. Describe the intraoperative nursing care and medical and anesthesia management for cesarean births. Page: 350 Heading: Intraoperative Care > Complications Integrated Processes: Nursing Process Client Need: Physiological Integrity: Reduction of Risk Potential Cognitive Level: Analysis [Analyzing] Concept: Ante/Intra/Post-partum Difficulty: Difficult Feedback 1 This is incorrect. The patient is not exhibiting manifestations of maternal respiratory depression related to anesthesia. Respiratory depression is not expected with regional anesthesia. 2 This is correct. The patient is exhibiting cardinal signs related to inadvertent injection of the anesthetic agent into the maternal bloodstream. The patient may also exhibit hypotension that can lead to loss of consciousness and cardiac arrest. 3 This is incorrect. The presence of maternal hypotension occurring during the administration is not alone an indication of an incident associated with the administration of anesthesia. 4 This is incorrect. The patient is not exhibiting expected manifestations related to the anesthetic medications. The symptoms are indicative of a medical emergency.

The nurse in the neonate nursery notices a neonate, born 45 minutes ago, is unresponsive to external stimuli, and has a respiratory and heart rate below normal range. Which action does the nurse take? 1. Picks up the neonate and tries to get a response. 2. Allows the neonate to naturally continue deep sleep. 3. Asks another nurse to assist with reassessment. 4. Notifies the caregiver of the neonate's condition.

NS: 2 Chapter: Chapter 15 Physiological and Behavioral Responses of the Neonate Chapter Learning Objective: 2. List the critical elements of neonatal assessment. Page: 467 Heading: Behavioral Characteristics > Periods of Reactivity > Period of Relative Inactivity Integrated Processes: Nursing Process Client Need: Physiological Integrity: Physiological Adaptation Cognitive Level: Analysis [Analyzing] Concept: Ante/Intra/Post-partum Difficulty: Moderate Feedback 1 This is incorrect. About 30 minutes after birth, and for a period of 2 hours, the neonate enters a period of relative inactivity. The nurse's findings are indicative of this period of behavioral characteristics and require no action by the nurse. 2 This is correct. The nurse needs to allow the neonate to continue to sleep deeply, which will last for approximately 2 hours. 3 This is incorrect. The behavior of the neonate is normal and does not require reassessment. 4 This is incorrect. The behavior of the neonate is normal and does not require notification of the caregiver.

The nurse is providing care to a 72-year-old female patient. While providing care, the nurse instructs the patient to slowly rise from a sitting or prone position. What is the pathophysiological reason for this instruction? 1. The patient is at higher risk for fractures due to postmenopausal osteoporosis. 2. The patient is at higher risk for hypotension due to decreased baroreceptor sensitivity. 3. The patient is at higher risk for falls due to decreased muscle strength and balance. 4. The patient is at higher risk for adverse drug reactions due to decreased hepatic function.

NS: 2 Chapter: Chapter 18 Well Women's Health Chapter Learning Objective: 7. Develop a health promotion-teaching plan for an older adult based on the normal physiological changes related to aging. Page: 569 Heading: Table 18-4: Age-Related Physiological Changes and Health Promotion Integrated Processes: Teaching and Learning Client Need: Physiological Integrity: Reduction of Risk Potential Cognitive Level: Analysis [Analyzing] Concept: Promoting Health Difficulty: Moderate Feedback 1 This is incorrect. Although the patient is at risk for falls, osteoporosis does not often cause falls. 2 This is correct. The normal physiological changes of aging include decreased baroreceptor sensitivity, thus increasing the risk for falls due to hypotension. 3 This is incorrect. Although the patient is at risk for falls due to decreased strength, this is not the best answer, as the patient may experience dizziness due to hypotension. 4 This is incorrect. There is no indication of a fall risk due to slower medication metabolism.

A patient in the second trimester of pregnancy is scheduled for a Doppler flow study because the health care provider (HCP) is concerned about an assessment finding during a routine prenatal visit. Which finding of concern does the nurse suspect? 1. Fetal movement count is less than 8 per hour. 2. Patient shows no weight gain in 2 weeks. 3. Patient exhibits mild lower extremity edema. 4. Fetal growth is below expectation for gestational age.

NS: 2 Chapter: Chapter 6 Antepartal Tests Chapter Learning Objective: 2. Identify the purpose and indication for key antenatal tests. Page: 137 Heading: Biophysical Assessment > Fetal Ultrasound Imaging > Doppler Flow Studies (Doppler Velocimetry) Integrated Processes: Nursing Process Client Need: Physiological Integrity: Reduction of Risk Potential Cognitive Level: Analysis [Analyzing] Concept: Ante/Intra/Post-partum Difficulty: Difficult Feedback 1 This is incorrect. Fetal count of less than 8 per hour is not a concern that would prompt a Doppler flow study. Four fetal kicks in 1 hour is considered reassuring. 2 This is correct. There are many variables that can cause a patient not to gain weight in a 2-week period: preexisting obesity or patient actions to maintain a normal weight gain. This finding would not cause the HCP to order a Doppler flow study. 3 This is incorrect. Mild lower extremity edema is not a concern; the patient needs to elevate legs periodically during the day. This finding does not warrant a Doppler flow study. 4 This is incorrect. A Doppler flow study is used in combination with other diagnostic tests to assess fetal status when fetal growth is below the expectation for gestational age (IUGR). Evaluating fetal circulation and uteroplacental blood flow with Doppler flow provides critical information regarding fetal reserves and adaptation.

A patient is scheduled for a contraction stress test (CST) at 36 weeks gestation. The nurse is aware that a successful testing is dependent on which factor? 1. Whether Braxton-Hicks contractions are occurring 2. Whether uterine contractions can be stimulated 3. If the mother is not overly tired or anxious 4. If the fetus is in an awake cycle and active

NS: 2 Chapter: Chapter 6 Antepartal Tests Chapter Learning Objective: 3. Describe the procedure, interpretation, advantages, and risks of common antenatal tests. Page: 144 Heading: Antenatal Fetal Surveillance > Contraction Stress Test Integrated Processes: Nursing Process Client Need: Physiological Integrity: Reduction of Risk Potential Cognitive Level: Analysis [Analyzing] Concept: Ante/Intra/Post-partum Difficulty: Moderate Feedback 1 This is incorrect. The success of a CST is not dependent on the presence of Braxton-Hicks contractions at the time of the testing. 2 This is correct. The success of a CST is dependent on the ability to stimulate uterine contractions. Contractions can be stimulated with careful administration of IV oxytocin or by having the mother brush her nipples for 10 minutes. 3 This is incorrect. The success of a CST is not affected if the mother is overly tired or anxious. 4 This is incorrect. The success of a CST is not dependent on the fetus being awake and active.

The nurse is using the Parer and Ikeda five-tier system. A co-worker is concerned about a patient whose fetus has an acceptably low risk of acidemia and evidence of impending fetal asphyxia. What is the next best step for the nurse? 1. Perform conservative measures. 2. Prepare for urgent delivery. 3. Assist provider in immediate delivery. 4. Increase surveillance of patient.

NS: 2 Chapter: Chapter 9 Fetal Heart Rate Assessment Chapter Learning Objective: Distinguish between Category I, II, and III patterns and appropriate nursing actions based on these interpretations. Page: 282 Heading: Fetal Reserves > NICHD Criteria for Interpretation of FHR Patterns Integrated Processes: Nursing Process Client Need: Physiological Integrity: Reduction of Risk Potential Cognitive Level: Application [Applying] Concept: Ante/Intra/Post-partum Difficulty: Moderate Feedback 1 This is incorrect. Blue indicates a 0 risk of acidemia. In these cases, the nurse should inform the health care provider and perform conservative measures. 2 This is correct. Orange indicates an acceptably low risk of acidemia and preparation for possible urgent delivery. 3 This is incorrect. Red indicates an unacceptably high risk of acidemia and the need for immediate delivery. 4 This is incorrect. Yellow indicates 0 risk of acidemia. The recommendation is for increased surveillance and conservative measures.

The nurse is assessing a newborn's reflexes. Which response will cause the nurse concern? 1. A fencing position when the head is turned 2. Strong Babinski reflex 3. Asymmetrical Moro reflex 4. Absence of rooting or sucking reflexes

NS: 3 Chapter: Chapter 15 Physiological and Behavioral Responses of the Neonate Chapter Learning Objective: 5. Describe the nursing care for neonates during the first week of life. Page: 463 Heading: Table 15-5: Newborn Reflexes Integrated Processes: Nursing Process Client Need: Physiological Integrity: Physiological Adaptation Cognitive Level: Application [Applying] Concept: Ante/Intra/Post-partum Difficulty: Moderate Feedback 1 This is incorrect. The nurse is not concerned when the neonate assumes a fencing position when the head is turned. Absence would indicate possible deafness or neurological deficit. This reflex disappears by age 4 months. 2 This is incorrect. The nurse is not concerned when the neonate demonstrates a strong Babinski reflex (hyperextension and fanning of toes) when the sole of the foot is stroked upward. Weak or absent response is indicative of a possible neurological deficit. This reflex disappears by age 1 year. 3 This is correct. The nurse is concerned if an asymmetrical response is noted when checking for a Moro reflex. This response may be related to temporary or permanent birth injury to clavicle, humerus, or brachial plexus. This reflex disappears by age 6 months. 4 This is incorrect. Before the nurse becomes concerned about the absence of a rooting or sucking reflex, the nurse needs to ascertain when the neonate was last fed. Recent feeding may cause an absence of either or both reflexes.

The nurse is assessing a term neonate delivered to a mother with a history of drug and alcohol abuse. Which finding does the nurse relate to the mother's history? 1. Chest circumference is less than the head circumference. 2. The neonate's pulse rate increases when the neonate cries. 3. When crying, the neonate exhibits an absence of tear production. 4. Head circumference is below the 10th percentile of normal for gestational age.

NS: 4 Chapter: Chapter 15 Physiological and Behavioral Responses of the Neonate Chapter Learning Objective: 2. List the critical elements of neonatal assessment. Page: 452 Heading: Table 15-3: Neonatal Assessment by Area/System Integrated Processes: Nursing Process Client Need: Physiological Integrity: Physiological Adaptation Cognitive Level: Application [Applying} Concept: Ante/Intra/Post-partum Difficulty: Difficult Feedback 1 This is incorrect. It is normal for the neonate's chest circumference to be 2 to 3 cm less than head circumference. This finding is not related to the mother's history. 2 This is incorrect. It is normal for a neonate's pulse rate to increase with crying; this finding is not related to the mother's history. 3 This is incorrect. It is normal for the neonate to have an absence of tear production; tears are not normally formed until an age of 2 months. 4 This is correct. Head circumference below the 10th percentile of normal for gestational age is indicative of microcephaly, which is often related to congenital malformation, maternal drug or alcohol ingestion, or maternal infection during pregnancy.

The nurse is presenting information to new parents regarding screening of their newborn. Which information does the nurse identify as being most important to the parents? 1. All babies born in the United States are screened for specific conditions. 2. Newborn screenings consist of a blood test and a hearing test. 3. Each state has statutes or regulations on newborn screening. 4. Screenings are for infections, genetic diseases, and inherited disorders.

NS: 4 Chapter: Chapter 15 Physiological and Behavioral Responses of the Neonate Chapter Learning Objective: 6. Describe the common laboratory and diagnostic tests for neonates. Page: 471 Heading: Screening Tests > Newborn Screening Integrated Processes: Nursing Process Client Need: Physiological Integrity: Reduction of Risk Potential Cognitive Level: Analysis [Analyzing] Concept: Ante/Intra/Post-partum Difficulty: Moderate Feedback 1 This is incorrect. It is true that all babies born in the United States are screened for specific conditions. However, this information is likely not to be the information that is most important to the parents of a newborn. 2 This is incorrect. It is true that newborn screening consists of a blood test and a hearing test. Some states are also including heart defect screening. The parents will be interested in this information, but other information will be most important. 3 This is incorrect. It is true that each state has statutes or regulations on newborn screening, and the degree of screening varies from state to state. However, this information is likely not to be the information that is most important to the parents of a newborn. 4 This is correct. The blood test screens for infections, genetic diseases, and inherited and metabolic disorders; this is the information the parents of a newborn will be most interested in. Parents are focused on the well-being of their newborn and will seek information that provides conditions and treatments if needed.

The nurse is providing information to a postpartum mother about circumcision of her neonate. The neonate's father states, "We have never done that to any baby boy in my family." Which statement is best for the nurse to make? 1. "Most families opt for the procedure for a variety of reasons." 2. "I can leave information for you to read over and then decide." 3. "I personally think that boys are cleaner and healthier if circumcised." 4. "I understand that family culture and beliefs form our way of life."

NS: 4 Chapter: Chapter 15 Physiological and Behavioral Responses of the Neonate Chapter Learning Objective: 9. Discuss the importance of incorporating knowledge of cultural beliefs, customs, and newborn variations in the care of the parents and newborn. Page: 473 Heading: Therapeutic and Surgical Procedures > Circumcision Integrated Processes: Culture and Spirituality Client Need: Psychosocial Integrity Cognitive Level: Application [Applying] Concept: Ante/Intra/Post-partum Difficulty: Moderate Feedback 1 This is incorrect. The nurse's comment is not completely truthful; newborn circumcisions declined from 1979 to 2010, from 64.5% to 58.3%. 2 This is incorrect. Leaving the information for the parent to read before making a decision is disrespectful to the family culture and beliefs. It also implies the parent lacks the knowledge to make a good decision. 3 This is incorrect. When the nurse interjects a personal opinion, especially when it addresses cleanliness and health, it is extremely disrespectful to the family culture and beliefs. 4 This is correct. The nurse should always respect the patient's culture and beliefs, even if the nurse does not agree with or share the opinions.

The nurse in labor and delivery notices an increase in the number of women requesting cesarean births. Which are the parameters and criteria used when making the decision to perform a cesarean delivery on maternal request (CDMR)? Select all that apply. 1. Patient is able to self-pay for the procedure. 2. Patient is willing to defer from legal litigation. 3. Mother is planning to only have one child. 4. Patient is aware of possible neonatal complications. 5. Procedure is performed after 39 weeks gestation.

NS: 4, 5 Chapter: Chapter 11 Intrapartum and Postpartum Care of Cesarean Birth Families Chapter Learning Objective: 2. Discuss the preoperative nursing care and medical and anesthesia management for cesarean births. Page: 346 Heading: Introduction Integrated Processes: Nursing Process Client Need: Physiological Integrity: Reduction of Risk Potential Cognitive Level: Analysis [Analyzing] Concept: Ante/Intra/Post-partum Difficulty: Difficult Feedback 1 This is incorrect. The ability to self-pay for a cesarean delivery is not part of the parameters and criteria relative to a CDMR. 2 This is incorrect. The willingness of a patient and partner to defer from legal litigation is not part of the parameters and criteria relative to a CDMR. 3 This is incorrect. The risk for complications increases with each cesarean birth. However, the intention of the mother to have only one child is not part of the parameters and criteria relative to a CDMR. 4 This is correct. Part of the parameters and criteria relative to a CDMR is that the patient recognizes that the fetus is at risk for respiratory distress, hypothermia, hypoglycemia, and/or NICU admission. 5 This is correct. Because of the increased risk for neonatal complications, the patient needs to understand that CDMR does not occur before 39 weeks gestation.

The nurse is providing dietary teaching to a patient in the first trimester of pregnancy who is overweight. Which daily dietary suggestions does the nurse make? Select all that apply. 1. One cup of 100% juice and cup of dried fruit. 2. Three cups of raw leafy and 1 cup cooked vegetables 3. One and a half cups of cooked pasta, rice, or cereal 4. Six ounces of lean meat, 2 eggs, and cup of beans 5. One cup of milk, 1 cup of yogurt, and oz of cheese

ANS: 1, 5 Chapter: Chapter 4 Physiological Aspects of Antepartum Care Chapter Learning Objective: 7. Identify the critical elements of assessment and nursing care during initial and subsequent prenatal visits. Page: 72 Heading: Preconception Health Care > Preconception Education > Nutrition Integrated Processes: Nursing Client Need: Health Promotion and Maintenance Cognitive Level: Analyzing [Analysis] Concept: Ante/Intra/Post-partum Difficulty: Difficult Feedback 1 This is correct. In the first trimester of pregnancy, the patient requires 2 cups of fruit; 1 cup of 100% juice and cup of dried fruit daily is a correct suggestion by the nurse. 2 This is incorrect. In the first trimester of pregnancy, the patient requires cups of vegetables daily; 3 cups of raw leafy vegetables and 1 cup of cooked vegetables is too much. 3 This is incorrect. One and a half cups of cooked rice, pasta, or cereal daily is too much; the patient in the first trimester of pregnancy needs 6 ounces of grains per day. 4 This is incorrect. A patient in the first trimester of pregnancy needs ounces of protein daily; the suggested amounts are too much. One ounce of protein consists of 1 ounce of lean meat, poultry, or seafood; cup of cooked beans, ounce of nuts or 2 tablespoons of peanut butter; or 1 egg. The suggested foods provide 10 ounces of protein. 5 This is correct. One cup of milk, 1 cup of yogurt, and ounces of cheese is equal to the recommended daily dairy intake. This suggestion by the nurse is correct.

The nurse in labor and delivery is preparing to initiate labor induction with the administration of oxytocin. After research about oxytocin, the nurse is aware of which fact about the drug? 1. Hypothalamus stimulation increases circulating oxytocin. 2. Synthetic oxytocin is identical to endogenous oxytocin. 3. The half-life of oxytocin is 1 hour, supporting close monitoring. 4. Action from IV oxytocin administration is less than 1 minute.

ANS: 2 Chapter: Chapter 10 High-Risk Labor and Birth Chapter Learning Objective: 2. Demonstrate understanding of knowledge related to induction of labor and augmentation of labor and vaginal birth after cesarean birth. Page: 311 Heading: Labor Interventions > Oxytocin Induction Integrated Processes: Nursing Process Client Need: Physiological Integrity: Reduction of Risk Potential Cognitive Level: Application [Applying] Concept: Ante/Intra/Post-partum Difficulty: Moderate Feedback 1 This is incorrect. Labor induction is not a result of endogenous oxytocin, which is a peptide synthesized by the hypothalamus that is transported to the posterior lobe of the pituitary gland. Oxytocin is naturally released in response to vaginal and cervical stretching. 2 This is correct. The nurse is aware from drug research that synthetic oxytocin is identical to endogenous oxytocin. 3 This is incorrect. The administration of oxytocin does require close monitoring, but the half-life of the drug is 10 minutes, not 1 hour. 4 This is incorrect. Uterine response to oxytocin usually occurs within 3 to 5 minutes after IV administration begins.

The nurse is providing care to a 35-year-old female patient who complains of low back pain, pain with defecation, pelvic pressure, and premenstrual spotting. The health care provider has prescribed the hormonal therapy Lupron for this condition. What is the goal of this prescription? 1. To prevent pregnancy at this time to promote healing 2. To suppress menstruation and further growth of the tissue 3. To prevent retrograde menstruation outside the uterine cavity 4. To increase blood flow to decrease the endometrial lining

ANS: 2 Chapter: Chapter 19 Alterations in Women's Health Chapter Learning Objective: 3. Describe common alterations in women's health, including medical management and nursing actions. Page: 584 Heading: Endometriosis > Medical Management Integrated Processes: Nursing Process Client Need: Physiological Integrity: Reduction of Risk Potential Cognitive Level: Application [Applying] Concept: Health Promotion Difficulty: Moderate Feedback 1 This is incorrect. This patient has the classic symptoms of endometriosis. Infertility is often a problem for patients with endometriosis. 2 This is correct. This is the goal of GnRH agonists, which suppresses menstruation and further growth of tissue. 3 This is incorrect. Although this medication is used to suppress menstruation, one theory for the cause of endometriosis is retrograde menstruation, which transports endometrial tissue outside the uterine cavity. 4 This is incorrect. The goal is to suppress menstruation, not increase blood flow and thinning of the endometrial tissue.

The nurse is counseling a couple in the third trimester of pregnancy and recommends the couple attend childbirth education classes. For which reason is the nurse least likely to recommend the classes? 1. The classes will affirm the normalcy of birth. 2. The techniques will enable a medication-free delivery. 3. The classes acknowledge a woman's ability to inherently give birth. 4. The classes explore ways to find strength and comfort during labor.

ANS: 2 Chapter: Chapter 5 The Psycho-Social-Cultural Aspects of the Antepartum Period Chapter Learning Objective: 6. Identify key components of childbirth preparation education for expectant families. Page: 124 Heading: Planning for Birth > Childbirth Education Integrated Processes: Nursing Process Client Need: Psychosocial Integrity Cognitive Level: Application [Applying] Concept: Ante/Intra/Post-partum Difficulty: Moderate Feedback 1 This is incorrect. Childbirth classes are designed to affirm the normalcy of childbirth and remove the ideas that it is an illness or the mother is sick. 2 This is correct. Childbirth classes do not focus on teaching techniques that enable a medication-free delivery. However, the classes may enable the mother to require less medication because of greater understanding of the birthing process. 3 This is incorrect. Childbirth classes focus on the ability of women to inherently give birth; it is a normal and natural function, which can often happen with minimal assistance or interference. 4 This is incorrect. Childbirth classes will help the mother and her coach to explore ways in which to promote strength and comfort during labor and delivery.

A patient who is at 39 weeks gestation is scheduled for amniotomy. The nurse is aware that which criteria must be met before the procedure? 1. Ultrasound indicates the umbilical cord is away from the cervix. 2. The nurse must have certification to perform the procedure. 3. The fetal head is currently engaged in the maternal pelvis. 4. Prior amniotic fluid leakage must be validated before the procedure.

ANS: 3 Chapter: Chapter 10 High-Risk Labor and Birth Chapter Learning Objective: 2. Demonstrate understanding of knowledge related to induction of labor and augmentation of labor and vaginal birth after cesarean birth. Page: 318 Heading: Labor Interventions > Amniotomy Integrated Processes: Nursing Process Client Need: Physiological Integrity: Reduction of Risk Potential Cognitive Level: Application [Applying] Concept: Ante/Intra/Post-partum Difficulty: Moderate Feedback 1 This is incorrect. A possible complication with amniotomy is for prolapse of the umbilical cords, and careful monitoring is required. However, the placement of the cord is not identified by ultrasound before the procedure. 2 This is incorrect. The amniotomy is performed by the primary health care provider who needs to be available in the event that an emergency intervention is necessary. Nurses do not perform this procedure. 3 This is correct. An important criterion for performing an amniotomy is for the fetal head to be engaged in the maternal pelvis. Lack of engagement will result in a prolonged labor and/or an increased risk for infection. 4 This is incorrect. Amniotic fluid leakage does not need to be confirmed before an amniotomy.

What is the common risk factor for hypertension, abnormal menstrual cycles, osteoarthritis, and high-risk pregnancies? 1. Polycystic ovarian syndrome 2. Diabetes mellitus 3. Body mass index over 32 4. Sedentary lifestyle

ANS: 3 Chapter: Chapter 18 Well Women's Health Chapter Learning Objective: 4. Discuss the effects of obesity on women's health. Page: 558 Heading: Health Promotion > Risk Reduction Integrated Processes: Teaching and Learning Client Need: Physiological Integrity: Physiological Adaptation Cognitive Level: Analysis [Analyzing] Concept: Promoting Health Difficulty: Moderate Feedback 1 This is incorrect. Although some of these symptoms are classic for polycystic ovarian syndrome, not all of the symptoms are indicative. 2 This is incorrect. Although some of these symptoms may indicate diabetes mellitus, not all of the symptoms are indicative. 3 This is correct. A BMI of >30 indicates obesity, which could cause the listed symptoms. 4 This is incorrect. A sedentary lifestyle may be the cause of obesity, but it does not indicate a disease process.

A nurse-preceptor is explaining to a new nurse about the tocodynamometer. The new nurse is looking at the EFM paper and sees that, of the two tracked heart rates, the one on the bottom is in the 80s; she is concerned that the fetal heart rate is bradycardic. Which of the following should the nurse do first? 1. Give the mother oxygen to increase the fetal heart rate. 2. Immediately call the provider into the room. 3. Check to make sure that the maternal radial pulse is being recorded correctly. 4. Adjust the monitor on the maternal abdomen.

ANS: 3 Chapter: Chapter 9 Fetal Heart Rate Assessment Chapter Learning Objective: Define terms used in electronic fetal monitoring (EFM). Page: 275 Heading: Modes or Types of Fetal and Uterine Monitoring > External Electronic Fetal and Uterine Monitoring Integrated Processes: Nursing Process Client Need: Physiological Integrity: Reduction of Risk Potential Cognitive Level: Application [Applying] Concept: Ante/Intra/Post-partum Difficulty: Moderate Feedback 1 This is incorrect. The maternal heart rate is usually significantly lower than the fetal heart rate and is therefore tracked underneath the fetal heart rate. It is therefore important to check and make sure that the maternal heart rate is being tracked correctly before beginning oxygen. 2 This is incorrect. The maternal heart rate is usually significantly lower than the fetal heart rate and is therefore tracked underneath the fetal heart rate. It is therefore important to check and make sure that the maternal heart rate is being tracked correctly before calling in the provider. 3 This is correct. The maternal heart rate is usually significantly lower than the fetal heart rate and is therefore tracked underneath the fetal heart rate. It is therefore important to check and make sure that the maternal heart rate is being tracked correctly before initiating any efforts for the fetus. 4 This is incorrect. The maternal heart rate is usually significantly lower than the fetal heart rate and is therefore tracked underneath the fetal heart rate. It is therefore important to check and make sure that the maternal heart rate is being tracked correctly before adjusting the fetal monitor.

The nurse is caring for a 15-year-old female who is pregnant with her first child. In her previous prenatal visit, the patient tested negative for chlamydia, syphilis, gonorrhea, and HIV. Based on the information provided, which condition is the patient's baby at higher risk for? 1. Intestinal problems 2. Neonatal conjunctivitis 3. Blindness 4. Pneumonia

ANS: 1 Chapter: Chapter 1 Trends and Issues Chapter Learning Objective: 4. Discuss current maternal and infant health issues. Page: 11 Heading: Issues > Teen Pregnancy Integrated Processes: Nursing Process Client Need: Physiological Integrity: Reduction of Risk Potential Cognitive Level: Application [Applying] Concept: Health Promotion Difficulty: Difficult Feedback 1 This is correct. Infants born to teen mothers are at increased risk for various conditions related to prematurity, including infant death, intestinal problems, and/or respiratory distress syndrome. 2 This is incorrect. Infants born to teen mothers who have gonorrhea are at increased risk of neonatal conjunctivitis and blindness. 3 This is incorrect. Infants born to teen mothers with syphilis and gonorrhea are at increased risk of blindness. 4 This is incorrect. Infants born to teen mothers with chlamydia may be at increased risk of developing chlamydial pneumonia.

A pregnant woman weighs 90.9 kg. The nurse is educating the patient on complications that the patient may be at risk for during pregnancy. Which response by the patient indicates that she understands? 1. "Due to my weight, there is a possibility that I may develop gestational diabetes." 2. "I am not overweight, but I am still at risk for gestational diabetes." 3. "My mother had preeclampsia during one of her pregnancies." 4. "I will need to do a glucose tolerance test in my second trimester."

ANS: 1 Chapter: Chapter 1 Trends and Issues Chapter Learning Objective: 4. Discuss current maternal and infant health issues. Page: 13 Heading: Issues > Obesity Integrated Processes: Nursing Process Client Need: Safe and Effective Care Environment: Management of Care Cognitive Level: Application [Applying] Concept: Ante/Intra/Post-partum Difficulty: Difficult Feedback 1 This is correct. The patient is at risk for gestational diabetes due to being obese during pregnancy. 2 This is incorrect. The patient is overweight. 3 This is incorrect. This response is not related to the question. 4 This is incorrect. The patient will need to get the glucose tolerance test in the second trimester, but this response does not relate to the question.

The nurse is educating the pregnant patient with a body mass index (BMI) of 33. The nurse knows that teaching has been effective when the patient states which of the following? 1. "My child may be at increased risk for birth injury." 2. "My child may have a decreased risk of developing childhood diabetes." 3. "I will probably give birth vaginally." 4. "I have a lower risk of developing gestational hypertension."

ANS: 1 Chapter: Chapter 1 Trends and Issues Chapter Learning Objective: 4. Discuss current maternal and infant health issues. Page: 13 Heading: Issues > Obesity Integrated Processes: Nursing Process Client Need: Safe and Effective Care Environment: Safety and Infection Control Cognitive Level: Application [Applying] Concept: Ante/Intra/Post-partum Difficulty: Difficult Feedback 1 This is correct. Shoulder dystocia and other birth injuries are associated with infant macrosomia (large size) due to maternal obesity. 2 This is incorrect. Children born to mothers who are obese are at increased risk of developing childhood obesity and diabetes. 3 This is incorrect. Pregnant patients who are obese are at increased risk of cesarean birth. 4 This is incorrect. Pregnant patients who are obese have an increased risk of developing gestational diabetes and gestational hypertension.

The nurse is providing care for a patient in the second phase of labor. After more than 4 hours of pushing, the nurse suspects fetal dystocia. Which is the greatest risk related to the nurse's suspected complication? 1. Neonatal asphyxia related to prolonged labor 2. Fetal injury confirmed by the presence of bruising 3. Greater risk for maternal lacerations 4. Increased consideration for a cesarean delivery

ANS: 1 Chapter: Chapter 10 High-Risk Labor and Birth Chapter Learning Objective: 3. Identify potential complications of dystocia in labor and related nursing and medical care. Page: 305 Heading: Dystocia > Fetal Dystocia Integrated Processes: Nursing Process Client Need: Physiological Integrity: Reduction of Risk Potential Cognitive Level: Analysis [Analyzing] Concept: Ante/Intra/Post-partum Difficulty: Difficult Feedback 1 This is correct. The greatest concern related to fetal dystocia is the complication of fetal asphyxia related to a prolonged labor. 2 This is incorrect. With fetal dystocia, the complication of fetal injury is possible and may be evidenced by the presence of bruising. 3 This is incorrect. A complication of fetal dystocia is a greater risk for maternal lacerations. 4 This is incorrect. The complication is fetal dystocia; a possible solution to the condition is cesarean delivery of the fetus.

The nurse is providing care in PACU for a patient who just delivered a neonate via cesarean section. The patient reports tightness in her chest. Assessment findings include tachypnea, hypotension, and decreasing oxygen saturation levels. Which complication does the nurse report to the health care provider? 1. Pulmonary embolism 2. Postpartum hemorrhage 3. Surgical-site infection 4. Developing endometritis

ANS: 1 Chapter: Chapter 11 Intrapartum and Postpartum Care of Cesarean Birth Families Chapter Learning Objective: 5. Identify potential intraoperative and postoperative complications related to cesarean birth and nursing actions to reduce risk. Page: 355 Heading: Postoperative Care > Complications Integrated Processes: Nursing Process Client Need: Physiological Integrity: Reduction of Risk Potential Cognitive Level: Analysis [Analyzing] Concept: Ante/Intra/Post-partum Difficulty: Moderate Feedback 1 This is correct. The patient's manifestations are classic for pulmonary embolus. Other assessment findings will include dyspnea, shortness of breath, and hypotension. 2 This is incorrect. The patient's manifestations are not indicative of postpartum hemorrhage. Symptoms would be hypotension, tachycardia, and probable evidence of bleeding. 3 This is incorrect. Surgical site infection is indicated by serous or purulent drainage, erythema, fever, pain, and wound dehiscence. However, the time line is incorrect; infection would not be evident in PACU. 4 This is incorrect. Endometritis is usually diagnosed within the first few days after delivery. Fever is the most common sign. Other signs include chills, uterine tenderness, and foul-smelling lochia.

1. A multiparous patient reports severe uterine cramps the first day after a vaginal delivery. The nurse is aware the patient is breastfeeding and associates the patient's pain primarily with which occurrence? 1. An increase in oxytocin release related to the newborn suckling 2. The presence of intense afterbirth pains related to multiparity 3. An expected response to the daily administration of oxytocin 4. The efforts of the uterus to return to a prepregnancy condition

ANS: 1 Chapter: Chapter 12 Postpartum Physiological Assessments and Nursing Care Chapter Learning Objective: 1. Describe the physiological changes that occur during the postpartum period. Page: 366 Heading: The Reproductive System > Uterus Integrated Processes: Nursing Process Client Need: Physiological Integrity: Physiological Adaptation Cognitive Level: Analysis [Analyzing] Concept: Ante/Intra/Post-partum Difficulty: Moderate Feedback 1 This is correct. The suckling of a newborn during breastfeeding will stimulate an increased release of oxytocin, which in turn stimulates the uterus to remain contracted. 2 This is incorrect. The presence of intense afterpains is frequently related to multiparity; however, in this scenario the nurse needs to recognize the severity is likely related to breastfeeding. 3 This is incorrect. Oxytocin is not administered unless the patient's uterus is not remaining contracted. The presence of severe cramps does not support the need of oxytocin. 4 This is incorrect. Afterpains or uterine contractions are the uterine effort to return the uterus nearly to the prepregnant size. The uterus never returns completely to the size prior to pregnancy.

Loutzenhiser, McAuslan, and Sharpe (2015) performed a study regarding maternal and paternal fatigue and factors associated with fatigue across the transition to parenthood. Which evidence-based conclusion is made regarding fatigue and the transitioning parents? 1. Levels of prenatal and postnatal fatigue are associated. 2. Postnatal fatigue remains consistent for a period of 1 year. 3. Poor sleep quality in fathers is related to depressive symptoms. 4. The length of couple relationship strongly impacts parenteral fatigue.

ANS: 1 Chapter: Chapter 13 Transition to Parenthood Chapter Learning Objective: 2. Identify factors that influence women and men in their role transitions to mother and father. Page: 398 Heading: Transition to Parenthood > Evidence-Based Practice > Maternal and Paternal Fatigue Integrated Processes: Nursing Process Client Need: Psychosocial Integrity Cognitive Level: Analysis [Analyzing] Concept: Family Dynamics Difficulty: Moderate Feedback 1 This is correct. The evidence-based conclusion for the study indicates that high levels of prenatal fatigue were associated with higher levels of postpartum fatigue for both the mother and father. 2 This is incorrect. Maternal and paternal fatigue increased following the birth and remained consistent over 6 months. 3 This is incorrect. Poor quality of sleep for fathers was associated with shorter duration of infant sleep. 4 This is incorrect. Neither family income nor length of couple relationship was associated with fatigue.

The nurse on a postpartum unit is focused on providing care that will assist the mother and father in making the transition to parenthood. For which reason does the nurse review the prenatal and labor records? 1. Pregnancy and birth experiences, which can either enhance or impede the process of becoming a mother. 2. Awareness of prenatal classes that will help identify and focus on learning needs of both parents. 3. Identification of preexisting maternal conditions that may interfere with parenting transitions. 4. Knowledge regarding questions and concerns the mother and father may have about neonate issues.

ANS: 1 Chapter: Chapter 13 Transition to Parenthood Chapter Learning Objective: 2. Identify factors that influence women and men in their role transitions to mother and father. Page: 400 Heading: Motherhood > Nursing Actions Integrated Processes: Nursing Process Client Need: Health Promotion and Maintenance Cognitive Level: Analysis [Analyzing] Concept: Family Dynamics Difficulty: Difficult Feedback 1 This is correct. The nurse reviews the maternal prenatal and labor records because pregnancy and birth experiences can either enhance or impede the process of becoming a mother. The nurse is looking for factors such as complications during pregnancy, labor, and birth. 2 This is incorrect. The nurse is not attempting to gain awareness of prenatal classes in order to help identify and focus on learning needs of both parents. 3 This is incorrect. The nurse is not attempting to gain information of preexisting maternal conditions that may interfere with parenting transitions. 4 This is incorrect. The nurse is not attempting to gain information regarding questions and concerns the mother and father may have about neonate issues.

The nurse is interested in promoting coparenting because of the high likeability that at some point, both parents will be working outside the home. After reading research by Davis, Schoppe-Sullivan, Mangelsdorf, and Brown (2009), the nurse learns that which factor impacts coparenting the most? 1. Infant temperament 2. Father's interest 3. Strength of support systems 4. Mother's expectations

ANS: 1 Chapter: Chapter 13 Transition to Parenthood Chapter Learning Objective: 4. Identify factors that affect the family dynamics. Page: 409 Heading: Family Dynamics Integrated Processes: Nursing Process Client Need: Psychosocial Integrity Cognitive Level: Analysis [Analyzing] Concept: Family Dynamics Difficulty: Moderate Feedback 1 This is correct. Infant temperament difficulty reported by fathers at 3.5 months was associated with a decrease in supportive coparenting behavior. Early interventions to enhance coparenting are essential for families with temperamentally difficult infants. 2 This is incorrect. The father's interest in coparenting can be negatively affected by an infant with who is temperamentally difficult. The father's interest is enhanced if the temperament difficulties decrease. 3 This is incorrect. The strength of the parent's support systems does not impact the coparenting experience. 4 This is incorrect. The mother's expectations may or may not affect the coparenting experience.

The nurse on a postpartum unit focuses on how to assist the father in identifying his role with the neonate. Which intervention by the nurse is most helpful? 1. Encourage the couple to identify mutual expectations of the fathering role. 2. Critique the father's methods of providing physical care for the neonate. 3. Provide written materials about the physical and emotional role of a father. 4. Observe for a competitive attitude between the parents about providing baby care.

ANS: 1 Chapter: Chapter 13 Transition to Parenthood Chapter Learning Objective: 5. Describe nursing actions that support couples during their transition to parenthood. Page: 402 Heading: Fatherhood > Nursing Actions Integrated Processes: Nursing Process Client Need: Physiological Integrity Cognitive Level: Analysis [Analyzing] Concept: Family Dynamics Difficulty: Difficult Feedback 1 This is correct. Mutually agreed-upon fathering expectations, shared by the couple, can decrease the level of stress within the relationship. 2 This is incorrect. The nurse does not need to critique the father's performance of providing physical care for the neonate unless the behavior has a potential for harm. Critiquing the father's efforts may cause feelings of inadequacy. 3 This is incorrect. Written materials may be helpful; however, the nurse should demonstrate infant care such as diapering, feeding, and holding. Demonstrating infant care skills enhances the father's comfort in caring for his infant. 4 This is incorrect. An attitude of competitiveness between the parents about providing baby care is not common or expected. Obvious signs of this attitude will require the nurse to implement interventions that will promote "team work" between the parents.

The nurse is aware the greatest source of bleeding during childbirth occurs following detachment of the placenta. Which physiological change takes place immediately after the expulsion of the placenta to decrease the amount of blood loss? 1. Contractions of the uterine myometrium 2. Factor VIII complex increases during gestation 3. Platelet activity increases before labor and delivery 4. Fibrin formation increases before the birth occurs

ANS: 1 Chapter: Chapter 14 High-Risk Postpartum Nursing Care Chapter Learning Objective: 1. Describe the primary causes of postpartum hemorrhage and the related nursing actions and medical care. Page: 419 Heading: Hemorrhage Integrated Processes: Nursing Process Client Need: Physiological Integrity: Reduction of Risk Potential Cognitive Level: Analysis [Analyzing] Concept: Ante/Intra/Post-partum Difficulty: Moderate Feedback 1 This is correct. After placenta detachment, contractions of the myometrium compress the blood vessels at the placental site, thus decreasing the amount of blood loss. 2 This is incorrect. Factor VIII complex increases during gestation and contributes to the hypercoagulability of the blood. However, the physiological change does not occur at the point of the placenta detaching. 3 This is incorrect. Platelet activity increases before labor and delivery and contributes to the hypercoagulability of the blood. However, the physiological change does not occur at the point of the placenta detaching. 4 This is incorrect. Fibrin formation increases before the birth occurs and contributes to the hypercoagulability of the blood. However, the physiological change does not occur at the point of the placenta detaching.

The nurse works in a postnatal nursery and is required by hospital policy to perform a gestational age assessment on specified neonates. On which neonate is the nurse most likely to perform this assessment? 1. The neonate with a birth weight of 4,100 g 2. The neonate born at 37 weeks gestation 3. The neonate born after an 18-hour labor 4. The neonate exposed to oxytocin in utero

ANS: 1 Chapter: Chapter 15 Physiological and Behavioral Responses of the Neonate Chapter Learning Objective: 3. List the critical elements of neonatal gestational age assessments. Page: 451 Heading: Neonatal Assessment: Gestational Age Assessment Integrated Processes: Nursing Process Client Need: Physiological Integrity: Reduction of Risk Potential Cognitive Level: Analysis [Analyzing] Concept: Ante/Intra/Post-partum Difficulty: Difficult Feedback 1 This is correct. Neonates who weigh less than 2,500 g or more than 4,000 g are most likely to be assessed for gestational age. The nurse will determine if the neonate is post-term. 2 This is incorrect. Neonates who are preterm, born before 37 weeks based on the maternal menstrual history, will be assessed for gestational age. The neonate born at 37 weeks gestational age does not fall into this category. 3 This is incorrect. Birth after an 18-hour labor does not require an assessment for gestational age on the neonate. 4 This is incorrect. All neonates are exposed to oxytocin in utero. Whether the oxytocin is natural or augmented, the effects are the same and do not require an assessment for gestational age.

The nurse is providing care for a neonate during the fourth stage of labor. Which action does the nurse take during this stage? 1. Dry the neonate immediately. 2. Compete neonate assessment within 1 hour. 3. Obtain neonate blood glucose levels. 4. Perform Apgar screening until scores are 7.

ANS: 1 Chapter: Chapter 15 Physiological and Behavioral Responses of the Neonate Chapter Learning Objective: 5. Describe the nursing care for neonates during the first week of life. Page: 468 Heading: Nursing Care of the Neonate > Nursing Actions During the Fourth Stage of Labor Integrated Processes: Nursing Process Client Need: Physiological Integrity: Physiological Adaptation Cognitive Level: Application [Applying] Concept: Ante/Intra/Post-partum Difficulty: Moderate Feedback 1 This is correct. The fourth stage of labor is from the birth of the neonate for 4 hours postpartum. The nurse will dry the neonate immediately to aid with thermoregulation and to prevent cold stress. 2 This is incorrect. The neonatal assessment is to be completed within 2 hours after birth of the neonate. 3 This is incorrect. Unless there is an indication of neonatal hypoglycemia, or a risk for hypoglycemia, the nurse does not routinely obtain a blood glucose level on the neonate. 4 This is incorrect. Apgar scores are obtained at 1 and 5 minutes, and the nurse will initiate appropriate actions based on the score. The score is not repeated until a score of 7 is attained.

A patient in the second trimester of pregnancy is discussing breastfeeding and other options with the nurse. Which question is most important for the nurses to ask? 1. "How does your partner feel about you breastfeeding?" 2. "Do you have family members who have breastfed their babies?" 3. "What are the reasons why you are considering breastfeeding?" 4. "At what point after childbirth do you plan to return to work?"

ANS: 1 Chapter: Chapter 16 Discharge Planning and Teaching Chapter Learning Objective: 6. Develop a teaching plan for breastfeeding. Page: 482 Heading: Newborn Nutrition and Feeding > Breastfeeding Integrated Processes: Teaching and Learning Client Need: Physiological Integrity: Basic Care and Comfort Cognitive Level: Analysis [Analyzing] Concept: Ante/Intra/Post-partum Difficulty: Difficult Feedback 1 This is correct. The woman's partner plays a significant role in her choice to breastfeed and to continue breastfeeding. Her feelings about and success at breastfeeding are enhanced by her partner's support. 2 This is incorrect. Family members, especially those who have breastfed, can be a good source of support; however, the woman's partner is a more significant source of support. 3 This is incorrect. It is important for the nurse to understand the woman's reasons for breastfeeding; however, this is not the most important question to ask. 4 This is incorrect. A woman's career plans and/or the time line for returning to work are important for the nurse to ask about; however, this is not the most important question.

A mother who is breastfeeding expresses concern about whether her infant is getting enough milk. Which concrete indicator does the nurse provide to the mother? 1. There are at least eight wet diapers and several stools per day. 2. The mother is physically and emotionally comfortable during feedings. 3. The newborn suckles and the mother can hear and/or see swallowing. 4. The newborn spontaneously releases the grip on the breast when satiated.

ANS: 1 Chapter: Chapter 16 Discharge Planning and Teaching Chapter Learning Objective: 8. Provide parents with information regarding newborn care that reflects the assessed learning needs of parents. Page: 487 Heading: Newborn Nutrition and Feeding > Breastfeeding > Patient Education > Determining Effective Feeding Integrated Processes: Nursing Process Client Need: Physiological Integrity: Physiological Adaptation Cognitive Level: Application [Applying] Concept: Ante/Intra/Post-partum Difficulty: Moderate Feedback 1 This is correct. The most concrete indicator that the breastfeeding baby is receiving enough milk is at least eight wet diapers and several stools per day. 2 This is incorrect. The mother being physically and emotionally comfortable during feedings is a subjective determination of successful breastfeeding. 3 This is incorrect. When the newborn suckles and the mother can hear and/or see swallowing, the mother knows there is a transfer of milk; however, the amount is still unknown. 4 This is incorrect. The newborn spontaneously releases the grip on the breast when satiated; however, this does not specifically indicate the amount of milk is adequate.

The nurse is providing care for a premature neonate born at 28 weeks gestation who is experiencing respiratory distress syndrome (RDS). Which assessment finding indicates to the nurse that the neonate's respiratory status is deteriorating? 1. Pao2 is 48 and Paco2 is 55 mm Hg on 90% oxygen. 2. Respiratory rate is 58 breaths per minute. 3. Breath sounds on auscultation are decreased. 4. Heart rate is 162 beats per minute.

ANS: 1 Chapter: Chapter 17 High-Risk Neonatal Nursing Care Chapter Learning Objective: 1. Describe the physiology and pathophysiology associated with selected complications of the neonatal period. Page: 512 Heading: Respiratory Distress Syndrome Integrated Processes: Nursing Process Client Need: Physiological Integrity: Reduction of Risk Potential Cognitive Level: Analysis [Analyzing] Concept: Ante/Intra/Post-partum Difficulty: Difficult Feedback 1 This is correct. A sign that the neonate's respiratory status is deteriorating is if increased oxygen levels fail to maintain a Pao2 and Paco2 within normal limits. The normal range of Pao2 is 60 to 70 mm Hg and the normal range of Paco2 is 35 to 45 mm Hg. The neonate is unable to maintain a normal range on 90% oxygen, which is a sign of deterioration. 2 This is incorrect. A respiration rate of 58 breaths per minute is considered normal. With RDS, the respiration rate is expected to be greater than 60 breaths per minute. 3 This is incorrect. An expected finding for a neonate with RDS is decreased breath sounds with auscultation. Rales are present as RDS progresses and the neonate deteriorates. 4 This is incorrect. With RDS, the heart rate is expected to be greater than 160 beats per minute. This is an expected finding and does not necessarily indicate deterioration.

The nurse is present in the delivery room when a mother is told her neonate was stillborn. The mother begins to wail loudly and pull at her hair. Which action does the nurse take? 1. Allow the mother to express grief in her own way. 2. Attempt to calm the mother and prevent self-harm. 3. Ask for a sedative to calm the mother's reaction. 4. Ask a family member to comfort the mother.

ANS: 1 Chapter: Chapter 17 High-Risk Neonatal Nursing Care Chapter Learning Objective: 4. Describe the loss and grief process experienced by parents whose infant has died. Page: 545 Heading: Loss and Grief Integrated Processes: Nursing Process Client Need: Psychosocial Integrity Cognitive Level: Application [Applying] Concept: Ante/Intra/Post-partum Difficulty: Moderate Feedback 1 This is correct. Culture, religion, and personal experience and beliefs will impact how individuals and families respond to loss. The nurse needs to support the mother and allow her to grieve in her own way. 2 This is incorrect. Attempts to calm the mother are likely to be ineffective. The nurse will protect the mother from self-harm, but hair pulling does not place the mother in physical danger. 3 This is incorrect. Asking for a sedative to calm the mother will just delay the mother's expression of grief. Nurses must keep in mind that each person experiences and expresses grief in his or her own way. 4 This is incorrect. Asking a family member to comfort the mother may not be effective due to the grief the family members will also experience. The mother needs support now, and the nurse will help fill this need immediately.

The nurse is assessing a 59-year-old female patient for her annual examination. The patient had a full hysterectomy, bilateral salpingo-oophorectomy 1 year ago for a noncancerous condition. The patient states that she wants to stop her menopausal hormone therapy at this time, and she will try black cohosh if the symptoms persist. Which response by the nurse is most appropriate? 1. "Hormone therapy is the most effective treatment for menopausal symptoms." 2. "Hormone therapy becomes less effective within 2 years of treatment." 3. "Hormone therapy is most effective with alternative medicines use." 4. "Hormone therapy is most effective when combined with lifestyle changes."

ANS: 1 Chapter: Chapter 18 Well Women's Health Chapter Learning Objective: 5 Discuss the physical and emotional changes related to perimenopause and menopause. Page: 563 Heading: Reproductive Changes Across the Life Span > Menopause > Treating Menopausal Symptoms Integrated Processes: Teaching and Learning Client Need: Physiological Integrity: Physiological Adaptation Cognitive Level: Analysis [Analyzing] Concept: Promoting Health Difficulty: Difficult Feedback 1 This is correct. Per the North American Menopause Society, hormone therapy is the most effective treatment for menopausal symptoms. 2 This is incorrect. There is no indication that hormone therapy decreases in 2 years. Length of use varies. 3 This is incorrect. Although alternative medicine use may enhance the hormone therapy, per the North American Menopause Society, hormone therapy is the most effective treatment for menopausal symptoms. 4 This is incorrect. Per the North American Menopause Society, hormone therapy is the most effective treatment for menopausal symptoms.

The nurse is providing care to a 46-year-old female patient. The patient appears hesitant when asked of her sexual history, and the nurse discovers that the patient is a lesbian. What education should the nurse provide to this patient pertaining to her health? 1. Lesbians often are at higher risk for menstrual disorders, abnormal insulin production, and infertility. 2. Lesbians often are at higher risk due to lower socioeconomic disparities than their heterosexual counterparts. 3. Lesbians often refuse choices in health care such as the HPV vaccine as many feel they are not at risk. 4. Lesbians often have fewer health issues than their heterosexual counterparts due to their strong support systems.

ANS: 1 Chapter: Chapter 18 Well Women's Health Chapter Learning Objective: 6. Describe the health care needs of lesbians and their barriers to health care. Page: 567 Heading: Specific Populations > Lesbian Health Integrated Processes: Teaching and Learning Client Need: Physiological Integrity: Reduction of Risk Potential Cognitive Level: Analysis [Analyzing] Concept: Promoting Health Difficulty: Moderate Feedback 1 This is correct. These are signs and symptoms of polycystic ovarian syndrome, which lesbians are at higher risk for than are their heterosexual counterparts. 2 This is incorrect. There is no indication of lower socioeconomic disparities among the LGBT population. 3 This is incorrect. There is no indication that LGBT population would refuse health care choices such as HPV vaccines. 4 This is incorrect. Due to the continued stigma, many members of LGBT population have lost the support of their blood family.

The nurse is providing care to the 24-year-old patient in the OB/GYN clinic. The patient states that she thinks she may be pregnant because she has missed three menstrual cycles. The patient says, "This couldn't be happening at a worse time. I have been training heavily for a triathlon in 4 months." Which response by the nurse is most appropriate? 1. "Secondary amenorrhea, or no menses for 3 months, is not always a sign of pregnancy; it may be due to your heavy athletic activity." 2. "Pregnancy during times of stress such as heavy athletic activity can occur when precautions are not followed." 3. "If the pregnancy test is negative, the physician will need to perform a diagnostic laparoscopy to rule out a neoplasm." 4. "Amenorrhea may occur at times of nutritional disturbances. Are you eating a well-balanced diet?"

ANS: 1 Chapter: Chapter 19 Alterations in Women's Health Chapter Learning Objective: 2. Discuss various causes of menstrual disorders. Page: 580 Heading: Table 19-2: Menstrual Disorders Integrated Processes: Communication and Documentation Client Need: Health Promotion and Maintenance Cognitive Level: Application [Applying] Concept: Patient-Centered Care Difficulty: Moderate Feedback 1 This is correct. Heavy athletic activity may cease ovulation and menstrual cycles. 2 This is incorrect. Stress may cause secondary amenorrhea, not pregnancy, and there is no indication that the patient has not taken precautions to prevent pregnancy. 3 This is incorrect. A laparoscopy is not indicated at this time. If the patient is not pregnant, the health care provider may order other diagnostics, such as an ultrasound. 4 This is incorrect. Although nutritional disturbances can often cause amenorrhea, there is no indication that this is the case with this patient.

The nurse is assessing a 33-year-old female patient who comes to the family practice clinic with complaints of excessive menstrual bleeding with bleeding between periods for the past 6 months. The health care provider has performed an endometrial biopsy and has discontinued the patient's oral contraceptives. What is the rationale for this treatment? 1. This condition is often associated with use of oral contraceptives and may subside upon discontinuation. 2. This condition is often associated with type 2 diabetes and must be controlled with a different type of birth control. 3. This condition is often associated with hyperplasia of the endometrial tissue, which is exacerbated with oral contraceptives. 4. This condition is often associated with ovarian cysts, which are exacerbated with the use of oral contraceptives.

ANS: 1 Chapter: Chapter 19 Alterations in Women's Health Chapter Learning Objective: 3. Describe common alterations in women's health, including medical management and nursing actions. Page: 581 Heading: Table 19-2: Menstrual Disorders Integrated Processes: Nursing Process Client Need: Physiological Integrity: Pharmacologic Therapies Cognitive Level: Analysis [Analyzing] Concept: Sexuality Difficulty: Moderate Feedback 1 This is correct. Metrorrhagia is the most significant form of menstrual disorder. If not caused by endometrial cancer, it is typically associated with use of an IUD and use of oral contraceptives. 2 This is incorrect. This is not associated with type 2 diabetes. PCOS is associated with type 2 diabetes, not metrorrhagia. 3 This is incorrect. Hyperplasia of endometrial tissue was ruled out with the biopsy. 4 This is incorrect. Metrorrhagia may be associated with ovarian cysts, and oral contraceptives may decrease ovulation or further trauma to the ovaries.

The nurse is preparing a teaching plan for a polycystic ovary syndrome (PCOS) support group. One of the teaching points will include diet and weight loss to decrease hirsutism and acne. What is the reasoning for this rationale? 1. Diet and exercise will promote weight loss, which will decrease hyperandrogenemia. 2. Diet and exercise will decrease the risk of type 2 diabetes and gestational diabetes. 3. Diet and exercise will decrease serum lipid levels and lower the risk for cardiovascular disease. 4. Diet and exercise will increase the frequency of ovulation and menstruation and increase fertility.

ANS: 1 Chapter: Chapter 19 Alterations in Women's Health Chapter Learning Objective: 3. Describe common alterations in women's health, including medical management and nursing actions. Page: 583 Heading: Polycystic Ovary Syndrome > Medical Management Integrated Processes: Teaching and Learning Client Need: Physiological Integrity: Physiological Adaptation Cognitive Level: Application [Applying] Concept: Health Promotion Difficulty: Moderate Feedback 1 This is correct. Hyperandrogenemia can lower testosterone levels, which can reduce the degree of acne and hirsutism. 2 This is incorrect. Hyperandrogenemia is a condition of high testosterone, not of abnormal blood sugars. 3 This is incorrect. Although weight loss can lower serum lipid levels, it does not directly affect testosterone levels. 4 This is incorrect. Although diet and exercise can increase frequency of ovulation and menstruation, the focus of the question is the level of testosterone.

The nurse is caring for a 33-year-old female patient who has just undergone laparoscopic removal of endometrial lesions. Which statement by the patient illustrates her understanding of the disease process and prognosis? 1. "I realize that this is not a cure for the issue, but I want to have more children within the next year." 2. "I'm happy to know that the surgeon has been able to cure me of this disease without a hysterectomy." 3. "I realize that my type 2 diabetes will be cured now that I have those lesions removed." 4. "I understand that I will not have to have a colostomy now that the growths were removed from my bowels."

ANS: 1 Chapter: Chapter 19 Alterations in Women's Health Chapter Learning Objective: 3. Describe common alterations in women's health, including medical management and nursing actions. Page: 584 Heading: Endometriosis > Medical Management Integrated Processes: Nursing Process Client Need: Health Promotion and Maintenance Cognitive Level: Application [Applying] Concept: Health Promotion Difficulty: Moderate Feedback 1 This is correct. Endometriosis may recur after surgical interventions, but the laparoscopic surgery is used for women with severe symptoms who are infertile and desire pregnancy. 2 This is incorrect. Endometriosis may recur after surgical interventions, but the laparoscopic surgery is used for women with severe symptoms who are infertile and desire pregnancy 3 This is incorrect. Type 2 diabetes is associated with polycystic ovarian syndrome, not endometriosis. 4 This is incorrect. There is no indication that the patient would have had to incur a colostomy for endometriosis.

The nurse is assessing a 72-year-old female patient in the women's health clinic. The mildly obese patient's history includes 7 vaginal births, 2 of which necessitated forceps deliveries, type 2 diabetes, and hypertension. The patient states that she has become incontinent of urine, has a history of frequent urinary tract infections (UTIs), and has a sense of fullness "down there." What would be an appropriate nursing intervention to help the patient with her symptoms? 1. Instruct the patient to stop urinating midstream at least twice a day. 2. Instruct the patient to eat a high-fiber diet and increase fluid intake. 3. Instruct the patient to add probiotics to their diet while taking antibiotics. 4. Instruct the patient to bear down effectively while having a bowel movement.

ANS: 1 Chapter: Chapter 19 Alterations in Women's Health Chapter Learning Objective: 3. Describe common alterations in women's health, including medical management and nursing actions. Page: 592 Heading: Pelvic Organ Prolapse > Cystocele and Rectocele > Nursing Actions Integrated Processes: Nursing Process Client Need: Physiological Integrity: Physiological Adaptation. Cognitive Level: Application [Applying] Concept: Elimination Difficulty: Moderate Feedback 1 This is correct. This is the Kegel exercise, which helps improve pelvic muscle strength for a mild cystocele. Key words in this question are frequent UTIs, urinary incontinence, multiple births, two of which were forceps deliveries, which further traumatizes the pelvic floor. 2 This is incorrect. This would be a teaching for a rectocele, not a cystocele. 3 This is incorrect. This measure would help prevent yeast infections while taking antibiotics, not a cystocele. 4 This is incorrect. This would exacerbate a rectocele and is not beneficial for a cystocele.

The nurse at a family practice clinic is providing care to a 47-year-old obese patient. The patient states that she realizes that she has put on extra weight but is reluctant to go to any exercise classes. She states that she often has to cross her legs when she sneezes and cannot do any exercises with her legs crossed. The health care provider has prescribed tolterodine (Detrol). What is the rationale for this medication? 1. Tolterodine is used to treat overactive bladders and to decrease urinary frequency, urgency, and urge incontinence. 2. Tolterodine is used to improve the tone and tissue in the urethral and vaginal areas. 3. Tolterodine is used to facilitate weight loss by acting as an appetite suppressant. 4. Tolterodine is used to improve blood flow to the pelvic muscles to decrease urinary tract infections.

ANS: 1 Chapter: Chapter 19 Alterations in Women's Health Chapter Learning Objective: 3. Describe common alterations in women's health, including medical management and nursing actions. Page: 593 Heading: Urinary Incontinence > Medical Management Integrated Processes: Nursing Process Client Need: Physiological Integrity: Pharmacological Therapies Cognitive Level: Application [Applying] Concept: Elimination Difficulty: Moderate Feedback 1 This is correct. Tolterodine (Detrol) inhibits cholinergic bladder contractions, thereby decreasing urinary frequency, urgency, and urge incontinence. 2 This is incorrect. Estrogen cream is applied to the genital tissues to improve tone and tissue in the urethral and vaginal areas. 3 This is incorrect. There is no indication that this would facilitate weight loss or suppress appetites. 4 This is incorrect. This does not decrease urinary tract infections, nor does it increase the blood blow to the pelvic muscles.

____ 4. The nurse is employed in a NICU. With each new admission, the neonate is classified in a specific category regarding care. The nurse is in the process of admitting a neonate at 22 weeks gestation with multiple life-threatening conditions involving both cardiac and respiratory systems. In which category of care does the nurse expect the neonate to be placed? 1. The category where aggressive care is probably futile. 2. The category where aggressive care is mostly uncertain. 3. The category where aggressive care is likely to be beneficial. 4. The category of "wait and see" to determine possible survival.

ANS: 1 Chapter: Chapter 2 Ethics and Standards of Practice Issues Chapter Learning Objective: 2. Debate ethical issues in maternity nursing. Page: 23 Heading: Ethics in Nursing Practice > Ethics in Neonatal Care Integrated Processes: Nursing Process Client Need: Physiological Integrity: Physiological Adaptation Cognitive Level: Analysis [Analyzing] Concept: Ante/Intra/Post-partum Difficulty: Moderate Feedback 1 This is correct. The neonate is notably premature with complex issues involving two major, life-sustaining body systems. Aggressive care is probably futile, and the prognosis for a meaningful life is extremely poor or hopeless. 2 This is incorrect. Aggressive care for this neonate is definitely uncertain; however, because of prematurity and profound life-threatening conditions, the neonate's prognosis is extremely poor or hopeless. 3 This is incorrect. The condition of the neonate does not support the possibility of aggressive care being beneficial. 4 This is incorrect. There is no "wait and see" category in the care of compromised neonates in NICU

A patient at 33 weeks gestation with a first pregnancy arrives at the labor and delivery unit with contractions. After monitoring the patient, the nurse determines the woman is in active labor and calls the health care provider (HCP), who prescribes a sleeping medication and sends the patient home. Which action does the nurse take? 1. Questions the HCP's prescribed treatment 2. Administers the medication and keeps the patient 3. Calls another HCP for a different prescription 4. Follows the HCP's instructions as prescribed

ANS: 1 Chapter: Chapter 2 Ethics and Standards of Practice Issues Chapter Learning Objective: 3. Explore standards of practice in maternity nursing. Page: 22 Heading: Ethics in Nursing Practice > ANA Code of Ethics Integrated Processes: Nursing Process Client Need: Safe and Effective Care Environment: Management of Care Cognitive Level: Application [Applying] Concept: Collaboration Difficulty: Moderate Feedback 1 This is correct. According to the ANA Code of Ethics, the nurse promotes, advocates for, and strives to protect the health, safety, and rights of all patients. The nurse needs to question the HCP's prescribed treatment. 2 This is incorrect. Administering the prescribed medication and keeping the patient on the unit is in direct conflict with the HCP's prescription. The nurse should question the HCP's prescribed treatment. 3 This is incorrect. It is unprofessional for the nurse to call another HCP for different orders. If the nurse is not successful in getting the prescribed treatment changed, the nursing supervisor is notified to discuss concerns with the prescribing HCP. 4 This is incorrect. Once the nurse determines the patient is in active labor, the nurse needs to use the ANA Code of Ethics to guide nursing actions. Following questionable treatment prescriptions does not relieve the nurse of responsibility.

The nurse is experiencing an ethical dilemma when confronted with a situation in which either the mother or fetus is predicted to die. The nurse feels bound by the ANA Code of Ethics to protect both patients. Which aspect of care during an ethical dilemma will guide the nurse? 1. Maternity nurses are bound to advocate first and foremost for the well-being of the mother. 2. The nurse is ethically bound to provide the best care for both the mother and fetus. 3. If the fetus is viable and healthy, its survival is the priority of the maternity nurse. 4. Survival of the mother is solely based on both patient and family decision making.

ANS: 1 Chapter: Chapter 2 Ethics and Standards of Practice Issues Chapter Learning Objective: 3. Explore standards of practice in maternity nursing. Page: 23 Heading: Ethics in Nursing Practice > Ethical Dilemmas Integrated Processes: Nursing Process Client Need: Physiological Integrity: Reduction of Risk Potential Cognitive Level: Application [Applying] Concept: Ante/Intra/Post-partum Difficulty: Difficult Feedback 1 This is correct. Practice dictates that the primary advocacy role of maternity nurses is on the behalf of the mother. 2 This is incorrect. Maternity nurses are expected to advocate for both maternal and fetal well-being; however, it is understood that it may not be to "do the right thing" for both. 3 This is incorrect. The loss of a healthy viable fetus is tremendous. However, during an ethical dilemma, the primary role of maternity nurses is on the behalf of the mother. 4 This is incorrect. It is never expected for the patient or family members to make a choice between the survival of the mother or the fetus.

The nurse works in a postpartum maternal-newborn unit and notices a newborn who is exhibiting signs of distress. Through investigation, the nurse learns the newborn's mother has a history of abusing street drugs. Which facility system does the nurse report as being deficient? 1. Risk management 2. Newborn monitoring 3. Patient information 4. Admission process

ANS: 1 Chapter: Chapter 2 Ethics and Standards of Practice Issues Chapter Learning Objective: 4. Describe legal issues in maternity nursing. Page: 29 Heading: Legal Issues in Delivery of Care > Risk Management Integrated Processes: Nursing Process Client Need: Safe and Effective Care Environment: Management of Care Cognitive Level: Application [Applying] Concept: Safety Difficulty: Moderate Feedback 1 This is correct. Of the options given, the deficiency is one related to risk management. A successful risk management program avoids preventable adverse outcomes and decreases the risk of liability through the use of appropriate, timely care, which accurately reflects maternal/fetal status before, during, and after interventions occur. 2 This is incorrect. The scenario does not contain information to support poor newborn monitoring. 3 This is incorrect. It can be assumed that patient information was available; however, the issue rests with the fact that it was not made readily to the postpartum nurse. 4 This is incorrect. The admission process is not likely the facility system that is deficient. Admission processes are not always performed by nurses; non-nursing personnel are not responsible for reporting concerns regarding risk management.

The nurse is counseling a couple who just learned their 16-week fetus tested positive for a serious genetic disorder. Which action by the nurse is inappropriate upon learning that the couple plan to continue the pregnancy? 1. Determine whether termination is still a consideration. 2. Explain they will experience grief, which is normal. 3. Provide additional information about the disorder. 4. Refer them to a disorder-specific support group.

ANS: 1 Chapter: Chapter 3 Genetics, Conception, Fetal Development, and Reproductive Technology Chapter Learning Objective: 1. Discuss the relevance of genetics within the context of the care of the childbearing family. Page: 39 Heading: Genetics and the Childbearing Family > Genetic Testing Integrated Processes: Nursing Process Client Need: Physiological Integrity: Physiological Adaptation Cognitive Level: Analysis [Analyzing] Concept: Family Dynamics Difficulty: Difficult Feedback 1 This is correct. Once the couple has decided to continue the pregnancy, it is inappropriate and unnecessary for the nurse to determine pregnancy termination is still a consideration. It is likely that counseling from family and medical personnel has helped with their difficult decision. 2 This is incorrect. The nurse needs to prepare the couple for the experience of grief over the loss of their "dream child." 3 This is incorrect. The nurse needs to provide the couple with additional information about the disorder, including reputable and accurate websites. 4 This is incorrect. The nurse can refer the couple to a disease-specific support group if one is available. The couple should at least consider attending a support group for parents with children born with defects.

Genomic medicine is an emerging medical discipline that involves using genomic information about an individual as part of the individual's clinical care. Which example does the nurse associate with genomic medicine? 1. Screening of neonates for inherited, treatable genetic diseases 2. Creating drugs specifically for the treatment of cancer 3. Conducting trial studies to determine how drugs effect individuals 4. Tracing and gaining knowledge about how genetic mutations occur

ANS: 1 Chapter: Chapter 3 Genetics, Conception, Fetal Development, and Reproductive Technology Chapter Learning Objective: 1. Discuss the relevance of genetics within the context of the care of the childbearing family. Page: 39 Heading: Genetics and the Childbearing Family > Genomic Medicine Integrated Processes: Nursing Process Client Need: Physiological Integrity: Reduction of Risk Potential Cognitive Level: Application [Applying] Concept: Patient-Centered Care Difficulty: Moderate Feedback 1 This is correct. An example of genomic medicine is screening of neonates for inherited, treatable genetic diseases. 2 This is incorrect. Genomic medicine focuses on the human genome to obtain valuable information to be used in the diagnosis, treatment, and prevention of genetically linked diseases. Cancer is not considered a genetic disease, but genomic medicine is looking for cell-free circulating DNA as a biomarker for cancer. 3 This is incorrect. Genomic medicine is not focused on trial studies to determine how drugs effect individuals. 4 This is incorrect. Genomic medicine does not focus on tracing and gaining knowledge about how genetic mutations occur.

A patient at 34 weeks gestation is undergoing an ultrasound. The nurse notes that the amniotic fluid is estimated at between 500 and 600 mL. Which deduction does the nurse make from this finding? 1. Oligohydramnios is present. 2. Fluid is normal for gestation age. 3. Polyhydramnios has formed. 4. Follow-up ultrasound is warranted.

ANS: 1 Chapter: Chapter 3 Genetics, Conception, Fetal Development, and Reproductive Technology Chapter Learning Objective: 3. Describe the development and function of the placenta and amniotic fluid. Page: 51 Heading: Placenta, Membranes, Amniotic Fluid, and Umbilical Cord > Amniotic Fluid Integrated Processes: Nursing Process Client Need: Physiological Integrity: Physiological Adaptation Cognitive Level: Analysis [Analyzing] Concept: Ante/Intra/Post-partum Difficulty: Difficult Feedback 1 This is correct. The volume of amniotic fluid at 34 weeks gestation should peak at 800 to 1,000 mL. The current volume indicates oligohydramnios, which is indicative of a decrease in placental function. The newborn is at increased risk for congenital renal problems. 2 This is incorrect. The amount of fluid is not normal; volume should be between 800 and 1,000 mL at 34 weeks gestation. 3 This is incorrect. Polyhydramnios refers to an excess amount of fluid between 1,800 to 2,000 mL. This condition increases the incidence of chromosomal disorders and gastrointestinal, cardiac, and neural tube disorders. 4 This is incorrect. This finding may or may not warrant a repeat of ultrasound testing.

A patient at 37 weeks gestation arrives at the labor and delivery unit and reports a rupture of her membranes. Which factor causes the nurse to anticipate the HCP will prescribe a medical method of labor induction? 1. The fetus is viable and the barrier for a sterile uterine environment is breached. 2. The fetus is at risk for "drying out" and causing the mother to have a dry birth. 3. The mother must be maintained on complete bedrest until contractions begin. 4. The mother is at risk for developing an infection and passing it to the fetus.

ANS: 1 Chapter: Chapter 3 Genetics, Conception, Fetal Development, and Reproductive Technology Chapter Learning Objective: 3. Describe the development and function of the placenta and amniotic fluid. Page: 51 Heading: Placenta, Membranes, Amniotic Fluid, and Umbilical Cord > Embryonic Membranes Integrated Processes: Nursing Process Client Need: Physiological Integrity: Reduction of Risk Potential Cognitive Level: Analysis [Analyzing] Concept: Ante/Intra/Post-partum Difficulty: Moderate Feedback 1 This is correct. The nurse anticipates a prescription for a medical method of labor induction because the fetus is viable and the sterile environment of the uterus is breached when the embryonic membranes rupture. 2 This is incorrect. Once the embryonic membranes rupture, amniotic fluid continues to be produced. The fetus is not at risk for "drying out" and the mother will not experience a "dry birth." 3 This is incorrect. The mother will be closely monitored for contractions and fetal well-being. However, she may not remain on bedrest until contractions begin. The HCP may delay inducing labor to see if contractions begin spontaneously within 24 hours; after 24 hours, labor is induced to prevent complications from the loss of uterine sterility. 4 This is incorrect. The fetus is at risk for bacteria from the vagina entering the uterus and causing an infection. The mother does not develop an infection and pass it to the fetus.

The nurse is collecting health information from a patient who is early in the first trimester of pregnancy. Which topic is most important for the nurse to discuss with the patient after learning that the patient works for a commercial cleaning company? 1. Risk related to exposure to environmental toxins 2. Weight limit for lifting during the patient's pregnancy 3. Importance of resting with feet up during the day 4. Reasons for the patient to look for a safer job

ANS: 1 Chapter: Chapter 4 Physiological Aspects of Antepartum Care Chapter Learning Objective: 1. Identify the major components of preconception health care. Page: 73 Heading: Preconception Health Care > Preconception Education > Self-Care Integrated Processes: Nursing Process Client Need: Physiological Integrity: Reduction of Risk Potential Cognitive Level: Analysis [Analyzing] Concept: Ante/Intra/Post-partum Difficulty: Moderate Feedback 1 This is correct. Exposure to environmental toxins increases the risk for miscarriage, preterm birth, and other complications. The patient's job may involve exposure to solvents and/or cleaning chemicals. 2 This is incorrect. The nurse needs to ascertain if the patient's job involves lifting and make appropriate recommendations; however, this is not currently the most important topic. 3 This is incorrect. Pregnancy sometimes causes edema in the feet and legs, which can be reduced by elevating the feet and legs periodically during the day. However, this is not currently the most important topic. 4 This is incorrect. The nurse needs to explain any identifiable risk the patient's job may create during pregnancy. The nurse will be open to discussion and suggestions for the patient's concerns. It is not appropriate for the nurse to suggest the patient find a safer job, which may not be possible since financial constraints will also cause risks.

A pregnant patient is at the prenatal clinic for a routine visit at 30 weeks gestation. The nurse monitors the patient for indications of physiological demands by the fetus on the patient. Which finding causes the nurse concern? 1. Hgb of 9.5 g/dL and Hct. of 30% 2. PT of 16.5 seconds 3. WBCs of 16,000 mm3 4. Heart rate up 20 bpm

ANS: 1 Chapter: Chapter 4 Physiological Aspects of Antepartum Care Chapter Learning Objective: 3. Identify the anatomical and physiological changes over the course of pregnancy. Page: 58 Heading: Table 4-1: Physiological Changes in Pregnancy Integrated Processes: Nursing Process Client Need: Physiological Integrity/Reduction of Risk Potential Cognitive Level: Analysis [Analyzing] Concept: Ante/Intra/Post-partum Difficulty: Difficult Feedback 1 This is correct. The patient's hemoglobin and hematocrit are below normal for the patient. This finding causes the nurse concern because the increased demand of iron for fetal development results in maternal iron deficiency anemia. 2 This is incorrect. The patient's PT is indicative of hypercoagulability, which is an expected physiological response to pregnancy in anticipation of blood loss during delivery. Normal PT is in the range of 11 to 13.5 seconds. This finding does not cause the nurse concern. 3 This is incorrect. WBC count of 16,000 mm3 is not abnormal in a patient who is at 30 weeks gestation, especially if there are no other indications of infection. The scenario does not indicate manifestations of infection. 4 This is incorrect. A 15 to 20 bpm increase in heart rate is expected due to a 40% increase in cardiac output. This finding does not cause the nurse concern.

A patient in the first trimester of pregnancy states, "I don't understand how a term baby can be accommodated by my uterus." Which information by the nurse specifically addresses the patient's comment? 1. The uterus size increases in size 20 times over a nonpregnant uterus. 2. The weight of the uterus increases from 7 g to 1,100 g during pregnancy. 3. About 80% of the increased capacity of the uterus is related to uteroplacental content. 4. About 75% of the increase in uterus size during pregnancy is related to stretching.

ANS: 1 Chapter: Chapter 4 Physiological Aspects of Antepartum Care Chapter Learning Objective: 7. Describe the elements of patient education and anticipatory guidance appropriate for each trimester of pregnancy. Page: 58 Heading: Table 4-1: Physiological Changes in Pregnancy Integrated Processes: Nursing Process Client Need: Physiological Integrity: Physiological Adaptation Cognitive Level: Analysis [Analyzing] Concept: Ante/Intra/Post-partum Difficulty: Difficult Feedback 1 This is correct. The information that specifically addresses the patient's comment about how her uterus will accommodate a term baby is clarified with the fact that the uterus increases in size 20 times over the non-pregnant size. 2 This is incorrect. The weight of the uterus does increase from 7 g to 1,100 g; however, this fact does not specifically address the patient's comment. 3 This is incorrect. Eighty percent of the increased uterine capacity is related to uteroplacental content; however, this information does not specifically address the patient's comment. A better factor is the increase in uterine capacity from 10 mL to 5,000 mL. 4 This is incorrect. Both stretching and growth are involved in the increase in the size of the uterus to accommodate the developing fetus. However, an increase from 7 g to 1,000 g supports a greater amount of growth instead of stretching.

The labor and delivery nursing staff is conducting research to determine the benefits of childbirth education (CBE). Which finding does evidence-based practice support? 1. Women who participated in CBE and/or had a birth plan had higher odds of a vaginal delivery. 2. Women who are considered to be at high risk had fewer complications if CBE or a birth plan was used. 3. Women of color, younger in age, and who are multipara respond best to CBE and/or a birth plan. 4. Women with a previous cesarean delivery are more likely to have a vaginal delivery after CBE.

ANS: 1 Chapter: Chapter 5 The Psycho-Social-Cultural Aspects of the Antepartum Period Chapter Learning Objective: 7. Analyze and critique current evidence-based research in psycho-social-cultural adaptation to pregnancy Page: 125 Heading: Planning for Birth > Childbirth Education Integrated Processes: Nursing Process Client Need: Health Promotion and Maintenance Cognitive Level: Analysis [Analyzing] Concept: Evidence-Based Practice Difficulty: Difficult Feedback 1 This is correct. According to Afshar et al., 2017, and Gagnon, 2011, women who participated in CBE and/or had a birth plan had higher odds of a vaginal delivery. 2 This is incorrect. The research did not address women who were at risk during pregnancy or labor and delivery. 3 This is incorrect. Afshar et al., 2017, and Gagnon, 2011, state that women who attended CBE or had a birth plan were older, more likely to be nulliparous, had a lower body mass index, and were less likely to be African American. 4 This is incorrect. Research by Afshar et al., 2017, and Gagnon, 2011, indicates that individualized prenatal education directed toward avoidance of a repeat caesarean birth does not increase the rate of vaginal birth after caesarean section

A couple announces to their parents that the couple is pregnant. One expectant grandmother says, "Grandchildren will call me by my first name. I am not ready to be a grandmother." Which feelings are being expressed by the grandmother? 1. The pregnancy presents undeniable evidence the grandmother is growing older. 2. The grandmother has specific wishes about how she is to be addressed as a person. 3. The grandmother is most likely teasing and actually feels overwhelming delight. 4. The grandmother has never thought of herself in this role and will adapt with time.

ANS: 1 Chapter: Chapter 5 The Psycho-Social-Cultural Aspects of the Antepartum Period Chapter Learning Objective: 2. Identify the major developmental tasks of pregnancy as they relate to maternal, paternal, and family adaptation. Page: 114 Heading: Family Adaptation During Pregnancy > Grandparent Adaptation Integrated Processes: Nursing Process Client Need: Psychosocial Integrity Cognitive Level: Analysis [Analyzing] Concept: Family Dynamics Difficulty: Moderate Feedback 1 This is correct. The feelings expressed in the grandmother's comment is in response to undeniable evidence that the grandmother is growing older. 2 This is incorrect. The grandmother does have specific wishes about how she wishes to be addressed as a person; however, those wishes are most likely attached to a denial of aging. 3 This is incorrect. The announcement of a new baby is an occasion for celebration. The grandmother's comment is inappropriate and will be a probable source of hurt feelings. 4 This is incorrect. Many grandparents cannot envision themselves as grandparents and will adapt over time to the role. However, the comment is most likely related to aging.

The nurse is providing care in a school clinic established for the care of adolescent mothers. When assessing a patient who is 11 years of age and pregnant, which deduction regarding the patient's psychosocial development will the nurse recognize? 1. The adolescent is self-centered and oriented toward the present. 2. At this age, pregnancy is likely a result of attachment to a first love. 3. Moving into the mothering role will be nearly impossible at this age. 4. The role of the grandmother will be as the baby's primary caretaker.

ANS: 1 Chapter: Chapter 5 The Psycho-Social-Cultural Aspects of the Antepartum Period Chapter Learning Objective: 3. Identify critical variables that influence adaptation to pregnancy, including age, parity, and social, cultural, and sexual orientation. Page: 105 Heading: Maternal Adaptation to Pregnancy > Factors That Influence Maternal Adaptation > Maternal Age > Adolescent Mothers Integrated Processes: Nursing Process Client Need: Psychosocial Integrity Cognitive Level: Application [Applying] Concept: Family Difficulty: Moderate Feedback 1 This is correct. Pregnancy in early adolescence is difficult because the adolescent is self-centered and oriented toward the present, which makes maternal adaptation to pregnancy difficult and interferes with mothering. 2 This is incorrect. In early adolescence, pregnancy is most likely related to coercion or abuse. 3 This is incorrect. Moving into the mothering role will be difficult for the adolescent; however, it will not be impossible if teaching, role models, and support are available. 4 This is incorrect. Grandmothers will play a significant role in caring for the infant as well as providing guidance to their daughter regarding mothering skills. However, it should not become the grandmother's role to become the primary care provider for the baby.

A patient in the second trimester of pregnancy seems distressed. With encouragement, the patient states, "I have been totally avoiding physical contact with my husband to avoid prompting any sexual activity." Which statement by the nurse is the best response? 1. "Nonsexual expressions of affection are important for both of you." 2. "Be honest and tell your husband the reason you are avoiding him." 3. "You need to agree to sex in order to prevent infidelity from occurring." 4. "Sex during pregnancy is a healthy and normal display of affection."

ANS: 1 Chapter: Chapter 5 The Psycho-Social-Cultural Aspects of the Antepartum Period Chapter Learning Objective: 7. Analyze and critique current evidence-based research in psycho-social-cultural adaptation to pregnancy. Page: 112 Heading: Sexuality in Pregnancy Integrated Processes: Nursing Process Client Need: Psychosocial Integrity Cognitive Level: Application [Applying] Concept: Family Dynamics Difficulty: Moderate Feedback 1 This is correct. The nurse needs to inform the patient that nonsexual expressions are important during pregnancy to both partners. 2 This is incorrect. If the nurse tells the patient to be honest and share her reasons for avoiding her husband, the patient will not change her behaviors. Continuation of avoidance can create tension in the relationship. 3 This is incorrect. The nurse should not tell the patient that having sex is a method to prevent infidelity. The patient has a right to avoid sex without guilt; however, displaying affection is still important. 4 This is incorrect. Sex can be a healthy, normal display of affection during pregnancy; however, both partners must be willing participants. The patient may have physical changes that make sex undesirable.

The nurse is providing care for a 45-year-old patient who has just learned she is in the second trimester of pregnancy. The patient thought she was experiencing manifestations of menopause until she recognized fetal movement. Which diagnostic test does the nurse expect to be prescribed for this patient? 1. Amniocentesis 2. Ultrasonography 3. Daily fetal movement count 4. Chorionic villi sampling

ANS: 1 Chapter: Chapter 6 Antepartal Tests Chapter Learning Objective: 2. Identify the purpose and indication for key antenatal tests Page: 133 Heading: Screening and Diagnostic Tests Integrated Processes: Nursing Process Client Need: Physiological Integrity: Reduction of Risk Potential Cognitive Level: Analysis [Analyzing] Concept: Ante/Intra/Post-partum Difficulty: Moderate Feedback 1 This is correct. Due to the age of the patient and the period of gestation, the nurse expects amniocentesis to be performed. The test is appropriate between 15 and 20 weeks of gestation and for detection of genetic disorders in mothers older than age 35 years. 2 This is incorrect. Ultrasonography is considered both a screening and diagnostic test; however, because of the patient's age, a test for genetic disorders is expected by the nurse. This test does not specifically identify genetic issues. 3 This is incorrect. A daily fetal movement count is a screening test to monitor for fetal well-being. This screening does not address concerns caused by maternal age. 4 This is incorrect. Chorionic villa sampling is a diagnostic test used for chromosomal analysis between 10 and 12 weeks gestation to detect fetal abnormalities caused by genetic disorders. The patient is past this time parameter.

A patient is in her first trimester of her second pregnancy. The patient's first child was born with a trisomy 21 defect. The patient is requesting testing to determine whether the current fetus has the same defect. Which initial testing does the nurse expect the HCP to prescribe? 1. Fetal ultrasound 2. Magnetic resonance imaging 3. Chorionic villa sampling 4. Amniocentesis

ANS: 1 Chapter: Chapter 6 Antepartal Tests Chapter Learning Objective: 2. Identify the purpose and indication for key antenatal tests. Page: 135 Heading: Biophysical Assessment > Fetal Ultrasound Imaging > Interpretation of Results Integrated Processes: Nursing Process Client Need: Physiological Integrity: Reduction of Risk Potential Cognitive Level: Analysis [Analyzing] Concept: Ante/Intra/Post-partum Difficulty: Difficult Feedback 1 This is correct. Fetal ultrasound in the first trimester of pregnancy can be performed for nuchal translucency, which measures the midsagittal plane with the neck of the fetus to assess the amount of fluid behind the neck. An elevated measurement is associated with trisomy 21. This is the initial test the nurse can expect; results may require further diagnostic testing. 2 This is incorrect. Magnetic resonance imaging (MRI) is primarily done to identify issues with brain development abnormalities. Trisomy 21 cannot specifically be detected with this test. 3 This is incorrect. Chronic villa sampling may be considered if the nuchal translucency test indicates a possibility of trisomy 21. This test is invasive and carries a 7% chance of interrupting the pregnancy; therefore, other initial screening is expected. 4 This is incorrect. Amniocentesis performed in the second trimester is effective in diagnosing genetic disorders. However, the test is invasive and involves some risk to the pregnancy. Initially, the screening test for nuchal translucency is expected.

An Eastern European Jewish couple had two children who died from Tay-Sachs disease. The couple is currently pregnant and have asked for genetic confirmation about this fetus with the intention of early termination if the fetus tests positively. For which reason does the nurse expect chorionic villa sampling to be prescribed? 1. The test is performed as early as 10 weeks gestation. 2. Risks to the fetus and mother are less than other tests. 3. A positive result allows termination during the test. 4. This is the only testing that is disease specific.

ANS: 1 Chapter: Chapter 6 Antepartal Tests Chapter Learning Objective: 3. Describe the procedure, interpretation, advantages, and risks of common antenatal tests. Page: 138 Heading: Biochemical Assessment > Chorionic Villus Sampling Integrated Processes: Nursing Process Client Need: Physiological Integrity: Reduction of Risk Potential Cognitive Level: Analysis [Analyzing] Concept: Ante/Intra/Post-partum Difficulty: Difficult Feedback 1 This is correct. Chorionic villa testing can be performed as early as 10 weeks gestation. Given the reproductive history of the couple and their expectations, this is the test the nurse should expect to be prescribed. 2 This is incorrect. The risks are higher for chorionic villa testing than for amniocentesis; there is a 7% fetal loss related to bleeding, infection, and rupture of membranes. However, amniocentesis is not performed until at least 15 weeks gestation. 3 This is incorrect. Results of chromosomal studies from a chorionic villa testing are available within 1 week. It is not possible to perform a termination during the initial testing. 4 This is incorrect. Chorionic villa testing is not specific to Tay-Sachs disease; detailed information is provided on any specific chromosomal abnormality detected. The testing is effective in detecting DNA or metabolic disorders.

When performing a physical assessment on a patient during the initial prenatal visit, the nurse notes spongy gums prone to bleeding during the oral exam. Which comment by the nurse is appropriate? 1. "Oral bleeding can contribute to anemia." 2. "Dental problems can interfere with nutrition." 3. "Periodontal disease is a risk factor for preterm labor" 4. "You need dental care because pregnancy causes dental problems."

ANS: 1 Chapter: Chapter 7 High-Risk Antepartum Nursing Care Chapter Learning Objective: 3. Identify potential antenatal complications for the woman, the fetus, and the newborn. Page: 153 Heading: Gestational Complications > Preterm Labor and Birth > Risk Factors for Preterm Labor and Birth Integrated Processes: Nursing Process Client Need: Physiological Integrity: Reduction of Risk Potential Cognitive Level: Application [Applying] Concept: Ante/Intra/Post-partum Difficulty: Difficult Feedback 1 This is incorrect. Oral bleeding would need to be consistent and significant to be a contributing factor to anemia. 2 This is incorrect. Periodontal disease may or may not interfere with meeting nutritional needs during pregnancy. Another comment is more applicable. 3 This is correct. Infection is considered a risk factor for preterm labor and birth. This is especially true for genitourinary infections and periodontal disease. 4 This is incorrect. It is important for all persons to see a dentist regularly, and patients who are pregnant should not neglect this care. Another comment specifically addresses the condition in the scenario.

The nurse in a prenatal unit is providing care for a patient who experienced PPROM at 32 weeks gestation. Which assessment does the nurse consider unnecessary? 1. Check for cervical dilation 2. Monitor for signs of infection 3. Assess for vaginal bleeding 4. Watch for fetal compromise

ANS: 1 Chapter: Chapter 7 High-Risk Antepartum Nursing Care Chapter Learning Objective: 4. Formulate a plan of care that includes the physical, emotional, and psychosocial needs of women diagnosed with pregnancy complications. Page: 159 Heading: Gestational Complications > Preterm Premature Rupture of Membranes/Chorioamnionitis > Medical Management Integrated Processes: Nursing Process Client Need: Physiological Integrity: Reduction of Risk Potential Cognitive Level: Application [Applying] Concept: Ante/Intra/Post-partum Difficulty: Moderate Feedback 1 This is correct. PPROM places the mother and fetus at risk for infection from migration of bacteria from the vagina and/or introduced from the environment. The nurse will not check the patient for cervical dilation. 2 This is incorrect. Due to PPROM, the nurse will assess the patient for signs of infection. Manifestations may include fever, uterine tenderness, malodorous vaginal drainage or discharge, and maternal and/or fetal tachycardia. 3 This is incorrect. After PPROM, the patient should be checked for vaginal bleeding, which can be indicative of placenta abruption. 4 This is incorrect. The nurse will monitor the fetus for signs of compromise, including changes in or absence of fetal heart rate, and decrease or absence of fetal movement.

The nurse is providing care to a postpartum patient after an emergency cesarean due to eclampsia. The patient received spinal anesthesia prior to delivery. Magnesium sulfate is infusing 2 g/hr in 100 mL of IV fluid. Which assessment finding will cause the nurse to administer calcium gluconate to the patient via IV push? 1. Serum magnesium level is 10 mg/dL. 2. Patella reflexes are rated at zero. 3. Respiratory rate is 18 breaths/min. 4. Urinary output remains at 30 mL/hr.

ANS: 1 Chapter: Chapter 7: High-Risk Antepartum Nursing Care Chapter Learning Objective: 2. Delineate clinical features indicative of pregnancy complications and tests to predict, screen for, diagnose, and manage high-risk conditions. Page: 172 Heading: Hypertension in Pregnancy Integrated Processes: Nursing Process Client Need: Physiological Integrity: Reduction of Risk Potential Cognitive Level: Analysis [Analyzing] Concept: Ante/Intra/Post-partum Difficulty: Difficult Feedback 1 This is correct. The therapeutic serum level of magnesium sulfate is 5 to 7 mg/dL, and the patient's laboratory result is 10 mg/dL. The nurse will give the antidote of calcium gluconate (5 to 10 mEq) by IV over a period of 5 to 10 minutes. 2 This is incorrect. Patella reflexes at zero are not a concern to cause the nurse to give the patient calcium gluconate. The lack of lower extremity reflexes is likely related to the residual effects of spinal anesthesia. 3 This is incorrect. A respiratory rate of 18 breaths/min is within normal limits. 4 This is incorrect. Urinary output of 30 mL/hour is within normal limits.

The nurse is caring for a 24-year-old woman who is G1P0 at 40 weeks, 1 day gestation and in active labor. She has just received an epidural and now complains of "an itchy feeling all over." Her vitals are as follows: HR 120, RR 12, BP 130/74, T 98.8, and O2 sat 98%. Which action should the nurse take first? 1. Call the health care provider regarding the patient's pruritus to order an antipruritic medication. 2. Activate emergency response due to the patient's pruritus and tachycardia postepidural placement. 3. Call the anesthesiologist regarding the patient's oxygen saturation level. 4. Take no further action regarding the patient's complaints, as they are normal after epidural placement.

ANS: 1 Chapter: Chapter 8 Intrapartum Assessment and Interventions Chapter Learning Objective: Demonstrate understanding of supportive care of the laboring woman. Page: 262 Heading: Pharmacological Management of Labor Discomfort > Epidural Anesthesia > Nursing Actions During Administration of Epidural Anesthesia Integrated Processes: Nursing Process Client Need: Safe and Effective Care Environment: Management of Care Cognitive Level: Analysis [Analyzing] Concept: Ante/Intra/Post-partum Difficulty: Difficult Feedback 1 This is correct. Ninety percent of women who receive opioids in epidural have itching, and the health care provider should be notified so the correct medications can be ordered and administered to treat the pruritus. 2 This is incorrect. The patient's vital signs do not indicate that she is having a critical response to the epidural placement. 3 This is incorrect. The patient's oxygen saturation is normal. 4 This is incorrect. The patient is tachycardic and pruritic. These symptoms require contacting a health care provider for further action.

The nurse is monitoring the fetal heart rate (FHR) tracing and sees that her patient has a tracing with a baseline of 120, moderate variability, with absence of decelerations and accelerations. According to the National Institute of Child Health and Human Development tier system, what category tracing does the patient's fall into? 1. A Category I tracing 2. A Category II tracing 3. A Category III tracing 4. A Category IV tracing

ANS: 1 Chapter: Chapter 9 Fetal Heart Rate Assessment Chapter Learning Objective: Distinguish between Category I, II, and III patterns and appropriate nursing actions based on these interpretations. Page: 281 Heading: Fetal Reserves > NICHD Criteria for Interpretation of FHR Patterns Integrated Processes: Nursing Process Client Need: Physiological Integrity: Reduction of Risk Potential Cognitive Level: Application [Applying] Concept: Ante/Intra/Post-partum Difficulty: Moderate Feedback 1 This is correct. This is a normal tracing. 2 This is incorrect. A Category II is an indeterminate tracing, and the patient's tracing has normal components 3 This is incorrect. A Category III tracing is an abnormal tracing, and the patient's tracing has normal components 4 This is incorrect. There is no such category as a Category IV tracing.

The nurse is providing care for a patient who delivered via cesarean 24 hours ago. Which teaching does the nurse provide for the patient and family? Select all that apply. 1. Signs and symptoms to report to health care provider 2. Comfortable positions for feeding the newborn 3. Encouragement for early dietary intake of solid foods 4. Encourage family to help with infant care and housework 5. Provide information on nutrition to promote tissue healing

ANS: 1, 2 Chapter: Chapter 11 Intrapartum and Postpartum Care of Cesarean Birth Families Chapter Learning Objective: 4. Discuss the postoperative nursing care of cesarean birth with women and their families. Page: 357 Heading: Postoperative Care > 24 Hours Postoperative to Discharge Integrated Processes: Nursing Process Client Need: Physiological Integrity: Reduction of Risk Potential Cognitive Level: Analyze [Analyzing] Concept: Ante/Intra/Post-partum Difficulty: Difficult Feedback 1 This is correct. The nurse teaches the patient and family about the signs and symptoms that need to be reported to the health care provider. Patients with cesarean deliveries are more prone to infection. 2 This is correct. Due to abdominal tenderness related to cesarean birth, the nurse will need to teach the mother and family about comfortable positions for feeding the newborn. Breastfeeding mothers may be more comfortable in a side-lying position. 3 This is correct. Following a cesarean, patients who eat solid foods early rather than waiting for the presence of bowel sounds have shown an earlier return of bowel function. 4 This is correct. Because the recovery for a cesarean will be 6 weeks or longer, the nurse will encourage family to help with infant care and housework. 5 This is correct. Because the patient has an abdominal incision, the nurse needs to provide teaching for a diet that will promote healing.

The nurse is providing teaching to a patient who is breastfeeding a newborn. The patient expresses interest in maintaining a healthy nutritional status for both her and her baby. Which information does the nurse present to meet the patient's need? Select all that apply. 1. Increase caloric intake by 500 to 1,000 per day. 2. Drink 2 to 3 liters of fluid each day. 3. Abstain from the intake of alcohol. 4. Eat fresh fruits and vegetables. 5. Avoid the intake of processed foods.

ANS: 1, 2 Chapter: Chapter 12 Postpartum Physiological Assessments and Nursing Care Chapter Learning Objective: 4. Describe the critical elements of discharge teaching. Page: 382 Heading: Health Promotion > Nutrition and Fluids Integrated Processes: Nursing Process Client Need: Physiological Integrity: Physiological Adaptation Cognitive Level: Analysis [Analyzing] Concept: Ante/Intra/Post-partum Difficulty: Difficult Feedback 1 This is correct. The lactating mother should increase her calorie intake by 500 to 1,000 calories daily. 2 This is correct. The lactating mother will need to drink approximately 2 to 3 liters of fluid daily. 3 This is incorrect. There is no evidence that an occasional alcoholic drink is harmful for the lactating mother; however, the long-term effects of daily alcohol use on breastfeeding infants is unknown, so alcoholic beverages should be kept to a minimum. The mother should wait 2 to 2.5 hours per drink before breastfeeding. 4 This is incorrect. Eating fresh fruits and vegetables is healthy for the mother; however, the mother needs to be alert that some fruits and vegetables may give the baby gas, cramps, and/or loose stools. 5 This is incorrect. There is no particular reason the mother should avoid processed foods completely; however, the mother should be conscious of consuming a healthy, balanced diet.

A nurse has recently transferred to a labor and delivery unit. During a scheduled cesarean, the nurse notices a prescription for the administration of 1,000 mL of prewarmed IV fluid. For which reason does the health care provider prescribe the fluid in this manner? Select all that apply. 1. Results in an increased maternal core temperature 2. Results in improved neonatal umbilical arterial pH 3. Results in improved Apgar scores 4. Results in decreased risk for maternal shock 5. Results in lower incidence of post-partum hypothermia

ANS: 1, 2, 3 Chapter: Chapter 11 Intrapartum and Postpartum Care of Cesarean Birth Families Chapter Learning Objective: 3. Describe the intraoperative nursing care and medical and anesthesia management for cesarean births. Page: 349 Heading: Perioperative Care > Scheduled Cesarean Birth Integrated Processes: Nursing Process Client Need: Physiological Integrity: Reduction of Risk Potential Cognitive Level: Analysis [Analyzing] Concept: Ante/Intra/Post-partum Difficulty: Difficult Feedback 1 This is correct. Prewarmed IV fluid administration in women with cesarean births results in an increased maternal core temperature. 2 This is correct. Prewarmed IV fluid administration in women with cesarean births results in improved neonatal umbilical arterial pH. 3 This is correct. Prewarmed IV fluid administration in women with cesarean births results in improved Apgar scores. 4 This is incorrect. Prewarmed IV fluid administration in women with cesarean births does not result in decreased risk for maternal shock. 5 This is incorrect. Prewarmed IV fluid administration in women with cesarean births does not result lower incidence of postpartum hypothermia.

The nurse in a postpartum unit evaluates new parents for risk factors that can indicate problems with bonding/attachment. Which situations does the nurse recognize as a cause for bonding/attachment problems? Select all that apply. 1. The mother experienced eclampsia in the third trimester of pregnancy. 2. The neonate is being treated for meconium aspiration syndrome. 3. The mother experienced dystocia in the second phase of labor. 4. The father of the neonate is in the military and not yet home on leave. 5. The mother's mother lives next door and is available to help with the baby.

ANS: 1, 2, 3 Chapter: Chapter 13 Transition to Parenthood Chapter Learning Objective: 3. Discuss bonding and attachment. Pages: 404 Heading: Bonding and Attachment > Bonding and Attachment Behaviors > Risk Factors for Delayed Bonding and/or Attachment Integrated Processes: Nursing Process Client Need: Psychosocial Integrity Cognitive Level: Analysis [Analyzing] Concept: Family Dynamics Difficulty: Difficult Feedback 1 This is correct. The nurse recognizes the mother who experienced eclampsia in the third trimester of pregnancy as being at risk for bonding/attachment problems. 2 This is correct. The nurse recognizes the mother whose neonate is being treated for meconium aspiration syndrome as being at risk for bonding/attachment problems. The mother is likely to be separated from the neonate. 3 This is correct. The nurse recognizes the parents of a neonate may experience bonding/attachment problems if the mother experienced dystocia in the second stage of labor. The condition is likely to have caused a long and exhaustive labor for both parents. 4 This is incorrect. When a neonate's father is delayed from contact with the neonate, problems with bonding/attachment will not necessarily occur. The mother is still available for the bonding process. 5 This is incorrect. The nurse recognizes that the close proximity of the mother's mother is not likely to cause bonding/attachment problems.

Which factors will facilitate the integration of evidence-based practice (EBP) in the maternity-newborn clinical setting? Select all that apply. 1. Frame clinical questions in PICOT format. 2. Collect the best and most relevant evidence. 3. Cultivate a spirit of inquiry in the workplace. 4. Encourage the use of trial and error methods. 5. Base practical decisions on nursing consensus.

ANS: 1, 2, 3 Chapter: Chapter 2 Ethics and Standards of Practice Issues Chapter Learning Objective: 5. Analyze concepts related to evidence-based practice. Page: 32 Heading: Evidence-Based Practice > Evidence-Based Nursing > Barriers to Evidence-Based Practice Integrated Processes: Nursing Process Client Need: Safe and Effective Care Environment: Management of Care Cognitive Level: Analysis [Analyzing] Concept: Evidence-Based Practice Difficulty: Difficult Feedback 1 This is correct. The PICOT question format will encourage unit nurses to think in scientific terms as they approach their daily work. The PICOT approach includes the important factors that need to be considered when implementing EBP information into the clinical setting. 2 This is correct. Collecting the best, most relevant evidence will facilitate implementing EBP research into the clinical setting. 3 This is correct. A spirit of inquiry is essential for fostering a desire to do things the right, evidence-based way. 4 This is incorrect. Trial and error is inappropriate in the clinical setting and may place patients, nurses, and facilities at ethical and legal risk. Changes in clinical practices need to be backed by research evidence. 5 This is incorrect. Deciding clinical practices by consensus may place patients, nurses, and facilities at both ethical and legal risk. Changes in clinical practices need to be backed by research evidence.

A patient in the third trimester of pregnancy expresses concern to the nurse about changes to her muscles, joints, and bones. Which conditions does the nurse reassure the patient are normal changes of pregnancy? Select all that apply. 1. Waddling gait 2. Low back pain 3. Increased risk of falls 4. Fractures 5. Severe muscle aches

ANS: 1, 2, 3 Chapter: Chapter 4 Physiological Aspects of Antepartum Care Chapter Learning Objective: 4. Link the anatomical and physiological changes of pregnancy to signs, symptoms, and common discomforts of pregnancy. Page: 58 Heading: Table 4-1: Physiological Changes in Pregnancy Integrated Processes: Nursing Process Client Need: Physiological Integrity: Physiological Adaptation Cognitive Level: Analysis [Analyzing] Concept: Ante/Intra/Post-partum Difficulty: Moderate Feedback 1 This is correct. A waddling gait is a normal change during pregnancy and related to increased progesterone and relaxin levels causing softening of joints and increased joint mobility. Widening and increased mobility of the sacroiliac and symphysis pubis result. 2 This is correct. Low back pain is expected during pregnancy and related to increased progesterone and relaxin levels leading to softening of joints and increased joint mobility, resulting in widening and increased mobility of the sacroiliac and symphysis pubis. 3 This is correct. Although it is hazardous, increased risk of falls is expected during pregnancy due to a shift in the center of gravity related to the enlarged uterus. The patient needs to take precautions to avoid falls or activities requiring balance. 4 This is incorrect. Fractures are not expected or normal during pregnancy. 5 This is incorrect. Muscle aches are not normal during pregnancy and may signal an electrolyte imbalance.

The nurse is caring for a 30-year-old woman who is G4P2012 at 38 weeks and 5 days gestation. The nurse is watching her EFM strip and notices that for the past 10 minutes the fetus has shown minimal variability. Which actions should the nurse perform? Select all that apply. 1. Check to see if the patients' membranes have been ruptured, as there could be potential cord compression. 2. Look to see what the patient's position is and is she supine; change her to left side lying. 3. Continue to watch the strip, but know that this could be due to fetal sleep. 4. Recognize that the fetus is 38 weeks and therefore this could be due to fetal maturity. 5. Check the FHR and connect the minimal variability to fetal bradyasystole.

ANS: 1, 2, 3 Chapter: Chapter 9 Fetal Heart Rate Assessment Chapter Learning Objective: Articulate the physiology of FHR patterns. Page: 286 Heading: FHR and Contraction Pattern Interpretation > Baseline Fetal Heart Rate Integrated Processes: Nursing Process Client Need: Physiological Integrity: Reduction of Risk Potential Cognitive Level: Application [Applying] Concept: Ante/Intra/Post-partum Difficulty: Moderate Feedback 1 This is correct. Minimal variability can be caused by cord compression. 2 This is correct. Minimal variability can be caused by maternal supine hypotension. 3 This is correct. Minimal variability can be caused by fetal sleep. 4 This is incorrect. Minimal variability can be caused by fetal prematurity. 5 This is incorrect. Minimal variability can be caused by tachysystole.

During a vaginal delivery, the primary care provider notices greenish yellow coloration on the fetal head during crowning. Intrapartum suctioning is performed as soon as the fetus's head is delivered. The nurse understands the aspiration of meconium will have which effects on the neonate's respiratory function? Select all that apply. 1. Result in airway obstruction 2. Contribute to pulmonary hypertension 3. Result in chemical pneumonitis 4. Cause surfactant dysfunction 5. Create strain on cardiac function

ANS: 1, 2, 3, 4 Chapter: Chapter 10 High-Risk Labor and Birth Chapter Learning Objective: 3. Identify potential complications of dystocia in labor and related nursing and medical care. Page: 328 Heading: Obstetric Complications > Meconium-Stained Fluid and Birth Integrated Processes: Nursing Process Client Need: Physiological Integrity: Reduction of Risk Potential Cognitive Level: Analysis [Analyzing] Concept: Ante/Intra/Post-partum Difficulty: Difficult Feedback 1 This is correct. Meconium aspiration syndrome (MAS) will result in airway obstruction, which is relative to the amount and consistency of the aspirated meconium. 2 This is correct. Due to airway obstruction related to MAS, there is a high risk for the development of pulmonary hypertension. 3 This is correct. Chemical pneumonitis is inflammation of the bronchial structures of the lungs caused by chemicals and/or substances that are foreign to the lungs. The presence of meconium or the medications used to treat MAS could be sources for the condition. 4 This is correct. If the meconium is aspirated deeply enough, the air sacs may be coated with the aspirate, which will interfere with the functioning of surfactant. 5 This is incorrect. The question specifically asks for identification of MAS on the fetus/neonate's respiratory function.

When assisting with a vacuum-assisted vaginal delivery, the nurse is aware that adherence to which guidelines for the vacuum device will minimize the nurse's liability in vacuum-assisted vaginal births? Select all that apply. 1. Pump up the vacuum manually to the pressure indicated on the pump. 2. Recognize that cup detachment (pop off) is a warning sign. 3. Understand that pressure should be released between contractions. 4. The procedure is timed from insertion of the cup into the vagina until the birth. 5. The cup should not be on the fetal head for longer than 5 to 10 minutes.

ANS: 1, 2, 3, 4 Chapter: Chapter 10 High-Risk Labor and Birth Chapter Learning Objective: 4. Identify and manage high-risk pregnancy, labor, and delivery to promote healthy outcomes for the mother and infant. Page: 321 Heading: Operative Vaginal Delivery > Vacuum-Assisted Delivery Integrated Processes: Nursing Process Client Need: Physiological Integrity: Reduction of Risk Potential Cognitive Level: Analysis [Analyzing] Concept: Ante/Intra/Post-partum Difficulty: Difficult Feedback 1 This is correct. Pump up the vacuum manually to the pressure indicated on the pump, not to exceed 500 to 600 mm Hg. 2 This is correct. The nurse needs to be aware that cup detachment (pop off) is a warning that too much ineffective force is being exerted on the fetal head. 3 This is correct. It is important to understand that pressure should be released between contractions; retention of unnecessary pressure increases the risk for fetal injury. 4 This is correct. The procedure is timed from point of insertion of the cup into the vagina until the birth. If the vacuum-assisted delivery does not occur after three attempts within 15 to 20 minutes, cesarean is indicated. 5 This is incorrect. The cup should not be on the fetal head for longer than 15 to 20 minutes. The nurse needs to be the reminder of the "three-pull rule" that specifies that vacuum-assisted efforts need to be stopped after three attempts in 15 minutes.

A postpartum patient informs the nurse of a frequent urge and burning when attempting to urinate. The nurse reviews the patient's medical record and associates which risk factors related to a possible urinary tract infection (UTI)? Select all that apply. 1. Neonatal macrosomia 2. Use of a vacuum extractor 3. Poor oral fluid intake 4. Urinary catheter during labor 5. Low-grade fever (101.3°F [38.5°C])

ANS: 1, 2, 3, 4 Chapter: Chapter 14 High-Risk Postpartum Nursing Care Chapter Learning Objective: 2. Describe the primary postpartum infections and the related nursing actions and medical care. Page: 429 Heading: Infections > Urinary Tract Infection Integrated Processes: Nursing Process Client Need: Physiological Integrity: Reduction of Risk Potential Cognitive Level: Analysis [Analyzing] Concept: Ante/Intra/Post-partum Difficulty: Difficult Feedback 1 This is correct. Neonatal macrosomia, which can cause edema around the urethra, is a risk factor for UTI. 2 This is correct. Operative vaginal deliveries, forceps, or vacuum extractor, which can cause edema around the urethra, is a risk factor for UTI. 3 This is correct. The postpartum patient needs to drink a minimum of 3,000 mL/day; poor oral fluid intake is a risk factor for UTI. 4 This is correct. Urinary catheter inserted during the labor process is a risk factor for UTI. 5 This is incorrect. A low-grade fever (101.3°F [38.5°C]) is a symptom of UTI and not a cause or risk factor.

The obstetric nurse is assessing the laboring patient for pain. Which of the following should the nurse identify in a pain assessment? Select all that apply. 1. Intensity of contractions 2. Presence of pain in the neck or back 3. Frequency and duration of contractions 4. Signs of anxiety 5. Presence of FHR with intermittent auscultation

ANS: 1, 2, 3, 4 Chapter: Chapter 8 Intrapartum Assessment and Interventions Chapter Learning Objective: Demonstrate understanding of supportive care of the laboring woman. Page: 258 Heading: Pharmacological Management of Labor Discomfort Integrated Processes: Nursing Process Client Need: Physiological Integrity: Physiological Adaptation Cognitive Level: Application [Applying] Concept: Ante/Intra/Post-partum Difficulty: Moderate Feedback 1 This is correct. In a pain assessment, the nurse should assess for intensity. 2 This is correct. In a pain assessment, the nurse should assess for location. 3 This is correct. In a pain assessment, the nurse should assess for pattern. 4 This is correct. In a pain assessment, the nurse should assess for degree of distress. 5 This is incorrect. Fetal heart tones are not part of a pain assessment.

A woman has recently given birth to an infant born at 35 weeks and 5 days gestation. What long-term effects should the nurse be concerned about with the infant being born at this gestation? Select all that apply. 1. Cerebral palsy 2. Respiratory disorders 3. Developmental delays 4. Visual impairments 5. Hearing impairments

ANS: 1, 2, 3, 4, 5 Chapter: Chapter 1 Trends and Issues Chapter Learning Objective: 1. Discuss current trends in the management of labor and birth. Page: 6 Heading: Trends > Preterm Births Integrated Processes: Nursing Process Client Need: Health Promotion and Maintenance Cognitive Level: Comprehension [Understanding] Concept: Health Promotion Difficulty: Moderate Feedback 1 This is correct. Cerebral palsy is a long-term effect of preterm birth. 2 This is correct. Respiratory disorders are a long-term effect of preterm birth. 3 This is correct. Developmental delays are a long-term effect of preterm birth. 4 This correct. Visual and hearing impairment is a long-term effect of preterm birth. 5 This is correct. Hearing impairments are a long-term effect of preterm birth.

A postpartum patient states, "I am really in pain." For which sources of pain will the nurse specifically assess the patient? Select all that apply. 1. Uterine contractions 2. Perineal trauma 3. Breast engorgement 4. Hemorrhoids 5. General soreness

ANS: 1, 2, 3, 4, 5 Chapter: Chapter 12 Postpartum Physiological Assessments and Nursing Care Chapter Learning Objective: 2. Identify the critical elements of assessment and nursing care during the postpartum period. Page: 372 Heading: The Reproductive System > Vagina and Perineum Integrated Processes: Nursing Process Client Need: Physiological Integrity: Physiological Adaptation Cognitive Level: Analysis [Analyzing] Concept: Ante/Intra/Post-partum Difficulty: Difficult Feedback 1 This is correct. The nurse will specifically assess for uterine contractions or afterpains being a source of pain. 2 This is correct. The nurse will specifically assess for perineal trauma being a source of pain. Perineal trauma includes episiotomy, lacerations, and/or ecchymosis. 3 This is correct. The nurse will specifically assess for breast engorgement as being a source of pain. Breastfeeding mothers may also have nipple pain caused by improper nipple latching by the neonate. 4 This is correct. The nurse will specifically assess for hemorrhoids as being a source of pain. 5 This is correct. The nurse will specifically assess for general soreness as being a source of pain.

A patient is experiencing pregnancy complications. Which factors will affect the client's ability to manage this situation? Select all that apply. 1. Current health status 2. Perceived threat to self or fetus 3. Previously used coping skills 4. Existence of a support network 5. Implemented nursing interventions

ANS: 1, 2, 3, 4, 5 Chapter: Chapter 5 The Psycho-Social-Cultural Aspects of the Antepartum Period Chapter Learning Objective: 4. Identify nursing assessments and interventions that promote positive psycho-social-cultural adaptations for the pregnant woman and her family. Page: 112 Heading: Sexuality in Pregnancy > Nursing Actions Integrated Processes: Nursing Process Client Need: Psychosocial Integrity Cognitive Level: Analysis [Analyzing] Concept: Patient-Centered Care Difficulty: Difficult Feedback 1 This is correct. The patient's physical condition will impact the patient's ability to manage this situation. 2 This is correct. The patient's perceived threat to herself or the fetus will affect the patient's ability to manage this situation. 3 This is correct. The patient's previously used coping skills will affect the patient's ability to manage this situation. 4 This is correct. The existence of a support network will assist the patient in her ability to manage this situation. 5 This is correct. The recognition of patient needs and the implementation of appropriate nursing interventions will increase that patient's ability to manage this situation.

The nursing staff in a labor and delivery unit has noticed an increase in the number of patients experiencing placental abruption. The nurses begin to review demographics for the patients involved. Which risk factors will the nurses expect? Select all that apply. 1. Hypertensive disorders 2. Uterine fibroids 3. Cigarette smoking 4. Methamphetamine use 5. Abdominal trauma

ANS: 1, 2, 3, 4, 5 Chapter: Chapter 7 High-Risk Antepartum Nursing Care Chapter Learning Objective: 3. Identify potential antenatal complications for the woman, the fetus, and the newborn Page: 183 Heading: Placental Abnormalities and Hemorrhagic Complications > Placental Abruption Integrated Processes: Nursing Process Client Need: Physiological Integrity: Reduction of Risk Potential Cognitive Level: Analysis [Analyzing] Concept: Ante/Intra/Post-partum Difficulty: Difficult Feedback 1 This is correct. Hypertensive disorders put women at risk for placental abruption during pregnancy. 2 This is correct. Uterine fibroids put women at risk for placental abruption during pregnancy. 3 This is correct. Cigarette smoking puts women at risk for placental abruption during pregnancy. 4 This is correct. Methamphetamine use puts women at risk for placental abruption during pregnancy. 5 This is correct. Abdominal trauma puts women at risk for placental abruption during pregnancy.

The nurse is taking the history of a gravida 2 para 1 patient. Which findings in the patient's history warrant further action? Select all that apply. 1. Anemia 2. Severe hemorrhage 3. Infections 4. Malnutrition 5. Eclampsia

ANS: 1, 2, 3, 5 Chapter: Chapter 1 Trends and Issues Chapter Learning Objective: 2. Discuss current trends in maternal and infant health outcomes. Page: 9 Heading: Trends > Maternal Death and Mortality Rates Integrated Processes: Nursing Process Client Need: Physiological Integrity: Reduction of Risk Potential Cognitive Level: Application [Applying] Concept: Ante/Intra/Post-partum Difficulty: Difficult Feedback 1 This is correct. Anemia is a primary cause of maternal death. 2 This is correct. Severe hemorrhage is a primary cause of maternal death. 3 This is correct. Infection is a primary cause of maternal death. 4 This is incorrect. Malnutrition is not a primary cause of maternal death. 5 This is correct. Eclampsia is a primary cause of maternal death.

A patient in labor receives high-level regional anesthesia, which inhibits her ability to push during the second state of labor. The primary care provider will use forceps to aid in the delivery of the fetus. Which fetal complications is the nurse aware of being related to a forceps birth? Select all that apply. 1. Intracranial hemorrhage 2. Cephalohematoma 3. Nerve injuries 4. Skin lacerations 5. Skull fracture

1, 2, 3, 4, 5 Chapter: Chapter 10 High-Risk Labor and Birth Chapter Learning Objective: 3. Identify potential complications of dystocia in labor and related nursing and medical care Page: 322 Heading: Operative Vaginal Delivery > Forceps-Assisted Delivery Integrated Processes: Nursing Process Client Need: Physiological Integrity: Reduction of Risk Potential Cognitive Level: Analysis [Analyzing] Concept: Ante/Intra/Post-partum Difficulty: Difficult Feedback 1 This is correct. Intracranial hemorrhage is a possible complication of a forceps delivery and results from pressure on the fetal head by the forceps during birth. 2 This is correct. Cephalohematoma is a possible complication of a forceps delivery and results from pressure on the head, which causes the rupture of small blood vessels and the collection of blood beneath the scalp. 3 This is correct. Nerve injuries are a possible complication of a forceps delivery and result from the pulling action of moving the fetus down the birth canal. 4 This is correct. Skin lacerations are a possible complication of forceps delivery and result primarily during the insertion of the forceps. 5 This is correct. Skull fracture is a possible complication of a forceps delivery and results from pressure on the fetal skull during insertion of the forceps and/or when exerting pressure during extraction of the fetus.

The nurse is encouraging cultural sensitivity among the nonmedical personnel in a prenatal clinic. Which type of family does the nurse identify as including children? Select all that apply. 1. Nuclear family 2. Extended family 3. Cohabitating family 4. Dyad family 5. Blended family

A patient is experiencing pregnancy complications. Which factors will affect the client's ability to manage this situation? Select all that apply. 1. Current health status 2. PerA patient is experiencing pregnancy complications. Which factors will affect the client's ability to manage this situation? Select all that apply. 1. Current health status 2. Perceived threat to self or fetus 3. Previously used coping skills 4. Existence of a support network 5. Implemented nursing interventionsceived threat to self or fetus 3. Previously used coping skills 4. Existence of a support network 5. Implemented nursing interventions

The nurse is providing care to a patient who is at 41 weeks gestation. Which factor about the patient does the nurse consider as an indication of late-term or post-term pregnancy? 1. Fetus is identified as a male 2. Patient's multiparity status 3. Delivered two babies at 38 weeks 4. History of regular menstruation

ANS: 1 Chapter: Chapter 10 High-Risk Labor and Birth Chapter Learning Objective: 5. Describe the key obstetrical emergencies and the related nursing and medical care. Page: 326 Heading: Obstetric Complications > Post-Term Pregnancy and Birth Integrated Processes: Nursing Process Client Need: Physiological Integrity: Reduction of Risk Potential Cognitive Level: Application [Applying] Concept: Ante/Intra/Post-partum Difficulty: Moderate Feedback 1 This is correct. Several risk factors for post-term pregnancy have been identified, including carrying a male fetus. 2 This is correct. The patient's multiparity status is not a factor related to post-term pregnancy. A history of post-term pregnancy is an identified risk factor. 3 This is correct. Having delivered two previous pregnancies at 38 weeks gestation is not an identified risk factor for a post-term pregnancy. 4 This is incorrect. A history of irregular menstruation may contribute to miscalculation about fetal age; a regular menstruation history is not a risk factor.

The nurse is providing postpartum care for a patient after a vaginal delivery. Which assessment finding causes the nurse to suspect endometritis from beta-hemolytic streptococcus? 1. Scant amount of odorless lochia 2. Presence of headache, malaise, and chills 3. Pain or discomfort in the midline lower abdomen 4. Elevated temperature greater than 100.4°F (38°C)

ANS: 1 Chapter: Chapter 14 High-Risk Postpartum Nursing Care Chapter Learning Objective: 2. Describe the primary postpartum infections and the related nursing actions and medical care. Page: 428 Heading: Infections > Endometritis Integrated Processes: Nursing Process Client Need: Physiological Integrity: Reduction of Risk Potential Cognitive Level: Application [Applying] Concept: Ante/Intra/Post-partum Difficulty: Moderate Feedback 1 This is correct. Endometritis from beta-hemolytic streptococcus specifically exhibits scant, odorless lochia in addition to the more universal signs of infection. 2 This is incorrect. The presence of headache, malaise, and chills is not specific to beta-hemolytic streptococcus infection. 3 This is incorrect. Pain or discomfort in the midline lower abdomen is not specific to beta-hemolytic streptococcus infection. 4 This is incorrect. Elevated temperature greater than 100.4°F (38°C) is not specific to beta-hemolytic streptococcus infection.

The nurse is teaching newborn care to an adolescent mother. When the nurse attempts to teach how to swaddle the newborn, the mother states, "What's the big deal about how to wrap up a baby?" The nurse needs to convey which reason as being most important for proper swaddling? 1. Improper swaddling can cause hip dysplasia. 2. Correct swaddling will increase the neonate's comfort. 3. Neonates are swaddled only until they can turn from front to back. 4. Two to three fingers need to fit between the infant's chest and the swaddle.

ANS: 1 Chapter: Chapter 16 Discharge Planning and Teaching Chapter Learning Objective: 8. Provide parents with information regarding newborn care that reflects the assessed learning needs of parents. Page: 500 Heading: Newborn Care > Swaddling Instructions Integrated Processes: Nursing Process Client Need: Physiological Integrity: Reduction of Risk Potential Cognitive Level: Analysis [Analyzing] Concept: Ante/Intra/Post-partum Difficulty: Moderate Feedback 1 This is correct. Improper swaddling can cause hip dysplasia. It is especially important to allow the hips to spread apart and bend up. In the womb, the legs are in a fetal position with the legs bent up across each other. Sudden straightening of the legs to a standing position can loosen the joints and damage the soft cartilage of the socket. This is the most important information for the nurse to convey. 2 This is incorrect. Correct swaddling will increase the neonate's comfort, and provides warmth and a sense of security that can have a calming effect, but another reason is most important. 3 This is incorrect. Infants are actually only swaddled until they can turn from back to front. Infants who are swaddled and roll from back to front while sleeping are at greater risk for SIDS. 4 This is incorrect. Check the tightness of swaddle. The parent should be able to get two to three fingers between the infant's chest and the swaddle, but this I not the most important reason to convey.

The nurse is providing support for the parents of a neonate born with anencephaly. The parents repeatedly state, "I don't believe this is happening to us. We were so careful during pregnancy." The nurse associates the parents' comments with which stage of grief? 1. Disbelief 2. Depression 3. Denial of reality 4. Anger with each other

ANS: 1 Chapter: Chapter 17 High-Risk Neonatal Nursing Care Chapter Learning Objective: 4. Describe the loss and grief process experienced by parents whose infant has died. Page: 545 Heading: Loss and Grief Integrated Processes: Nursing Process Client Need: Psychosocial Integrity Cognitive Level: Analysis [Analyzing] Concept: Ante/Intra/Post-partum Difficulty: Difficult Feedback 1 This is correct. The parents' comments indicate that they are experiencing the stage of grief associated with avoidance, disbelief, shock, or guilt. 2 This is incorrect. Pain, physical discomfort, depression, difficulty concentrating, and anger at self or partner are stages of grief. The parents' comments do not reflect depression. 3 This is incorrect. Denial of reality is not expressed in the parents' comments. Denial is not specifically relative to loss and grief associated with a child. 4 This is incorrect. The parents' comments do not reflect anger with each other; the emotion expressed is more closely related to disbelief.

A patient arrives to the family practice clinic for her annual examination. The nurse's assessment data includes thin, 35-year-old female, history of weight loss surgery and total hysterectomy, BMI is 19, patient has been taking corticosteroids for severe asthma. Which of the following is an expected diagnostic screening for a potential health problem? 1. Dual-energy x-ray absorptiometry scan 2. Serum electrolyte levels and vitamin D 3. Serum cholesterol and diabetic screening. 4. Papanicolaou (Pap) Smear

ANS: 1 Chapter: Chapter 18 Well Women's Health Chapter Learning Objective: 5. Discuss the physical and emotional changes related to perimenopause and menopause; 2. Discuss preventative screenings for women across the life span. Page: 563 Heading: Osteoporosis > Risk Factors for Osteoporosis Integrated Processes: Teaching and Learning Client Need: Health Promotion and Maintenance Cognitive Level: Analysis [Analyzing] Concept: Promoting Health Difficulty: Moderate Feedback 1 This is correct. The dual-energy x-ray absorptiometry (DXA) is indicated for this patient because she has the following risk factors: corticosteroid use, low BMI, and surgical menopause. 2 This is incorrect. Although it may be a screening tool for the risk factors, the DXA screen is used to screen for osteoporosis. 3 This is incorrect. There is no indication for this screening assessment. 4 This is incorrect. The patient has had a total hysterectomy and there is no need for a Pap test.

The nurse on a maternity unit is an Orthodox Jew whose rabbi teaches that pregnancy terminations are permitted only to save the life of the mother. Which situation becomes possible if the nurse puts aside personal values and assists with elective terminations? 1. Moral distress 2. Legal actions 3. Loss of spirituality 4. Professional dilemma

ANS: 1 Chapter: Chapter 2 Ethics and Standards of Practice Issues Chapter Learning Objective: 1. Define key terms. Page: 23 Heading: Ethics in Nursing Practice > Ethical Dilemmas Integrated Processes: Nursing Process Client Need: Safe and Effective Care Environment: Management of Care Cognitive Level: Application [Applying] Concept: Ethics Difficulty: Moderate Feedback 1 This is correct. When a nurse puts aside values and carries out an action believed to be wrong, it creates a situation of moral distress. 2 This is incorrect. If the nurse is assisting with elective pregnancy terminations in a legally identified location and manner, it is unexpected for the nurse to face legal actions. 3 This is incorrect. The nurse may or may not experience a loss of spirituality. The development of moral distress is most likely. 4 This is incorrect. The nurse may experience some type of personal dilemma; however, professional dilemma is not likely.

A patient who is pregnant asks the nurse when her baby is due to be born. The patient reports her last menstrual period (LMP) date as April 14. Using Naegele's rule, the nurse will set the estimated date of delivery (EDD) as what date? 1. July 21 2. January 7 3. July 14 4. January 21

ANS: 1 Chapter: Chapter 4 Physiological Aspects of Antepartum Care Chapter Learning Objective: 2. Describe methods for diagnosis of pregnancy and determination of estimated date of delivery. Page: 76 Heading: Pregnancy > Due Date Calculation Integrated Processes: Nursing Client Need: Health Promotion and Maintenance Cognitive Level: Application [Applying] Concept: Ante/Intra/Post-partum Difficulty: Moderate Feedback 1 This is correct. A fetal ultrasound will provide information about the fetal development, allowing for an accurate estimated date of delivery (EDD). The nurse expects this manner of determination. 2 This is incorrect. The gestational wheel can only be used if the date of the LMP is known. 3 This is incorrect. The birthdays of the child's siblings are irrelevant. 4 This is incorrect. Whether a mother carries to term is secondary to determining the normal EDD.

The nurse is helping the patient assume a position for placement of an epidural. Which of the following positions are appropriate for placing an epidural? Select all that apply. 1. Lateral position with head flexed toward chest 2. Trendelenburg position 3. Lithotomy position 4. Sitting position with elbows on knees 5. Prone position

ANS: 1, 2 Chapter: Chapter 8 Intrapartum Assessment and Interventions Chapter Learning Objective: Demonstrate understanding of supportive care of the laboring woman. Page: 261 Heading: Pharmacological Management of Labor Discomfort > Epidural Anesthesia > Nursing Actions During Administration of Epidural Anesthesia Integrated Processes: Nursing Process Client Need: Safe and Effective Care Environment: Management of Care Cognitive Level: Application [Applying] Concept: Ante/Intra/Post-partum Difficulty: Moderate Feedback 1 This is correct. The lateral position would give good access to the spine. 2 This is incorrect. This does not give access to the spine. 3 This is incorrect. This does not give access to the spine. 4 This is correct. The sitting position with head flexed would give good access to the spine. 5 This is incorrect. The mother will not be able to easily assume this position, and it will not give access to the spine.

The nurse is aware of concern about the increasing numbers of severe maternal morbidity (SMM). It is believed to be related to changes in the overall health of the population of women giving birth. Which reasons does the nurse identify as causes of SMM? Select all that apply. 1. Increases in maternal age 2. Prepregnancy obesity 3. Cesarean deliveries 4. Inability to pay for health care 5. Preexisting chronic medical conditions

ANS: 1, 2, 3, 5 Chapter: Chapter 14 High-Risk Postpartum Nursing Care Chapter Learning Objective: 1. Describe the primary causes of postpartum hemorrhage and the related nursing actions and medical care. Page: 418 Heading: Introduction Integrated Processes: Nursing Process Client Need: Physiological Integrity: Reduction of Risk Potential Cognitive Level: Analysis [Analyzing] Concept: Ante/Intra/Post-partum Difficulty: Difficult Feedback 1 This is correct. Documented increases in maternal age is a likely cause for SMM; older women have increased risk. 2 This is correct. Obesity is a general health risk in the United States; prepregnancy obesity causes increased incidences of SMM. 3 This is correct. Due to improved diagnostic technology and increased litigation related to childbirth, cesarean deliveries are increasing. Surgical procedures always carry a risk for complications. 4 This is incorrect. The inability to pay for health care may or may not impact the increasing incidence of SMM. 5 This is correct. Preexisting chronic medical conditions are a contributor to the increasing rates of SMM. Due to a decrease in overall general health of women, complications are more likely.

A new mother expresses severe frustration with an infant that is exhibiting symptoms of colic. Which suggestions from the nurse are aimed at infant safety? Select all that apply. 1. Hold the infant and sway from side to side or walk around with the infant. 2. Place the infant in a car seat and take him or her for a ride in the car. 3. Place the baby in a safe place and allow the baby to cry for 10 to 15 minutes. 4. Do simple household chores, such as vacuuming or washing the dishes. 5. Place the infant (abdomen down) over the knees and gently rub or pat the back.

ANS: 3, 4 Chapter: Chapter 16 Discharge Planning and Teaching Chapter Learning Objective: 8. Provide parents with information regarding newborn care that reflects the assessed learning needs of parents. Page: 494 Heading: Newborn Care > Colic Integrated Processes: Nursing Process Client Need: Physiological Integrity: Reduction of Risk Potential Cognitive Level: Analysis [Analyzing] Concept: Ante/Intra/Post-partum Difficulty: Difficult Feedback 1 This is incorrect. Holding the infant and swaying from side to side or walking around with the infant is an appropriate action for calming the infant with colic. However, the nurse is focusing on infant safety; this does not address the concern. 2 This is incorrect. Placing the infant in a car seat and taking him or her for a ride in the car can be very effective in calming an infant with colic. However, the mother who expresses severe frustration with a colicky baby may not be safe driving a car. 3 This is correct. When a mother expresses severe frustration with a colicky baby, she needs to place the baby in a safe place and allow the baby to cry for 10 to 15 minutes. The mother can check on the baby when she has calmed down. 4 This is correct. A mother who expresses severe frustration with a colicky baby needs to find a distraction. Doing simple household chores, such as vacuuming or washing the dishes can be effective and allow the mother time to calm down. 5 This is incorrect. Placing the infant (abdomen down) over the knees and gently rubbing or patting the back can be effective in calming a colicky baby. However, the mother who is extremely frustrated needs to prevent the opportunity to shake, throw, hit, slam, or jerk a child of any age.

The nurse uses research from Greenberg and Morris (1974) as a guideline for identifying the presence of engrossment in a new baby by the father. Which behaviors exhibit paternal-infant bonding related to engrossment? Select all that apply. 1. Seeing the baby as attractive 2. Perceiving the baby as being perfect 3. Having a desire to touch the baby 4. Indicating an increasing sense of self-esteem 5. Positively commenting about the baby's features

NS: 1, 2, 3, 4, 5 Chapter: Chapter 13 Transition to Parenthood Chapter Learning Objective: 3. Discuss bonding and attachment. Page: 407 Heading: Parent-Infant Contact > Paternal-Infant Contact Integrated Processes: Nursing Process Client Need: Psychosocial Integrity Cognitive Level: Analysis [Analyzing] Concept: Family Dynamics Difficulty: Difficult Feedback 1 This is correct. Evidence of paternal engrossment involves a visual awareness of the baby and seeing their baby as attractive. 2 This is correct. Evidence of paternal engrossment involves a perception that their baby is perfect. 3 This is correct. Evidence of paternal engrossment involves a tactile awareness of the baby and having a desire to touch the baby. 4 This is correct. Evidence of paternal engrossment involves an increase of self-esteem. 5 This is correct. Evidence of paternal engrossment involves an awareness of and positive comments about their infant's distinct features.

A patient with a history of hypertension is giving birth. During delivery, the staff was not able to stabilize the patient's blood pressure. As a result, the patient died shortly after delivery. This is an example of what type of death? 1. Early maternal death 2. Late maternal death 3. Direct obstetric death 4. Indirect obstetric death

PTS: 1 CON: Evidence-Based Practice 2. ANS: 4 Chapter: Chapter 1 Trends and Issues Chapter Learning Objective: 2. Discuss current trends in maternal and infant health outcomes. Page: 7 Heading: Trends > Maternal Death and Mortality Rates Integrated Processes: Nursing Process Client Need: Physiological Integrity: Reduction of Risk Potential Cognitive Level: Application [Applying] Concept: Ante/Intra/Post-partum Difficulty: Hard Feedback 1 This is incorrect. Early maternal death is not an example of maternal death. Examples of maternal death include late maternal death, indirect obstetric death, direct obstetric death, and pregnancy-related death. 2 This is incorrect. Late maternal death occurs 42 days after termination of pregnancy from a direct or indirect obstetric cause. 3 This is incorrect. Direct obstetric death results from complications during pregnancy, labor, birth, and/or postpartum period. 4 This is correct. Indirect obstetric death is caused by a preexisting disease, or a disease that develops during pregnancy.

The nurse is caring for a baby who is experiencing fetal tachycardia. Which action should the nurse take next? 1. Perform fetal scalp stimulation for 5 seconds. 2. Check maternal allergies in the patient chart. 3. Apply heat packs to the maternal chest and head. 4. Stimulate the fetus with a vibroacoustic device.

S: 2 Chapter: Chapter 9 Fetal Heart Rate Assessment Chapter Learning Objective: Articulate the physiology of FHR patterns. Page: 286 Heading: FHR and Contraction Pattern Interpretation > Baseline Fetal Heart Rate > Medical Management Integrated Processes: Nursing Process Client Need: Physiological Integrity: Reduction of Risk Potential Cognitive Level: Application [Applying] Concept: Ante/Intra/Post-partum Difficulty: Moderate Feedback 1 This is incorrect. Fetal scalp stimulation should be used for fetal bradycardia. 2 This is correct. Fetal tachycardia can be caused by maternal or fetal infection and antibiotics may be administered; typically, penicillin is administered and a penicillin allergy would require a different medication. 3 This is incorrect. Fetal tachycardia can be caused by maternal fever. Heat application would increase maternal temperature. 4 This is incorrect. Fetal movement can cause fetal tachycardia; a vibroacoustic device would cause fetal movement and is therefore unnecessary.


Ensembles d'études connexes

Ch 52 Nursing Management: Patients w/ Dermatologic Problems

View Set

Consumer Behavior Exam II Chapter 9

View Set